Você está na página 1de 733

JC-2 Examination Papers

2013

Maths
College
Yishun Junior College
Victoria Junior College
Tampines Junior College
Temasek Junior College
Serangoon Junior College
St Andrews Junior College
River Valley High School
Raffles Institution
Pioneer Junior College
Nanyang Junior College
National Junior College
Meridian Junior College
Millennia Institute
Jurong Junior College
Innova Junior College
HWA Chong Institute
Dunman High School
Catholic Junior College
Anderson Junior College
Anglo Chinese Junior College

H2
P1
P1
P1
P1
P1
P1
P1
P1
P1
P1
P1
P1
P1
P1
P1
P1
P1
P1
P1
P1

P2
P2
P2
P2
P2
P2
P2
P2
P2
P2
P2
P2
P2
P2
P2
P2
P2
P2
P2
P2

YISHUN JUNIOR COLLEGE


2013 JC2 PRELIMINARY EXAMINATION

9740/01

MATHEMATICS
Higher 2
Paper 1

20 AUGUST 2013

Additional materials :
Answer paper
Graph paper
List of Formulae (MF15)

TUESDAY 0800h 1100h

YISHUN JUNIOR COLLEGE YISHUN JUNIOR COLLEGE YISHUN JUNIOR COLLEGE YISHUN JUNIOR COLLEGE YISHUN JUNIOR COLLEGE
YISHUN JUNIOR COLLEGE YISHUN JUNIOR COLLEGE YISHUN JUNIOR COLLEGE YISHUN JUNIOR COLLEGE YISHUN JUNIOR COLLEGE
YISHUN JUNIOR COLLEGE YISHUN JUNIOR COLLEGE YISHUN JUNIOR COLLEGE YISHUN JUNIOR COLLEGE YISHUN JUNIOR COLLEGE
YISHUN JUNIOR COLLEGE YISHUN JUNIOR COLLEGE YISHUN JUNIOR COLLEGE YISHUN JUNIOR COLLEGE YISHUN JUNIOR COLLEGE
YISHUN JUNIOR COLLEGE YISHUN JUNIOR COLLEGE YISHUN JUNIOR COLLEGE YISHUN JUNIOR COLLEGE YISHUN JUNIOR COLLEGE
YISHUN JUNIOR COLLEGE YISHUN JUNIOR COLLEGE YISHUN JUNIOR COLLEGE YISHUN JUNIOR COLLEGE YISHUN JUNIOR COLLEGE
YISHUN JUNIOR COLLEGE YISHUN JUNIOR COLLEGE YISHUN JUNIOR COLLEGE YISHUN JUNIOR COLLEGE YISHUN JUNIOR COLLEGE
YISHUN JUNIOR COLLEGE YISHUN JUNIOR COLLEGE YISHUN JUNIOR COLLEGE YISHUN JUNIOR COLLEGE YISHUN JUNIOR COLLEGE
YISHUN JUNIOR COLLEGE YISHUN JUNIOR COLLEGE YISHUN JUNIOR COLLEGE YISHUN JUNIOR COLLEGE YISHUN JUNIOR COLLEGE
YISHUN JUNIOR COLLEGE YISHUN JUNIOR COLLEGE YISHUN JUNIOR COLLEGE YISHUN JUNIOR COLLEGE YISHUN JUNIOR COLLEGE
YISHUN JUNIOR COLLEGE YISHUN JUNIOR COLLEGE YISHUN JUNIOR COLLEGE YISHUN JUNIOR COLLEGE YISHUN JUNIOR COLLEGE
YISHUN JUNIOR COLLEGE YISHUN JUNIOR COLLEGE YISHUN JUNIOR COLLEGE YISHUN JUNIOR COLLEGE YISHUN JUNIOR COLLEGE
YISHUN JUNIOR COLLEGE YISHUN JUNIOR COLLEGE YISHUN JUNIOR COLLEGE YISHUN JUNIOR COLLEGE YISHUN JUNIOR COLLEGE
YISHUN JUNIOR COLLEGE YISHUN JUNIOR COLLEGE YISHUN JUNIOR COLLEGE YISHUN JUNIOR COLLEGE YSIHUN JUNIOR COLLEGE
YISHUN JUNIOR COLLEGE YISHUN JUNIOR COLLEGE YISHUN JUNIOR COLLEGE YISHUN JUNIOR COLLEGE YSIHUN JUNIOR COLLEGE
YISHUN JUNIOR COLLEGE YISHUN JUNIOR COLLEGE YISHUN JUNIOR COLLEGE YISHUN JUNIOR COLLEGE YSIHUN JUNIOR COLLEGE
YISHUN JUNIOR COLLEGE YISHUN JUNIOR COLLEGE YISHUN JUNIOR COLLEGE YISHUN JUNIOR COLLEGE YSIHUN JUNIOR COLLEGE
YISHUN JUNIOR COLLEGE YISHUN JUNIOR COLLEGE YISHUN JUNIOR COLLEGE YISHUN JUNIOR COLLEGE YSIHUN JUNIOR COLLEGE
YISHUN JUNIOR COLLEGE
3 hoursYISHUN JUNIOR COLLEGE YISHUN JUNIOR COLLEGE YISHUN JUNIOR COLLEGE YSIHUN JUNIOR COLLEGE

TIME

READ THESE INSTRUCTIONS FIRST


Write your name and CTG in the spaces provided on all the work you hand in.
Write in dark blue or black pen on both sides of the paper.
You may use a soft pencil for any diagrams or graphs.
Do not use staples, paper clips, highlighters, glue or correction fluid.
Answer all the questions.
Give non-exact numerical answers correct to 3 significant figures, or 1 decimal place in the case
of angles in degrees, unless a different level of accuracy is specified in the question.
You are expected to use a graphic calculator.
Unsupported answers from a graphic calculator are allowed unless a question specifically states
otherwise.
Where unsupported answers from a graphic calculator are not allowed in a question, you are
required to present the mathematical steps using mathematical notations and not calculator
commands.
You are reminded of the need for clear presentation in your answers.
At the end of the examination, write down the question number of the questions attempted,
model of calculator used on the spaces provided on the cover page. Tie your cover page on top
of the answer scripts before submission.
The number of marks is given in brackets [ ] at the end of each question or part question.

This question paper consists of 4 printed pages.

Yishun Junior College 2013 JC2 Preliminary Exam H2 Mathematics 9740/01


1

Use the method of mathematical induction to prove that for all n ,


n
sin 2n 1 sin
cos 2r
,
0 .

2sin
r 1

[6]

Hence find

cos r .
2

[2]

r 1

(i)

(ii)

Using the method of differences, show that


n
1
B
C
A

n2 n3
r 1 r 1 r 3
where A, B, and C are constants to be determined.
[4]

1
Explain why the series
converges, and write down its value. [2]
r 1 r 1 r 3

(iii) Hence show that

r 1

r 2

5
.
12

[2]

Four housewives bought three different types of meat from the same meat seller. The
mass of meat and the total each housewife paid are shown in the table below.
Mrs Lee
Mrs Nasri
Mrs Vicnesh
Mrs Parker
Chicken (kg)
0.3
0.5
0.6
0.8
Mutton (kg)
0.4
0
0.5
a
Duck (kg)
0.6
0.4
0
1
Total amount ($)
10.7
6.5
7
18
Find the value of a.
[4]

(i)

Without using a calculator, solve the inequality

(ii)

Hence solve 1

1 5ln x
0.
ln x 2 ln x 6

5x 1
1.
x x6
2

[3]
[3]

ex
A
C , where C is an
Find the value of A such that 2 x
dx 2 x
e 1
e 1
arbitrary constant.
[2]
2x
e
The region under the curve y 2 x
, 0 x ln 2 is rotated 2 radians
e 1
about the x-axis to form a solid. Using the result in (i), find the exact
volume of the solid.
[4]
2

(a)

(i)

(ii)

(b)

A curve has parametric equations


x 4sin ,
y cot , where 0 .
(i) Sketch the curve.
[2]
(ii) Find the exact area of the region bounded by the curve, the 2 axes and the
line y 1 .
[4]

Page 2 of 4

Yishun Junior College 2013 JC2 Preliminary Exam H2 Mathematics 9740/01


6

The points A, B and C have position vectors i k , 4i j 6k , 7i xj 4k , where


x 0.
(i) Evaluate x, given that BC 17 .
The point P lies on AC such that BP is perpendicular to AC. Find
(ii) the coordinates of P,
(iii) the coordinates of B ' , the reflection of B in the line AC,
(iv) the exact area of the quadrilateral ABCB ' .

[2]
[5]
[2]
[3]

The equation of a curve is y 2 xy 1 .


(i) Find the equations of all tangents to the curve that are parallel to the y-axis. [4]
(ii) State and justify whether the curve has any stationary points.
[2]
(iii) Find the area of the region bounded by the axes, and the normal to the curve at
the point where the y-coordinate is 2.
[5]

(i)

(ii)

(i)

4 x
in ascending powers of x, up to and including the term in x 2 .
1 x
State the set of values of x for which the expansion is valid.
[5]
1
By substituting x , obtain an estimate for 390 , leaving your answer as a
10
fraction.
[2]
Expand

An open container is made in the form of an upright


prism with an equilateral triangular base of side x, and
height h.
Each unit of area of the base costs a and each unit of
area of the sides costs b, where a and b are constants.
The total cost of the container is a fixed amount c.
Prove that the capacity, V, of the container is
3
x 4c 3ax 2 .
[3]
48b

(ii)

10

(i)
(ii)

Hence show that V is maximum when the cost of the base is

c
.
3

[4]

w
Given that w 2 , Re w 0 , arg * , prove that w 3 i .
[3]
w 3
Hence find the equation of the locus z a r , where a, r , which contains
the points representing i and w.
[2]
(a) Find the exact maximum and minimum possible values of z 3i .
[3]
(b)

3
7.
Find the range of arg z

Page 3 of 4

[2]

Yishun Junior College 2013 JC2 Preliminary Exam H2 Mathematics 9740/01


11

(a)

Find the equation of the curve in the form y f x , given that it passes through

d2 y 6x4 2
.
[4]

dx 2
x3
Perry is jogging in a park. The distance covered, x kilometres, varies with time, t
hours. As he jogs, the difference between his initial speed and speed at time t, is
proportional to the distance he has covered. Given that he starts his jog with a
speed of 5 km/h, and his speed is 4 km/h when he has jogged 2 km, find the exact
time he takes to complete a 5 km jog.
[6]
the points 1, 0 , 1, 2 , and

(b)

12

x2
.
x2
Find the equation(s) of the asymptote(s) of C.
[1]
Sketch the curve C, labelling the equation(s) of its asymptote(s) and coordinates
of any axial intercepts and turning points.
[2]

The curve C has equation y


(i)
(ii)

(iii) Hence find the range of values of k for which the equation x 2 k x 2 4 has no
real roots.

[2]
~ End of Paper ~

Page 4 of 4

YISHUN JUNIOR COLLEGE


Mathematics Department
SOLUTION
Examination : JC2 Preliminary Examination
: JC2 H2 Maths

Subject
Qn
1
(i)

Let Pn be the statement cos 2r

n 1:

Solution
sin 2n 1 sin
2sin

r 1

LHS cos 2r cos 2


r 1

sin 3 sin
2sin
1
1

2 cos 3 sin 3
2
2

2sin
2 cos 2 sin

2sin
cos 2

RHS

LHS RHS , P1 is true.


Assume Pk is true for some k
k

cos 2r

i.e.

sin 2k 1 sin
2sin

r 1

To show Pk 1 is true.
k 1

cos 2r

i.e.

sin 2k 3 sin

r 1

2sin

k 1

LHS cos 2r
r 1

cos 2r cos 2k 2
r 1

sin 2k 1 sin

cos 2k 2
2sin
sin 2k 1 sin 2 cos 2k 2 sin

2sin
sin 2k 1 sin sin 2k 3 sin 2k 1

sin 2k 3 sin
2sin

2sin

RHS

Pk 1 is true

Hence by induction, Pn is true for all n

Page 1 of 11

, n

Date

: 20/08/2013

Paper No.

: 1

Qn
(ii)

2cos r 1
2

r 1

2 cos 2 r n
r 1
n

2 cos 2 r
r 1

cos r
2

Solution
sin 2n 1 sin

sin 2n 1 sin

r 1 r 3

2sin

sin 2n 1 sin 2n sin


2sin

sin 2n 1 2n 1 sin
4sin

r 1

2
(i)

2sin

a
b

r 1 r 3

1
1
, b
2
2
n
n
1
1 1
1

r 3
r 1 r 1 r 3
n 1 2 r 1

By cover-up method, a

1 1 1

2 2 4
1 1

3 5

1 1

4 6

1 1

5 7

1
1

n 1 n 1

1
1

n n2

1
1

n 1 n 3
5
1
1

12 2 n 2 2 n 3

1
5
, BC
2
12
1
1
0,
0
As n ,
n2
n3

1
Hence
converges and
r 1 r 1 r 3
Hence A

(ii)

(iii)

r 1 r 3 12
r 1

r 2 r 2 4r 4
r 1 r 3 r 2 4r 3
2
Hence r 2 r 1 r 3 0
2

Page 2 of 11

Solution

Qn
1

r 2

r 1

r 2

r 1 r 3

r 1

r 1 r 3

5
12

Let x, y, z be cost (per kg) of chicken, mutton, and duck respectively.


0.3x 0.4 y 0.6 z 10.7

0.4 z 6.5
0.5 x
0.6 x 0.5 y
7

From GC, x 5 , y 8 , z 10
0.8 5 a 8 110 18

4
(i)

Hence a 0.5
5x 1
1
2
x x6
x 2 x 6 5 x 1
0
x2 x 6
x2 4 x 5
0
x2 x 6
x 1 x 5 0
x 3 x 2

ve

(ii)

ve

ve

ve

ve

3
2
1
5
Hence x 3 , 1 x 2 , or x 5 .
1 5ln x
5ln x 1
1
0
1
2
2
ln x ln x 6
ln x ln x 6

5 y 1
1 , where y ln x
y y 6
Hence ln x 3 , 1 ln x 2 , or ln x 5 .
i.e. 0 x e3 , e1 x e2 , or x e5 .
2

5
(a)
(i)

e2 x
ex
d
x
dx

2x
2
2x
e 1
e 1
2
1
2e 2 x e 2 x 1 dx

2
1

C
2x
2 e 1

Page 3 of 11

Qn
(ii)

Solution
ln 2

ln 2

e2 x
Volume = 2 x
dx

0 e 1
0
u e2 x

u 2e 2 x

e4 x

e2 x 1

dx

e2 x
2x

2 e 1
2x

Volume
ln 2

ln 2

e2 x
e2 x



dx
2x
2x
0 e 1
2 e 1 0

ln 2
4
1 1

2x

ln e 1 0

2 4 1 2 1 1 2

1 2 1

ln 5 ln 2
4 5 2

1
5
3

ln
2 2 20
y

(b)
(i)

(ii)

y cot

dy
cosec2
d

y 1 / 4
y 0 / 2

Area x dy

Area 2 2

4 4sin cosec 2 d
2

4 4 cosec d

4 ln

2 1 ln 1 0

2 1

y dx

2 2 2 cot 4 cos d
4

2 cos
2 2 4
d

4 sin
2

2 2 4 2 cosec sin d

4 ln cosec cot 4
2

2 2

4 ln

2
2 2 4
ln cosec cot cos
4

4 ln 2 1

2 2 4 ln

1
2 1
2

Page 4 of 11

Qn
6
(i)

Solution

7 4 3

BC x 1 x 1
4 6 2

32 x 1 2 17
2

x 1
(ii)

(iii)

(iv)

x 1 2
x 3 or x 1 (rej. since x 0 )
7 1 6
2


AC 3 0 3 3 1
4 1 3
1


1
2


l AC : r 0 1 ,
1
1


1
2


Since P lies on AC, OP 0 1 , for some
1


1
1
2 4 2 3

BP 0 1 1 1
1
1 6 5

BP perpendicular to AC
2 3 2


1 1 0 2
5 1


1 2 5

OP 0 2 1 2
1 1 3

Thus P(5, 2, 3)

OP

OB OB '
2

5 4 6

OB ' 2OP OB 2 2 1 3
3 6 0

Thus B (6, 3, 0)
Since ABCB ' is a kite
(AC cuts BB ' at the midpoint perpendicularly),
Area of quadrilateral ABCB '

Page 5 of 11

Qn

2 Area of triangle ABC

Solution

1
BC AC
2
3 6
3 2
4



2 3 3 2 1 3 7



2 3
2 1
1
2

3 16 49 1

7
(i)

3 66
y 2 xy 1
dy
dy
2y y x 0
dx
dx
dy
2 y x y
dx
dy
y

dx 2 y x

Tangent parallel to y-axis implies 2 y x 0


x
y
2
2

(ii)

(iii)

x
x
x 1
2
2
1
x 2 1
4
2
x 4
x 2
Hence equation of tangents are x 2 and x 2
dy
y

dx 2 y x
dy
0 implies y 0 .
Hence
dx
But y 2 xy 1 .
Thus y 0 LHS 0 RHS
Hence there are no stationary points.
22 2 x 1
5
x
2

dy
2
4
5

, y2
dx 2 2 5 3
2

2
3
Hence gradient of normal is .
4
x

Page 6 of 11

Solution

Qn

3
5
y2 x
4
2

8
(i)

3
31
Equation of normal: y x
4
8
31
x 0 y 3.875
8
31
y 0 x 5.16
6
1 31 31
Hence area of region
2 6 8
961

10.0 3 s.f.
96
4 x
1 x
1

1
x 2
4 1 1 x
4

1 1
1 x 2 2 x 2

2 1

2
4
2

x x2

2 1
1 x x 2
8 128

1 x x 2

x x2

x2
2 1
x x2
8
8 128

9
143 2
2 x
x
4
64
x
Valid for:
1 and x 1 .
4
Hence x : 1 x 1
(ii)

9
(i)

1
2
10 2 9 1 143 1

1
4 10 64 10
1
10
10
39 11503

11 10 6400
126533
390
6400
Cost of base a Area of base
1
a x 2 sin 60
2
3 2

ax
4
4

Page 7 of 11

Qn

Solution

Cost of sides b Area of sides


b 3xh
3bhx

3 2
Hence
ax 3bhx c , i.e. h
4
V Area of base height

3 2
ax
4
3bx

3 2
c
ax
3 2
4

x
4
3bx
3

x 4c 3ax 2
48b

(ii)

3
4cx 3ax3
48b
dV
3

4c 3 3ax 2
dx 48b
dV
0 4c 3 3ax 2
dx

Hence cost of base

3 2 c
ax
4
3

d 2V
3

6 3ax 0 for x 0
2
dx
48b

Hence V is maximum when cost of base is


10
(i)

w
arg *
w 3

arg w arg w*
2arg w

arg w

c
3

Hence w 2 cos i sin


6
6

3 1
2
i
2
2
3 i

Page 8 of 11

Solution

Qn
(ii)

Let A, B, C represent a, w, i respectively on an Argand diagram.


Im

B
Re
A

Then AB AC r , thus A lies on perpendicular bisector of BC.

Im w Im i 1 . Thus perpendicular bisector of BC is vertical. Hence a


Thus
r i

3
2
2

3
3

1
1
2
4

(ii)
(a)

7
2
3

Im

Re
A

3
7
Maximum z 3i 3

2
2
2

39
7

4
2

1
39 7
2

Page 9 of 11

3
2

Solution

Qn
2

3
7
Minimum z 3i 32

2
2

(b)

39
7

4
2

1
39 7
2

Im

7
2

3
7
2

3
2

Re


2
6

3
Hence arg z1
7
6

2
6
sin

11
(a)

d2 y 6 x4 2

6 x 2 x 3
2
3
dx
x
dy
3x 2 x 2 C
dx
y x3 x 1 Cx D, C , D
1, 0 0 1 1 C D
C D 2 - - - - - (1)
1, 2 2 1 1 C D

(b)

C D 4 - - - - - (2)
Hence C 3 , D 1 .
Thus y x3 x 1 3x 1
dx
dx
5 x , initial
5
dt
dt
dx
Hence 5 kx, k 0
dt
dx
4 . Thus 4 5 k 2 .
x 2,
dt
1
k
2
dx
1
5 x
Hence
dt
2
dx 10 x

dt
2
Page 10 of 11

Qn

dt
2

dx 10 x
2
t
dx

10 x
2 ln 10 x C
x 0 , t 0 . Thus 0 2ln10 C
C 2ln10
Hence t 2ln10 2ln 10 x

Solution

x 5 t 2 ln10 2 ln 5
10
5
2 ln 2
2 ln
12
(i)

x2
4
x2
x2
x2
Asymptotes: y x 2 , x 2
y

(ii)

(4, 8)
x

y x2

x2
(iii)

x2 k x2 4

x2
k x 2
x2
For all k , y k x 2 cuts 2, 0 .
No real roots, hence y
Thus 0 k 1.

x2
does not intersect y k x 2 .
x2

Page 11 of 11

YISHUN JUNIOR COLLEGE


2013 JC2 PRELIMINARY EXAMINATION

9740/02

MATHEMATICS
Higher 2
Paper 2

21 AUGUST 2013

Additional materials:
Answer paper
List of Formulae (MF15)

WEDNESDAY 1400h 1700h

YISHUN JUNIOR COLLEGE YISHUN JUNIOR COLLEGE YISHUN JUNIOR COLLEGE YISHUN JUNIOR COLLEGE YISHUN JUNIOR COLLEGE
YISHUN JUNIOR COLLEGE YISHUN JUNIOR COLLEGE YISHUN JUNIOR COLLEGE YISHUN JUNIOR COLLEGE YISHUN JUNIOR COLLEGE
YISHUN JUNIOR COLLEGE YISHUN JUNIOR COLLEGE YISHUN JUNIOR COLLEGE YISHUN JUNIOR COLLEGE YISHUN JUNIOR COLLEGE
YISHUN JUNIOR COLLEGE YISHUN JUNIOR COLLEGE YISHUN JUNIOR COLLEGE YISHUN JUNIOR COLLEGE YISHUN JUNIOR COLLEGE
YISHUN JUNIOR COLLEGE YISHUN JUNIOR COLLEGE YISHUN JUNIOR COLLEGE YISHUN JUNIOR COLLEGE YISHUN JUNIOR COLLEGE
YISHUN JUNIOR COLLEGE YISHUN JUNIOR COLLEGE YISHUN JUNIOR COLLEGE YISHUN JUNIOR COLLEGE YISHUN JUNIOR COLLEGE
YISHUN JUNIOR COLLEGE YISHUN JUNIOR COLLEGE YISHUN JUNIOR COLLEGE YISHUN JUNIOR COLLEGE YISHUN JUNIOR COLLEGE
YISHUN JUNIOR COLLEGE YISHUN JUNIOR COLLEGE YISHUN JUNIOR COLLEGE YISHUN JUNIOR COLLEGE YISHUN JUNIOR COLLEGE
YISHUN JUNIOR COLLEGE YISHUN JUNIOR COLLEGE YISHUN JUNIOR COLLEGE YISHUN JUNIOR COLLEGE YISHUN JUNIOR COLLEGE
YISHUN JUNIOR COLLEGE YISHUN JUNIOR COLLEGE YISHUN JUNIOR COLLEGE YISHUN JUNIOR COLLEGE YISHUN JUNIOR COLLEGE
YISHUN JUNIOR COLLEGE YISHUN JUNIOR COLLEGE YISHUN JUNIOR COLLEGE YISHUN JUNIOR COLLEGE YISHUN JUNIOR COLLEGE
YISHUN JUNIOR COLLEGE YISHUN JUNIOR COLLEGE YISHUN JUNIOR COLLEGE YISHUN JUNIOR COLLEGE YISHUN JUNIOR COLLEGE
YISHUN JUNIOR COLLEGE YISHUN JUNIOR COLLEGE YISHUN JUNIOR COLLEGE YISHUN JUNIOR COLLEGE YISHUN JUNIOR COLLEGE
YISHUN JUNIOR COLLEGE YISHUN JUNIOR COLLEGE YISHUN JUNIOR COLLEGE YISHUN JUNIOR COLLEGE YSIHUN JUNIOR COLLEGE
YISHUN JUNIOR COLLEGE YISHUN JUNIOR COLLEGE YISHUN JUNIOR COLLEGE YISHUN JUNIOR COLLEGE YSIHUN JUNIOR COLLEGE
YISHUN JUNIOR COLLEGE YISHUN JUNIOR COLLEGE YISHUN JUNIOR COLLEGE YISHUN JUNIOR COLLEGE YSIHUN JUNIOR COLLEGE
YISHUN JUNIOR COLLEGE YISHUN JUNIOR COLLEGE YISHUN JUNIOR COLLEGE YISHUN JUNIOR COLLEGE YSIHUN JUNIOR COLLEGE
3 hoursYISHUN JUNIOR COLLEGE YISHUN JUNIOR COLLEGE YISHUN JUNIOR COLLEGE YSIHUN JUNIOR COLLEGE
YISHUN JUNIOR COLLEGE
YISHUN JUNIOR COLLEGE YISHUN JUNIOR COLLEGE YISHUN JUNIOR COLLEGE YISHUN JUNIOR COLLEGE YSIHUN JUNIOR COLLEGE

TIME

READ THESE INSTRUCTIONS FIRST


Write your name and CTG in the spaces provided on all the work you hand in.
Write in dark blue or black pen on both sides of the paper.
You may use a soft pencil for any diagrams or graphs.
Do not use staples, paper clips, highlighters, glue or correction fluid.
Answer all the questions.
Give non-exact numerical answers correct to 3 significant figures, or 1 decimal place in the case
of angles in degrees, unless a different level of accuracy is specified in the question.
You are expected to use a graphic calculator.
Unsupported answers from a graphic calculator are allowed unless a question specifically states
otherwise.
Where unsupported answers from a graphic calculator are not allowed in a question, you are
required to present the mathematical steps using mathematical notations and not calculator
commands.
You are reminded of the need for clear presentation in your answers.
At the end of the examination, write down the question number of the questions attempted,
model of calculator used on the spaces provided on the cover page. Tie your cover page on top
of the answer scripts before submission.
The number of marks is given in brackets [ ] at the end of each question or part question.

This question paper consists of 4 printed pages.

Yishun Junior College 2013 JC2 Preliminary Exam H2 Mathematics 9740/02


Section A: Pure Mathematics [40 marks]
1

The functions f and g are defined by


f : x 1 2ln x 1 ,

x , x 1,

g:x
x2 4 x 6
,
x , x 2.
1
(i) Explain why the function g exists and express g 1 in a similar form.
(ii) Show that the composite function fg exists. Define fg and find its range.
(iii) State a sequence of transformations which transform the graph of
y ln 1 x to the graph of y f x .
2

Find the equation of the plane 1 in scalar product form.


[3]
Find the Cartesian equation of the plane 2 that also contains the line l1 but is
perpendicular to the plane 1 .
[2]

(iii) The line l2 with equation r 5 i 7 2 j 3 a k,

[3]

The plane 1 contains the point (1, , 1) and the line l1 with equation y 1, x 1 z .
(i)
(ii)

[4]
[4]

(i)
(ii)

(i)

intersects

1 at 30 . Find the possible value(s) of a in exact form.

[3]

Find the series expansion of e4 x sin x , up to and including the term in x 4 .


Using your answer in (i), find the series expansion of e4 x cos x , up to and
including the term in x 3 .

[3]

Let f z z 3 z 2 bz 4, b

[2]

. Given f 1+i 3 0 , obtain all the roots of

f z 0 .

[3]

(ii) Hence find the exact roots of the equation 4w3 bw2 w 1 0 .
[2]
(iii) For the value of z found in (i) for which Im z 0 , find the smallest positive
integer n such that z n . State the value of z n when n takes this value.
5

[4]

Andy and Bob played a computer game last year.


(a)

Andy gained 450 points on the first day he played. On each subsequent day, he
gained 13 points less than what he gained on the previous day. He played daily up
to and including the day he gained less than 55 points. Find the total points Andy
gained.
[4]

(b)

Bob gained 400 points on the first day he played. On each subsequent day, he
gained 0.9 times of the points he gained on the previous day. Find the minimum
number of days for Bob to gain at least 95% of the theoretical maximum points
he can earn.
[3]

Page 2 of 4

Yishun Junior College 2013 JC2 Preliminary Exam H2 Mathematics 9740/02


Section B: Statistics [60 marks]
6

Betty has one black, one yellow, two red, two green and three blue beads. The nine
beads are arranged randomly in a line. Find the probability that
(i) the two red beads are next to each other,
[2]
(ii) all the red and green beads are separated,
[2]
(iii) either the two red beads are next to each other or the two green beads are next to
each other or both.
[3]
The nine beads are now threaded randomly on a ring.
(iv) Find the probability that the two red beads are next to each other and the two
green beads are next to each other.
[2]

I have a choice of two routes to get to school. The probability that I am punctual for
43
3
school is
and the probability that I choose the first route is . If I get to school
5
50
27
punctually, the probability that the first route is chosen is
. Find the probability that
43
I get to school punctually if
(i) I choose the first route,
[2]
(ii) I choose the second route.
[2]

In a factory, a machine fills and seals tin cans of milk automatically. It is given that 5 %
of the cans are dented.
(i) Given a random sample of 10 cans, find the probability that more than two cans
[2]
are dented.
(ii) Given 30 random samples of 100 cans, find the probability that the mean number
of dented cans per sample does not exceed four.
[3]
(iii) Find the least number of cans that must be taken such that the probability for
more than one can to be dented exceeds 0.95.
[3]
It is found that 99.7 % of the cans are not rusty.
(iv) Given a random sample of 1000 cans, use a suitable approximation to find the
probability that at most 998 cans are not rusty.
[3]

The random variable X has the distribution N , 2 . It is known that


P X 10 0.1 and P 9 X 10 0.2 . Calculate the values of and .

[4]

1.5 , find P X 2Y .

[3]

Another random variable Y has the distribution N 4.8, 1.32 . Given that 10 and

10

A journalist intends to conduct a survey to find out the perception of different age
groups of citizens on the effectiveness and relevance of the education system. Describe
how a quota sample of size 50 might be carried out in this context. Explain a
disadvantage of quota sampling in the context of your answer.
[3]
State, with a reason, whether stratified sampling is realistic to be used in this case.

Page 3 of 4

[1]

Yishun Junior College 2013 JC2 Preliminary Exam H2 Mathematics 9740/02


11

At a car rental firm, there are 5 cars and 3 vans available each day. The demand for cars
follows a Poisson distribution with a mean of 2 on a weekday and a mean of 5 per day
on weekends. The demand for vans follows a Poisson distribution with a mean of 2 on
any day of the week. The demand for cars and the demand for vans on any day are
independent.
(i) Find the probability that, on a particular weekday, the demand for cars is 5. [1]
(ii) Find the least number of cars the rental firm should have to meet the demand on a
particular weekday with at least a probability of 0.9.
[2]
(iii) Using a suitable approximation, find the probability that, out of 104 Saturdays,
there are at least 50 days in which the total demand for vehicles exceeds 6 each
day.
[4]
(iv) Find the probability that, on a Wednesday, 5 cars are hired out given that at most
6 vehicles were hired out on that day.
[4]

12

The college wishes to find out the number of hours spent per week, on average, by a
typical student on social media. A survey was conducted on a random sample of 80
students and the time spent per week, x hours, was recorded and summarised by
2
x 18 8967 .
x 18 208 ,
(i) Find, correct to 1 decimal place, the unbiased estimates of the population mean
and variance.
[2]
(ii) The Head of Department of Student Well-being claims that the population mean
weekly time spent on social media exceeds 0 , where 0 is a constant.
Use the earlier sample to determine, correct to 1 decimal place, the least value of
0 in order for the claim not to be valid at 5% level of significance.
[4]

13

An experiment was conducted to investigate how the mass of a drug in a human body
varies with time, measured from when the drug is given. The results are summarised in
the following table.
t (minutes)
x (milligrams)

10

15

20

30

40

55

70

90

1.36

0.93

0.72

0.50

0.39

0.31

0.26

0.21

(i)
(ii)

Draw a scatter diagram for the data.


[2]
It is desirable to predict the mass of the drug in the long run. Explain why, in this
context, neither a linear nor a quadratic model is likely to be appropriate.
[2]
(iii) It is decided that a model of the form t x a b will be used. Calculate the

least squares estimates of a and b and estimate the mass of the drug after two
hours.
[3]
(iv) Give an interpretation, in context, of the value of a.
[1]
~ End of Paper ~

Page 4 of 4

YISHUN JUNIOR COLLEGE


Mathematics Department
SOLUTION
Examination : JC2 Preliminary Examination

Date

: 21/08/2013

Subject

Paper No.

: 2

Qn
1
(i)

: JC2 H2 Maths
Solution

Graph of y g x , x 2
y

(2, 2)

Any horizontal line y k , k

cuts the graph of y g x , x 2 at most once. Hence g

is one-one.
Thus g 1 exists.
Let y x 2 4 x 6
y x 2 2
2

x2 y2
x 2 y 2 (rej.
g 1 : x

(ii)

y 2 since x 2 )

2 x 2, x 2

Rg 2, 1, Df

Hence fg exists.
fg( x) 1 2ln x 2 4 x 5 , x 2
Rfg , 1

(iii)

Translation of 2 units in the positive x-direction


Scaling parallel to the y-axis by a factor of 2,
followed by a translation of 1 unit in the positive
y-direction.
OR
Translation of 0.5 unit in the positive y-direction,
followed by scaling parallel to the y-axis by a
factor of 2.

2
(i)

0
1


l1 : r 1 0 ,
1
1


Page 1 of 9

Solution

Qn

(ii)

1 0 1

3 1 2
1 1 2

1 1 2

0 2 1
1 2 2

2 1 2

1 : r 1 3 1 3

2 1 2
1 2 1

0 1 4
1 2 1

1 0 1

2 : r 4 1 4 3
1 1 1

2 : x 4 y z 3 (Cartesian equation)

(iii)

sin 30

2 1

1 2
2 a

4 1 4 1 4 a2

1
2a

2
9 5 a2

3
(i)

(ii)

3 5 a2 4 a
9 5 a 2 16a 2
7a 2 45
45
a
7
4x
e sin x
2
3

4x 4x

1 4 x

x x3

2!
3!
3!

32

1
1 4 x 8 x 2 x3 x 1 4 x x3
3

6
47 3
x 4x2
x 10 x 4
6
47 2
4e4 x sin x e4 x cos x 1 8 x
x 40 x3
2
47 2
47 3


e4 x cos x 1 8 x
x 40 x3 4 x 4 x 2
x
2
6


15
26 3
1 4x x2
x
2
3

Page 2 of 9

Qn
4
(i)

Solution
By conjugate root theorem, z 1 i 3 is also a root.

Hence z 1 i 3

z 1 i 3 z

2 z 4 is a factor of f z .

Az B z 2 2 z 4 z 3 z 2 bz 4
(ii)

By comparison of coefficients, A 1 , B 1 .
Hence z 1 is also a root.
4w3 bw2 w 1 0
2

1 1 1
4 b 0
w w w
1
Hence w
z
1
1
1
, or
,
1 1 i 3
1 i 3

1,

(iii)

5
(a)

1 i 3
1 i 3
, or
4
4

z 1 i 3 2e 3

z n n k , k
3
Hence n 3k
Thus least n 3 and z 3 23 e i 8
an 55 450 13 n 1 55
13n 408
n 31.4 3 s.f.
Hence Andy stopped playing on day 32.
Total points
S32
32
2 450 13 32 1
3
7952
Sn 0.95S

(b)

6
(i)

400 1 0.9n

0.95

400
1 0.9

1 0.9
1 0.9n 0.95
0.9n 0.05
ln 0.05
n
28.4 3 s.f.
ln 0.9
Minimum number of days = 29 days.

Number of arrangements without restrictions =


Required probability =

8!
9!
2

2!3! 2!2!3! 9

9!
2!2!3!

Page 3 of 9

Qn
(ii)

Solution
No. of ways to slot in red beads and green beads = 6C4
Required probability =

(iii)

5! 6
4!
9!
5
C4

3!
2!2! 2!2!3! 42

P (2 red beads together and 2 green beads together) =


Required probability =P A B
P A P B P A B

=
(iv)

2 2 1
7

9 9 18 18

Number of arrangements without restrictions =


Required probability =

7
(i)

43/50

7/50

8!
2 2!2!3!

6!
8!
1

2(3!)2! 2(2!2!3!)2! 14
27/43

1st route

16/43

2nd route

Punctual

Late

1st route
2nd route

P punctual | 1st route

P punctual and 1st route


P 1st route

43 27

50 43
3
5
9

10
P punctual | 2nd route

(ii)

P punctual and 2nd route


P 2nd route

43 16

50 43
2
5
4

Page 4 of 9

4!
2!2!
7!
9!
1

3! 2!2!3! 18

Qn
8
(i)

Solution
Let X be the r.v. number of tin cans that are dented out of 10 cans.
X ~ B 10,0.05
P X 2 1 P X 2 0.0115

(ii)

Method 1
Let Y be the r.v. number of tin cans that are dented out of 100 cans.
Y ~ B 100,0.05
E(Y) = 5 and Var(Y) = 4.75
4.75
By CLT, Y ~ N 5,
approximately
30

P Y 4 0.00598

Method 2
Let T be the r.v. number of tin cans that are dented out of 3000 cans.
T ~ B 3000,0.05
Required probability = P T 120 0.00551
(iii)

Let W be the r.v. number of tin cans that are dented out of n cans.
W ~ B n,0.05
P W 1 0.95

Method 1
P W 1 0.05
When n 92, P W 1 0.0521
When n 93, P W 1 0.04998
Hence least n = 93
Method 2
1 P W 0 P W 1 0.95
0.05 0.95 n 0.05 0.95
n

n 1

Using GC, n > 92.98857.


Hence least n = 93
(iv)

Let V be the r.v. number of cans that are rusty out of 1000 cans.
V ~ B 1000,0.003
Since n = 1000 is large, p = 0.003 is small and np = 3 < 5
V ~ Po 3 approximately P V 2 1 P V 1 0.801

Page 5 of 9

Qn
9

P X 10 0.1

Solution

10

PZ
0.1

10 1.281551567 (equation 1)

P X 9 0.7

PZ
0.7

9 0.5244005101 (equation 2)
Solving, we have 8.31, 1.32

X 2Y ~ N 0.4, 9.01
P X 2Y 0 0.553

10

11
(i)

Quota sampling might be carried out by


- dividing the range of ages into several groups (strata such as 13-16, 17-19, 20-22, 2330, 31-50 etc)
- assigning a quota for each age group with a total of 50
- by waiting outside a library and picking citizens at the journalists discretion until all
the quotas are met.
One disadvantage is that the journalist may likely have collected a biased sample of
citizens due to his/her non random selection process. For example, he/she might choose
people that he/she thought are more approachable.
Stratified sampling is not realistic in this context because it is difficult to obtain the
sampling frame.
Let C be the r.v. demand for cars on a weekday.
C ~ Po 2
P C 5 0.0360894089 0.0361

(ii)

P C n 0.9

Using GC and listing all probabilities,


P C 3 0.85712 0.9
P C 4 0.94735 0.9

(iii)

Least number of cars required = 4


Let A be the r.v. demand for vehicles on a Saturday.
A ~ Po 7
P A 6 1 P A 6

0.5502889
Let W be the r.v number of Saturdays (out of 104) in which the total demand for
vehicles exceeds 6 each day.
W ~ B 104,0.5502889
Since n = 104 (large) and np = 57.2300456 > 5 and
nq =46.7699544 > 5,
W ~ N 57.2300456, 25.73698676 approximately
P W 50 P W 49.5 0.936 using continuity correction
Page 6 of 9

Qn
(iv)

Solution
Let V be the r.v. demand for vans on a weekday.
V ~ Po 2

P 5 cars hired| 6 vehicles hired


P 5 cars hired and 6 vehicles hired
P( 6 vehicles hired)
P 5 cars hired and 1 van hired

P(V 0) P C 0 P(V 1) C 0 P(V 2) P C 4 P(V 3) P C 3

P C 5 P V 1
P(V 0) P C 0 P(V 1) C 0 P(V 2) P C 4 P(V 3) P C 3

0.052653 0.4060058
0.9395530118

0.0228
12
(i)

Unbiased estimate of population mean


x 18 18 20.6
x
80

x 18
1
2
s
x 18

79
80

H 0 : 0
2

(ii)

42131

106.7
395

H1 : 0
X
~ N 0, 1 approximately by CLT, where
s
n
42131
x 20.6, s 2
, n 80 , 0
395

Under H 0 , Z

At 5% level of significance and one tail test, critical value = 1.645


In order not to reject
,
Test value < 1.645
20.6 0
1.645
41231
39580

0 18.7006
Least 0 18.8 . (1 d.p.)

Page 7 of 9

Qn
13
(i)

Solution
x

(10 , 1.36)

(90 , 0.21)
t
(ii)

(iii)

The mass of the drug could not continue to decrease linearly to a negative value. Thus, a
linear model is not appropriate.
A quadratic model would eventually lead to an increase in the mass of the drug.
However, the mass of the drug is likely to plateau off or stay constant after many hours.
Thus a quadratic model would also not be appropriate.
t x a b

1
x a b
t
Using GC, a 0.07177273 0.0718 and b 12.8879265 12.9

(iv)

When t = 120 mins,


We have x 0.1791721175 0.179 .
The amount of drugs that remains in the body after a long period of time will
approximate to a.

Page 8 of 9

VICTORIA JUNIOR COLLEGE


Preliminary Examination

MATHEMATICS
(Higher 2)

9740/01
September 2013
3 hours

Paper 1

READ THESE INSTRUCTIONS FIRST


Write your name and CT group on all the work you hand in.
Write in dark blue or black pen on both sides of the paper.
You may use a soft pencil for any diagrams or graphs.
Do not use staples, paper clips, highlighters, glue or correction fluid.
Answer all the questions.
Give non-exact numerical answers correct to 3 significant figures, or 1 decimal place in the case of angles in
degrees, unless a different level of accuracy is specified in the question.
You are expected to use a graphic calculator.
Unsupported answers from a graphic calculator are allowed unless a question specifically states otherwise.
Where unsupported answers from a graphic calculator are not allowed in a question, you are required to present
the mathematical steps using mathematical notations and not calculator commands.
You are reminded of the need for clear presentation in your answers.
At the end of the examination, fasten all your work securely together.
The number of marks is given in brackets [ ] at the end of each question or part question.

This document consists of 5 printed pages


VJC 2013

VICTORIA JUNIOR COLLEGE

[Turn over

Without using a calculator, solve the inequality


15 11x
1.
x2 6 x 9
Deduce the range of values of x that satisfies
15 11ln x

ln x

6ln x 9

[4]

1.

It is given that y 1 ln(1 x).


(i)

By differentiating successively, find the Maclaurins series for y, up to and


including the term in x 2 .

(ii)

[3]

1
4
By substituting x , approximate ln , leaving your answer in exact form. [3]
4
5

At the beginning of January 2010, Robert borrowed $200 000 from a bank that charges
0.4% interest at the end of every month. Robert pays back $1500 at the beginning of
every month, starting from February 2010. Show that the amount Robert still owes the
bank at the end of n months is 376500 175000(1.004)n .
[3]
How much did he pay on his last instalment?

[2]

[3]

A sequence of positive real numbers x1, x2, x3, ... satisfies the recurrence relation

xn1 xn 4 4 xn
for n 1 .
Given that x1 = 9, find the values of x2, x3 and x4. Hence, conjecture an expression for
xn in terms of n.
[2]
Prove your conjecture using mathematical induction.

[4]

Referred to the origin O, the points A, B and C have position vectors 6i 4 j 3k ,

3i 3j k and 3i j 4k respectively.
(i) Find the size of angle ABC.
(ii)

[3]

If P is a point on the line AB such that CP 83, find the possible coordinates
of P.

[4]

x=2
y = f(x)
y=5

2
0

The diagram shows the graph of y f ( x) which has a minimum point at (0, 2) and
asymptotes x 2 and y 5.
On separate diagrams, sketch the graphs of
1
(i) y
,
f ( x)

[3]

(ii) y 2 f ( x),

[3]

(iii) y f '( x).

[2]

The variables x and y are connected by the differential equation


By using the substitution u y x , show that

dy y 1 3

.
dx 2 x 2 2 x

du 1
3

x
.
dx 2
2 x

[3]

Sketch, for x 0, three typical members of the family of solution curves of the
differential equation

dy y 1 3

.
dx 2 x 2 2 x

[5]

(i)

r 2 3r 1 1
1

.
Show that
r 2 ! r ! r 2 !

(ii)

Hence find

r 1

r 2 3r 1
. (There is no need to express your answer as a single
r 2 !

algebraic fraction.)
(iii)

[2]

[3]

Using the result in part (ii) and the standard series expansion for e x , find the
exact value of

r 1

r 2 3r 3
.
r 2 !

[4]
[Turn over
3

The complex numbers w and z satisfy the relations w 6 8i w 8 6i and

z 8 6i 5 respectively.
(i) Illustrate both of these relations on a single Argand diagram.

[4]

(ii) Find, in either order,


(a) the least value of z w , and
(b)

the value of z in the form x iy when z w attains its least value,

leaving your answers in exact form.

[5]

(iii) Find the range of values for such that the system of equations

z 8 6i 5,
arg z 4i
has more than one solution for z.
10

(a)

[4]

The curve C has parametric equations


x 3 cos t , y 3sin t , for

(i)

.
2
2
Sketch C, indicating clearly the exact coordinates of the axial intercepts. [2]
t

(ii) Find the exact value of the area of the region bounded by C and the y axis.
[5]
(b)

(i)

Differentiate 1 x 2 with respect to x.

1
2

cos 1 x dx

[2]

1
.
2

(ii)

Show that

(iii)

The diagram below shows the curve with equation y cos1 x . The

4 2

region bounded by the curve, the line y

[3]

and the y-axis is denoted by

R.
y

y cos1 x

x
Find the exact volume of revolution when R is rotated completely about the
x-axis.
[4]
4

11

x3 y 2 z

; the line l2 passes through the points with


2
1
k
coordinates (1, 4, 2k ) and (3, 1, k ) , where k is a constant.

The line l1 has equation

(i)

Determine a cartesian equation of l2.

(ii)

Show that there are no real values of k for which l1 and l2 are perpendicular to
each other.

(iii)

[2]

[2]

The plane p has equation x 4 y z a, where a is a constant. If l1 and p have


no common points, what can be said about the values of k and a?

[4]

For the value of k found in part (iii) and for a 2, find the shortest distance between l1
and p.

[3]

It is given instead that k 3 and that l1 and l2 intersect. is the plane containing l1
and l2 . By first finding a vector perpendicular to or otherwise, find an equation of

in the form r u v w, such that v is perpendicular to w.

[4]

Solutions to VJC Prelim 2013 Paper 1


Q
1)

Solutions

15 11x
1
x2 6x 9
15 11x x 2 6 x 9
0
x2 6x 9
x2 5x 6
0
2
x 3
x2 5x 6

x 3
x 6 x 1
0
2
x 3
2

x 6 or 1 x 3 or x 3

Replace x with ln x
ln x 6 or 1 ln x 3 or ln x 3
2i)

0 x e6 or e x e3 or x e3
y 1 ln 1 x
y 2 1 ln 1 x
2y

dy
1

dx 1 x

d2 y
1
dy
2 2 y 2
2
dx
dx
1 x
When x = 0,
dy 1
y 1,
,
dx 2
2
d2 y 1
3
1


dx 2 2
2
4

2
1
3 x
y 1 x
2
4 2!
1
3
x x2
2
8
1
When x ,
4
2
1 1 3 1
1
1 ln 1 1
2 4 8 4
4
1

ii)

5 141
1 ln
4 128
4 19881
1 ln
5 16384
3497
4
ln
16384
5

Q
3)

Solutions

n Amt owed (beginning)


1 200 000

Amt owed (end)


1.004 200 000

1.004 200 000 1500

1.0042 200 000

1.004 200 000

1500 1.004
1.0043 200 000

1500 1.0042 1.004

1500 1.004 1500

1.004n 200 000

1500 1.004n 1

Amount owed after n months


1.004n 200 000 1500 1.004n 1
1.004 200 000 1500
n

1.004

1.004 1 1.004n 1

1 1.004
1.004 200 000 375000 1.004 1.004n
n

376500 175000 1.004

At the last instalment,

376500 175000 1.004 1500


n

15
7
15
n ln 1.004 ln
7
15
ln
7
n
ln 1.004

1.004

n 190.92

His last instalment is on the 191st month.


Amount he owed at end of 191st month

$ 376500 175000 1.004

4)

191

$1373.97
x1 9
x2 9 4 4 9 25
x3 25 4 4 25 49
x4 49 4 4 49 81
xn 2n 1

Let Pn be the statement xn 2n 1 , n


When n = 1,
LHS u1 9
2

RHS 2 1 1 9 = LHS
2

P1 is true
Assume Pk is true for some k

1.004

Solutions
i.e. xk 2k 1

LHS of Pk 1 xk 1
xk 4 4 xk
2k 1 4 4 2k 1
2

2k 1 0

4 k 4 k 1 4 8k 4
4k 2 12k 9
2k 3

RHS of Pk 1
Pk is true Pk 1 is true.

5i)

P is true
Since 1
Pk is true Pk1 is true,
by Mathematical Induction, Pn is true for all n
6 3 3

BA 4 3 1
3 1 2

3 3 0

BC 1 3 4
4 1 5

ii)

3 0

1 4
2 5
cos ABC
14 41
14
ABC cos1
125.8
14 41
6 3

Equation of line AB is r 4 s 1 , s
3 2

P is on line AB
6 3

OP 4 s 1 for some s
3 2

6 3 3 3 3s

CP 4 s 1 1 5 s
3 2 4 7 2 s

CP

3 3s 5 s 7 2 s
2

83 9 18s 9 s 2 25 10 s s 2 49 28s 4s 2
14 s 2 56 s 83
s s 4 0

s 0 or 4

Solutions
6
When s = 0, OP 4
3

6
3 6

When s = 4, OP 4 4 1 0
3
2 5

The coordinates of P are 6,4, 3 or 6,0,5 .


6i)

1
2

x3
ii)

y 5
2
O

y 5
x2
iii)

y0

O
x2
7)

uy x
du dy
1

x
y
dx dx
2 x

dy y 1 3


dx 2 x 2 2 x
dy
y
x
3
x

dx 2 x
2 2 x
du 1
3

x
dx 2
2 x

Solutions
1
1 3
u C x 2 3x 2
3
1
1 3
y x x 2 3x 2 D
3
D
1
y x 3
3
x
dy
y 1 3
0,

When
2x 2 2x
dx
y = x +3 --- locus of stationary points
1
When x is large, y x 3 (D = 0)
3
1
asymptote is y x 3
3
When x 0 , y when D 0

D0

y x3

D0
D0

O
x0
8i

8ii

r 1 r 2 1
1
1

r ! r 2 !
r 2 !

r 2 3r 1

r 2 !
r 1

r 2 3r 2 1
r 2 !

r 2 3r 1
r 2 !

n
1
1

r 2 !
r 1 r !
1

1!
1

2!
1

3!

1!
1
n!

1
3!
1
4!
1
5!
1
n

1!
1

n 2 !

1 1
1
1
3
1
1

1! 2! n 1! n 2 ! 2 n 1! n 2 !

Q
8iii

Solutions

r 2 3r 1

r 2 !

r 1

r 2 3r 3

r 2 !
r 1

3
2

2
r 3r 1
2

r 2 !
r 1 r 2 !
3
1 1 1

2
2
3! 4! 5!

3
12
2 e 11
2
2!

2e

9i

9iia
9iib

7
2

Geometry Method
Least z w

8 1 6 1
2

7 2 5

x-coordinate = 8 5cos

y-coordinate = 6 5sin

5
2
5

6
i
2
2
Algebraic Method
Alternative

x 8 y 6
2

52

y x 2 2
Sub (2) into (1)

x 8

x 8 52
2

x 8
x 8

z 8

9iia
9iib

5
2
or x 8

y 6

5
2

5
2

NA

8,6
5

Solutions
5
5

z 8
i6

2
2

5
5

Least z w 8
1 6
1
2
2

2 7

2 7
7 2 5
2

9iii

5
sin 1
2
2
8 2

tan 1 tan 1
8
4

1 5
sin

0, 4
68

5
1
1 1
1 5
tan 1 sin 1
tan sin

4
4
68
68
0.406 0.896

10ai

0,3

3, 0

0, 3

10aii

Area x dy
3

2 3 cos t 3cos t dt

3 3 2 cos 2 t dt

2 cos 2t 1
dt
3 3

2
2

sin 2t t 2
3 3

2
4


3 3
4 4

3 3
2

3 3
unit 2
2

8, 6

Q
10bi

Solutions
1
1

d
1
1 x 2 2 1 x 2 2 2 x

dx
2

x 1 x 2

10bii

1
2

1
2

cos x dx x cos x 2
0

x
2

dx
0
1 x2
1

1
2 2
1 x 0
24


1
4 2 2

1
4 2
2

10biii

Volume

0
1

4 2

11i

1
y dx

2 2
2

cos 1 x dx

2
4 2

2
4 2

unit 3

1 3 2


4 1 5
2k k k


1 2


Vector equation of l2 is r 4 s 5 , s
2k k


x 1 y 4 z 2k
Cartesian equation of l2 is
.

2
5
k

11ii

2 2

2
5 1 1 k
k k

Since k 2 0 , 1 k 2 0 for all real values of k .
2 2

5 1 0 for all real values of k, hence l1 and l2 cannot be perpendicular to each other.
k k

Q
11iii

Solutions

Since l1 // p, l1 normal vector of p


2 1

1 4 0
k 1

24k 0

k 6
Since there are no common points, (3, -2, 0) does not lie in p.
3 1

2 4 a
0 1

a 5

2 3 1

BC 0 2 2
0 0 0

1
1
1
BF 2
4
0 18 1


7

18

B 3, 2,0
n

C 2,0,0

2 2 12
1


A vector is 1 5 12 12 1
3 3 12
1


2

Let v be 1 .
3

2 1 4

w 1 1 5
3 1 1

3
2
4



r 1 1 5 , ,
3
3
1


VICTORIA JUNIOR COLLEGE


Preliminary Examination

MATHEMATICS
(Higher 2)

9740/02
September 2013

Paper 2
3 hours

READ THESE INSTRUCTIONS FIRST


Write your name and CT group on all the work you hand in.
Write in dark blue or black pen on both sides of the paper.
You may use a soft pencil for any diagrams or graphs.
Do not use staples, paper clips, highlighters, glue or correction fluid.
Answer all the questions.
Give non-exact numerical answers correct to 3 significant figures, or 1 decimal place in the case of angles in
degrees, unless a different level of accuracy is specified in the question.
You are expected to use a graphic calculator.
Unsupported answers from a graphic calculator are allowed unless a question specifically states otherwise.
Where unsupported answers from a graphic calculator are not allowed in a question, you are required to present
the mathematical steps using mathematical notations and not calculator commands.
You are reminded of the need for clear presentation in your answers.
At the end of the examination, fasten all your work securely together.
The number of marks is given in brackets [ ] at the end of each question or part question.

This document consists of 7 printed pages


VJC 2013

VICTORIA JUNIOR COLLEGE

[Turn over

Section A: Pure Mathematics [40 marks]


1

A trekker is standing at point A, on the edge of a very slow-moving river which is 1 km wide.
He wishes to return to his camp-ground at point D on the opposite side of the river, located
1 km downstream from the point B that is directly across where he is currently situated (refer
to the diagram below). He swims at a speed of 4 km per hour and walks at a speed of 6 km
per hour. It may be assumed that the current has insignificant impact on his swim across the
river.
1 km
B

x km

1 km

A
The trekker swims across the river from A to C, which is x km from B. He then walks along
the bank of the river from C to his camp-ground at D.
Using differentiation, calculate, in hours, the shortest travelling time taken by the trekker to
travel from A to D.
[5]

ax 2 bx c
, where a, b and c are non-zero constants.
x 1
23
(a) It is given that C passes through the point 3, and has a minimum point at 2, 10 .
2
(i)
Find the values of a, b and c.
[3]

The curve C has equation y

(ii)

(iii)

Sketch C, giving the coordinates of any turning points, points of intersection with the
axes and the equations of any asymptotes.
[3]
Find the set of values of

m
, where m and k are positive constants, such that the
k

x 2 y 1
1 does not intersect C.
curve with equation 2
k
m2
2

[2]

(b) In the case when a 1 and b 1 , find the set of values of c, where c 2 , such that C
has no stationary point.
[3]

The function f is defined by


f :x

x2 2 x 2 , 0 x 1.

(i)

Sketch the graphs of y f x , y f 1 x and y ff 1 x on a single diagram,

(ii)

indicating clearly the domains of the respective functions.


Find f 1 x .

[3]
[2]

The function g is defined by


g:x

where a is a constant and a 1.


(iii)
(iv)

xa
, 0 x 3,
x 1

Show that the composite function gf exists and find, in exact form, the range of gf.
Given that h' x gf x , show that h is an increasing function for 0 x 3.

[4]
[3]

A graphic calculator is not to be used in answering this question.


(i)

Find the roots of the equation z 3 4 3 4i , giving each root in the form rei , where
[4]
r 0 and .
Show the roots obtained on a single Argand diagram.
2
1

[2]
2

Hence, or otherwise, find the exact value of z z2 where z1 and z2 are any two

(ii)

of the roots.

[3]

Hence solve the equation z 6 8 3z 3 64 0 .

[3]

Section B: Statistics [60 marks]


5

2
1
and P( B) .
3
2
State an inequality satisfied by P( A B) .

For events A and B, it is given that P( A)


(i)

It is given further that A and B are independent. Find


P( A B) ,
(ii)
(iii)

P( A ' B) .

[2]

[1]
[2]
[Turn Over

A student has 2 ten-cent coins, 3 twenty-cent coins, 4 fifty-cent coins and 1 one-dollar coin.
(i)

He arranges the coins in a line. Find the number of ways of arranging the coins such
that any two adjacent coins do not add to a total value of $1.
[3]

(ii)

He puts all the coins in his coin pouch and randomly picks 4 coins, one after another
without replacement. Assuming that each coin has an equal chance to be chosen, find
the probability that the total value of the 4 coins is $1.
[3]

During the dry and hot months of June and July, forest fires are prevalent in the forested areas
of a particular country. Smoke from the forest fire affects air quality and causes the country
and its surrounding regions to be shrouded by haze. In a neighbouring country A, the air
quality is exacerbated by the direction of seasonal monsoon winds. It was found that during
this period, the number of patients admitted per week to the National General Hospital in
country A due to respiratory problems follows a Poisson distribution with mean 4.9.
(i)

Calculate the probability that in a randomly chosen week in June or July, the
National General Hospital admits more than 5 patients with respiratory problems. [1]

(ii)

Calculate the probability that in a week in June or July where the National General
Hospital admits more than 5 patients with respiratory problems, there will be exactly
8 patients with respiratory problems admitted at the hospital.
[2]

(iii)

Using a suitable approximation, find the probability that the National General
Hospital admits more than 22 patients with respiratory problems in a 4-week period
in June or July.
[3]

(iv)

The probability that not more than 1 patient admitted to the National General
Hospital due to respiratory problems in n consecutive days in June or July is at most
0.38. Find the smallest value of n.
[3]

(v)

Explain why the Poisson distribution may not be a good model for the number of
patients admitted to the National General Hospital due to respiratory problems in a
year.
[1]

Eight randomly selected students were asked to indicate the number of hours they spent
studying Mathematics per week during the June holiday. The corresponding Mathematics
Mid-year examination scores marked out of 100 marks of the eight students were recorded.
The eight pairs of data are given in the table below.
Time spent (x hours)

2.0

2.4

3.0

3.8

4.8

7.2

8.2

9.4

Mathematics score (y) 10

19

47

64

78

76

84

(i)

Given that the regression line of y on x is y 1.9263 9.6223x , find the value of p.
[3]
Draw a scatter diagram to illustrate the data. Calculate the product moment
correlation coefficient between x and y.
[2]

(ii)

Calculate the least squares estimates of a and b for the model y a b ln x . Find the
product moment correlation coefficient between ln x and y.

(iii)

Using your answers to parts (i) and (ii), explain which of y 1.9263 9.6223x or
y a b ln x is the better model.

(iv)

[2]

[2]

Using the appropriate regression line, estimate the Mathematics Mid-year


examination score of a student who spent 5.0 hours studying Mathematics during the
June holiday. Comment on the reliability of your estimate.
[2]

[Turn Over

The manager of a fast food restaurant in a small town decides to launch a lunch-time
promotion. During the promotion period, each customer can choose to purchase either one set
meal at a discounted price from counter A, or one set meal with a limited edition soft toy from
counter B. The waiting times, in minutes, at counter A and counter B follow independent
normal distributions with means and standard deviations as shown in the table.
Mean waiting time
(minutes)

Standard deviation
(minutes)

Counter A

12

2.7

Counter B

20

4.4

(i)

5 customers at the restaurant are randomly selected and their waiting times recorded.
3 of them made their purchase at counter A and 2 of them made their purchase at
counter B. Find the probability that the average waiting time for the 5 customers is
less than 14 minutes.
[3]

(ii)

Using a suitable approximation, calculate the probability that out of 50 customers


who queued at counter A, there were at least 43 customers who waited for more than
[4]
8.5 minutes.

(iii)

The manager wishes to provide quality service by promising customers at counter B


that no more than 5% of customers need to queue for more than k minutes. Find the
least integer value of k.
[3]

(iv)

Annie and Betty arrive together at the fast food restaurant and start to queue
simultaneously, one at each counter. The two of them leave the restaurant together
after both have completed their purchases. Calculate the probability that the first
person to complete her purchase needs to wait no more than 5 minutes for the other
person.
[4]

10

A machine in a factory operates 7 hours a day and produces 50 steel rods in an hour. The
length of a rod is denoted by x cm. Over a long period of time, it is known that the mean
length of a rod is 22.0 cm.
(a)

On his first day of work, a quality surveyor takes the first 7 rods produced and
calculates their mean length.
(i) Explain why the sample collected could be biased.

[1]

(ii) Suggest a more appropriate sampling method and describe briefly how this can
be done in the context of the question.
[2]
(b)

The machine is reset when it is found that the mean length of the rods produced is
more than 22.0 cm. The quality surveyor took a random sample of 10 rods produced
and measured their lengths. The results are summarized by

x 228.66 , x

5234.58 .

(i) Test, at the 5% significance level, whether the machine has to be reset.

[6]

(ii) State clearly any assumption that is necessary in order for the test to be valid. [1]
An upgrade of the machine gives a population variance of 2 cm2. A random sample
of 100 rods selected after the upgrade gives a mean of 22.5 cm. The machine is reset
whenever the mean length of rods produced is more than 22.0 cm. A test at the 10%
significance level indicates that the machine need not be reset.
(iii) Find the set of values of .

[4]

End of Paper

Solutions to H2 Mathematics Prelims Paper 2

Section A
No
1

Solution
Let t be the time taken in hours by the trekker to get from point A to point D.
AC 1 x 2

1 x2
hours
4

Time taken from A to C

1
1 x 2 hours
4

CD 1 x

1 x
hours
6
1
1 x hours
6

Time taken from C to D

1 x 2 1 x

4
6
1

dt 1 1
1
1 x 2 2 2 x
dx 4 2
6
x
1

4 1 x2 6

Hence, t

dt
0
dx

x
4 1 x

1
0
6

6x 4 1 x2
36 x 2 16 16 x 2
20 x 2 16
x

4
5

Sign of

since x 0
+

Hence, t is indeed a minimum when x


So the minimum traveling time is
2i)

4
5

1 54 1

4
5

0.353 hours (3 s.f.)

ax 2 bx c
x 1

23 23 a 3 3b c

9a 3b c 23
Since C passes through the point 3, ,
3 1
2 2
2

C passes through the point 2, 10 too, so 10


Since (2, 10) is a minimum point,

a 2 2b c
2

2 1

dy
0 when x 2 .
dx

4a 2b c 10

2
dy 2ax b x 1 ax bx c 1

2
dx
x 1

ax 2 2ax b c

x 1

So 0 4a 4a b c b c 0.
Solving the three equations using the GC, a 3, b 2, c 2.
(ii)

3x 2 2 x 2
x 1

Performing long division,


3x 1
2
x 1 3x 2 x 2

3x 2 3x
x2
x 1
3
Hence, y 3x 1

3
x 1

2,10
y

3x 2 2 x 2
x 1

x 2 y 1

1
k2
m2
2

0, 2
y 3x 1
iii)

iv)

x 1

x 2 y 1
m

1 describes a hyperbola that has asymptotes with equations y 1 x .


2
2
k
m
k
From the graph, comparing asymptotes, we can observe that the gradient of the asymptote with positive
gradient of the hyperbola must be at most 3 if the two graphs are not to intersect. Hence, the set of values
of mk is mk : 0 mk 3.
2

When c = 2, y

x2 x 2
becomes, y x 2, x 1 and has no stationary point.
x 1

From our working in part (i), when a b 1,

dy x 2 2 x 1 c

2
dx
x 1

dy
0, i.e. x2 2 x 1 c 0.
dx

Stationary points of c occur when

For there to be no stationary points, the equation x2 2x 1 c 0 must have no real solutions for x.
Hence, the discriminant 22 4 11 c 0.

4 4 1 c 0
c2
So the set of values of c is c

y
2

3(i)

: c 2.

2, 2
y f x

yff

1,1

yf

O
(ii)

Let y f x x 2 2 x 2

y x 1 1
2

x 1

y 1

x 1 y 1
x 1 y 1
Since 0 x 1 from the domain of f, x 1 y 1
So f 1 x 1 x 1
(iii)

Rf 1, 2

Dg 0, 3

Since Rf Dg , the composite function gf exists.


The range of gf is equivalent to the range of g when the domain is set to [1, 2]

y = g(x)

y=1

1
2a
1 a
and g 2
2
3
2

a
1

,
So Rgf
2
3
g 1

(iv)

h ' x gf x
x2 2x 2 a
x2 2x 2 1

Since

x 1 1 a
2
x 1 2
2
x 1 0 for

and a 1. So h ' x 0 for all x , and in particular for 0 x 3. Hence, h is an increasing

all x , we have both

x 1

2 0 and

x 1

1 a 0 for all

function for 0 x 3.
4

z 3 4 3 4i
8e

, where k

, k 0, 1

i 6 2 k

z 2e
2e

2 k
i 18
3

1318 i

, 2e

i
18

11

, 2e 18

Im

(i)

B
2
Re
2

C
z12 z2 2 z1 z2 z1 z2

Let the points A, B, and C represent the complex numbers z1, z2, and z2 respectively on the Argand
diagram above. By symmetry of the problem, any two roots will give the same result, so we take
i

11

z1 2e 18 and z2 2e 18 .
Using parallelogram law of vector addition and cosine rule,
z1 z2

22 22 2 2 2 cos

2
D z2

z1 z2
22 22 2 2 2 cos
2 3

2
3

z12 z2 2 z1 z2 z1 z2
4 3
Alternatively,
Take z1 2e

i
18

11

O
4

and z2 2e 18 . Then z12 4e

E z12

9 i

11

and z2 2 4e 9 .

z12 z2 2
4e

F z2 2

9 i

11

4e 9

42 42 2 4 4 cos

2
3

4 3
(ii)

z 6 8 3z 3 64 0

z3

8 3

8 3

4 1 64

8 3 64

2
8 3 8i

2
4 3 4i
13 i

11 i

When z 3 4 3 4i, from our previous answer, we have z 2e 18 , 2e 18 , 2e 18 . Since the coefficients
of the polynomial in the equation are all real, the remaining three roots must be the conjugates of these.
Hence, the roots of the equation are z 2e

1318 i

, 2e

1118 i

, 2e

i
18

11 i

13 i

, 2e18 , 2e 18 , 2e 18 .

Section B
No

Solution
General Comments
Remember to leave non exact answers correct to 3 significant figures.
Use , , ,... for population parameters and capital letters

A, B, C,... for random variables. Avoid using P, B, N , Z as random variables when there is a possibility
of confusion with probability, Binomial, Normal and Standard normal.
5(i)

P( A B) P( A) & P( A B) P( B),
P( A B)

1
2

P A B P A P B P A B
23 12 P A B
76 P A B

Since P A B 1, we have
So
(ii)

1
6

P A B 12 .

7
6

P A B 1 P A B 16 .

Since A and B are independent, P A B P A P B

1
.
2

(iii)

P A ' B P A ' P B P A ' B


P A ' P B P A ' P B
1 1 1 1

3 2 3 2
2

3
Alternatively,

P A ' B 1 P A P A B
1

6(i)

2
3

2 1

3 3

For no two adjacent coins to have a total value of exactly $1, we require that no two fifty-cent coins are
adjacent. Hence, we may first arrange the other six coins, then choose four out of the seven places
between the six coins to insert the four fifty-cent coins.

The number of ways is therefore


(ii)

6! 7
C4 2100.
2!3!

The only possible combination of coins that have a total value of $1 is a single fifty-cent coin, two
twenty-cent coins, and one ten-cent coin.
Hence, the required probability is

Alternative :
4/10 x 3/9 x 2/8 x 2/7 x 4!/2! = 4/35
7(i)

C1 3C2 2C1 4

10
C4
35

Let X be the number of patients admitted in a randomly chosen week in June or July. Then X ~ Po(4.9)

P X 5 1 P X 5
0.3664985
0.366 (3 s.f.)
(ii)

P X 8 | X 5

P X 8 and X 5
P X 5

P X 8

P X 5

0.061377
0.3664985
0.167 (3 s.f.)

(iii)

Let Y X1 X 2 X 3 X 4 .

Y ~ Po 4.9 4.9 4.9 4.9 , i.e. Y ~ Po 19.6 .

Since E(Y) = 19.6 > 10, Y ~ N 19.6, 19.6 approximately.

P Y 22 P Y 22.5

(continuity correction)

0.256 (3 s.f.)
(iv)

Let U be the number of patients admitted in n consecutive days in June or July.

U ~ Po 4.97n , i.e. U ~ Po 0.7n

P U 1 0.38
P U 0 P U 1 0.38
e 0.7 n e 0.7 n 0.7 n 0.38
e 0.7 n 1 0.7 n 0.38
y

2.998

From the GC, n 2.998.


Hence, the smallest value of n is 3.
Alternatively,
n

P U 1

2
3
4

0.59183 > 0.38


0.37961 < 0.38
0.23108

So from the GC, the smallest value of n such that P U 1 0.38 is 3.


(v)

8(i)

The Poisson distribution may not be a good model for the number of patients admitted in a year because
the mean number of admission per week may not be constant throughout the year it fluctuates
according to the severity of the haze which may be less severe outside of the June and July months.
From the given data, x 5.1.
Since the point x , y lies on the regression line of y on x, y 1.9263 9.6223 5.1 51.00003
But y
Hence,

10 19 p 47 64 78 76 84 378 p

8
8

378 p
51.00003
8
p 30.00024
p 30 (integer value)

(ii)

x
hours
From the GC, the product moment correlation coefficient, r = 0.944 (3 s.f.)
The least squares regression line of y on ln x is given by y 21.334 48.670ln x (to 5 s.f.)
So a = 21.3 and b =48.7 (to 3 s.f.)
The product moment correlation coefficient for this model is 0.986 (3 s.f.)

(iii)

Since the product moment correlation coefficient in part (ii) is closer to 1, as compared to the answer
found in part (i), a linear relationship between y and ln x would be a better model.

(iv)

We use y 21.334 48.670ln x from part (ii) since it better fits the data.
When x = 5.0, y 21.334 48.670ln 5.0

56.997 (5 s.f.)
So the estimated score of a student who spent 5.0 hours studying Mathematics during the June holiday is
57.0 (3 s.f.)
This estimate is reliable since x = 5.0 is within the given range 2.0 x 9.4, and the linear model used
is an appropriate one.
9(i)

Let X and Y be the random variables denoting the waiting times in minutes for a randomly chosen
customer at counters A and B respectively.

X ~ N 12, 2.72 and Y ~ N 20, 4.42


X1 X 2 X 3 Y1 Y2
5
X 1 X 2 X 3 Y1 Y2
E M E

1
3E X 2 E Y
5
1
3 12 2 20
5
15.2

Then M

X X 2 X 3 Y1 Y2
Var M Var 1

1
2 3Var X 2 Var Y
5
1
3 2.7 2 2 4.42
25
2.4236
So M ~ N 15.2, 2.4236 .

P M 14 0.220 (3 s.f.)
(ii)

Let C be the number of customers out of 50 randomly chosen customers at counter A that waited for
more than 8.5 minutes.

C ~ B 50, P X 8.5 , i.e. C ~ B 50, 0.90256

So C ' ~ B 50, 0.097437 where C C ' 50.


Since the number of customers, 50, is sufficiently large and E C ' 50 0.097437 4.87 is less than
5, C ' ~ Po 4.8718 approximately.

P C 43 P 50 C ' 43
P C ' 7
0.880 (3 s.f.)
(iii)

Having no more than 5% of customers needing to queue for more than k minutes is equivalent to stating
that the probability that a customer needing to queue for more than k minutes is no more than 5%, i.e.
that P Y k 0.05.

0.05

20
k 27.237

So the least integer value of k is 28.


(iv)

P X Y 5 P 5 X Y 5
X Y ~ N 12 20, 2.7 2 4.42
X Y ~ N 8, 26.65

P X Y 5 P 5 X Y 5
0.275 (3 s.f.)

27.237

10a
i

The method of sampling fails to account for possible changes in production conditions throughout the
day which may affect the lengths of the rods. Hence, the sample collected may not be representative of
the population of the rods produced.

(ii)

The surveyor could pick a randomly produced rod in the first hour, say the kth rod where 1 k 50,
and then pick every 50th rod thereafter. This is called systematic sampling.

(bi)

228.66
22.866
10
1
228.662
s 2 5234.58

9
10
x

0.67116
Let cm be the population mean length of the rods.

H0 : 22.0 vs H1 : 22.0

Level of significance: 5%

X 22.0
S
10
Using a t-test with 9, p value 0.0043111 0.05.
Test statistic: T

Since the p-value is less than 5%, H0 is rejected at the 5% significance level. Hence there is sufficient
evidence that 22.0 and the machine has to be reset.
(ii)

The length of a rod is assumed to be normally distributed.

(iii)

H0 : 22.0 vs H1 : 22.0

Level of significance: 5%
Test statistic: Z

X 22.0

100

0.10

1.281
65

Since H0 is not rejected, z 1.2816.

22.5 22.0

1.2816

100

5 1.2816
3.9015

The set of values that can take is thus

: 3.90.

Tampines Junior College


H2 Mathematics 9740/01
Preliminary Examination 2013
Answer All Questions.

1.

Without using a graphing calculator, solve the inequality

2.

It is given that

4 x 2 3x 4
1.
1 x

[4]

b
f ( x) a x cx d
x
for non-zero constants a , b, c and d.

15
The curve y = f(x) passes through the points with coordinates 4, and
2
229
9
9,
; and has a stationary point at 1, . Determine the equation of the
18
2
[4]
curve y = f(x).

3.

4.

By using the substitution x 2sec t , show that

x2 4
3
dx
.
4
x
32

[5]

The equation y e2 x 2 x has exactly one root denoted by .


(i)

Sketch the graph of y e2 x 2 x and find the value of correct to 3


decimal places.

[2]

A sequence of real numbers x1 , x2 , x3 ,... satisfies the recurrence relation


xn 1

for n 1.

1
ln(2 xn )
2

Prove algebraically that, if the sequence converges, then it converges to .


[2]
(iii) Prove that
xn1 xn if xn .
[2]
(ii)

5.

By considering uk uk 1 , where uk
n

Sn
k 1

1
, find the sum
k (k 1)

1
.
k (k 1)(k 2)

Hence, find the smallest integer n such that Sn differs from

[4]

1
by less than 10-4.
4

[2]

6.

A manufacturer needs to make a closed cylindrical can that will hold 1500 cm3 of
liquid. The radius of the circular base is r cm and the height of cylinder is h cm.
Given that the amount of material required in making the can is A cm2, find, using
[7]
differentiation, the values of r and h which give a minimum A cm2.

7.

A skydiver opens her parachute when her speed is 20 m s1. Her speed t seconds
after this is w m s1, and is modelled by the differential equation
dw
1
( w 8)( w 10) .
dt
2

8.

w8
0.4e9t .
w 10

(i)

Using this result, show that

[5]

(ii)

According to this model, what is the speed of the skydiver in the long
term?
[2]

A graphing calculator is not to be used in answering this question.


(i)

Find the cube roots of 2 + 2i in the form rei where r > 0 and
< .
[3]

These cube roots z1 , z2 , z3 are represented by points A, B and C respectively on an


Argand diagram, with arg z3 arg z1 arg z2 .
(ii)

Show the roots on an Argand diagram.

[2]

The midpoint of AB is M, and M represents the complex number w.


(iii)

Find the modulus and argument of w.

[3]

9.

A sequence u1 , u2 , u3 ,... is such that u1 5 and

un1 3un 4 , for all n 1.


(i)

Write down the first four terms of the sequence.

[2]

(ii)

Use your answers to part (i), obtain a conjecture for the nth term of the
sequence such that un = a n 2 where a is a constant to be determined. [2]

(iii)

10.

11.

Use the method of mathematical induction to prove your conjecture to part


(ii).
[4]

Referred to the origin O, the position vector of the point A is 2i 2j 6k and the
cartesian equation of the line l is x 1 2 y z 6 . Find
(i)

the position vector of the foot of the perpendicular from A to l,

[3]

(ii)

the perpendicular distance from A to l,

[2]

(iii)

the cartesian equation of the plane which contains l and A.

[3]

Given that y sin e x 1 .

d 2 y dy
ye2 x .
2
dx
dx

(i)

Show that

(ii)

By further differentiation, find the first three non-zero terms of the


Maclaurin series for y.
[5]

(iii)

Given that x is sufficiently small, using the result in part (ii), show that
1
1 1 3
2 .
x
x 4
sin e 1 x
2

[2]

[3]

12.

A curve C has parametric equations


x 4cos t ,

where
(i)

t .
2
2

P is a point on C with parameter t


P.

13.

y tan t ,

. Find the equation of the normal at


3
[5]

(ii)

Sketch C, indicating clearly any asymptotes and axial intercept(s) if any.


[2]

(iii)

Find the exact area of the region bounded by C, the x-axis and the line
[5]
x 2.

The curve C has equation 4 y 4 y 16 19 3x(2 x) .


dy
in terms of x and y.
dx

(i)

Find

(ii)

Show that the equation of C can be expressed in the form

( x 1)2 ( y 2)2

1
a2
b2
where a and b are constants to be determined.

[2]

[3]

(iii)

Sketch C, stating the coordinates of any points of intersection with the


axes.
[3]

(iv)

By sketching another line on the same diagram, find the values of x which
satisfy

3 x 3 x 16 19 3x(2 x) .
(v)

[3]

Find the numerical value of the volume of revolution when the smaller
region bounded by C and the line in part (iv) is rotated completely about
the x-axis. Give your answer correct to 3 decimal places.
[4]

END OF PAPER

Tampines Junior College


H2 Mathematics 9740/01
Preliminary Examination 2013
Answer All Questions.

1.

Without using a graphing calculator, solve the inequality

4 x 2 3x 4
1.
1 x

[4]

Solution:
4 x 2 3x 4
1
1 x
4 x2 4 x 3
0
1 x
(2 x 1) 2 2
0
1 x
Since (2 x 1)2 2 0 x ,
1 x 0
x 1

2.

It is given that

b
f ( x) a x cx d
x
for non-zero constants a , b, c and d.

15
The curve y = f(x) passes through the points with coordinates 4, and
2
229
9
9,
; and has a stationary point at 1, . Determine the equation of the
18
2
curve y = f(x).
[4]

Solution

b
y a x cx d
x
dy
a
b

2 c
dx 2 x x

9
a b c d ......(1)
2
15
b
2a 4c d ......(2)
4
2
b
229
3a 9c d
......(3)
9
18
a
b c 0......(4)
2
By GC, a 3, b 2, c

1
, d 1
2

2 x
y 3 x 1
x 2

3.

By using the substitution x 2sec t , show that

x2 4
3
dx
.
4
x
32

[5]

Solution:

x2 4
4sec 2 t 4
3
dx
2sec t tan t dt
0
x4
16sec 4 t

tan 2 t
3
dt
0 4sec3 t
1 3 sin 2 t
cos3 t dt

2
0
4 cos t
1
3 sin 2 t cos t dt
4 0

dx
2sec t tan t
dt
when x 2, t 0
when x 4, t

1 sin t 3
4 3 0

sin
3

1
3 sin 0

4
3
3

1 3

12 2

3
(Shown)
32

_______________________________________________________________________________

4.

The equation y e2 x 2 x has exactly one root denoted by .


Sketch the graph of y e2 x 2 x and find the value of correct to 3

(i)

decimal places.

[2]

A sequence of real numbers x1 , x2 , x3 ,... satisfies the recurrence relation


xn 1

for n 1.

1
ln(2 xn )
2

Prove algebraically that, if the sequence converges, then it converges to .


[2]
(iii) Prove that
[2]
xn1 xn if xn .
(ii)

Solution:

y e2 x 2 x
(i)

= 0.273

(i)

As n , xn L and xn1 L .
1
L ln(2 L)
2
2 L e2 L
e2 L 2 L 0
From (i), L = = 0.273

(ii)

If xn , then
y < 0 from graph
e2 x 2 x
1
x ln(2 x)
2
1
xn ln(2 xn )
2
xn xn 1

______________________________________________________________________________

5.

By considering uk uk 1 , where uk
n

Sn
k 1

1
, find the sum
k (k 1)

1
.
k (k 1)(k 2)

Hence, find the smallest integer n such that Sn differs from


Solution:

1
1

k (k 1) (k 1)(k 2)
k 2k

k (k 1)(k 2)
2

k (k 1)(k 2)

uk uk 1

1
k 1 k ( k 1)(k 2)
1 n
(uk uk 1 )
2 k 1
1
(u1 u2
2
+ u2 u3
n

Sn

+ u3 u4
+ ......
+ un 2 un 1
+ un 1 un
+ un un 1 )
1
(u1 un 1 )
2
1 1
1
(
)
2 2 (n 1)(n 2)

Sn

1
1 1
1
1
104 (
) 104
4
2 2 (n 1)(n 2) 4

1
2
4
(n 1)(n 2) 10

(n 1)(n 2) 5000

By GC, least n = 70

[4]

1
by less than 10-4.[2]
4

6.

A manufacturer needs to make a closed cylindrical can that will hold 1500 cm3 of
liquid. The radius of the circular base is r cm and the height of cylinder is h cm.
Given that the amount of material required in making the can is A cm2, find, using
differentiation, the values of r and h which give a minimum A cm2.
[7]

Solution:

A 2rh 2r 2
V r 2 h 1500
1500
h
r 2
1500
A 2r 2 2r 2
r
3000

+ 2r 2
r
dA
3000
2 + 4r 0
dr
r
3000 750
r3
=
4

750
r3

2
d A 6000
3 + 4
dr 2
r
6000
=
+ 4 12 37.699 0
750

A is minimum when
750
1500
r3
= 6.20 and h
12.4
2

750
3

(3 s.f.)

7.

A skydiver opens her parachute when her speed is 20 m s1. Her speed t seconds
after this is w m s1, and is modelled by the differential equation
dw
1
( w 8)( w 10) .
dt
2
w8
0.4e9t .
w 10

(i)

Using this result, show that

(ii)

According to this model, what is the speed of the skydiver in the long
term?
[2]

Solution:
(i)

(w 8)(w 10) dw 2 dt
1
1
1
18(w 8) 18(w 10) dw 2 dt

1
ln | w 8 | ln | w 10 | t C
9
1
w8
ln
t C
9 w 10
w8
ln
9t 9C
w 10
w8
where A e 9C
Ae 9t
w 10
When t =0, w = 20,
20 8
A
20 10
A 0.4
w8

0.4e9t
w 10

(ii)

18
0.4e9t
w 10

9t
As t , 0.4e 0

18
0
w 10
18

1
w 10
w 10 18
w8

Hence the speed of the skydiver approaches 8 ms1.

[5]

8.

A graphing calculator is not to be used in answering this question.


(i)

Find the cube roots of 2 + 2i in the form rei where r > 0 and
< .
[3]

These cube roots z1 , z2 , z3 are represented by points A, B and C respectively on an


Argand diagram, with arg z3 arg z1 arg z2 .
(ii)

Show the roots on an Argand diagram.

[2]

The midpoint of AB is M, and M represents the complex number w.


(iii)

Find the modulus and argument of w.

Solution:
(i)

z3 = 2 + 2i = 2 2e

z 2 2e

1
2

=2 e
1

i 2 k
4

3
4

1
3

2 k
i

4 3
i

z1 2 2 e 4 , z2 2 2 e

11
12

(ii)

(iii) Midpoint of AB :
arg(w) =

|w| =

1 11 7

2 12 4 12

2 cos

3
2

, z3 2 2 e

-i

5
12

[3]

9.

A sequence u1 , u2 , u3 ,... is such that u1 5 and

un1 3un 4 , for all n 1.


(i)

Write down the first four terms of the sequence.

[2]

(ii)

Use your answers to part (i), obtain a conjecture for the nth term of the
sequence such that un = a n 2 where a is a constant to be determined. [2]

(iii)

Use the method of mathematical induction to prove your conjecture to part


(ii).
[4]

Solution:
(i)

u1 5;

u2 3 5 4 11

u3 3 11 4 29;

u4 3 29 4 83

u1 31 2
u2 32 2
u3 33 2
u4 34 2
.
.
(ii)

.
un 3n 2, a 3
Let Pn be the statement un 3n 2 for n 1.
When n 1, LHS u1 5 (given)
RHS 31 2 5
LHS = RHS
P1 is true.
Assume that Pk is true for some positive integers k . i.e. uk 3k 2.
We now prove that Pk 1 is true.
When n k 1, uk 1 3uk 4

= 3 3k 2 4
3k 1 6 4
3k 1 2

Pk is true Pk 1 is true.
By Mathematical Induction, Pn is true for all n 1.
_________________________________________________________________________

10.

Referred to the origin O, the position vector of the point A is 2i 2j 6k and the
cartesian equation of the line l is x 1 2 y z 6 . Find
(i)

the position vector of the foot of the perpendicular from A to l,

[3]

(ii)

the perpendicular distance from A to l,

[2]

(iii)

the cartesian equation of the plane which contains l and A.

[3]

Solution:
(i) Let B be the foot of perpendicular.
1
1

OB 2 AB 4
6

1

AB. 1 0
1

8
5
1
OB 1
3
3

13

(ii)

2
1 1
AB 7
78 or 2.94
3 3
5

(iii)

1 2 12
4


1 7 3 3 1
1 5 9
3


4 14

r. 1 2 . 1
3 6 3

4

r. 1 24
3

4 x y 3 z 24
_________________________________________________________________________

10

Given that y sin e x 1 .

11.

d 2 y dy
Show that
ye2 x .
2
dx
dx

(i)
(ii)

By further differentiation, find the first three non-zero terms of the


Maclaurin series for y.
[5]

(iii)

Given that x is sufficiently small, using the result in part (ii), show that
1
1 1 3
2 .
x
x 4
sin e 1 x
2

Solution:
(i)

[2]

y sin e x 1
dy
e x cos e x 1
dx
d2 y
e x cos e x 1 e 2 x sin e x 1
dx 2
dy

ye 2 x
dx

(ii)

d 3 y d 2 y 2 x dy

e
2 ye 2 x
dx 3 dx 2
dx
d2 y
dy

2 e2 x 2 y
dx
dx

d 4 y d3 y 2 x d 2 y
dy
dy

3 e 2 2 2e 2 x 2 y
4
dx
dx
dx
dx

dx
When x 0, y 0,

d4 y
dy
d2 y
d3 y
1, 2 1, 3 0, 4 5
dx
dx
dx
dx

By Maclaurin's expansion,
x2
x4
5 ...
2!
4!
2
4
x 5x
x
2 24

y 0 x

[3]

11
(iii)

1
sin e x 1
2
x
sin e 1

x2 5x4
x

2 24

x 5 x3
x 1

2 24

x 5 x 3 2 3 x 5 x 3
1

2 1 2

...

x
2 24
2!
2 24

1
x
2 1 x 3 ...

x
2

12.

1 1 3
(Shown)
x2 x 4

A curve C has parametric equations


x 4cos t ,

where
(i)

t .
2
2

P is a point on C with parameter t


P.

. Find the equation of the normal at


3
[5]

(ii)

Sketch C, indicating clearly any asymptotes and axial intercept(s) if any.


[2]

(iii)

Find the exact area of the region bounded by C, the x-axis and the line
[5]
x 2.

Solution:
(i)

y tan t ,

x 4cos t
y tan t

dx
4sin t
dt
dy
sec2 t
dt

12

dy
sec 2 t

dx 4sin t
1
=
4sin t cos 2 t

When t

, x 2, y 3,

dy
2

dx
3

Equation of normal: y 3

3
x
2

y
(ii)

4
x=0
(iii)

3
( x 2)
2

Required area
4

ydx
2

tan t (4sin t )dt


3

4 3 tan t sin tdt


0

sin 2 t
dt
0 cos t

1 cos 2 t
4 3
dt
0
cos t
4 3

4 3 (sec t cos t )dt


0

4(ln sec t tan t sin t ) 03


3
)
2
2(2 ln(2 3) 3)
4(ln(2 3)

13

13.

The curve C has equation 4 y 4 y 16 19 3x(2 x) .


dy
in terms of x and y.
dx

(i)

Find

(ii)

Show that the equation of C can be expressed in the form

( x 1)2 ( y 2)2

1
a2
b2
where a and b are constants to be determined.

Sketch C, stating the coordinates of any points of intersection with the


axes.
[3]

(iv)

By sketching another line on the same diagram, find the values of x which
satisfy

(v)

[3]

Find the numerical value of the volume of revolution when the smaller
region bounded by C and the line in part (iv) is rotated completely about
the x-axis. Give your answer correct to 3 decimal places.
[4]

Solution:

(ii)

[3]

(iii)

3 x 3 x 16 19 3x(2 x) .

(i)

[2]

16 y 2 64 y 19 6 x 3x 2
Differentiating implicitly w.r.t x,
dy
dy
32 y 64 6 6 x
dx
dx
dy
3 3x
3(1 x)

=
dx 16 y 32
16( y 2)

16 y 2 4 y 19 3 x 2 2 x
16 y 2 4 y 4 4 19 3 x 2 2 x 1 1
16 y 2 45 3 x 1 3
2

3 x 1 16 y 2 48
2

x 1
42

y 2

a 4, b 3

14
(iii)

(iv)

Let 4 y 3 x.
x 3
y
4 4
line drawn correctly on graph
From G.C.,
x 1 or x 5

(v)


1
4 4

5
x 1

dx
3 1
2 dx
1

16

25.198055
25.198 (3 d.p.)

END OF PAPER

Tampines Junior College


H2 Mathematics 9740/02
Preliminary Examination 2013

Section A: Pure Mathematics [40 marks]


1.

Functions f and g are defined by

f : x x2 6 x 9, for x , x 0,
g : x 2 3x 1, for x , x k.

2.

(i)

Sketch the graph of f and state its range.

[2]

(ii)

Explain why the inverse function f 1 does not exist.

[1]

(iii)

State a maximal domain of f for which the inverse function exists.

[1]

(iv)

By considering the graph of g, find the least value of k for which the composite
[2]
function fg exists.

Do not use a graphing calculator in answering this question.


The planes p1 , p2 and p3 have equations r 2i k 3 , r i 3j 3 and

r i j respectively, where and are constants.

The planes p1 and p2 intersect in a line l.


(i)

Show that the point (0,1,-3) lies on p1 and p2 .

[1]

(ii)

Find the vector equation of l.

[2]

(iii)

Given that the three planes have no point in common, find the value of . What
can be said about the value of ?
[3]

(iv)

Point P has position vector i + 2j + 4k. Given that PQ is perpendicular to p1 and


distance PQ is 10 units. Find the possible coordinates of Q.
[4]

3.

Do not use a graphing calculator in answering this question.


(a)

(b)

The complex number z satisfies the relations |z 4| 3 and |z| = |z 4|.


(i)

Sketch, on a single Argand diagram, the locus of points representing the


complex number z.
[3]

(ii)

Find the greatest and least possible values of arg z, giving your answers
in radians correct to 3 decimal places.
[3]

(i)

Given that w = 1 + i is a root of the equation


w3 + 3w2 + pw +q = 0,

(ii)

4.

(a)

(b)

find the values of the real numbers p and q.

[4]

For these values of p and q, solve the equation in part (i).

[2]

Dini was given 10 stickers for her birthday. She then starts to buy stickers for a
total period of 52 weeks. The number of stickers she buys each week is 2 more
than the number she bought the previous week.
(i)

If she bought 70 stickers in the 26th week, find the number of stickers
she would have at the end of 52 weeks.
[3]

(ii)

If she can only afford to buy 420 stickers in the last 4 weeks, find the
maximum number of stickers she can buy in the first week.
[3]

The sum of the first n terms of a sequence is given by 2(3n1 3) . By finding the
nth term of the sequence, or otherwise, show that this is a geometric
progression, and state the values of the first term and common ratio.
Explain, whether this geometric progression converges.
[6]

Section B: Statistics [60 marks]


5.

1
1
1
Events A and B are such that P( B) , P( A B) and P( A B) .
3
5
6
Find
(i)
P( A B) ,
P( A | B) .
(ii)

[1]
[3]

Hence determine whether events A and B are independent.

[2]

6.

7.

(a)

Ten points are marked on the circumference of a circle. Find the number of
triangles that can be formed.
[1]

(b)

Twelve points are marked on the circumference of a circle and labelled with
digits from 1 to 12. Find the number of possible arrangements such that points
labelled with digits divisible by 3 are separated.
[3]

(c)

Fifteen points are marked on the circumference of a circle. All possible chords
that join any two of these points are drawn. A pair of such chords is defined as
free if the two chords in the pair do not intersect and have no common
endpoints. Find the possible number of free chords.
[2]

Mechanical faults occur at random on a speed train, at an average rate of per week.
(i)

Given that the probability of at least 3 mechanical faults occurring in a period of


3 weeks is 0.7, show that 1.2 , correct to 1 decimal place.
[2]

(ii)

Using a suitable approximation, find the probability that there are exactly 35
[3]
mechanical faults occurring in a period of 30 weeks.

Electrical faults also occur at random on the speed train, at an average rate of 2.3 per
week.
(iii)

8.

Find the probability that there are exactly 2 electrical faults occurring in a week,
given that there is a total of less than 4 faults occurring in a week.
[3]

A school has used an established leadership appraisal score to determine whether a


student has displayed leadership qualities. The school claims that the mean leadership
score of a student is 3.8 and the population variance is 0.1. The Dean wants to test
whether the mean score has improved at 5% level of significance. A random sample of 10
students is examined and the sample mean is denoted by .
(i)

Explain the meaning of 5% level of significance in the context of the question.


[1]

(ii)

Find the range of values of the mean score if the claim is not rejected at the 5%
level of significance. State any assumptions you have made.
[5]

(iii)

Another sample of 10 students is taken and the total score is found to be 45.
State the conclusion of the test.
[2]

9.

A particular hospital in Town A observed that the number of patients seeking medical
treatment for respiratory problems has changed due to changes in air quality. The
hospital decided to monitor the situation and the table below shows the results:

Pollutants Index (x)


Number of patients (y)

200
140

138
134

124
115

112
87

83
k

73
48

66
33

42
10

The least square regression line of y on x has equation


y 14.894 0.89875x

correct to 5significant figures.


The line passes through ( x , y ) and the mean pollutants index, x , is 104.75.

10.

(i)

Show that k = 67.

[2]

(ii)

Draw a scatter diagram to illustrate the data.

[2]

(iii)

Explain, giving your reasons, why a model of the form y = a ln x + b fits the
data better.
[1]

(iv)

Using the model in part (iii), calculate the equation of the least square regression
[3]
line and the value of r.

(v)

The hospital decided that they will need to purchase new equipment if the
number of patients reaches 120. Explain, with reason, whether it is advisable for
the hospital to purchase new equipment if the pollutant index is 150.
[1]

The time taken for a car to be serviced at Workshop A follows a normal distribution with
mean 120 minutes and standard deviation 10 minutes. The time taken for a car to be
serviced at Workshop B follows a normal distribution with mean 130 minutes and
standard deviation 7 minutes. The time taken for a car to be serviced at Workshop A is
independent of the time taken for a car to be serviced at Workshop B.
(i)

Find the probability that the time taken for a car to be serviced at Workshop A is
shorter than the time taken for a car to be serviced at Workshop B.
[3]

(ii)

Find the probability that the average time taken for 4 cars to be serviced at
Workshop A differs from the average time taken for 3 cars to be serviced at
Workshop B by at least 15 minutes.
[4]

(iii)

A random sample of 60 cars is taken from Workshop A. Using a suitable


approximation, find the probability that at least 49 cars each take less than 130
minutes to be serviced at Workshop A.
[4]

11.

A company employs 35 representatives who work from the same office. The working
hours are such that any one representative spends just 25% of his day in the office.
(i)

Find the minimum number of rooms needed in the office so that each
representative will have a room at least 90% of the time.
[3]

(ii)

Find the probability that there are fewer than 6 representatives in the office in a
randomly chosen day, and calculate the percentage error in using a Poisson
distribution as an approximation in this case.
[4]

There are N such companies in the building, where N 50 , with the same employment
strength and working hours. If the probability that the total number of representatives in
these N companies exceeding 500 in a randomly chosen working day exceeds 0.9, show
that

1.282 6.5625N 500 8.75 N 0 .


Hence find the least value of N.

END OF PAPER

[5]

Tampines Junior College


H2 Mathematics 9740/02
Preliminary Examination 2013

Section A: Pure Mathematics [40 marks]

1.

Functions f and g are defined by

f : x x2 6 x 9, for x , x 0,
g : x 2 3x 1, for x , x k.

(i)

Sketch the graph of f and state its range.

[2]

(ii)

Explain why the inverse function f 1 does not exist.

[1]

(iii)

State a maximal domain of f for which the inverse function exists.

[1]

(iv)

By considering the graph of g, find the least value of k for which the composite
function fg exists.
[2]

Solution:

f(x) x2 6 x 9 = ( x 3)2 18

(i)

-3

R f [18, )

-9

-18
(-3,-18)

(iii)

Any horizontal line y = k, 18 k 9 cuts the graph of f at 2 points, hence f is not


one-one function. f-1 does not exist.
Maximal domain of f for which f-1 exists = (, 3]

(iv)

fg exists if R g Df . i.e. R g (, 0]

(ii)

Let g(x) = 0,

2 3x 1 0
3x 1 4
x

Least value of k =

5
3

5
3

2.

Do not use a graphing calculator in answering this question.


The planes p1 , p2 and p3 have equations r 2i k 3 , r i 3j 3 and

r i j respectively, where and are constants.


The planes p1 and p2 intersect in a line l.
(i)

Show that the point (0,1,-3) lies on p1 and p2 .

[1]

(ii)

Find the vector equation of l.

[2]

(iii)

Given that the three planes have no point in common, find the value of . What
can be said about the value of ?
[3]

(iv)

Point P has position vector i + 2j + 4k. Given that PQ is perpendicular to p1 and


distance PQ is 10 units. Find the possible coordinates of Q.
[4]

Solution:
(i)
02

1 . 0 0 0 3 3
3 1

0 1

1 . 3 0 3 0 3
3 0

Thus the point (0,1,-3) lies on p1 and p2..


(ii) p1 : 2 x z 3
p2 : x 3 y 3

2 1 3

0 3 1
1 0 6

0

Since 1 lies on p1 and p2, equation of l is
3

0 3

r 1 s1, s
~
3 6

(iii)

3

1 . 1 0
0 6

3 1 0
1

3
0

1 . 1
3 0

1

PQ 10n1
2
1
OQ OP 10
0
5
1
2 1

OQ 2 5 0 2
1 4

1 4 5 1 4 5

OQ 2 or 2

42 5 4 2 5
Coordinates of Q are (1 4 5 , 2, 4 2 5 ) and (1 4 5 , 2, 4 2 5 )

3.

Do not use a graphing calculator in answering this question.


(a)

(b)

The complex number z satisfies the relations |z 4| 3 and |z| = |z 4|.


(i)

Sketch, on a single Argand diagram, the locus of points representing the


complex number z.
[3]

(ii)

Find the greatest and least possible values of arg z, giving your answers
in radians correct to 3 decimal places.
[3]

(i)

Given that w = 1 + i is a root of the equation


w3 + 3w2 + pw +q = 0,

(ii)

find the values of the real numbers p and q.

[4]

For these values of p and q, solve the equation in part (i).

[2]

Solution:
(a)
(i)

(ii)

(b)

32 2 2 5
5
tan 1 0.841
2
Greatest arg(z) = 0.841 rad
Least arg(z) = 0.841

(i) 1 + i is a root of the equation w3 + 3w2 + pw +q = 0, then


(1 + i)3 + 3(1 + i)2 + (1 + i)p + q = 0
2 + 2i + 3(2i) + (1 + i)p + q = 0
2 + 8i + p + ip + q = 0
Comparing real and imaginary parts:
2 + p + q = 0
and 8 + p = 0
Hence p = 8 and q = 10

(ii) 1 + i is a root of the equation w3 + 3w2 + pw +q = 0, then 1 i is also a root since


the coefficients are all real numbers.
(w (1 + i))(w (1 i))(w A) = 0
(w2 2w + 2)(w A) = 0
A = 5
The roots are 1 i, 1 + i, 5

4.

(a)

(b)

Solution:
(a)
(i)

Dini was given 10 stickers for her birthday. She then starts to buy stickers for a
total period of 52 weeks. The number of stickers she buys each week is 2 more
than the number she bought the previous week.
(i)

If she bought 70 stickers in the 26th week, find the number of stickers
she would have at the end of 52 weeks.
[3]

(ii)

If she can only afford to buy 420 stickers in the last 4 weeks, find the
maximum number of stickers she can buy in the first week.
[3]

The sum of the first n terms of a sequence is given by 2(3n1 3) . By finding the
nth term of the sequence, or otherwise, show that this is a geometric
progression, and state the values of the first term and common ratio.
Explain, whether this geometric progression converges.
[6]

a 25d 70
a 20

Number of stickers at the end of 52 weeks = 10

52
[2(20) 51(2)] 3702
2

52
48
[2(a) 51(2)] [2(a) 47(2)] 420
(ii) 2
2
a6

Maximum number of stickers = 6


(b)

Tn Sn S n 1
2(3n 1 3) 2(3n 3)
2(3n )(3 1)
4(3n )
12(3n 1 )

Tn
12(3n1 )

3 which is a constant
Tn1 12(3n2 )
The sequence is a G.P with first term 12 and common ratio 3.
Since |r| = 3 > 1. The G.P. does not converge.

Section B: Statistics [60 marks]


5.

1
1
1
Events A and B are such that P( B) , P( A B) and P( A B) .
3
5
6
Find
(i)
P( A B) ,
(ii)
P( A | B) .

[1]
[3]

Hence determine whether events A and B are independent.

[2]

Solution:
1 5

6 6

(i)

P( A B) 1

(ii)

P( A B) P( A B) P( B)

5 1 1

6 3 2

1
P( A B)
3
P( A | B)
2
2
P( B)
4
3
P( A) P( A B) P( B) P( A B)

7
10

Since P( A) P( B) P( A B) , A and B are not independent.

6.

(a)

Ten points are marked on the circumference of a circle. Find the number of
triangles that can be formed.
[1]

(b)

Twelve points are marked on the circumference of a circle and labelled with
digits from 1 to 12. Find the number of possible arrangements such that points
labelled with digits divisible by 3 are separated.
[3]

(c)

Fifteen points are marked on the circumference of a circle. All possible chords
that join any two of these points are drawn. A pair of such chords is defined as
free if the two chords in the pair do not intersect and have no common
endpoints. Find the possible number of free chords.
[2]

Solution:
10
(a)
120
3
(b)
(c)

8! 8
4! 8467200
8 4
15
2! 2730
4

7.

Mechanical faults occur at random on a speed train, at an average rate of per week.
(i)

Given that the probability of at least 3 mechanical faults occurring in a period of


3 weeks is 0.7, show that 1.2 , correct to 1 decimal place.
[2]

(ii)

Using a suitable approximation, find the probability that there are exactly 35
mechanical faults occurring in a period of 30 weeks.
[3]

Electrical faults also occur at random on the speed train, at an average rate of 2.3 per
week.
(iii)

Find the probability that there are exactly 2 electrical faults occurring in a week,
given that there is a total of less than 4 faults occurring in a week.
[3]

Solution:
(i)
Let X be the number of mechanical faults in 3 weeks.
X ~ Po(3)

P(X 3) = 0.7
1 P(X 2) = 0.7
P(X 2) = 0.3
From G.C., = 1.2051 1.2 (1 d.p.)
(ii)

Let Y be the number of mechanical faults in 30 weeks


Y ~Po(36)
Since 36 10 , Y ~ N(36, 36) approximately
c .c .

P Y 35 P 34.5 Y 35.5 0.065499 0.0655


(iii)

Let W be the number of electrical faults occurring in a week.


W ~ Po(2.3)
Let T be the total number of faults per week
T ~ Po(3.5)

P W 2 | T 4

P W 2 P X 0 P W 2 P X 1
P T 3

0.32744
0.327

8.

A school has used an established leadership appraisal score to determine whether a


student has displayed leadership qualities. The school claims that the mean leadership
score of a student is 3.8 and the population variance is 0.1. The Dean wants to test
whether the mean score has improved at 5% level of significance. A random sample of 10
students is examined and the sample mean is denoted by .
(i)

Explain the meaning of 5% level of significance in the context of the question.


[1]

(ii)

Find the range of values of the mean score if the claim is not rejected at the 5%
level of significance. State any assumptions you have made.
[5]

(iii)

Another sample of 10 students is taken and the total score is found to be 45.
State the conclusion of the test.
[2]

Solution:
(i) 5% level of significance refers to a 0.05 probability of wrongly rejecting the mean
leadership score is 3.8 when the actual leadership score is 3.8.
(ii)

Let X and be the leadership score and mean leadership score of a student.
H0:
= 3.8
H1:
> 3.8
Assume that the leadership score follows a normal distribution
x 3.8
Under H0,
Z=
~ N(0, 1)
s / 10
x 3.8
For H0 to be not rejected,
Z=
< 1.64485
0.1 / 10

x 3.96
(iii) Since x 4.5 3.96 , we reject H0. Hence, we conclude that there is sufficient
evidence at 5% level of significance that the mean leadership score has improved.

9.

A particular hospital in Town A observed that the number of patients seeking medical
treatment for respiratory problems has changed due to changes in air quality. The
hospital decided to monitor the situation and the table below shows the results:

Pollutants Index (x)


Number of patients (y)

200
140

138
134

124
115

112
87

83
k

73
48

66
33

42
10

The least square regression line of y on x has equation


y 14.894 0.89875x

correct to 5significant figures.


The line passes through ( x , y ) and the mean pollutants index, x , is 104.75.
(i)

Show that k = 67.

[2]

(ii)

Draw a scatter diagram to illustrate the data.

[2]

(iii)

Explain, giving your reasons, why a model of the form y = a ln x + b fits the
data better.
[1]

(iv)

Using the model in part (iii), calculate the equation of the least square regression
line and the value of r.
[3]

(v)

The hospital decided that they will need to purchase new equipment if the
number of patients reaches 120. Explain, with reason, whether it is advisable for
the hospital to purchase new equipment if the pollutant index is 150.
[1]

Solution:

567 k
, x 104.75
8
567 k
14.894 0.89875(104.75)
8
k = 67.0005 67 (showed)
y

(i)

xy

(ii)
140

10

x
42

200

(iii) A model of the form y = a ln x + b fits the data better as the scatter diagram shows that as
the pollutant index x increases, the number of patients y increases at a decreasing rate
which could be due to people slowly adjusting to the air quality after a period of time.

10

(iv) The equation of the least square regression line is


y = 95.183 ln x 353.84
y = 95.2 ln x 354
r = 0.972
(v)

When x = 150, y = 123


It is advisable for the hospital to purchase new equipment since the predicted value of
y = 123 > 120 is a reliable prediction as x = 150 lies within the data range given.

10. The time taken for a car to be serviced at Workshop A follows a normal distribution with
mean 120 minutes and standard deviation 10 minutes. The time taken for a car to be
serviced at Workshop B follows a normal distribution with mean 130 minutes and
standard deviation 7 minutes. The time taken for a car to be serviced at Workshop A is
independent of the time taken for a car to be serviced at Workshop B.
(i)

Find the probability that the time taken for a car to be serviced at Workshop A is
shorter than the time taken for a car to be serviced at Workshop B.
[3]

(ii)

Find the probability that the average time taken for 4 cars to be serviced at
Workshop A differs from the average time taken for 3 cars to be serviced at
Workshop B by at least 15 minutes.
[4]

(iii)

A random sample of 60 cars is taken from Workshop A. Using a suitable


approximation, find the probability that at least 49 cars each take less than 130
minutes to be serviced at Workshop A.
[4]

Solution:
Let X and Y be the time taken for a car to be serviced at Workshop A and B respectively
X N(120, 102)and

Y N(130, 72)

(i)

X Y N(-10, 149)
P(X < Y) = P(X Y< 0) = 0.794

(ii)

Let

and

N(120,

~ N(-10,
P(

15) = P(

~N(130,

)
15) + P(

15) = 0.218

(iii) Let W be the number of cars taking less than 130 minutes to be serviced at Workshop A
out of 60 cars.
P(X < 130) = 0.8413447
W B(60, 0.8413447)
Since np = 50.4986 > 5 and n(1 p) = 9.501318 > 5,
W N(50.4986, 8.01108) approximately

11
c.c

P(W 49) P(W > 48.5) = 0.760

11.

A company employs 35 representatives who work from the same office. The working
hours are such that any one representative spends just 25% of his day in the office.
(i)

Find the minimum number of rooms needed in the office so that each
representative will have a room at least 90% of the time.
[3]

(ii)

Find the probability that there are fewer than 6 representatives in the office in a
randomly chosen day, and calculate the percentage error in using a Poisson
distribution as an approximation in this case.
[4]

There are N such companies in the building, where N 50 , with the same employment
strength and working hours. If the probability that the total number of representatives in
these N companies exceeding 500 in a randomly chosen working day exceeds 0.9, show
that

1.282 6.5625N 500 8.75 N 0 .


Hence find the least value of N.
Solution:
(i)
Let X be the number representatives in the office in a day.
X ~ B(35, 0.25)
Let m be the number of rooms needed in the office

P X m 0.9
Using the GC,

Minimum number of rooms is 12

(ii)

P X 6 P X 5 0.097620
If Poisson distribution is used, then X ~ Po(8.75) approximately.

P X 6 P X 5 0.13173

[5]

12

Percentage error

0.13173 0.09762
100% 34.9%
0.09762

np 8.75 and np(1 p) 6.5625

Since N 50 is large, by Central Limit Theorem,

X1 X 2 ... X N

N 8.75 N , 6.5625 N approximately

P X1 X 2 ... X N 500 0.9

P X1 X 2 ... X N 500 0.9


P X1 X 2 ... X N 500 0.1
500 8.75 N

P Z
0.1
6.5625 N

500 8.75 N
1.282
6.5625 N

1.282 6.5625N 500 8.75 N 0

Using the GC,

Least value of N is 61

END OF PAPER

TEMASEK JUNIOR COLLEGE, SINGAPORE


Preliminary Examination 2013
Higher 2

9740/01

MATHEMATICS
Paper 1

18 September 2013
Additional Materials:

Answer paper
List of Formulae (MF15)

3 hours

READ THESE INSTRUCTIONS FIRST


Write your Civics group and name on all the work that you hand in.
Write in dark blue or black pen on both sides of the paper.
You may use a soft pencil for any diagrams or graphs.
Do not use staples, paper clips, highlighters, glue or correction fluid.
Answer all the questions.
Give non-exact numerical answers correct to 3 significant figures, or 1 decimal place in the case of
angles in degrees, unless a different level of accuracy is specified in the question.
You are expected to use a graphic calculator.
Unsupported answers from a graphic calculator are allowed unless a question specifically states
otherwise.
Where unsupported answers from a graphic calculator are not allowed in a question, you are required
to present the mathematical steps using mathematical notations and not calculator commands.
You are reminded of the need for clear presentation in your answers.
The number of marks is given in brackets [ ] at the end of each question or part question.
At the end of the examination, fasten all your work securely together.

This document consists of 6 printed pages.

TJC 2013

[Turn over

If two curves y = ax3 +

bx and y = ln (px2) intersect at the points where x = 1, x = 2

and x = 4, find the values of a, b and p, correct to 2 decimal places.

[4]

A police officer discovered the dead body of a murdered man. The body was located in
a room that was kept constant at 20oC. According to Newtons law of cooling, the body
will radiate heat energy into the room at a rate proportional to the difference in
temperature between the body and the room. Let T oC be the body temperature t
minutes after cooling starts.
(i)

Write down a differential equation relating T and t.

(ii)

By solving this differential equation, show that T 20 Ae k t where A and k are


constants.

[1]
[3]

The coroner arrived at the scene at 10.40 pm and the body temperature of the victim
was measured to be 34oC. He made another measurement of the temperature after 90
minutes and temperature was 30oC. It is assumed that the victims temperature was
37oC at the time of death. Find the time of the victims death.

[3]

8r
, for r = 1, 2, 3, ..
(2r 1)2 (2r 1) 2
The sum of the first n terms is denoted by Sn.

The rth term of a sequence is given by ur

8
24
and S2
, find the exact values of S3 and S4.
9
25

(i)

Given that S1

(ii)

By considering the values of 1 Sn for n = 1, 2, 3 and 4, or otherwise, make a


conjecture for a formula for Sn in terms of n.

(iii) Prove your conjecture by the method of mathematical induction.

[1]

[1]
[4]

(iv) Find the smallest integer n for which the sum Sn differs from 1 by less than 105.
[2]

TJC/MA 9740/Preliminary Exam 2013

(a)

The sum,

S n , of the first n terms of a sequence is given by

Sn 3n 4n 2 where n 2 . Show that this sequence is an arithmetic


progression.
[3]
2

(b)

A large volume of water consists of x kg of impurities. In a water purification


1
process,
of the impurities, where r , r 1 is removed in the first stage. In
r
1
each subsequent stage, the amount of impurities removed is
of that removed in
r
the preceding stage. Assume no other impurities are added in the process.
(i)

Find the amount of impurities removed in the second and third stages of
purification in terms of x and r.

(ii)

[1]

Find the total amount of impurities removed by the end of the nth stage. [2]

(iii) Find the range of values of r such that at least half of the impurities will
remain no matter how many stages are used.

[2]

A graphic calculator is not to be used in answering this question.


p
.
q
Express z in the form x + yi, where x and y are exact real values to be determined.
[2]

Two complex numbers p and q are given by p 1 i and q 1 3i , and z


(i)
(ii)

By considering the moduli and arguments of p and q, find the exact values of |z|
and arg z, where arg z .

[4]

6 2
11
(iii) Hence, show that sin
.
4
12

[3]

Let f (r ) r 1 r r 1 . Show that f (r ) f (r 1) 3r r 1 .

[1]

(i)

Find

r r 1 .

[2]

r 1

(ii)
(iii)

Using the result obtained in part (i), deduce that

Find the sum of the series


12 3 22 32 3 42 52 3 62

Give your answer in a fully factorised form.

r 1

1
n(n 1)(2n 1) .
6

[2]

3 n 1 n2 , where n is odd.
2

[4]

[Turn over
TJC/MA 9740/Preliminary Exam 2013

(a)

The diagram below shows the graph of y = f(x). The graph has a minimum point
at (1, 3) and intersects the axes at x = 2 and y = 4. The equations of the
asymptotes are y = 2x and x = 1.
y
y = 2x
4
2

(1, 3)
x

x= 1
On separate diagrams, sketch the graphs of
(i)

y = f (x),

(ii)

[2]

1
,
f ( x)

[2]

(iii) y2 = f (x).

[2]

In each case, give if possible, the equations of the asymptotes, the coordinates of
the turning points and the coordinates of the points where the graph crosses the xand y-axes.
(b)

A curve undergoes the transformations A, B and C in succession:


A: a translation of 2 units in the negative direction of the x-axis.
B: a scaling parallel to the x-axis by a factor 2.
C: a translation of 1 unit in the positive direction of the y-axis.
The equation of the resulting curve is y 2 x 9

2
.
x4

Find the equation of the curve before the three transformations were effected.[3]

TJC/MA 9740/Preliminary Exam 2013

A curve is defined by the parametric equations


1 t2
4t
x
, y
, 0 t 1.
2
1 t
1 t2

(i)

Show that the equation of the normal to the curve at the point where t
given by y

(ii)

2
6
x .
3
5

1
is
2
[5]

Sketch the curve and the normal in part (i) on the same diagram.

[2]

(iii) Show that the area bounded by the curve, the normal and the axes can be
expressed in the form

a f (t ) dt c ,

where a, b and c are constants to be

determined. Hence evaluate this area.

2x

(i)

Find

(ii)

Sketch the curves y = ex sin 2x and y = ex for 0 x

[3]

cos 4 x dx .

[4]

on a single diagram. Find


2
the exact x-coordinate of the point of intersection between the two curves.
[3]

(iii) Find the exact volume of the solid generated when the region bounded by the two
curves and the y-axis is rotated through 2 radians about the x-axis.

10

[4]

The functions f, g and h are defined by


f :x

g:x
h:x

x 2 , x , x k ,

1
, x , x 0,
x2
1
, x , x k .
2
x 4x 4

State the largest value of k such that f 1 exists.

[1]

Use this value of k for the following parts.


(i)

(ii)

(iii)

Find f 1 in a similar form. Hence find the range of values of x such that
f x f 1 x .
[4]
Determine the range of values of x for which f x g x , giving your
answers in exact form.
[4]
Show that the composite function gf exists and detemine if the functions h and
gf are equal.
[2]
[Turn over
TJC/MA 9740/Preliminary Exam 2013

11

The equations of two planes p1 and p2 are


2 x y 2 z 0 ,
3x + y + z = ,

respectively, where and are constants.


The line l is the line of intersection between p1 and p2 and has equation
1 x y 1 z

.
3
4
5
(i)

Show that = 1 and = 2.

[3]

(ii)

Find the exact value of the cosine of the acute angle between p1 and p2.

[2]

(iii) The point A with position vector 4j +2k lies on p2. Find the exact shortest
distance from A to l. Hence or otherwise, the exact value of the perpendicular
distance from A to p1.

[5]

(iv) Another plane p3 contains the origin.


(a)

When p3 and p2 have no common point, state a vector equation of p3.

(b)

When p3 is perpendicular to p2, A lies on p3. Find the position vector of the
common point of intersection between p1, p2 and p3.

End of Paper

TJC/MA 9740/Preliminary Exam 2013

[1]
[3]

2013 Preliminary Examination H2 Mathematics 9740 Paper 1 (Solutions)


[System of Equations] [4 marks]
1
At the points of intersection, ax3 +
When x = 1, a +

b = ln p

2 b = ln (4p)
When x = 4, 64a + 2 b = ln (16p)
When x = 2, 8a +

bx = ln (px2)
a + b ln p = 0
8a +

------- (1)

2 b ln p = ln 4 ------- (2)

64a + 2 b ln p = ln 16 ------- (3)

Using GC, a = 0.01


b = 3.557 b = 12.65
ln p = 3.545 p = 34.63
[Differential Equation] [7 marks]
dT
k T 20 , k 0
2
(i)
dt
(ii)

T 20 dT k d t
ln T 20 kt c or ln T 20 kt c

T > 20

T 20 e k t c
T 20 ec e k t
T 20 Ae k t where A= ec
Let x be the time from death to the point where his body temperature reaches
340C.
When t = 0, 37 = 20 + A A = 17
When t = x, 34 20 17e kx 17e kx 14 (1)
When t = x + 90, 30 20 17e k x90 17e

Taking

k x 90

(1)
e kx
14
, we have k x 90
(2)
10
e

e90 k

14
10

7
90k ln
5
1 7
ln
90 5
Sub into (1) and solving for x,
1 14
x ln = 51.9 52
k 17
The victim died at 9.48pm.
k

Page 1 of 15

10 (2)

Mathematical Induction [8 marks]


8
24
24
24
48
32
48
80
S1 , S2
, S3 = 2 2
and S4 = 2 2
3 (i)
9
25
25 5 7 49
49 7 9 81
(ii)

1 1

9 32
1
1
1 S2
2
25 5
1
1
1 S3
2
49 7
1
1
1 S4 2
81 9
1
1 Sn
2
2n 1
1 S1

Hence, a conjecture for Sn 1


(iii) Let Pn be the statement: Sn 1
When n = 1, LHS S1

2n 1

1
, for all positive integers n.
(2n 1) 2

8
1 8
and RHS 1 2 .
9
3
9

P1 is true.

Assume Pk is true for some positive integers k. i.e. Sk 1


When n = k +1, need to prove that Pk+1 is true,
1
1
1
i.e. Sk 1 1
2
2
2k 3
2 k 1 1
LHS Sk 1 Sk uk 1

1
8 k 1

(2k 1) 2 2k 12 2k 32

1
(2k 1) 2

8 k 1
1
2
2k 3

2k 32 8 k 1
1
1

(2k 1)2
2k 3 2

4k 2 12k 9 8k 8
1

(2k 1)2
2k 3 2

4k 2 4k 1
1
(2k 1)2 2k 32

1
1
(2k 1) 2
1

2k 3 2

2k 12

2
2k 3
= RHS

Hence, Pk is true Pk+1 is true.


Page 2 of 15

1
(2k 1)2

Since P1 is true and Pk is true Pk+1 is true, by Mathematical Induction, Pn is true for all
n+.
(iv)

1 Sn < 105
1
105

2
(2n 1)
(2n 1)2 105
Method 1:
Using GC, when
n = 157, (2 157 1)2 99225 105
n = 158, (2 158 1)2 100489 105
Least n = 158
Method 2:
By sketching the graph of y (2n 1)2 105
y

157.6

For (2n 1)2 105 0 , n 157.6


Least n = 158
Method 3:
(2n 1)2 105 0
4n2 4n 1 105 0
(n +158.6) (n 157.6) > 0

158.6

157.6

n > 157.6
or
Least n = 158

n < 158.6 (rejected)

Page 3 of 15

AP & GP [8 marks]
Tn Sn Sn1 3n2 4n 2 3(n 1)2 4(n 1) 2 6n 1
(a)

Tn Tn1 6n 1 6(n 1) 1
= 6, which is independent on n
this progression is an arithmetic progression.

x
r2
x
In the 3rd stage, amount of impurities removed = 3
r

(b)(i) In the 2nd stage, amount of impurities removed =

(ii)

Total amount of impurities removed by the end of the nth stage of purification
x x
x
2 n
r r
r
1

1
x rn

r 1 1
r

(iii)

x r n 1
r

r 1

Assuming that we can carry out the process of purification indefinitely.


Total amount of impurities remained
x x x

x 2 3
r
r r

x 1
x
x
x

r 1 1
r 1
r
x
x
1
1
x , [or alternatively r x ]
For x
1 2
r 1 2
1
r
x
1
x r 3
r 1 2

Page 4 of 15

Complex Numbers 1 [9 marks]


p 1 i 1 3i 1 3i i 3 1
1
1 3 1 3 i
5
(i) z

q 1 3i 1 3i
4
4
4

arg p = tan 1 1

p 12 12 2 ,

(ii)

2
.
3
3
p
2
2
11
z
, arg z arg p arg q

4 3
12
2
q

q 12 3 2 , arg q tan 1 3
2

From part (i) and (ii),

(iii)

1
1
2
11
1 3 1 3 i
cos

4
4
2
12
Comparing the imaginary parts,

2
11 1
sin
1 3
2
12 4

11
Since sin
12

11
i sin

12

11
sin

12

2 3 1
3 1
6 2
11
we get sin

4
4
12 2 2
Alternative Method (Using Argand Diagram)
Im(z)
1
4

2
2

From diagram,

11
sin
12

3 1

11
12

2
2

OR

1
4

11
12

1
4

Re(z)

1 3

3 1

3 1
4

6 2
4
1
4

1 3

11
sin
12

11
sin
12

1 3

2 2

11
sin
12

Page 5 of 15

2
2

1 3

1112

1
4

Re(z)

Im(z)

2
2

2
2

3 1
6 2

4
2 2


11
sin
sin
12
12

Im(z)

OR

12
2
2

1
4

11
12

Re(z)

1 3

Page 6 of 15

14 1 3

2
2

3 1
2 2

6 2
4

Sigma Notation & Method of Difference [9 marks]


6

f (r ) f (r 1) r 1 r r 1 r 2 r 1 r
r 1 r r 1 r 2
3r r 1
n

(i)

r r 1
r 1

1 n
f r f r 1
3 r 1

1
f (1) f 0
3
1
f (2) f 1
3

1
f (n 1) f n 2
3
1
f (n) f n 1
3
1
f (n) f 0
3
1
1
n 1 n n 1 0 n n 1 n 1
3
3
1

(ii)

r r 1 3 n n 1 n 1
r 1
n

r
r 1
n

r 1
n

r 1
n

r 1
n

r 1

(iii)

1
r n n 1 n 1
3
n
1
n n 1 n 1 r
3
r 1
1
1
n n 1 n 1 n(n 1)
3
2
1
n n 1 2 n 1 3
6
1
n n 1 2n 1 (deduced)
6

12 3 22 32 3 42 52 3 62
12 22 32 42 52
n

n 1
2

r 1

r 1

r 2 2 2 r

3 n 1 n2
2

n 2 2 22 42 62

n 1

1
1 n 1 n 1 n 1
n n 1 2n 1 2 22
1 2

1
6
6 2 2
2
1
4 n 1 n 1
n n 1 2n 1

n
6
3 2 2
Page 7 of 15

1
1
n n 1 2n 1 n n 1 n 1
6
3
1
1
n n 1 2n 1 2 n 1 n n 1 4n 1
6
6
Transformation of Graphs [9 marks]
y

7(a) (i)

y = f (x)
y=2
0

x= 1
y

(ii)

1
4

(1, 1 )
3

1 0

x= 2
(iii)

y
y2 = f (x)

2
2
2

(1, 3 )
0
(1, 3 )

1
f ( x)

x= 1

Page 8 of 15

(b)

y 2x 9

2
x4

replace y by y + 1

y 2x 8

2
x4

replace x by 2x

y 2 2x 8

2
1
4x 8
2x 4
x2

replace x by (x2)

y 4( x 2) 8

1
1
4x
( x 2) 2
x

The equation of the curve before the transformations were effected is y 4 x

Parametric Equations [10 marks]


8

(i)

2
2
dx 2t 1 t 2t 1 t 2t 2t 3 2t 2t 3
4t

2
2
2
dt
1 t 2
1 t 2
1 t 2

2
dy 4 1 t 2t 4t 4 4t 2 8t 2
4 4t 2

=
=
2
2
2
dt
1 t 2
1 t 2
1 t 2
2 2

dy dy dx 4 4t 2 1 t t 2 1

dx dt dt 1 t 2 2
4t
t

1
3
8
,x= ,y=
2
5
5
2
1
1
dy 2
3
2

Gradient of the normal =


1
dx
2
3
2
1
Equation of the normal at t = :
2
8 2
3
y x
5 3
5
2
6
y x (Shown)
3
5
At t =

Page 9 of 15

1
x

(ii)

53 , 85

2
8
5
6
5

(iii)

x
3
1
1 65 8 3 1
Area = 3 y dx
2 5 5 5 5

0
21
4t
4t

dt
25 1 1 t 2 1 t 2 2

= 0.447295 + 0.84
= 1.29 units 2 (Using GC)
1
21
a= , b=0, c=
2
25
Integration & its Application [11 marks]
9

(i)

1 2x
1
e cos 4 x e2 x 4sin 4 x dx
2
2
1
e2 x cos 4 x + 2 e2 x sin 4 x dx
2
1
1
1

e2 x cos 4 x + 2 e2 x sin 4 x e2 x 4cos 4 x dx


2
2
2

1
e2 x cos 4 x + e2 x sin 4 x 4 e2 x cos 4 x dx
2
1
5 e2 x cos 4 x dx e2 x cos 4 x 2sin 4 x + c
2
1
2x
2x
e cos 4 x dx 10 e cos 4 x 2sin 4 x + c

2x

cos 4 x dx

Alternatively,

1
1

cos 4 x dx sin 4 x e2 x sin 4 x 2e2 x dx


4
4

1
1
e2 x sin 4 x e2 x sin 4 x dx
4
2
1
1 1

e2 x sin 4 x cos 4 x e2 x cos 4 x 2e2 x dx


4
2 4

1
1
1
e2 x sin 4 x + e2 x cos 4 x e2 x cos 4 x dx
4
8
4
5 2x
1
e cos 4 x dx e2 x cos 4 x 2sin 4 x + c

4
8
1
2x
2x
e cos 4 x dx 10 e cos 4 x 2sin 4 x + c

2x

Page 10 of 15

(ii)

y = ex

y = ex sin 2x

At the point of intersection, ex sin 2x = ex


ex (sin 2x 1) = 0
x
e =0
or sin 2x = 1
no solution as ex > 0

(iii)

2x

x
4

y = ex
y = ex sin 2x

1
0

Volume of solid generated

e
x

dx

sin 2 x

dx

e2 x dx e2 x sin 2 2 x dx
4

1 cos 4 x
4
4
e2 x dx e2 x
dx
0
0
2

1
4
e2 x e2 x cos 4 x dx
0
2

1 1 2x 4 1 2x
4
e + e cos 4 x 2sin 4 x
2 2 0 10
0

1 1 2
e 1 e 2 e 0
10

2 2

1
2e 2 3 units3

10

Page 11 of 15

using (i) answer

Function & Inequalities [11 marks]


10

Largest k 2
(i)

Since x 2 , f ( x) 2 x
Let y 2 x x 2 y

Alternative solution:
Let y x 2
y x2
x = y +2 (rejected since x 2 )

f 1 : x

or y = (x 2)
or x = y +2

2 x, x 0

Df = , 2 and Df 1 0, .
For f x f 1 x , 0 x 2

(ii)

Method 1: Algebraic Method


1
x 2 2
x
3
2
x 2x 1 0
x 1 x2 x 1 0 by long division

1 5 1 5
x
0

2
2

x 1 x

1 5
2

1 5
1 5
x 0 or 0 x 1 or
x2
2
2

Method 2: Graphical Method


y

To find the intersection points:


1
x 2 2
x
3
2
x 2x 1 0
x 1 x2 x 1 0

y = g(x)

y = f(x)

1 5
2

1 5
2

x 1 or x
x

From the graph, for f x g x ,


1 5
1 5
x 0 or 0 x 1 or
x2
2
2

(iii)

Rf = 0, , Dg = \{0}
Since Rf Dg, gf exists.
Since Dh = , 2 = Dgf = Df = , 2 ,
h( x) gf ( x)
Page 12 of 15

1 5
2

1 5
2

Vectors [14 marks]


11

p1 : 2 x y 2 z 0 and
2
i.e. r 1 0
2

P2 :

and

3x + y + z = ,
3

r 1

1 3
1 x y 1 z

Equation of line l is
. i.e. r 0 t 4 , t
3
4
5
1 5

Since l lies on p2, l is perpendicular to the normal of p2.
3 3

4 1 0 9 4 5 0 1 (shown)
5

(i)

Since (1, 0, 1) lies on p2, 3(1) 0 1(1) 2

(shown)

Alternative method 1
Let t = 0 and t = 1 Points (1, 0, 1) and (4, 4, 6) lie on line l and plane p2.
3(1) + 0 + (1) = and
= 1 and = 2

3(4) + (4) + (6) =

Alternative method 2
2 3
3

Let 1 1 k 4 for some k


2
5


2
3

2 6 k 4
5
5

k 1, 2 3

1 (shown)
Since (1, 0, 1) lies on p2, 3(1) 0 1(1) 2 (shown)
(ii)

Let be the acute angle between p1 and p2.

2 3

11
2 1
7
7 11

cos

33
9 11
3 11

Page 13 of 15

A(0, 4, 2)
1
(iii) Let OB 0 and N be the foot of from A to line l.
1

3
l
Perpendicular distance from A to l,
d 4
N
B(1, 0, 1)
5

3
1 3
3



AB 4
8 1
4 4
5
1 5
1
8 11
4 22


AN =

5
50
50
50
50
Hence, shortest distance from A to p1 , |AF|
= AN sin

7 11
4 22
1

5
33

8
3

P2

O
P1

Alternatively,
1 3t
Since N lies on l, ON 4t for some t
1 5t

A(0, 4, 2)
d

1 3t

AN ON OA 4t 4
5t 1

l
B(1, 0, 1)

1 3t 3
7
Since AN is perpendicular to line l, 4t 4 4 0 t
25
5t 1 5

25
AN 72
25

12
5

Perpendicular distance from A to l = AN

352 4 22

5
5

Otherwise Method:

Shortest distance from A to p1=

0

4
2

2

1
2
8

3
9

Page 14 of 15

3
d 4
5

(iv) (a)

(b)

p3 is parallel to p2 and contains the origin.


3
Equation of plane p2: r 1 0
1

p3 contains points O & A OA is parallel to p3 .
3
p3 is perpendicular to p2 p3 parallel to 1
1

0 3
1


n 3 = 4 1 6 1
2 1
2


1
Equation of p3 is r 1 0
2

p1 : 2 x y 2 z 0

Using GC, solve

p2 : 3x y z 2
p3 : x y 2 z 0

8
4
1
2
Position vector of the point of intersection =
12 or
6
17
17
2
1

p2

p3

O
l

Page 15 of 15

p1

TEMASEK JUNIOR COLLEGE, SINGAPORE


Preliminary Examination 2013
Higher 2

9740/02

MATHEMATICS
Paper 2

23 September 2013
Additional Materials:

Answer paper
List of Formulae (MF15)

3 hours

READ THESE INSTRUCTIONS FIRST


Write your Civics group and name on all the work that you hand in.
Write in dark blue or black pen on both sides of the paper.
You may use a soft pencil for any diagrams or graphs.
Do not use staples, paper clips, highlighters, glue or correction fluid.
Answer all the questions.
Give non-exact numerical answers correct to 3 significant figures, or 1 decimal place in the case of
angles in degrees, unless a different level of accuracy is specified in the question.
You are expected to use a graphic calculator.
Unsupported answers from a graphic calculator are allowed unless a question specifically states
otherwise.
Where unsupported answers from a graphic calculator are not allowed in a question, you are required
to present the mathematical steps using mathematical notations and not calculator commands.
You are reminded of the need for clear presentation in your answers.
The number of marks is given in brackets [ ] at the end of each question or part question.
At the end of the examination, fasten all your work securely together.

This document consists of 8 printed pages.

TJC 2013

[Turn over

Section A: Pure Mathematics [40 marks]

The diagram below shows the points P and Q on the circumference of a circle with
centre O, and radius 2a cm, where POQ . Points P and Q are moving on the
circumference so that is increasing at a constant rate.

Find the acute angle at the instant when the rate of change of the area of the shaded
segment is

a
times the rate of change of the length of the minor arc PQ.
2

[5]

The position vectors of the points A, B and C, relative to the origin O are
a i 2k , b 3i j 3k and c i j k respectively.

(i)

Determine whether the points A, B and C are collinear.

(ii)

The point D on the line segment AB is such that AD : DB k :1 k. Find the value of
k such that CD bisects the angle ACB.

TJC 2013

TJC/MA 9740/Preliminary Exam 2013

[2]

[4]

Rubick, the sorcerer, has created a virus that infects people in Town A and Town B
according to the recurrence model given by

xn 2 xn2 5 , for all n 3 ,


where: x1 a

represents the number of people infected in Town A on Day 1,

x2 b

represents the number of people infected in Town B on Day 1,

x2 n 1

represents the total number of infected people in Town A on Day n,

x2n

represents the total number of infected people in Town B on Day n.

(i)

Show that x2n (b 5)2n1 5 , for all n 1.

(ii)

Find the value of b if the number of infected people in Town B is constant for all days.
[1]

[3]

(iii) It is given that x2n1 (a 5)2n1 5 , for all n 1. The number of infected people in
Town B on Day 7 is 93 times the number of infected people in Town A on Day 3.
Find the least possible values of a and b.

[3]

(a) (i) Express ln(1 x) as a series in ascending powers of x, up to and including the term
in x3.

[1]

(ii) Obtain the expansion of

ln(1 x)6
in ascending powers of x, up to and including
(1 3x) 4

the term in x3.

[2]

(iii) State the range of values of x for which the expansion in part (ii) is valid.

[2]

(b) If y cos(ln( x 1)) sin(ln( x 1)) where x 0, show that

x 1

d2 y
dy
x 1 y 0.
2
dx
dx

[2]

By further differentiation of this result, find the Maclaurin's expansion for y up to and
including the term in x3.

[3]
[Turn Over

TJC 2013

TJC/MA 9740/Preliminary Exam 2013

(a) It is given that w m(m 2) (m2 2m 9)i, where m

Find the value of m if arg w = and the point representing w lies on the line
2
[3]
x + y + 9 = 0.
(b) The complex number z satisfies the relations
z 2 3i z 2 i

and arg( z 2 3i) 0 .


3

(i) Illustrate both of these relations on a single Argand diagram and indicate the locus
of the point representing z.

[3]

(ii) Find z when z is the least. Give your answer in the form a + bi where a and b
are exact real numbers.

[2]

(c) Find the fourth roots of 8(1 3 i) , giving your answers in the form rei where r > 0
and .

[4]

Section B: Statistics [60 marks]

On a particular day, an insurance company receives 90 claims for flight delays. Due to
staff shortage, it is only able to process 9 of these claims.
(i)

Describe how you would choose a systematic random sample of size 9 from the
received claims.

(ii)

[2]

Comment on whether this method of sampling gives a better indication of the value of
the 90 claims as compared to simply choosing as the sample the first 9 claims
received.

TJC 2013

[1]

TJC/MA 9740/Preliminary Exam 2013

(a) (i) At a wedding dinner, 6 men and 4 women are to be seated at a round table with 10
identical seats. Find the number of different arrangements if there are no
restrictions.

[1]

(ii) At another round table, 2 of the 10 identical seats are each tied with a red ribbon
and are adjacent to each other. How many ways can 7 people be seated at the
table?

[2]

(b) Eight students participate in the semi-final round of a Mathematics Quiz. They are
randomly paired up to compete with each other and the winner of each pair will
advance to the final round.
(i) Find the number of possible sets of results from the semi-final round. For example,
one possible set of results is A beats C, B beats H, D beats F, and E beats G.

[2]

(ii) How many ways can the prizes be awarded to the four finalists if there are one
$500, one $200 and two $100 book prizes?

[1]

A company uses a machine to produce washers of average thickness 0.50 mm. To


determine whether the machine is in proper working order, a random sample of 10 washers
is chosen and it has a standard deviation of 0.03 mm.
Let x be the sample mean thickness. Find the range of values of x such that there is
insufficient evidence to reject the machine supervisors claim that the machine is in proper
working order, using a significance level of 5%. State any assumptions that you need to
make.

[7]

[Turn Over
TJC 2013

TJC/MA 9740/Preliminary Exam 2013

A particular brand of orange juice is sold in bottles of two sizes. The amount in each bottle,
in millilitres, is normally distributed as shown in the table below.
Size

(i)

Mean (ml)

Standard deviation (ml)

Small

305

Medium

600

30

It is given that 80% of small bottles contains between 300 ml and 312 ml of orange
juice. Find the value of .

(ii)

[2]

Find the probability that the average amount of orange juice in three randomly chosen
small bottles is more than half the amount of orange juice in a randomly chosen
medium bottle.

[3]

(iii) Clara bought fifteen medium bottles of orange juice for a party. Find the least number
of paper cups each of maximum capacity 120 ml that she must buy such that the
probability that there are sufficient paper cups to contain the total amount of orange
juice is at least 95%. Assume that the paper cups are not to be reused and there is no
wastage of orange juice.

10

[4]

(a) In a box, there are 12 marbles of which 5 are red, 4 are black and 3 are white.
4 marbles are taken at random from the box, each marble being replaced before the
next one is taken. Find the probability that
(i) at least one marble of each colour is obtained,

[2]

(ii) at least one marble is white.

[2]

(b) Everyday, Gerald has a choice of 2 routes to get to school. The probability that he gets
43
to school without being delayed is
. The probability that he chooses the first route
50
3
is . If he gets to school without being delayed, the probability that the first route is
5
27
chosen is
. Find the probability that he gets to school without being delayed if
43

TJC 2013

(i) he chooses the first route,

[3]

(ii) he chooses the second route.

[3]

TJC/MA 9740/Preliminary Exam 2013

11

(a)

For each of the following two statements, state, giving a reason, whether the
statement is highly possibly correct, highly possibly incorrect, definitely correct or
definitely incorrect.

[2]

Statement 1: The linear product moment correlation coefficient between the


average journey time to school and the distance travelled is 1.12.
Statement 2: The linear product moment correlation coefficient between the term
test marks of students and the number of days of absence in the term is
0.75.

(b) May travels regularly from home to the hospital for treatment. She leaves home at
x minutes after 8 am and takes y minutes to arrive at the hospital. Ten pairs of data
are recorded in the table below.
x

10

15

20

25

30

35

40

45

31

42

33

48

47

53

68

65

71

70

(i)

Draw a scatter diagram to illustrate the data, labelling the axes clearly.

[1]

(ii)

Calculate the equation of the least squares regression line of y on x and the value
of the linear product moment correlation coefficient for the data.

[3]

(iii) Interpret the regression coefficient in the context of the question.

[1]

(iv) Draw the regression line of y on x on the scatter diagram.

[1]

(v)

May has a hospital treatment appointment at 8.45 am on a particular day. If she


leaves home at 8.10 am, explain whether she is likely to be on time for her
hospital treatment. Comment on the reliability of your answer.

[3]

[Turn Over
TJC 2013

TJC/MA 9740/Preliminary Exam 2013

12

A factory produces a particular type of electronic component. The probability of a


component being acceptable is 0.96. The components are packed in boxes of 24. A box is
rejected if it contains fewer than 22 acceptable components.
(i)

Show that the probability that a randomly chosen box is rejected is 0.0693.

[1]

(ii)

5 boxes are chosen at random. Find the probability that two of the boxes each contain
exactly 22 acceptable components and one box is rejected.
[3]

The factory produces 70 boxes each day. The production is being monitored for n days,
where n is large.
(iii) Find the expected number of rejected boxes in a day.

[2]

(iv) Using a suitable approximation, find the probability that at least 65 boxes are not
rejected in a day.
(v)

[2]

Find the least value of n such that there is a 0.8 chance that the mean number of
boxes being rejected per day is at most 5.

[4]

It is proposed to introduce an alternative policy with regard to packing and inspection, as


follows:
The daily production of components is to be packed in 140 boxes, each containing
12 components, and boxes containing fewer than 11 acceptable components are to be
rejected.
Explain clearly if the alternative policy is more profitable for the factory.

END OF PAPER

TJC 2013

TJC/MA 9740/Preliminary Exam 2013

[2]

1
1

Section A: Pure Mathematics [40 marks]


The diagram below shows the points P and Q on the circumference of a circle with centre O, and

POQ
radius 2a cm, where
. Points P and Q are moving on the circumference so that is
increasing at a constant rate.

Find the acute angle at the instant when the rate of change of the area of the shaded segment is

a
times the rate of change of the length of the minor arc PQ.
2
[Solution]
Let s be the length of the minor arc PQ and A be the area of the shaded segment.
ds
d
s 2a
2a
dt
dt
1
A 4a 2 sin
2
d
dA
d

2a 2
cos

dt
dt
dt
d
2a 2
1 cos
dt
dA a ds
Given
,
dt 2 dt
d
d
2a 2
1 cos = a 2
dt
dt
1
cos
2

or 60

TJC/MA 9740/02/Preliminary Exam 2013

[5]

2
2

The position vectors of the points A, B and C, relative to the origin O are
a i 2k , b 3i j 3k and c i j k respectively.
(i) Determine whether the points A, B and C are collinear.
(ii)

[2]

The point D on the line segment AB is such that AD : DB k :1 k. Find the value of k
such that CD bisects the angle ACB.

[4]

[Solution]
(i)

3 1 2
AB OB OA 1 0 = 1
3 2 1

1 1 0
AC OC OA 1 0 = 1
1 2 1

Since AB m AC for all m

, AB not parallel to AC .

Hence, A, B and C are not collinear.

(ii)

0
2

CA = 1 , CB = 0
1
2

By Ratio Theorem,
CD kCB 1 k CA
2
0
2k

= k 0 1 k 1 = k 1
2
1
k 1

cos =

CA CD
CA CD

CB CD
CB CD

0 2k
2 2k

1 1 k 0 1 k
1 1 k 2 1 k


2
2 2
1 k 1 k
k

4k 2k 2
2

1
3

TJC/MA 9740/02/Preliminary Exam 2013

1k

3
3

Rubick, the sorcerer, has created a virus that infects people in Town A and Town B according to
the recurrence model given by
xn 2 xn2 5 , for all n 3 ,
where: x1 a

represents the number of people infected in Town A on Day 1,

x2 b

represents the number of people infected in Town B on Day 1,

x2 n 1

represents the total number of infected people in Town A on Day n,

x2n

represents the total number of infected people in Town B on Day n.

(i)

Show that x2n (b 5)2n1 5 , for all n 1.

(ii)

Find the value of b if the number of infected people in Town B is constant for all days.
[1]

[3]

(iii) It is given that x2n1 (a 5)2n1 5 , for all n 1. The number of infected people in Town
B on Day 7 is 93 times the number of infected people in Town A on Day 3. Find the least
possible values of a and b.
[3]
[Solution]
(i)
x2 b
x4 2 x2 5 2b 5

x6 2 x4 5 2 2b 5 5 22 b 2(5) 5
x8 23 b 22 (5) 2(5) 5
.
.
.

x2n 2n1 b 2n2 (5) 2n3 (5) ... 5

2n1 b 5 2n2 2n3 .... 21 1

2n1 1
2n 1 b 5

2 1
= (b 5)2n1 5
(ii)

If the number of infected people in Town B is constant for all days,


x4 x2
2b 5 b b 5

(iii)

Given that x2n1 (a 5)2n1 5 and x2 7 93x231 .

b 5 26 5 93 a 5 22 5
64b 315 93 4a 15

93a 270
16
Using GC, least integer a 6 , b 18
b

TJC/MA 9740/02/Preliminary Exam 2013

4
Alternative solution to 3(i)
Let Pn be the statement x2n b 5 2n1 5 n

Consider n 1 .
LHS of P1 x2 b (given)
RHS of P1 b 5 211 5 b

P1 is true.
Assume Pk is true for some k

. Then

x2 k b 5 2k 1 5

x2k 2 2 x2k 5 2 b 5 2k 1 5 5 b 5 2k 11 5 .

Pk 1 is true.
Pk is true Pk 1 is true
P1 P2
By mathematical induction x2n b 5 2n1 5 n

TJC/MA 9740/02/Preliminary Exam 2013

5
4

(a) (i) Express ln(1 x) as a series in ascending powers of x, up to and including the term
in x3.

[1]

(ii) Obtain the expansion of


term in x3.

ln(1 x)6
in ascending powers of x, up to and including the
(1 3x) 4
[2]

(iii) State the range of values of x for which the expansion in part (ii) is valid.

[2]

(b) If y cos(ln( x 1)) sin(ln( x 1)) where x 0, show that

d2 y
dy
x 1 y 0.
[2]
2
dx
dx
By further differentiation of this result, find the Maclaurin's expansion for y up to and
including the term in x3.
[3]
[Solution]

x 1

(a) (i) ln(1 x) x

x 2 x3
...
2 3

ln(1 x)6
(ii)
6ln(1 x) (1 3x)4
4
(1 3x)

x 2 x3
4(5)
3x 2 ...
6 x ... 1 4(3x)
2 3
2!

6 x 3x2 2 x3 ...1 12 x 90 x2 ...

6 x 69 x2 506 x3 ...
(iii) For the expansion of ln(1 x) to be valid, 1 x 1 .

1
1
For the expansion of (1 3x)4 to be valid, 1 3x 1 i.e. x .
3
3
6
1
1
ln(1 x)
So, for the expansion of
to be valid, x .
4
3
3
(1 3x)
(b)

y cos(ln( x 1)) sin(ln( x 1))

dy
1
1

sin(ln( x 1))
cos(ln( x 1))
dx
x 1
x 1
dy
( x 1) sin(ln( x 1)) cos(ln( x 1))
dx

( x 1)

d 2 y dy
1
1

cos(ln( x 1))
sin(ln( x 1))
2
dx
dx
x 1
x 1

( x 1)2

d2 y
dy
( x 1) cos(ln( x 1)) sin(ln( x 1))
2
dx
dx

d2 y
dy
( x 1) y 0
( x 1)
2
dx
dx
2

TJC/MA 9740/02/Preliminary Exam 2013

6
( x 1)2

d3 y
d2 y
dy

3(
x

1)
2 0
3
2
dx
dx
dx

When x = 0, y = 1,

y 1 x

d2 y
d3 y
dy
,
1,

2
4.
dx 2
dx3
dx

x2
x3
(2) (4) ....
2!
3!

i.e. y 1 x x 2

2 x3
....
3

TJC/MA 9740/02/Preliminary Exam 2013

7
5

(a) It is given that w m(m 2) (m2 2m 9)i, where m

Find the value of m if arg w = and the point representing w lies on the line
2
x + y + 9 = 0.

[3]

(b) The complex number z satisfies the relations


and arg( z 2 3i) 0 .
3
(i) Illustrate both of these relations on a single Argand diagram and indicate the locus of
the point representing z.
[3]
z 2 3i z 2 i

(ii) Find z when z is the least. Give your answer in the form a + bi where a and b are
exact real numbers.

[2]
i

(c) Find the fourth roots of 8(1 3 i) , giving your answers in the form re where r > 0 and
.

[4]

[Solution]
(a)

w m(m 2) (m2 2m 9)i, m


arg w = Re(w) 0 m(m 2) 0 m 0 or m 2
2
When m = 0, w 9i . i.e. x = 0, y = 9. It satisfies x + y + 9 = 0.
When m = 2, w i . i.e. x = 0, y = 1. It does not satisfy x + y + 9 = 0.
m=0

TJC/MA 9740/02/Preliminary Exam 2013

8
(b)

(i)

arg( z 2 3i) 0

Locus of z

z 2 3i z 2 i
O

(ii)

(c)

[
G
1
]
n
e
Least |z| = OB
e
BC
d 2
AC
tan( / 3)
3

arg( z 2 3i)
3

Hence, z = o2 2 i
3

s
1+ 3 i = 1+ 3 = 2 , arg(1+ 3i) = tan -1 3 = . 1+ 3 i = 2e 3
h
3

i
a
( OR 8(1 3 i) 16e 3 )
d
e 3 i)
Let z = 4th root of 8(1
i

i( +2 k)
3

z 4 8(1 3 i) z 4 16e 3 24 e

z 2e

1+6 k

12

12

, k = 0, 1 , 2
5
i
12

= 2e , 2e

, 2e

7
12

11
i

12

2e

TJC/MA 9740/02/Preliminary Exam 2013

9
Section B: Statistics [60 marks]
On a particular day, an insurance company receives 90 claims for flight delays. Due to staff
shortage, it is only able to process 9 of these claims.
(i) Describe how you would choose a systematic random sample of size 9 from the received
claims.
[2]
(ii) Comment on whether this method of sampling gives a better indication of the value of the
90 claims as compared to simply choosing as the sample the first 9 claims received.[1]
[Solution]
6

(i)

Arrange the 90 claims in an ordered list based on the value of claims and assign numbers
from 1 to 90 to them.
90
As
10 , generate a random number from 1 to 10, and select the claim with this
9
number. Then select every 10th claim down the list until a sample of 9 claims is obtained.

(ii)

Answer 1 (The 90 claims have been arranged based on the value of the claims in (i))
Systematic sampling is better as it gives an even spread on the different values of claim.
The first nine claims received may not cover a good range of values of claim.
Answer 2: (The 90 claims have been arranged based on the time the claims are received or
based on the name of the people making the claims in (i))
Systematic sampling may not be better as it may not necessarily gives an even spread on
the different values of claim than the sample of the first 9 claims received.

TJC/MA 9740/02/Preliminary Exam 2013

10
7

(a) (i) At a wedding dinner, 6 men and 4 women are to be seated at a round table with 10
identical seats. Find the number of different arrangements if there are no restrictions.
[1]
(ii) At another round table, 2 of the 10 identical seats are each tied with a red ribbon and are
adjacent to each other. How many ways can 7 people be seated at the table?
[2]
(b) Eight students participate in the semi-final round of a Mathematics Quiz. They are randomly
paired up to compete with each other and the winner of each pair will advance to the final
round.
(i) Find the number of possible sets of results from the semi-final round. For example, one
possible set of results is A beats C, B beats H, D beats F, and E beats G.
[2]
(ii) How many ways can the prizes be awarded to the four finalists if there are one $500, one
$200 and two $100 book prizes?
[1]

[Solution]
(a) (i)
(ii)

No of ways = (10 1)! = 9! =362880


No of ways = 10 P7 = 604800

(b) (i) Number of possible sets of results =

OR

C2 6C2 4C2 2C2 4


2 1680
4!

(Split the students into pair groupings first, then decide on the results)

8
Number of possible sets of results = 4! 1680
4
8
(There are ways of choosing 4 winners. And there are 4! ways of matching these
4
winners to losers.

(ii) Number of ways to award the four finalists =

4!
= 12
2!

TJC/MA 9740/02/Preliminary Exam 2013

11
8

A company uses a machine to produce washers of average thickness 0.50 mm. To determine
whether the machine is in proper working order, a random sample of 10 washers is chosen and it
has a standard deviation of 0.03 mm.
Let x be the sample mean thickness. Find the range of values of x such that there is insufficient
evidence to reject the machine supervisors claim that the machine is in proper working order,
using a significance level of 5%. State any assumptions that you need to make.

[Solution]
Let X be the thickness of a washer and be the mean thickness of a washer.
Ho : 0.5 (machine is in proper working order)
H1 : 0.5
Level of significance = 5%
Given sample variance is 0.032 (Population variance 2 is unknown)
n
10
s2
(sample variance) 0.032 0.001
n 1
9
Assume that X is normally distributed.
X
Test statistic:
t9
s n

x 0.5
0.001 10

If H o is true,

tcal

and

tcritical 2.26216

To have insufficient evidence to reject supervisors claim, it means do not reject H o ,

2.26216

x 0.5
2.26216
0.001 10

0.47738 x 0.52262
0.477 x 0.523 (3 s.f.)

TJC/MA 9740/02/Preliminary Exam 2013

[7]

12
9

A particular brand of orange juice is sold in bottles of two sizes. The amount in each bottle, in
millilitres, is normally distributed as shown in the table below.
Size

Mean (ml)

Standard deviation (ml)

Small

305

Medium

600

30

(i)

It is given that 80% of small bottles contains between 300 ml and 312 ml of orange juice.
Find the value of .
[2]

(ii)

Find the probability that the average amount of orange juice in three randomly chosen small
bottles is more than half the amount of orange juice in a randomly chosen medium bottle.
[3]

(iii) Clara bought fifteen medium bottles of orange juice for a party. Find the least number of
paper cups each of maximum capacity 120 ml that she must buy such that the probability
that there are sufficient paper cups to contain the total amount of orange juice is at least
95%. Assume that the paper cups are not to be reused and there is no wastage of orange
juice.
[4]
[Solution]
(i) Let S be the amount of orange juice in a small bottle. y
y = 0.8
S ~N(305, 2)
P(300 < S < 312) = 0.8
y = P(300 < S < 312)
Using GC, = 4.5726412 = 4.57 (3 s.f.)

4.57
(ii) Let M be the amount of orange juice in a medium bottle.
M ~N(600, 302)
S S S3
Let S 1 2
3
1
1

E S M E( S ) E( M ) 5
2
2

1 Var( S ) 1
4.57264122 1

Var S M
Var( M )
302 231.9697
2
3
3

2
2
1
S M ~ N(5, 231.9697)
2
1
1

P S M P S M 0 = 0.629
2
2

TJC/MA 9740/02/Preliminary Exam 2013

13
Alternatively
3
3

E S1 S2 S3 M 3E( S ) E( M ) 15
2
2

3
Var S1 S2 S3 M 3Var(S ) Var(M ) 2087.727
2

2
3
S1 S2 S3 M ~ N(15, 2087.727)
2
3
S S S3 1

P 1 2
M P S1 S2 S3 M 0 = 0.629
3
2
2

(iii) Let T = M1+ M2+ M3+..+M15 and n be the number of paper cups required.
T ~N(9000, 13500)
P(T 120n) 0.95
Using GC, P(T 12076) = 0.84915
P(T 12077) = 0.98057
she must buy at least 77 paper cups.
Alternatively

120n 9191.114
n 76.59
she must buy at least 77 paper cups.

TJC/MA 9740/02/Preliminary Exam 2013

14
10

(a) In a box, there are 12 marbles of which 5 are red, 4 are black and 3 are white. 4 marbles are
taken at random from the box, each marble being replaced before the next one is taken. Find
the probability that
(i) at least one marble of each colour is obtained,

[2]

(ii) at least one marble is white.

[2]

(b) Everyday, Gerald has a choice of 2 routes to get to school. The probability that he gets to
43
3
school without being delayed is
. The probability that he chooses the first route is . If
5
50
27
.
he gets to school without being delayed, the probability that the first route is chosen is
43
Find the probability that he gets to school without being delayed if
(i) he chooses the first route,

Need to consider the


order of the colours of
the marbles drawn.

(ii) he chooses the second route.


[Solution]
(i)

P at least one marble of each colour

5 4 3
4! 5
1
12 12 12
2! 12

Alternative method 1
P at least one marble of each colour
2
2
2
2
2
2
3 4 4 4 5 4
4! 3 4 3 5 4 5


12 12 12 2!2! 12 12 12 12 12 12
1

3
3
3
3 9 4 8 5 7

4 12 12 12 12 12 12

5
12

Answer needs to be simplified

Note: The draws are with replacement.

3 4 5 4 3 5 5 3 4

5 4 3
4!
1 or 2 1 1 2 1 1 2 1 1
Cannot use
12 11 10
2!
12

4

Alternative method 2

P at least one marble of each colour

P(4R)

P(4B)

P(4W)

9 4 8 4 7 4
5 4 4 4 3 4 5
1
12 12 12
12 12 12 12
P(no W), which
includes P(4R)+P(4B).

P(no B), which


includes P(4R)+P(4W).

P(no R), which


includes P(4B)+P(4W).

TJC/MA 9740/02/Preliminary Exam 2013

[3]
[3]

15
4

175
9
(ii) P at least one marble is white 1 P no white 1
256
12
Alternative method 1
P at least one marble is white P 4W P 3W P 2W P 1W
4

3
3 9 3 9
3 9 175
4 6 4
12
12 12 12 12
12 12 256
Alternative method 2
P at least one marble is white
P 1st marble is white P W' W P W' W' W P W' W' W' W
2

3 9 3 9 3 9 3 175

12 12 12 12 12 12 12 256
Alternative method 3

P at least one marble is white


3
3
2
2
4 4 5 4
4 5 5 4
4 5 175
1 4 4 6
12 12 12 12
12 12 256
12 12

Given P 1st route chosen

(a)

P no delay

(i)

43
50

3
;
5

Delay
3
5

27
P 1st route chosen | no delay
43
P 1st route chosen and no delay 27

43
43
50
P 1st route chosen and no delay
P no delay | 1st route

(ii)

1st
route

No delay
Delay

2
5

27
0.54
50

2nd
route

No delay

Note: This a is a conditional probability


=P(D given that 2nd route is chosen)
i.e. a = P(D|2nd route)

P 1st route chosen and no delay


P 1 route
st

27

50 9
3
10
5

Let P no delay | 2nd route a Alternatively,

43 27 2
= a
50 50 5
4
a
5

P no delay

P(no delay) =P(1st route no delay) + P(2nd route no delay)


P 2nd route no delay =
P no delay | 2nd route

P 2nd route no delay


P 2st route

TJC/MA 9740/02/Preliminary Exam 2013

43 27 16 8

50 50 50 25

8
4
25
2
5
5

16
11

For each of the following two statements, state, giving a reason, whether the statement is

(a)

highly possibly correct, highly possibly incorrect, definitely correct or definitely incorrect.
[2]
Statement 1: The linear product moment correlation coefficient between the average
journey time to school and the distance travelled is 1.12.
Statement 2: The linear product moment correlation coefficient between the term test
marks of students and the number of days of absence in the term is 0.75.
(b) May travels regularly from home to the hospital for treatment. She leaves home at

minutes after 8 am and takes y minutes to arrive at the hospital. Ten pairs of data are
recorded in the table below.
x

10

15

20

25

30

35

40

45

31

42

33

48

47

53

68

65

71

70

(i)

Draw a scatter diagram to illustrate the data, labelling the axes clearly.

[1]

(ii)

Calculate the equation of the least squares regression line of y on x and the value of the
linear product moment correlation coefficient for the data.

[3]

(iii) Interpret the regression coefficient in the context of the question.

[1]

(iv) Draw the regression line of y on x on the scatter diagram.

[1]

(v)

May has a hospital treatment appointment at 8.45 am on a particular day. If she leaves
home at 8.10 am, explain whether she is likely to be on time for her hospital treatment.
Comment on the reliability of your answer.

[3]

[Solution]
(a) Statement 1: Definitely incorrect as r cannot exceed 1.
Statement 2: Highly possibly incorrect. We expect a negative correlation coefficient.
(b) (i) (iv)

TJC/MA 9740/02/Preliminary Exam 2013

17
(ii)Equation of the least squares regression line of y on x :
y = 31.527 + 0.94545x
i.e. y = 31.5 + 0.945x (to 3 s.f.)
and r = 0.949 (to 3 s.f.)

(iii) Interpretation of the regression coefficient 0.945: The increase in the amount of time (in
min) taken to arrive at the hospital per minute delay after 8am in leaving home.
(v)

If May leaves home at 8.10 am, x =10


y = 31.527 + 0.94545(10) = 40.98 41
Estimated arrival time at the hospital = 8.51 am
May is likely to be late for her hospital treatment.
Since x = 10 lies within the given data range and r = 0.949 is close to 1, the above
answer is reliable.

TJC/MA 9740/02/Preliminary Exam 2013

18
12

A factory produces a particular type of electronic component. The probability of a component


being acceptable is 0.96. The components are packed in boxes of 24. A box is rejected if it
contains fewer than 22 acceptable components.
(i)

Show that the probability that a randomly chosen box is rejected is 0.0693.

[1]

(ii)

5 boxes are chosen at random. Find the probability that two of the boxes each contain
exactly 22 acceptable components and one box is rejected.
[3]

The factory produces 70 boxes each day. The production is being monitored for n days, where n
is large.
(iii) Find the expected number of rejected boxes in a day.

[2]

(iv) Using a suitable approximation, find the probability that at least 65 boxes are not rejected
in a day.
(v)

[2]

Find the least value of n such that there is a 0.8 chance that the mean number of boxes
being rejected per day is at most 5.
[4]

It is proposed to introduce an alternative policy with regard to packing and inspection, as


follows:
The daily production of components is to be packed in 140 boxes, each containing
12
components, and boxes containing fewer than 11 acceptable components are to be rejected.
Explain clearly if the alternative policy is more profitable for the factory.
[2]
[Solution]
Let X be the number of electronic components that are acceptable in a box of 24 components.
X ~ B(24, 0.96)
(i)

P(a randomly chosen box is rejected) = P(X < 22)


= P X 21
= 0.069288 0.0693 (to 3 s.f.)

(ii)

Required probability

2 5!
2
= P X 22 P X 21 1 P X 22 P X 21

2!2!
5!
2
2
= 0.179886 0.069288 1 0.179886 0.069288
2!2!

= 0.0379

TJC/MA 9740/02/Preliminary Exam 2013

19
(iii)

Let Y be the number of boxes (out of 70) which are rejected in a day.
Y ~ B(70, 0.069288)
Expected number of rejected boxes in a day = 70 0.069288 = 4.85

(iv)

Y ~ B(70, 0.069288)
Since n = 70 is large and np = 4.85016 <5,
Y ~ P0(4.85016) approximately
P( 65 boxes are not rejected) = P( 5 boxes are rejected)
= P(Y 5)
= 0.642

(vi)

Since n is large, by Central Limit Theorem,


Y

Y1 Y2
n

Yn

4.51410

N 4.85016,
approx.
n

P Y 5 0.8

Using GC,
when n = 142, P Y 5 0.79966 0.8


when n = 143, P Y 5 0.80046 0.8
Hence, least integer n = 143
Let W be the number of electronic components that are acceptable in a box of 12 components.
W ~ B(12, 0.96)
P(a box is rejected) = P(W < 11)
= P W 10
= 0.080935
Under the alternative policy, the probability that a box is being rejected is higher.
Hence the alternative policy is not more profitable.

TJC/MA 9740/02/Preliminary Exam 2013

Serangoon Junior College


Mathematics
H2 P1

Let y e2 x sin 3x .
(i)

Using the standard series expansions of ex and sin x, show that the Maclaurins
series for y is given by
y ax bx 2 cx3 5x 4 ... , where a, b and c are constants to be
determined.
[3]

(ii)

The graph of y e2 x sin 3x for 0 x 1 is given below.


y

y0

y e2 x sin 3x

x0

A student wants to use a polynomial of degree up to 4 as an approximation for


y = e2 x sin 3x . Explain with a clear diagram, which of the following
polynomials he should use if he wants to use it to estimate y0, the maximum

value of y in the above diagram.


y ax bx 2
y ax bx2 cx3
y ax bx2 cx3 5x4

8x2 1
dx .
4 x2 1

(a)

Find

(b)

Find, using integration by parts,

3x

tan 1 e 3 x dx

[2]

[2]

[4]

2
( x a)( x b)
d
, x , a, b, c, d ,
cx d
c
is shown below. The asymptotes are y = x + k and x = 2 where k is a constant. The curve cuts
the x-axis at 3 and 2 and the y-axis at 3 .

The graph of the function y f ( x) where f ( x)

x = 2

y=x+k

y = f(x)

(i)

If a < b, find the values of a, b, c, d and k.

(ii)

Sketch on a separate diagram, the graph of y =

1
f ( x)

[5]
[3]

Your sketch should clearly show any axial intercepts and equations of asymptotes.

The complex numbers z1 2 and z2 i are roots of the cubic equation


z3 + Az2 + Bz + C = 0, where A, B and C are real constants.
(i)

State the third root and find the values of A, B and C.

(ii)

Hence, with a suitable substitution, find the roots of the equation


2w3 w2 + 2w 1 = 0.

[3]

[3]

3
5

(a)

Without using a graphic calculator, find the set of values of x such that
1
4
.
2
2 x x
9

(b)

The functions f and g are defined by


f :x
g:x

[4]

x 1 for x 1
4 x 2 4 x 2 for 3 x 0

(i)

Show that the composite function fg exists;

[2]

(ii)

Find the composite function fg in a similar form and write down its
range.
[3]

It is given that f ( x) x3 x 1 .
(i)

Show that f(x) is an increasing function for all values of x.

[1]

The equation f ( x) 0 has only one real root, which lies between 1 and 1. To
approximate the value of , a student proposes to generate a sequence of real numbers
x1 , x2 , x3 , given by
x1 k and

xn 1

2 xn3 1
,
3xn2 1

for n +and k .

(ii)

Show that if the sequence converges, it converges to .

(iii)

By considering xn1 xn and using the result in part (i), show that
if xn , then xn1 xn ;

[2]

[3]

if xn , then xn1 xn .
(iv)

Hence, predict the behaviour of the sequence for x1 = 1. Justify your


prediction with results from your graphic calculator.
[2]

4
7

(a)

A sequence u1 , u2 , u3 , is given by

u1 0 and

+
un1 un 2n 1 n .

Use the method of mathematical induction to prove that


un n 1 .
2

(b)

(i)

By using the identity 4sin3 3sin sin 3 , show that


n

r 0

(ii)

[4]

sin 3 3r

1
1

3sin n sin 3n1

4
3

Hence, find the infinite sum


2
3
1
1 3 3 1
33
33
sin sin 2 sin
3 sin
...
2 3
2 3
2 3
2
3

[2]

The points A and B are equidistant from the origin, O and have position vectors a and

b (referred to O) such that the acute angle AOB is . The point N on AB is such that
4
AN : NB = 1: 2 and the point M is the foot of perpendicular of N on OB.

(i)
(ii)

[3]

Show that the position vector of point M is

1 2 b .

3
Given that b is a unit vector, find the exact area of OMN .

[4]
[4]

In a model of loan repayment offered by ABC Bank, the amount of money owed by
the borrower, x (dollars) at time t (months), increases, due to interest, at a rate
proportional to the amount of money owed. The borrower repays the loan at a
constant rate r. Theoretically, the model suggests that when x = a, the interest and
repayment balance, i.e. the amount of money owed by the borrower remains constant.
It is assumed that both x and t are continuous variables.
(i)

Show that

dx r
x a .
dt a

[3]

A man borrows $A from the bank.


(ii)
(iii)

Find the amount owed by the man at time t, in terms of t, r, a and A.


[3]
Determine with a suitable diagram if the loan can be repaid in a finite time
if A < a.
[2]

5
10

(a)

Find the coordinates of the point(s) to the curve

2 x2 xy y 2 9
at which the tangent is parallel to the y axis.
(b)

[5]

The curve C has parametric equations

x t 2 t , y 4 t.

11

(i)

The point P on the curve has parameter p. Show that the equation of the
tangent at P is
[2]
(2 p 1)(4 p y) x p 2 p .

(ii)

Hence, show that every tangent to the curve C does not meet the curve
again.
[3]

The curve C has equation 3 y 2 9 x2 .


(i)

Sketch the graph of C , labelling the exact coordinates of all turning points and
points of intersection with the axes.
[2]

The region R is bounded by the curve C, the positive x axis and the positive y axis.
(ii)
(iii)

12

Find the exact dimensions of the rectangle of largest area that can be inscribed
[7]
in region R such that the origin is one of the vertices of the rectangle.
Hence, find the exact maximum area of a rectangle inscribed in an ellipse with
equation 3( y 2)2 9 ( x 3)2 .
[2]

The points P and Q have position vectors 3i 4 j k and 5i 7 j 6k respectively.


The plane 1 contains the point Q and is perpendicular to PQ. The equation of plane

1 is r a b c where , and the vectors b and c are perpendicular.

(i)

State a possible vector a and verify that b can be taken as 3i 2 j .


Hence find a suitable vector c .

[5]

The plane 2 has equation x 2 y 3z 6 .


(ii)
(iii)
(iv)

Find the cosine of the acute angle between the two planes 1 and 2 .
[2]
The line of intersection of the two planes is l. Find the vector equation of l.
[2]
Find the length of the perpendicular from the point P to the line l.
[3]

(i)

e2 x

1 2 x

2
3
2 x
2 x

...

2!
3!
4
2
1 2 x 2 x 2 x3 x 4 ...
3
3

sin 3x

3
3x

3x
...

3!
9 3
3x x ...
2
4
9
81 5

x ...
e2 x sin 3x 1 2 x 2 x 2 x3 ... 3x x3
3
2
40

9 3
3x x 6 x 2 9 x 4 6 x3 4 x 4 ...
2
3
3x 6 x 2 x3 5 x 4 ...
2
(ii)

3
y 3 x 6 x 2 x3 5 x 4
2

y 3x 6 x 2

3 3
x
2

y 3x 6 x 2

3 3
x
2

y e2 x sin 3x

x0

At x = x0, the error between the y value of the curve y e2 x sin 3x and the y values
3
of the other 3 curves is the least for y = 3x 6 x 2 x3 .
2
3
Hence, the student should use y = 3x 6 x 2 x3
2

(a)

8x2 1
dx
4 x2 1
3

2 2 dx
4 x 1

3
2
dx or
2
(2 x) 1

1
1

2 3
2(2 x 1) 2(2 x 1)

3 2 x 1
3
3
2 x ln
C or 2 x ln 2 x 1 ln 2 x 1 C
4
4
4 2 x 1

(b)

3x

tan 1 e3 x dx

1
1
3e3 x
e3 x tan 1 e3 x e3 x
dx
3
3 1 e3 x 2
1
1
dx
e3 x tan 1 e3 x
3
(1 e6 x )

1 3x
e6 x
1
3 x
dx
e tan e 6 x
3
e 1
1
1
e3 x tan 1 e3 x ln e6 x 1 C
3
6

dx

As the x intercepts are x = 3 and x = 2,


a=2
b=3

y = f ( x)

( x 2)( x 3) x 2 x 6

(cx d )
x2

xk
c=1

(By long division)

A
4
x 1
( x 2)
( x 2)

d=2
k = -1
a = 2, b = 3, c = 1, d = 2 and k = -1 or
a = -3, b = -2, c = 1, d = 2 and k = -1.

(i)

y=

1
f ( x)

x = 3

x=2

(i)

The third root is z3 i


Let P( z ) z 3 Az 2 Bz C
( z 2)( z i)][ z (i)]

( z 2)( z 2 i 2 ) ( z 2)( z 2 1)
z3 2z 2 z 2

By comparing coefficients:
A 2, B 1 and C 2

(ii)

2w3 w2 + 2w 1 = 0

1
,
z
1
1
1
2( )3 ( )2 2( ) 1 0
z
z
z
2
1
2
1
+ 2 3 =0
z
z
z
3
2
z 2z z 2 0
From part (i), the roots of z3 2z2 + z 2 = 0 are 2, i.
1
1
w= ,
2
i
Replacing w by

i.e. w =

1
, i
2

(a)

4
1

0
9 2 x x2
4 ( 2 x x2 ) 9
0
2 x x2
(4 x 2 4 x 1 )
0
2 x x2

2 x 1 0

2 x 1 x
2

Using sign test, x 2 or x

(b)

(i)

( 2 x 1) 2
0
2 x x2

1
2

1
or x 1 .
2

y
(3, 26)
y = g(x)

(1,1)
x
From the GC, R g 1, 26
Since R g Df [1, ) , fg exists.
(ii)

fg x 4 x 2 4 x 1 ( 2 x 1)2 2 x 1
fg : x

Or fg : x

2 x 1 , 3 x 0

(2 x 1),

2 x 1,

3 x
1
x 0
2

1
2

From GC,
y

5
-3
R fg [0,5]

(i)

f '( x) 3x2 1 0 for all x


f(x) is an increasing function for all values of x.

(ii)

Let the limit of the sequence be l.


As n l , xn l , xn1 l

2l 3 1
3l 2 1
3l 3 l 2l 3 1
l3 l 1 0
Since the root of l 3 l 1 0 is the same as that of f (x) x3 x 1 0 ,
l (shown)
l

(iii)

Consider xn 1 xn

2 xn3 1
xn
3xn2 1

xn3 xn 1
3xn2 1

xn3 xn 1

3xn2 1
f ( x )
2 n
3xn 1

Since 3xn2 1 0 and f ( xn ) f ( ) = 0 for xn (because f is an increasing


f ( x )
function), we have 2 n 0 if xn .
3xn 1
xn1 xn 0 if xn
xn1 xn if xn (shown)
Similar, 3xn2 1 0 and f ( xn ) f ( ) = 0 for xn (because f is an
f ( xn )
increasing function), we have
0 if xn .
3xn2 1
xn1 xn 0 if xn
xn1 xn if xn (shown)

(iv)

Since x1 = 1 < , from part (iii), the sequence of numbers increases


and from part, I predict that the sequence of numbers converges to the root .
This is seen from the results generated from the GC below.
n xn
1
-1
2
-0.75
3 -0.68605
4 -0.68234
5 -0.68233
6 -0.68233

(a)

Let Pn be the statement un n 1 , n


2

When n = 1, LHS = u1 0 (given)


RHS = 1 1 0 = LHS
2

P1 is true.

Assume that Pk is true for some k


i.e. uk k 1

To show that Pk+1 is true,


i.e. uk 1 k 1 1

Consider LHS = uk 1

uk 2k 1 (from un1 un 2n 1 )
k 1 2k 1
2

k 2 2k 1 2k 1
k 2 k 1 1 = RHS
2

Pk+1 is true if Pk is true.

Since P1 is true and Pk+1 is true if Pk is true, by Mathematical Induction, Pn is true for
all n .

(b)

(i)

sin 3 3r

r 0

1 1

3sin 3r sin 3r 1
r

r 0 3 4
1 n 1
1

r 1 sin 3r r sin 3r 1
4 r 0 3
3

1
3sin sin 3
4
1
sin 3 sin 9
3
n

1
sin 3n
n 1
3
3
1
1

n 1 sin 3n n sin 3n 1
3
3

1
1

3sin n sin 3n1


4
3

n2

sin 3n 1

2
1 3 3 1
3 3
(ii) sin sin
sin

2 3
2 32
2

1
3 1
32
= lim sin 3 sin 3
2 sin 3
N
2 3
2 3
2

1
1
3N 1
= lim 3sin N sin

N 4
2 3
2

3

3
= sin 0
4
2
4
3

N
1
33
sin

3N
2

(i)

Using ratio theorem,

ON

2a b
3

OM ON b b
1
b
2a + b
b
3

1 b
2 2a b + b b
3 b
1 b

2
2 2 a b cos + b
3 b
4

1 b
2
2
2
2 2 b
+
b

3 b
2

(ii)

1 2 b (Shown)
3

Area of OMN

1
2

1
ON OM
2

2a b 1 2

3
3

1 2 2a b b

1 2 2a b b b

1 2 a b sin n

18

18

where n is a unit vector a & b

(i)

1 2 b
9

22
18

units

(Since b is a unit vector, b 1 )

dx
= kx r
dt
When x = a,
k=

(ii)

dx
= ka r = 0
dt

r
a

dx r
x a
dt a

1
r
dx dt
xa
a

ln|x a| =

(Shown)

r
t C
a

r
t
a

x = Be + a
Given that, when

, we get B = A a
r
t
a

Therefore, x = (A a ) e + a

(i)

When the loan is repaid,

10

(a)

dy
dy

4x x y 2 y
0
dx
dx

4 x y dy

2 y x dx
When the tangent is parallel to the y axis,
2y x 0 x 2y
Hence, 2(2 y)2 (2 y) y y 2 9

8 y2 2 y2 y2 9
Solving, y 1 or y 1.
Since x = 2y,
x = 2 or x = 2.
Hence, the coordinates are (2, 1) and (2, 1).
dx
dy
(b)
(i)
2t 1,
1
dt
dt
dy
1

dx 2t 1

x p 2 p, y 4 p ,

At the point P,
Hence, equation of tangent at P:
1
y (4 p)
x ( p 2 p)

2 p 1

dy
1

.
dx 2 p 1

(2 p 1)(4 p y) x p 2 p (shown)
(ii)

For the tangent to meet the curve again, there must be another value of t
such that:

(2 p 1)(4 p (4 t )) (t 2 t ) p 2 p .
(2 p 1)t 2 p 2 p t 2 t p 2 p
t 2 2 pt p 2 0
(t p)2 = 0
Since t = p is the only solution, then every tangent to the curve C does
not meet the curve again.

11
Solution
y

(0, 3)

(3, 0)

(3, 0)

(0, 3)

(ii) Consider a rectangle inscribed in the ellipse, touching the curve at the point (x, y).
Area of rectangle, A = xy x

9 x2
3

1 9 x 2 1/2 2 x 9 x 2 1/2
dA

2 3 3 3
dx

dA 9 x 2

dx 3

1/2

9 2 x2

At maximum area,
9 x2
i.e.

1/2

dA
0
dx

9 2 x2

0
3

9 2 x2
0 9 2 x2 0
3
Hence, x

dA
dx

Slope

3 2
3 2
(reject as x > 0)
or x
2
2

3 2
2
+

3 2
2
0

3 2
2

Thus, area is a maximum at x

Sub. x

3 2
.
2

3 2
into 3 y 2 9 x2 .
2
2

9
3
3y 9
9 2
2
2

So, y 2

3
6
6
y
or y
(rej. as y 0)
2
2
2

Thus, rectangle of maximum area has dimensions

6
units.
2

3 2
units by
2

11. (ii) Alternative solution

-----------------------------------------------(1)

----------------------------------------------(2)
Substituting (1) into (2), we obtain
.
When area is maximum,
Thus,
-----------------------------------------------(3)
Substituting equation (3) into the equation of the curve

Since

3 2
.
2

9
3
3y2 9
9 2
2
So, y 2

3
6
6
y
or y
(rej. as y 0)
2
2
2

3 2
2
+

dA
dx

3 2
2
0

3 2
2

Slope

Thus the dimensions

3 2
units by
2

6
units make area a maximum.
2

(iii) The ellipse is obtained by a translation of the curve C without scaling.


Hence, by symmetry, the maximum area will be 4 times the area of the rectangle in (ii).
Maximum area =

3 2
6

4
2
2

Maximum area 6 3 units2

12

(i)

Given 1 : r a b c and b c .

Since a can be any position vector of point on plane 1 and that since Q is on

5
plane 1 a OQ 7 .

6

5 3 2
Consider PQ OQ OP 7 4 3

6 1

(ii)

3 3 2
Since 2 PQ 2 3 0


0
0 7


3
2 is parallel to 1 .
0

3
Hence b 2

0

14
3 2
To find c , consider b PQ 2 3
21

0 7
13

14
Thus c 21 .

13

2
1

Normal of 1 is 3 . Normal of 2 is 2 .


7
3


If is the angle between the two planes, then
2 1
3 2 62 14 cos

7 3

13
cos
2 217

(iii)

Method 1 (using GC)


2 5 2
1 : r 3 7 3 73
7 6 7

2 : x 2 y 3z 6

2 x 3 y 7 z 73

164
23

By using GC, l: r 85 13

0
1

Method 2
2 1 23
23
3 2 13 1 13

1
7 3 1

2 5 2
1 : r 3 7 3 73
2 x 3 y 7 z 73
7 6 7

2 : x 2 y 3z 6

Let x, y,0 lies on l x, y,0 lies on both planes 1 and 2 .

2 x 3 y 73
x 2 y 6
Solving, we have x 164, y 85
Thus we have,

(iv)

164
23

l: r 85 13

0
1

3
OP 4
1

3 164 161
AP OP OA 4 85 89

1 0

Shortest distance

= AP unit vector // to l

161 23


1
= 89 13
1 1 699

102
1

=
184

669

46
= 8.15 (corr to 3 s.f)

Serangoon Junior College


Mathematics
H2 P2
Section A: Pure Mathematics [40 marks]
1

[You are expected to indicate if any, all intercepts, asymptotes and turning points
on your diagrams.]
(a)

The graphs of y = f(|x|) and y2 = f(x), not drawn to scale, are shown below.
y = f(|x|)

y=

y = f(x)

9
4

(2,

2)
y=

y=

(2, 2 )

x = 1

x=1

x=1

Sketch the graph of y = f(x).

(b)

[3]

The diagram below shows the graph of y f x with asymptotes y = 1 and


x = 0. The curve has turning points at (2, 1) and (2, 1).
y

y=1

2,1
1
x=0
Sketch the graph of y f ' x

3
2

2, 1
[3]

3
2

2
2

(a)

To prepare for the annual Will run, an SRJC student plans a training
programme such that he runs 800 m on Day 1, 960m on Day 2, ,
i.e. for each successive day, the distance to be covered is multiplied by a
6
constant factor of .
5
(i)
(ii)

(b)

If the N th day is the first time that he is able to cover the required
distance of at least 10 km in a single run, find the value of N.
[3]
The student adhered to his plans initially but lost his momentum after
the first week. For the remaining days, he realised that his daily
3
distance was only of the distance covered the previous day. Show
5
that the total distance run by the student in his entire training
programme will never exceed 16 km.
[3]

A sheet of graph paper is marked in 1 mm squares. A triangular region R is


1
bounded by the x-axis, the line x = 100 mm and the line y x . Find the
5
number of complete 1 mm squares which have all their vertices either inside R
or on the boundary of R.
[3]

3
3

(a)

In an Argand diagram, the points A, B, C, D and E represent the complex


numbers

z1 2(cos 1 i sin 1 ) where


z2 cos2 i sin 2 where

2
, where z1* is the conjugate of z1,
*
z1

z1 z2 ,
and

z1 z2 respectively.

Mark and label the points A, B, C, D and E in the same Argand diagram.
Show your working clearly in your answer.

(b)

[4]

.
2
In an Argand diagram, a is represented by the point A and the variable
complex number z is represented by P.
Sketch in an Argand diagram the set of all possible positions of P satisfying
both of the following inequalities:
The fixed complex number a is such that 0 < arg a <

z a ia*

and

0 arg ( z a ) arg (ia) ,

where a* is the conjugate of complex number a.

[3]

Hence find, in terms of a,


(i)
(ii)

the range of values of z ,


the range of values of arg z.

[2]
[2]

The loci given by arg ( z a) 0 and arg ( z a) arg (ia) intersect the locus of
z a ia* at B and C respectively.

(iii)

Find, in terms of a, the complex numbers representing the points B and


C.
[2]

4
4

The diagram shows a curve C defined by the parametric equations

x 4cos2 1, y 4cos2 1 tan , where .


2
2
The curve C intersects the x axis at the origin and at the point (3, 0).
y

x 1

(i)

(ii)

dy 8 sin 2 sec2 4

.
dx
8cos sin
What can be said about the tangent to C at 0 ?

[3]

Find the values of at the origin.

[2]

Show that

The region enclosed by C is denoted by R.


(iii)

Find the exact area of R.

[5]

(iv)

Find the volume of revolution when R is rotated 180 about the x-axis.
Give your answer correct to 3 significant figures.

[2]

5
Section B: Statistics [60 marks]
5

A fair die is thrown twice. The events A and B are defined as follows:
A: an even number is obtained on the first throw.
B: the sum of the two throws is more than 4.
(i) Determine if A and B are independent. You should justify your conclusion with
relevant working.
[3]
(ii) Find P( A ' B) where A is the complement of the event A.
[3]

Mr Ahmad, Miss Banu and the families of Lim and Tan were invited to a wedding
banquet. The Lim family consists of a man and both his parents while the Tan family
consists of a woman and both her parents. The invited guests sit at a round table for
eight.
Find the number of ways of arranging the eight people if
(i) there are no restrictions;
[1]
(ii) the men and women are seated alternately;
[2]
(iii) Mr Ahmad and Miss Banu sit separately, and the members of the same family are
seated together;
[3]
(iv) the members of the same family are seated together and the seats are numbered.
[3]

The boys and girls in a school participate in an obstacle course as part of a fitness
programme. The time taken by a girl to complete the obstacle course can be assumed
to follow a normal distribution with mean 48 minutes and standard deviation 7
minutes. The time taken by a boy to complete the same obstacle course can also be
assumed to follow an independent normal distribution with mean 33 minutes and
standard deviation 5 minutes.
(i)

It was noted that 80% of the boys completed the obstacle course within k
minutes of the mean time of 33 minutes. Find the value of k, correct to three
significant figures.
[3]

(ii)

Find the probability that the total time taken by two girls is more than three
times that taken by a boy in completing the obstacle course.
[4]

(iii)

A group of 60 girls is randomly selected from those who completed the


obstacle course. By using a suitable approximation, find the probability that at
most 10% of them took more than 57 minutes each to complete the obstacle
course.
[4]

6
8

On a random day, the Pollutants Standard Index (PSI) readings are updated every
1
hour. The chance that a random PSI reading in a day will hit more than 200 is .
8
A day is considered a hazy day if the PSI readings hit more than 200 for more than
seven times in a day.
Show that the probability that a day will be considered a hazy day is

(i)

0.00683, correct to 5 decimal places.

[2]

(ii)

State an assumption needed for the distribution used in part (i) to be valid. [1]

(iii)

Find the least number of days such that the probability of having less than one
hazy day is less than 0.5.

[4]

Using a suitable approximation, find the probability that the number of hazy

(iv)

days in a year with 365 days is not less than three.

[3]

A new weather model predicted that the total number of hazy days in the

(v)

next five years will be at most 6. Assuming that the model is accurate, use the
approximate distribution in part (iv) to find the probability that the number of
hazy days in each of the five years is at least one.

[3]

It is standard procedure to measure the length of babies born in a maternity unit. The
lengths (in centimetres) of 13 babies born in a particular week were measured and
recorded. However, due to an unknown computer error, the lengths of two babies were
lost. The remaining lengths were recorded as follows:
49

50

51

47

49

48

54

53

45

50

48

Fortunately, the sample mean and variance were recorded prior to the computer error.
7
3
The sample mean and variance were recorded as 49
cm and 9
cm2. It is
13
169
assumed that the length of a randomly chosen baby is normally distributed.
(i)
(ii)

(iii)

Find the values of the lost data.


[4]
Past records show that the mean length of a baby born during the same period
of the year is 50 cm. Carry out a 2-tail test at the 5% significance level to test
if the mean length has changed.
[3]
The population variance of the length of babies in a different week is 2 . A
2-tail test conducted at the 5% significance level will reject the hypothesis that
the population mean length is o cm. Find, in terms of , the possible range
of values of o .

[3]

7
10

(a)

The marketing manager of a company that makes a particular model of


running shoes, conducted a study to determine the relation between
weekly advertising expenditure (x) and the sales of the shoe (y). She recorded
the weekly advertising expenditure and sales data, and plotted the following
scatter diagram.

A popular celebrity had endorsed the particular running shoe for free in a
weekly column of a widely read newspaper. Identify the point on the scatter
diagram that represents the week that was most likely affected by the weekly
column and explain why this point is an outlier that should not be considered
for the study.
[2]

8
10

(b)

The average running speed for various distances by ten joggers are shown in
the table below.
Distance,
x (km)
Average
speed, y
(km/hour)

1.8

3.2

10

7.2

4.8

42

21

8.35

7.98

7.88

7.46

8.21

7.59

7.79

7.62

7.14

The regression lines of y on x and x on y are given as follows:


y = 8.021 0.0311x
x = 196.154 24.128y
(i)

Find the values of k and l, leaving your answers correct to 2 decimal


places.
[3]

(ii)

Plot the data in the table with the values found in part (i) on a scatter
diagram and calculate the product-moment correlation coefficient. [2]

(iii)

A student suspects that the relation between x and y can be modelled


after one of the following formulae:
(A)

y = a + bx

(B)

y = axb

(C)

y=a+

b
x

where a, b and b < 0.


Explain, with reasons, why Formula (C) is not a suitable choice and
which of the remaining two formulae is the better model.
[3]
(iv)

A jogger wants to participate in a 217 km ultra-marathon. If he wants


to use a regression line based on part (iii) to estimate his average
speed, explain why using this estimate may not be a good one.
[1]

(a)

y = f(x)

9/4
2

(b)

y f ' x

y=0

x=0

n 1

(i)

6
Let 800 10000
5
25
ln
2
n1
6
ln
5

n 14.9
The value of N is 15.

(ii)

7 1
6 7
3
6
800 1
800
5

5
5
The total distance run =
+
6
3
1
1
5
5

= 13.xxx < 16 km (Shown)


(b)

The number of complete 1 mm squares on the 1st row =

95

The number of complete 1 mm squares on the 2nd row =

90

The number of complete 1 mm squares on the 3rd row =

85

The number of complete 1 mm squares on the 19th =

The number of complete 1 mm squares on the 3rd row =

The total number of complete 1 mm squares required


= 95 + 90 + 85 + + 5 + 0
=

19
95 5
2

= 950

3
Im
E
D
2
z1z2

B
1

z2

z1+z2
1

C
1

C:

2
) arg 2 arg z1* 0 ( 1 ) 1
*
z1

C = mid-point of OA

E: z1 z2 2 , OE = OA

arg ( z1 z2 ) 1 2 where

Re

2
2
2
* 1
*
z1
z1
z1

arg (

z1

1 2

3
4

(b)

Im(z)

|a|

A
arg(a)

(i)

(ii)

az 2a

AOB

arg a and COA


2
4

arg a arg z arg a


2
4

(iii)

B: z a a
C: z a ia

B
Re(z)

dx
8cos sin
d
dy
8cos sin tan 4cos2 1 sec2
d
8 sin 2 sec2 4
dy 8 sin 2 sec2 4 8 sin 2 sec2 4

dx
8cos sin
8 cos sin

(i)

dy
is undefined.
dx
The tangent is vertical at 0 .

When 0 ,

(ii)

When x 0 , 4cos2 1 0
1
cos 2
4
1
1

or
(no solution for )
cos
cos
2
2
2
2

(iii)

or

The part of curve C which forms the region R is defined for


Area of R

3
dx

d
y
d
3

3 4cos 2 1 tan 8cos sin d

8 3 sin 2 4sin 2 cos 2 d

3 1
2
8
cos 2 1 sin 2 d
2
3

3 1
1
8
cos 2 1 cos 4 1 d
2
2
3

4 3 cos 2 cos 4 d

1
1
3
4 sin 2 sin 4
4
2

1
2 1
4 1 2
4 sin
sin
sin
3 4
3 2 3
2

1 4
sin
4 3

2 1
4

4 sin
sin

3 2
3

1
4 sin sin
3 2
3

3 1 3
4

2
2 2

3 3
(iv)

Volume of revolution
0
2 dx
2
d
y

d
3

4cos 1 tan
2

8sin cos d

16.0 (to 3 s.f.)

(i)
Sum
1
2
3
4
5
6

1 2
2 3
3 4
4 5
5 6
6 7
7 8
1
6 5
P( A) , P( B) 1
2
36 6
1 5 5
P( A) P( B)
2 6 12

3
4
5
6
7
8
9

4 5 6
5 6 7
6 7 8
7 8 9
8 9 10
9 10 11
10 11 12

4
9
Since P( A) P( B) P( A B), A and B are not independent.
P( A B)

(ii)

P A

1
5
, PB
2
6

7
18
Hence, P A B = P A + PB P A B
1 5 7
=
2 6 18
17
=
18
P( A ' B)

(i)

Number of arrangements

= (8 1)!
= 5040

(ii)

Number of arrangements

= (4 1)! 4!
= 144

(iii)

Number of arrangements

= [(4 1)! 3! 3!] [(3 1)! 3! 3! 2!]


= 216 144
= 72

(iv)

Number of arrangements

= (4 1)! 3! 3! 8
= 1728

(i)

Let X be rv representing the time in minutes taken by a boy in completing the


obstacle course.
X ~ N (33, 52 )
Given P(| X 33| k ) 0.80
k

P | Z | 0.80
5

P Z 0.90
5

Using GC,

or

P Z 0.10
5

k
1.28155
5

k 6.40776 6.41 (3 s.f.)


(ii)

Let Y be the rv representing time in minutes taken by a girl in completing the


obstacle course
Y ~ N (48, 72 )

Y1 Y2 3 X ~ N (96 99, 2(49) 32 (25))


Y1 Y2 3 X ~ N (3, 323)
P Y1 Y2 3 X

P Y1 Y2 3 X 0
= 0.43371 = 0.434
(iii)

(3 sf)

Now P(Y 57) 0.09927

Let W be the rv representing the number of girls, out of 60 who took 57


minutes or more each to complete the obstacle course.
W ~ B(60, 0.09927)
Since n 60 is large, np 5.956 5 , nq 54.04 5 ,
then W ~ N (5.956, 5.365)

approx.

cc
P W 6
P(W < 6.5)

= 0.593
cc
Or P 0 W 6
P(-0.5 <W < 6.5)

= 0.590

(i)

Let X be the r.v. no. of PSI readings which hit more than 200 out of 24
readings

1
B 24,
8

P "hazy day" P X 7 1 P X 7
0.00683 , correct to 5 dp (Shown)

(ii)

The PSI readings in a day are independent of one another.

(iii)

Let Y be the r.v. no. of hazy days out of n days

B n,0.00683

P Y 1 0.5
P Y 0 0.5

1 0.00683
0.99317

0.5

0.5

n ln 0.99317 ln 0.5
n

ln 0.5
101.14
ln 0.99317

Least number of days = 102

(iv)

Let W be the r.v. no. of hazy days out of 365 days

B 365,0.00683

Since 365 0.00683 2.49295 5 ,

Po 2.49295 approx.

P W 3 1 P W 2
0.454 , correct to 3 sf

(v)

W1 W2 W3 W4 W5

Po 12.46475

Required probability

4
5!
P W 1 P W 2

4!
P W1 W2 W3 W4 W5 6

P W 1

0.0762

Let X denote the random variable representing the length of a randomly chosen baby
in cm.

(i)

x 49

7
13

544 a b 644

13
13

a b 100 (1)

sample variance = 9

s2

3
1524

169 169

n
(sample variance)
n 1

644
1
26970 a 2 b2
12
13

13 1524

12 169

644
1
2
2
OR: 26970 a b
13
13

1524

169

a 2 b2 5050 (2)

Solving (1) and (2), a 45 and b 55

(ii)

H o : 50
H1 : 50
(2 tailed test at 5% significance level)

X
~ t (12)
Under H o , Ttest
S
13
From GC, p value = 0.604
Since p value = 0.604 > 0.05, we do not reject H o . Here we have insufficient
evidence at 5% significance level that the population mean length of babies is not 50.

(iii)

H o : o
H1 : o
(2 tailed test at 5% significance level)

644
o

2
ztest 13
Under H o , X ~ N o ,

13

13
Critical region: ztest 1.95996

644
o
13

1.95996

13

644

o 1.95996
13
13

or

644

o 1.95996
13
13

o 49.5 0.544

or

o 49.5 0.544

10

(a)

The point required is (250, 6000).


The study is on the relationship between weekly advertising expenditure and
the sales of the shoe. It is likely that there are more people buying the shoes
because of what they have read in the weekly column. Because the
endorsement by the celebrity was not done throughout the entire period of
study and there was no advertisement expenditure on the part of the company,
the marketing manager should not use that particular weeks data for her
study.

(b)

(i)

0.0311x + y = 8.021
x + 24.128y = 196.154

x = 10.5093

From the GC,


and

y = 7.6942

103 k
= 10.5093
10

and

70.02 l
= 7.6942
10

k = 2.09 (2 dp) and l = 6.92 (2 dp)


(ii)

y
8.35

6.92

1.8

From the GC, r = 0.86573 = 0.866 (3 sf)

42

x (km)

(iii)

(C) is not suitable since b < 0 would mean that y increases as x increase which
is not the case in the scatter diagram.
For (B), we have y = axb ln y = ln a + b ln x
From the GC, the product-moment correlation coefficient between ln y and ln
x is 0.99517. Since 1 < 0.99517 < 0.866, then (B) would be the best fit to
the data and hence, the better model.

(iv)

The estimate may not be a good one as this is an extrapolation and it is not
known if the relationship between ln y and ln x is linear there.

ST ANDREWS JUNIOR COLLEGE


PRELIMINARY EXAM
MATHEMATICS
Higher 2
Paper 1

Friday

9740/1

30 Aug 2013

3 hours

READ THESE INSTRUCTIONS FIRST


Write your name, civics group and index number on all the work you hand in.
Write in dark blue or black pen on both sides of the paper.
You may use a soft pencil for any diagrams or graphs.
Answer all the questions. Total marks is 100.
Give non-exact numerical answers correct to 3 significant figures, or 1 decimal place in the case of
angles in degrees, unless a different level of accuracy is specified in the question.
You are expected to use a graphic calculator.
Unsupported answers from a graphic calculator are allowed unless a question specifically state
otherwise.
Where unsupported answers from a graphic calculator are not allowed in a question, you are required
to present the mathematic steps using mathematical notations and not calculator commands.
You are reminded of the need for clear presentation in your answers.
The number of marks is given in brackets [ ] at the end of each question or part question.
At the end of the examination, fasten all your work securely together.

This document consists of 6 printed pages including this page.

[Turn Over

The line l1 has Cartesian equation 2 x

y2
, z 1 . Another line l2 with vector equation
7

1
0


r 6 a , where , intersects the line l1 . Find the value of a and calculate
1
1


the acute angle between the lines l1 and l2 .
[4]
2

(i) Find, in terms of a , the range of values of x for which the inequality
is satisfied, where a is a positive constant
(ii) Hence, deduce the solution to the inequality

x
1
.
2
x a x 2

1
x
2
xa x 2
[3]

[2]

The graph y f ( x) , where f ( x) a cos x sin x b sin x c , passes through the points
3 5
3

and , 4 .
,
, , 2

2 4
2
2
6

(i) Find the values of a, b and c, where a, b, c .


(ii) Evaluate exactly
4

/2

/4

f '( x) dx.

[4]
[2]

The function f is defined as follows:

f :x

e x a, x , where a >1

(i) Explain why f has an inverse.

[1]

(ii) Sketch the graph of y f ( x) and y f 1 ( x) on the same axes, showing clearly all
axial intercepts, asymptotes, and the relationship between the two graphs. Hence, write
down an equation in terms of x and e x which has the same roots as the equation

f ( x) f 1 ( x) .

[4]

Another function g is defined as follows:


g:x

e x a , x b , where b is an unknown constant.

(iii) State, in terms of a, the smallest value of b such that g is a function.

[1]

(iv) Using the answer found in (iii), solve g( x) f ( x) , leaving your answers in terms of
a.

[2]

[Turn Over

The figure shows a right pyramid VABCD with a square


base ABCD, standing horizontally on a cuboid
ABCDEFGH. It is given that VA = VB = VC = VD = 5 cm,
EF = FG = 4 cm and AE = 2 cm, as shown in the diagram.
O is the centre of the square base EFGH. Perpendicular
unit vectors i, j, k are parallel to EF, FG, EA respectively.

5
D

A
2

B
k

H
O

(i) Show that the height of the figure, OV is 2 17 .


(ii) State the geometrical meaning of

VA OV
2 17

(iii) Find the equation of the line passing through B and V.

i
F

[2]
[1]
[2]

(iv) A plane has the equation 3x 2 y z . Given that the planes , VBC and
VAB have no point in common, what can be said about the values of and ? [3]

With respect to the origin O, the position vectors of the points A and B are 3i 2 j mk
and 12i 8j 4k respectively, where m 0 . The point P is on line AB such that
AP:PB=2:1.

(i) Find the coordinates of point P in terms of m.

[2]

(ii) It is given that the area of triangle OAP is 4 13 units2. Show that m =2.

[4]

(iii) Hence, or otherwise, the shortest distance from O to the line AP.

[3]

(a) Let f ( x) sin( x3 ) , where x

[3]

. Using standard series, evaluate f (15) (0) .

(b) By repeated differentiation, find the first three non-zero terms of the Maclaurins
x
series of the expression y
. Use binomial expansion to express y as a
1 x x2
series of ascending powers of x to verify your answer.
[7]

[Turn Over

Gossipboy.com is interested to find out how fast gossip spreads in Northern Country. The
variable y is defined as the proportion of the population who had heard the gossip after t
days. According to their research finding, one model for the spread of gossip is that the
rate of change of y is proportional to the product of the proportion of the population who
had heard the gossip and the proportion who had not heard it.
(i) Write down a differential equation relating y and t, where t is in terms of days.

[1]

(ii) If 10% of the population knew the gossip initially, show that the general solution for
1
, where k is a positive
the differential equation can be written as y kt
9e 1
constant.
[5]
(iii) It is given that 90% of the population will know the gossip after 30 days. Show that
2
k ln A , where A is a positive constant to be determined and hence, sketch a
15
[4]
curve to illustrate the relationship between y and t.

(a) At the start of a new year, a mother wants to save up money for her childs
education. The bank offers her a choice of two plans: a savings plan and an
investment plan. Both require a one-time, initial amount to be placed in the plan.
Savings plan
Investment plan
Fixed interest of 3% per For odd-numbered
annum
(e.g. 2013): Interest
per annum
For even-numbered
(e.g. 2014): Interest
per annum

years
of 5%
years
of 1%

For each plan, the interest is credited back into the plan at the end of the year to earn
further interest.
(Give all numerical answers to the nearest dollar for this question.)
(i) Find the value of the savings plan after 2 years, given that its value after 6 years
is $35,000.
[2]
(ii) If she placed $30,000 in the savings plan initially, show that the interest earned
[3]
in the 2nd year is $927. Find the interest earned in the 10th year.
(iii) She begins saving at the start of 2013. Explain whether it is more profitable for
her to place her money in the savings or investment plan if she saves for an even
number of years
[3]

[Turn Over

(b) The sum of the first n terms of a sequence is denoted by S n . Given that Sn en 1 ,
(i) prove that the sequence is a geometric progression,
(ii) find

S
r 1

[5]

10

In the diagram above, the curve C has parametric equation x 2a cos3 t , y 2a sin 3 t ,
where 0 t

.In the x-y plane, the origin is O and the variable point P lies in the first
2
quadrant such that OP = a and OP makes an angle with the positive x-axis.

(i) Find the equation of the tangent to C at the point where t .

[4]

(ii) Find the Cartesian equation of the locus of P. Hence, show that, as varies from 0

to , the locus of P touches C at the point where t = .


[5]
2
4
The variable point Q is such that it lies on the positive x-axis and OP = PQ = a.

, the rate of change of the area of triangle POQ when


6
is increasing at a rate of 0.5 radian/sec.
[4]

(iii) Find at the point where =

[Turn Over

11

1
3
(a) The complex number w1 is given by w1
i
2 2

(i) Find w1 and arg( w1 )

[2]

Let z1 be a complex number such that

arg( z1 )

. On an Argand diagram, the

points P , Q and R represent the complex numbers z1 , w1 z1 and w12 z1 respectively.


(ii) Sketch P , Q and R on the Argand diagram, showing clearly the relationship
between the three points, and explain why triangle PQR is equilateral.
[3]
(iii) Find the Cartesian equation of the locus z z1 z w1 z1 if it is given that
arg z1

[3]

(b) (i) Given that z p is a solution of the equation


az 3 bz 2 cz d 0 ,

where a and c are real constants while b and d are purely imaginary constants,
show algebraically that z p* is another solution.
[You may use the result z w z* w* .]
*

[3]

(ii) Hence, without using a calculator, solve the equation


z 3 11iz 2 64 z 170i 0 ,

given that z 5 3i is one of the solutions.

The End

[3]

Paper 1 Solutions
1

1
0 2

l1 : r 2 s 7 , s
1 0


Since the two lines intersect,
0
1 1
0



2 14 6 a for some ,
1
0 1
1



1
2 14 6 a
1 1
Solving, we get 2, 1, a 3 .
Thus a 3 .

Acute angle between the two lines


1 0


14 3
0 1

1
cos
197 10
42
cos 1
18.90
1970

x 2 2 x 2 ax
1
x
1
x
0
2
2
0

xa x 2
xa x 2
x a x2 2

ax 2
0
x a x2 2
1

Since x 2 2 0 for all x

ax 2
0
xa

Multiplying both sides by x a , we have


2

ax 2 x a 0

x a or x
Alternative method

2
a

1
x
2
xa x 2

2
2
2
2
Multiply both sides by this positive term ( x 2)( x a) , we get ( x a)( x 2) x( x a) .
2
2
This leads us to ( x a)( x 2) x( x a) 0

( x a)( x2 2 x( x a)) 0
( x a)(ax 2) 0
2
x a or x
a
2ii

Let y x .
Then

x
1
1
y
2
2
becomes
.
x a x 2
ya y 2
2

From (i), y a or y

2
.
a

2
(no solution)
a
Since a 0 and x 0 for all x , we must have x a , i.e. x a or x a .
i.e. x a or x

3i

Form three equations:

3 5
6 , 2 ;

At , 2
;
2
4
At

3
1
3 5
:
a bc
4
2
2

1
1
3
a
bc 2
2
2
2


At , 4 ; b c 4
2
Solving using GC, a=2, b=3, c=1
3ii

/2

/4

f '( x) dx

f ( / 2) f ( / 4)

4i

= 42

3
2
2

ii

From graph, all lines y k where k Rf cut the graph y f ( x) exactly once. Thus, by the horizontal line test, f is 1-1 and the inverse
exists.

From graph, the graphs of y f ( x) and y f 1 ( x) intersect along the line y x . Thus, solving f ( x) f 1 ( x) is equivalent to solving
f ( x) x , i.e.
Iii

x ex a
Smallest value of b= ln a

iv

The graphs of y f ( x) and y f ( x) intersect when y=0 and 1. Thus, when solving for g( x) f ( x) , we get
e x a 0 or e x a 1
e x a or e x 1 a
x ln a or x ln 1 a

Alternatively

ex a ex a
ex a ex a

e x a e2 x 2ae x a 2
e2 x 2a 1 e x a 2 a 0
e
x

2a 1

2a 1 4(1)(a 2 a)
2 1
2

2a 1 1
2
a 1 or a

Thus x ln a 1 or ln a
5(i)

1 2
1
4 42
32 8
2
2
Drop a perpendicular line from V to plane ABCD, and call the foot of perpendicular M.
Consider the triangle AVM.
OE

VM 52 8 17
Thus, height of figure OM is 2+ 17

Alternative
If we drop a perpendicular line down from V to BC, call the foot of the perpendicular X.
VX 52 22 21 .

The height of the cone portion = 21 22 17 .


Thus the height of the figure is 2+ 17 .
5(ii)

VA OV
2 17

(ii)

iv

VA OV
OV

. Thus it represents the length OE.

Alternative answer:
It represents the shortest distance of A to OV.
0
2


OV 0 and OB 2
2
2 17

VB OB OV 2
17

Thus equation of line VB:


0
2

r 0 2 ,
2 17
17

Note the planes VBC and VAB intersect along the line VB.
6

Since the planes , VBC and VAB have no point in common, then line VB does not intersect .
Thus, VB will be parallel to .
2 3


2 2 0
17


Thus,
6 4 17 0
10

17

6i

Since VB does not intersect , (0,0, 2 17) does not lie on .


Thus,

0 3

2 17 10

17
20

10
17
By Ratio Theorem

3
12 9


OP 2 2 8 6
3

m
4 8 m
3
7

The coordinates of point P are (9, 6,


6ii

8 m
)
3

1
OA OP = 4 13
2

3 9

4 13

m 8 m

3
3 27

2 18 4 13
m 8 m

Area of OAP

1
2

1
6

1
6

16 16m

24 24m 4 13

8
6

2 2m

3 3m 4 13
0

2 2m 3 3m
2

1 m

3 13

32 3 13

1 m 13 3 13
1 m 3
8

1 m 3 or m 1 3
m 2
6iii

or

m 4 (rej since m 0)

9 3 6

AP 6 2 4
2 2 4

AP 68

1
h 68 4 13
2
208
h
3.50
17

Alternative(1):

3

2

9

2

| 6 |
17
2

12
0
208

3.50
17
17

Alternative(2):

3
3


Find: ON 2 2 for a fixed
2
2


3
3 3


2 2 . 2 =0
2 2
2

9
17
9

ON
7a

208
3.50
17

Using standard series


x3 x5
sin( x) x ...
3! 5!

( x 3 )3 ( x 3 )5
```(*)

3!
5!
f (15) (0) 1
The coefficient of x15 =
= from (*)
15!
5!
15!
Therefore, f (15) (0)
6 7 8 9 10 11 12 13 14 15 =10897286400.
5!
x
y
1 x x2
(1 x x2 ) y x
Differentiating wrt x:
dy
(1 2 x) y (1 x x 2 ) 1 --- (1)
dx
Now,
f(0) = 0,
from (1): f '(0) = 1 by substituting x = 0
Differentiating (1) wrt x again:
dy
d2 y
2 y 2(1 2 x) (1 x x 2 ) 2 ) 0 ---(2)
dx
dx
(2)
Substituting x = 0, f (0) = 2
Hence, f ( x) sin( x3 ) = x3

Differentiating (2) wrt x again:


10

dy
d2 y
d3 y
3(1 2 x) 2 (1 x x 2 ) 3 0 --- (3)
dx
dx
dx

Substituting x = 0, f (3) (0) = 12

f ( x) f (0) xf (1) (0) x 2


= x x2 2 x3 +

f (2) (0) 3 f (3) (0)


x
...
2!
3!

Using binomial expansion


2
x
x 1 x x 2
2
1 x x

1 2 x x 2 2 ...
x 1 1 x x 2


2!

x 1 x x 2 x 2 2 x3 x 4 ...

x 1 x 2 x 2 ... x x 2 2 x3 ...

8 (i)

dy
k ( y )(1 y )
dt

(ii)

dy
k ( y )(1 y )
dt
1
y(1 y) dy k dt

11

y 1 y

dy kt C

ln y ln(1 y ) kt C ,

or

1
2

1 2
(y )
2
2

dy kt C

y 0, (1 y ) 0

y
ln
kt C
1 y
y
ekt C
1 y
y (1 y ) Aekt ,

A eC

When t 0, y

1
9
Ae0
10 10
1
A
9
1
y (1 y )e kt
9
9 y (1 y )e kt

1
10

y (9 e kt ) e kt
y

e kt
9 e kt

e kt y

(iii)

1
9e

1
9
When t 30, y
10
kt

(Shown)

12

9
1
k (30)
10 9e
1
30 k
81e
9 10
1
e 30 k
81
1
30k ln
81
1
k ln(34 )
30
2
k ln 3
15

0.1

13

9ai

9aii

9aiii

9b (i)

Value after 6 years


a(1.03)6 35000
35000
a
(1.03)6
Value after 2 years
a(1.03) 2
35000

(1.03) 2 $31097
6
(1.03)
Or
V6 V2 (1.03) 4
35000 V2 (1.03) 4
V2 $31097
Interest in 2nd year 0.03 (30000 1.03) $927
Interest in 10th year
V10 V9
30000 (1.03)10 30000 (1.03)9
$1174
Or
Interest in 10th year
0.03 V10
0.03 30000 (1.03)9
$1174
After 2n years, the savings plan will have $a(1.03)2 n whereas the investment plan will have
$a(1.05)(1.01) (1.05)(1.01) $a(1.05) n (1.01) n
Savings plan: a(1.03)2n a[(1.03)2 ]n a(1.0609)n
Investment plan: a(1.05)n (1.01)n a[(1.05)(1.01)]n a(1.0605) n
The savings plan is more profitable.
Sn en 1

14

Tn S n Sn 1

en 1 en 1 1

en en 1
en 1 e 1
Tn
en en1

e = constant
Tn1 en 1 en 2 en 2 e 1
Thus, it is a GP.
9b (ii)

S
r 1

S1 S 2 ...S n
e1 1 e 2 1 ... e n 1
e1 e 2 .. e n n

10(i)

e e n 1

n
e 1
C: x 2a cos3 t , y 2a sin 3 t

dy dy dt
2a(3sin 2 t cos t )

tan t.
dx dt dx 2a(3cos 2 t )( sin t )
Equation of tangent to C where t .
y 2a sin 3
tan
x 2a cos3
y tan ( x 2a cos3 ) 2a sin 3
y ( tan ) x 2a cos 2 sin 2a sin 3

10(ii)

y ( tan ) x 2a cos3 tan 2a sin 3 ( tan ) x 2a sin (cos 2 sin 2 )


( tan ) x 2a sin
For the point P,
x a cos , y a sin
15

x2 y 2 a2 (cos2 sin 2 ) a2
x2 y 2 a 2 is the equation of the locus of P.

To find pt of intersection of locus of P with C,


x 2 y 2 a 2 --- (1)
Substitute x 2a cos3 t , y 2a sin 3 t into (1), we have
4a 2 cos 6 t 4a 2 sin 6 t a 2

cos 6 t sin 6 t

1
4

(1 sin 2 t )3 sin 6 t

1
4

1 3sin 2 t 3sin 4 t sin 6 t sin 6 t


1
0
4
4(sin 2 t ) 2 4sin 2 t 1 0

1
4

sin 4 t sin 2 t

(2sin 2 t 1) 2 0
1
2
1
sin t
2
sin 2 t

since P lies in the first quadrant.


10(iii)

(Shown)
4
Let A be the area of triangle OPQ.
t

16

1
(2a cos )(a sin ) 1 2
A= 2
= a sin 2
2

dA 1 2
a (2cos 2 ) a 2 cos 2
d 2
When

11(a)(i)

a(ii)

,
6
dA dA d
1 a2
units2/sec.

a 2 cos 2( ) =
dt d dt
6 2 4
2

2
1
3
1 3
w1
i
1
2 2
2 2

2
1
2

tan 1 3 . Since w1 lies on the second quadrant, arg( w1 )


tan
.
1
3
3
3

2

Observe that since OPQ , OQR and ORP are isosceles triangles, so
17

OPQ OQP OQR


ORQ ORP OPR

2
3 .
2
6

Hence, PQR QRP RPQ

, and therefore triangle PQR is an equilateral triangle.

Alternative Explanation:
Triangles OQP, ORQ, OPR are identical since OQ=OP=OR= z1 and angles QOP, QOR and ROP are equal (SAS).
Thus, QP=PR=RQ
a(iii)

The locus is bisector of line segment PQ, and passes through the origin.
Since arg z1

, the bisector makes an angle of


4
1 2 7

with the positive x-axis.
4 2 3 12

18

Gradient of the line is tan(

5
).
12

Thus equation of the line is y x tan(


or y x tan(
11bi

7
) y 3.73x
12

5
)
12

Since z p is a solution of the equation az 3 bz 2 cz d 0 ,

ap3 bp 2 cp d 0 .
Taking complex conjugates on both sides and using the properties of complex conjugates,

(ap 3 bp 2 cp d )* 0*
(ap 3 )* (bp 2 )* (cp)* d * 0*
a* ( p* )3 b* ( p* )2 c* p* d * 0*.

19

Since 0, a and c are real, 0* 0, a* a, c* c .


Since b and d are purely imaginary, b* b, d * d .
Hence,

a( p* )3 b( p* )2 cp* d 0
(a( p* )3 b( p* )2 cp* d ) 0
a( p* )3 b( p* ) 2 c( p* ) d 0.
Hence, z p* is another solution of the equation.
11bii

For z 3 11iz 2 64 z 170i 0 ,


Since z 5 3i is one of the solutions, by using (i), we conclude that z (5 3i) 5 3i is another solution.
Let q be the final solution of the equation. We then have

z 3 11iz 2 64 z 170i
( z (5 3i))( z (5 3i))( z q)
( z 2 (5 3i 5 3i) z (25 15i 15i 9))( z q)
( z 2 6iz 34)( z q).
By comparing the constant term, 34q 170i , and thus q 5i .
Therefore, the roots are z 5 3i, 5 3i, 5i .

20

ST ANDREWS JUNIOR COLLEGE


PRELIMINARY EXAM
MATHEMATICS
Higher 2
Paper 2

Tuesday

9740/2

17 Sep 2013

3 hours

READ THESE INSTRUCTIONS FIRST


Write your name, civics group and index number on all the work you hand in.
Write in dark blue or black pen on both sides of the paper.
You may use a soft pencil for any diagrams or graphs.
Answer all the questions. Total marks is 100.
Give non-exact numerical answers correct to 3 significant figures, or 1 decimal place in the case of
angles in degrees, unless a different level of accuracy is specified in the question.
You are expected to use a graphic calculator.
Unsupported answers from a graphic calculator are allowed unless a question specifically state
otherwise.
Where unsupported answers from a graphic calculator are not allowed in a question, you are required
to present the mathematic steps using mathematical notations and not calculator commands.
You are reminded of the need for clear presentation in your answers.
The number of marks is given in brackets [ ] at the end of each question or part question.
At the end of the examination, fasten all your work securely together.

This document consists of 6 printed pages including this page.

[Turn Over

2
Pure Mathematics (40 marks)
1

(i) On an Argand diagram, sketch the locus of the point representing z such that both
[4]
z 2 i 2 and z 2 i iz 3 are satisfied.
(ii) Find the range of arg( z 1 i) .

[3]

x2 x 2
, showing clearly any axial intercepts, coordinates
x2
of turning points and asymptote(s).
[4]

(i) Sketch the graph of y

(ii) Find the range of values of m, where m > 0, such that the equation
x2 x 2
mx 1
x2
has exactly 2 roots.

[2]

The functions f and g are defined as follows:


x2 x 2
f :x
, x , x 2,
x2
g : x ex , x .

(iii) Find the range of f.

[1]

(iv) Find the exact range of the composite function gf.

[2]

(a) (i) Using the method of differences, find

r 1
n

r 1

r 3 in terms of n.

[2]

(ii) Express (r 1)3 r 3 in the form of ar 2 br c where a, b and c are constants


to be determined.
[1]
(iii) Hence show that

r
r 1

1
nn 12n 1 .
6

[4]

(b) Prove by mathematical induction that


n

r r ! (n 1)! 1 for all positive integers n.

[4]

r 1

[Turn Over

(a) Find, in terms of n and e,

2e
1

x n1 ln x dx .

[4]

1
(b) The diagram shows the curve C with equation y sec x where 0 x .
6
y

2
6,

y sec x

1
(i) The region R is bounded by the axes, the curve C and the line x .
6
Find the exact area of the region R .
(ii) Hence, or otherwise, find the exact value of

2
3
1

cos 1

1
dy .
y

[3]
[2]

2
.
3
Find the exact volume of the solid of revolution formed when S is rotated

(iii) The region S is enclosed by the curve C, the y-axis and the line y

through 2 radians about the x-axis.

[4]

Statistics (60 marks)

A cinema wishes to conduct a survey of the cinema-goers on a particular day to


investigate how gender and age affect the types of food preferred.
(i) Describe how you would carry out quota sampling to obtain a sample of 50 cinemagoers.
[3]
(ii) State one disadvantage of quota sampling, in the context of the question.

[1]

[Turn Over

4
6

During the college carnival, a class decides on a game which requires a player to pick
some coloured cards which are indistinguishable except for their colour. In total, there
are 10 cards: 3 blue, 5 red and 2 yellow cards.
(i) Find the number of ways for a player to arrange 9 cards in a row.

[3]

(ii) The class decides that the game is played by picking 4 cards from the 10 cards
without replacement and the total points are added from the 4 cards with each
coloured card assigned a different score as follows:
Blue: no point

Red: 1 point

Yellow: 3 points

Find the probability that a player obtains exactly 4 points.

[3]

A teacher conducted a survey on a large number of students to determine the choice of


colours for painting the school hall from 3 colour options of white, green and blue. Of
the students surveyed, 40% were boys and 60% were girls. Of the boys, 50% chose
white, 20% chose green and the rest chose blue. Of the girls, 25% chose white, 45%
chose green and the rest chose blue.
Draw a probability tree diagram to illustrate the above information.

[1]

(i) One student is randomly selected. Find the probability that the student chose white.
[1]
(ii) Two students are randomly selected. Find the probability that the two students are of
the same gender or chose different colours (or both).
[3]
(iii) Three girls are randomly selected. Find the probability that exactly 1 girl chose
white, given that none of them chose blue.
[3]

[Turn Over

5
8

The radiation intensity I at time t, from a radioactive source, is given by the formula

I I 0e kt , where I 0 and k are constants.


The following data were obtained from a particular source. The values of t may be
considered to be exact, while the values of I are subject to experimental error.
t

0.2

0.4

0.6

0.8

1.0

3.22

1.63

0.89

0.41

0.36

(i) Find the product-moment correlation coefficients between


(a) I and t, and
[2]

(b) ln I and t.

(ii) Would the regression line of ln I on t or I on t be better in establishing a model for


the data? Give a reason for your choice.
[1]
(iii) Find the equation of the regression line of your choice in part (ii). Hence plot on the
same graph, the scatter diagram and regression line.
[3]
(iv) By using your choice of the regression line in part (ii), give estimates for I 0 and k.
Explain why it is reasonable to use this regression line to estimate the value of t
when I = 1.5. Obtain this value.

[3]

In a 24-hour operating MacDuck shop, the number of Hello Daniel toys sold per hour, S,
follows a Poisson distribution with mean .
(i) If the probability that a shop sells exactly one Hello Daniel toy in a randomly chosen
minute is 0.1, show that the value of corrected to 3 significant figures is 6.71. [2]
(ii) Find the least number of Hello Daniel toys that the shop needs to stock up on a given
day so that the probability that it is sold out is less than 80%.
[3]
(iii) Find the probability that, in an hour, there are at most 10 Hello Daniel toys sold
given that there are more than 6 sold.
[3]
(iv) The number of Hello Daniel toys sold was recorded for n 1-hour periods, where n is
large. Find n such that the probability of the mean number of Hello Daniel toys sold
per hour is more than 7 is approximately 5%.
[2]

[Turn Over

10

A manufacturer claims that his new pill helps people lose weight. A random sample of
20 people took the pill for a month and the loss in weight (initial weight final weight)
after a month, x kg, were summarized as follows.

x 20.49

88.4253

(i) Find unbiased estimates of the population mean and variance.

[2]

(ii) Test whether the manufacturers claim is justified at the 5% level of significance.
State an assumption necessary for the test to be carried out.
[5]
(iii) Suppose that the population standard deviation is known to be 1.6 kg and the
assumption made in part (ii) is still valid. The manufacturer claims that the average
weight loss for people taking the pill is at least 0 . A test, using the same set of data
above at the 10% level of significance, indicates that the manufacturers claim is
valid. Find the largest value of 0 .
[4]
11

At ABC supermarket, the mass of a randomly chosen apple is normally distributed with a
mean of g and a standard deviation of 30.5g.
(i) An apple is labelled as large if its mass is at least 300g. What is the greatest integer
value of given that the probability of picking up a large apple is at most 0.7?
[3]
Subsequently, it is found that 320 g.
(ii) Apples are packed into cartons of 45. A carton is considered to have passed the
quality test if it contains more than 35 large apples. Using a suitable approximation,
calculate the probability that in a randomly chosen batch of 80 cartons of apples,
more than 30 of them passed the quality test.
[4]
(iii) The mass of a randomly chosen durian is normally distributed with a mean of 1600g
and a standard deviation of 300g. Find the probability that the average mass of 3
randomly chosen durians exceeds twice the mass of two randomly chosen apples by
less than 50g.
State an assumption needed for your calculation.

The End

[5]

Paper 2 Marking Scheme


1i

z 2 i iz 3
z 2i i z

3
i

z 2 i z 3i .

ii

Let E (1,1) . Observe that AC CF 2 and CE 3 . Hence,

AC
1 2
AEC tan 1
tan
CE
3
0.588 (3 s.f.)
CF
1 2
CEF sin 1
sin
CE
3
0.730 (3 s.f.)

2(i)

Hence, the range of arg( z 1 i) is 0.730 arg( z 1 i) 0.588


By long division,
x2 x 2
4
y
x 1
x2
x2
To find the stationary points, consider

dy
0
dx

dy
4
1
2
dx
x 2
1

x 2

x 0 or 4
The stationary points are (0, -1) and (4,7)

(ii)

Sketch the graph of y mx 1 on the same axes. The number of intersections of this graph and y
the given equation.
The graph of y mx 1 is a straight line passing through (0,1) .
Thus for 2 roots, m 1
Alternative solution

x2 x 2
is equal to the number of roots of
x2

x2 x 2
mx 1
x2
x 2 x 2 mx 1 x 2

m 1 x 2 (2 2m) x 4 0
For the equation to have two roots, discriminant >0
Thus,

2 2m

4 m 1 4 0

4 8m 4m2 16m 16 0
4m2 8m 12 0
(iii)

m 3(rejected since m 0) or m 1
R f (, 1] [7, )

(iv)

Graph of y g( x)

g
R f (, 1] [7, )
Rgf (, e7 ] [e1 ,0)

Rgf (, e7 ] [e1 ,0)


3ai

r 1
n

r 1

r3

23

13

..

..

..

..
(n 1)3

n3

(n 1)3

n3

= (n 1)3 1 (or n3 3n2 3n )


aii
aiii

(r 1)3 r 3 r 3 3r 2 3r 1 r 3
= 3r 2 3r 1
From (i),

r 1
n

r 1
n

3r

r 3 (n 1)3 1

3r 1 (n 1)3 1 using result of (ii)

r 1
n

r 1

r 1

3 r 2 3 r 1 (n 1)3 1
r 1
n

3n(1 n)
n n3 3n 2 3n
2
r 1
n
3n(1 n)
3 r 2
n n3 3n 2 3n
2
r 1
n
3n(1 n)
3 r 2
n3 3n 2 2n
2
r 1
n
3n(1 n)
3 r 2
n(n 1)(n 2)
2
r 1
n
n(n 1)
3 2(n 2)
3 r 2
2
r 1
3 r 2

n(n 1)
2n 1
2
r 1
n
1
Hence r 2 nn 12n 1
6
r 1
n

3 r 2

r r ! (n 1)! 1 , where n

Let Pn be the proposition:

r 1

When n = 1, LHS= 11! 1 , RHS = (1 1)! 1 2! 1 1


Therefore P1 is true.
Assume that Pk is true for some positive integers k,
k

r r ! (k 1)! 1

i.e.

r 1

k 1

We want to prove that

r r ! (k 2)! 1
r 1

LHS
k

r r ! (k 1)(k 1)!

r 1

= (k 1)! 1 (k 1)(k 1)!


= (k 1)!(k 2) 1
= (k 2)! 1
= RHS
4a

Since P1 is true and that Pk is true implies Pk+1 is also true, by mathematical induction, Pn is true for all positive integer values of n.
Using integration by parts,

2e

x n 1 ln x dx

Let u ln x

2e

x n ln x

2e

xn

1
dx
x

du 1

dx x

dv
x n 1
dx
xn
v
n

2e

ln 2e
n

2e ln 2 1
n

2e ln 2 1
n

b(i)

2e

x n 1 dx

2e

1 xn

n n 1

n
1 2e
1


n n
n

1
n
n
n 2e ln 2 1 2e 1
2

Area R 6 sec x dx
0

ln sec x tan x 06
ln

2
1

ln1
3
3

ln 3 or
b(ii)

b(iii)

2
3
1

cos 1

When x

1
ln 3
2

1
2 1
dy
ln 3
y
36 2

ln 3
3 3 2

2
3

2
2
6
Volume of S
6 0 sec x dx
3

2 2

tan x 06
9
2

5(i)

2 2
2 1

or

9
3
3
9

Decide on the strata and number to be selected for each stratum. For example, we could have
Age (below 21)

Age (21 and above)

Female

10

10

Male

20

10

Select the required numbers in a non-random way, for example, pick the first 10 females aged below 21 who leave the cinema.
5(ii)
6i

One disadvantage would be that it produces a biased sample as only those who leave earlier will be surveyed.

9!
1260
4!3!2!
9!
756
Case 2 : without 1 of the blue card =
2!5!2!
9!
504
Case 3 : without 1 of the yellow card =
5!3!
Case1: without 1 of the red card =

Total = 2520
ii

P(total score 4)

OR P(total score 4)

= P(1Y1R2B)+P(4R)

= P(1Y1R2B)+P(4R)

3 2 5 2 4! 5 4 3 2

10 9 8 7 2! 10 9 8 7
1
(or 0.167)
6

C1 5C1 3C2 5C4


10
10
C4
C4

1
6

0.5

White

0.2

Green

0.4

0.3

Blue

0.6

0.25

White

0.45

Green

0.30

Blue

Girls

P(student chose white) = 0.4 0.5 0.6 0.25


= 0.35 (or

ii

Probability tree diagram:

Boys

7
)
20

Required probability
= 1 P(Students are of different gender and chose same colour)

(0.4 0.5 0.6 0.25) (0.4 0.2 0.6 0.45)

(0.4 0.3 0.6 0.3)

1067
= 0.8536 (or
)
1250
= 1 2

Direct Method (Alternative)

P( A B ) P A P B P A B
P(a student chooses white)= 0.35
P(a student chooses green)= 0.4 0.2 0.6 0.45 0.35
P(a student chooses blue)= 1-0.35-0.35=0.3
P(both students same gender)= 0.42 0.62 0.52
P(both students choose different colour)
= P(White, Green) + P(White, Blue) + P(Green, Blue)

2 0.35 0.35 0.35 0.3 0.35 0.3 =0.665

P(both students same gender and choose different colour)


=P(both boys,white, green) + P(both boys, white, blue) + P(both boys, green, blue)
+ P(both girls, white, green) + P(both girls, white, blue) + P(both girls, green, blue)

0.42 2 0.5 0.2 0.5 0.3 0.2 0.3

0.62 2 0.25 0.45 0.25 0.3 0.45 0.3


0.3314
iii

P(exactly 1 Girl chose White | none of the Girls chose Blue)

P({exactly 1 Girl chose White} {none of the Girls chose Blue})


P(none of the Girls chose Blue)

P(1 out of the 3 Girls chose White and the other 2 chose Green)
P(none of the Girls chose Blue)

C1 (0.25)(0.45)(0.45)
(0.7)(0.7)(0.7)

0.151875
0.343

= 0.443 (or

1215
)
2744

8 (i)

Using the GC,


Product-moment correlation coefficient between I and t, r = - 0.924.
Product-moment correlation coefficient between ln I and t, r = - 0.984.

(ii)

The regression line of ln I on t is better.


Reason: r = - 0.984 is closer to - 1.
Alternative reason: Since
I I 0e kt ln I ln I 0 kt , hence, ln I on t follows a linear relation.

(iii)
ln I

(0.2, 1.17)

t
ln I = - 2.88t + 1.65
12.5

(iv)

(1, - 1.02)
)
Using the regression line of ln I on t,
k = 2.88 (to 3 s.f.) and ln I 0 1.6543 I 0 5.23 (to 3 s.f.)
Since I = 1.5 is within the given range for I, it is reasonable to use the regression line of ln I on t to estimate the value of t.
Alternative Reason: Since t is the independent variable (values are exact) and ln I is the dependent variable (values are subject to error), it is
reasonable to use the regression line of ln I on t to estimate the value of t when I = 1.5.

ln(1.5) 2.8811t 1.6543


t
9i

1.6543 ln(1.5)
0.433 (to 3 s.f.)
2.8811

Let X be the random variable no of Hello Daniel toys sold in a minute.

Po (

v
)
60

P( X 1) 0.1

v 60v
e 0.1
60

From GC v 6.70995 6.71 (3 s.f)


ii

(Shown)

Let N be the number of Hello Daniel toys that the shop has to stock up and Y be the number of Hello Daniel toys sold in a day.

Po (6.70995 24)

Po (161.04)

P(having a shortage)<0.80
P(Y N ) 0.80

P(Y N ) 0.20
From GC

P(Y 149) 0.18213


P(Y 150) 0.20423

iii

Least value of N is 150


Po (6.70995)

P( S 10 S 6)

iv

P(6 S 10)
1 P( S 6)

0.42703
0.844
0.50625

Since n is large, by Central Limit Theorem,

N (6.70995,

6.70995
) approximately
n

P( S 7) 0.05
P( S 7) 0.95
7 6.70995
) 0.95
6.70995
n
0.29005
1.64485
6.70995
n

P( Z

n 1.64485

6.70995
0.29005

n 14.6897
n 215.79
n 216
10(i)

Let X be the r.v. loss in weight for a randomly chosen person who took the pill for a month.

20.49
1.0245
20
1
20.492
3.5491 3.55
s 2 88.4253
19
20
x

10(i)

Assume X is normally distributed.

Ho : 0
H1 : 0
Under H o , T

X
s/ n

t 19

Using a 1-tailed t-test at 5% level of significance,


p-value = 0.0125 < 5%

Reject H0. There is sufficient evidence at the 5% level of significance that the pill helps people to lose weight.
(iii)

H 0 : = o

vs

H1 : o

Assuming that H 0 is true, X ~ N 0 ,

test statistic Z =

X o
1.6 / 20

1.62

20

~ N(0,1)

(If the manufacturers claim is valid, it means that we do not reject H 0 )


Hence, using the one-tailed z-test at the 10% level of significance, we will not reject H 0 if z 1.2816

1.0245 o

1.2816
1.6 / 20
1.0245 o 0.45850

o 1.483
Largest value of 0 is 1.48.
11
(i)

Let X be the r.v. the mass of an apple in g.


X ~ N ( , 30.52 )

P( X 300) 0.7
P( X 300) 0.3
X 300
P(

) 0.3
30.5
30.5
300
) 0.3
P( Z
30.5
300
0.52440
30.5
315.994
Greatest value of is 315g

(ii)

P( X 300) 0.7440036
Let Y be the number of apples out of 45 which are large.

Y ~ B 45,?.7440036

P Y 35 ? P Y 35 ? .250053

Let C be the number of cartons out of 80 which pass the quality test.

C ~ B 80 , 0.250053

Since n is large, np 20.004 5 , nq 59.996 5

C~ N

20.004 , 15.002 approx.

c .c
P C 30

(iii)

30.5 ? .0337 (3s.f )

Let M be the r.v. the mass of a durian.

M1 M 2 M 3
2( X1 X 2 ) N (320,37442)
3
M M2 M3
3(1600)
E 1
2( X 1 X 2 )
2(2 320)
3
3

2
M M2 M3
3(300 ) 2
Var 1
2( X 1 X 2 )
2 (2 30.52 )
2
3
3

Alternative Method

3002
N (1600,
)
3

E M 2( X1 X 2 ) 1600 2(2 320)

Var M 2( X 1 X 2 )

(252 ) 2
2 (2 30.52 )
3

M 2( X1 X 2 ) N (320,37442)
M M2 M3

P0 1
2( X 1 X 2 ) 50
3

0.0324 (3 s.f)
Assumption: The mass of the apples and durians are independent of each other.

River Valley High School


Mathematics
H2 P1
1

In the National Hockey League Competition, 2 points are awarded to the winning team in
a match while the losing team takes no point. However, if the game ends in a draw, then
each team will receive 1 point. Final position in the competition is determined by the total
points obtained by each team upon completion of all their matches for the season.
At the end of the regular season in 2013, the top 3 teams
3 y of
1 the completion emerged
z with
1
the following results:
Position
Team
No. of Wins No. of Draws No. of Loses Points
2 y 1
st
z 1
1
Boon Lay Rovers
0
x
y
2nd
Malan United
a
1.5 x
z
rd
3
West Coast Rangers
b
x 1
Find the value of x, y, z and determine the team with the most number of loses among the
top 3 teams.
[4]

The series expansion of 1 ax up to and including the term in x 2 is given by


b

3
3
1 x x 2 . Find the values of a and b. Explain why the substitution x 2 may not be
2
8
7
4

suitable in estimating the value of 5 using the above series.

n2 n 4
1
A sequence u1, u2, u3, ... is such that u1 = and un1 un
, for n 1.
(n 3)2 (n 2) 2
9
(i)

(ii)

Use the method of mathematical induction to prove that un

Hence find

n 1

[5]

(i)

Given that y cos ln x 2 , find

dy
.
dx

Find the exact value of the integral

2013 RVHS H2 Maths Prelim Paper 1

n 2

n2 n 4
in terms of N and state its value as N .
(n 3)2 (n 2) 2

[4]

[3]

[2]

(ii)

e2

ln x
sin ln x 2 dx .
x

[5]

The point

P x, y moves along a curve of which equation is given by

2 x y 2 3 x y 2 16 . Find

(i)

the gradient of the curve at P,

[2]

(ii)

the relationship between x and y if the normal at P is parallel to the x-axis,

[2]

(iii)

the equation of the normal at P where the curve cuts the negative y-axis.

[3]

(a)

The graphs of y f ( x) and y f x are shown below:


y

y
y=4

x
y = 2

(4, 3)

x=2

x = 2

x=2

yf x

y f ( x)

On separate diagrams, sketch the graphs of


(i)

y f ( x) ,

[2]

(ii)

y f ' ( x) ,

[2]

showing clearly in each case the axial intercepts, the asymptotes and the
coordinates of the turning points, where possible.
(b)

The curve C1 has equation x 2 y 2 6 x 8 y 16 0 . Express the equation in the


form ( x a) 2 ( y b) 2 c 2 , where a, b and c are constants to be found.
[1]

y2
1 . State a sequence of transformations
Another curve C 2 has equation x
9
that transforms C 2 to C1 .
[3]
2

2013 RVHS H2 Maths Prelim Paper 1

The complex numbers w and z satisfy the equations

w 3
i
z 2
z * 2w 4 4i.
Find w and z.

[4]

The complex numbers w, z and v are represented by the points W, Z and V on an Argand
diagram. Given that arg (v) < 0 and WVZ forms an equilateral triangle, show that v can be
expressed as

1 a w az,

where a is to be found in the form rei .

[3]

Find the exact value of arg (w v).

[2]

At the end of December 2012, Johns company managed to secure a long term city
development contract with the local government. He thus decided to approach Companies
A and B for supply of construction raw materials.
(a)

(b)

After the first supply of 200 units of raw materials at the end of January 2013,
Company A supplied 20 units more than it supplied at the end of the previous
month. For example, Company A supplied 220 units at the end of February 2013
and 240 units at the end of March 2013. However, John decided that he would end
the partnership with Company A at end of February 2017. Determine
(i)

the total amount of raw materials that would be supplied by Company A


by the end of February 2017,
[2]

(ii)

the time when Company A would have first supplied more than half of the
total amount supplied for the whole duration.
[2]

Company B also supplied Johns company with raw materials at the end of each
month from January 2013. The amount supplied for each month is in a geometric
progression. In particular, Company B would supply 387.24 units in December
2013 and 109.37 units in December 2014. Find, correct to the nearest unit,
(i)

the amount of raw materials supplied by Company B at the end of January


2013,
[3]

(ii)

the total amount of raw materials Company B could supply in the long run.
[2]

2013 RVHS H2 Maths Prelim Paper 1

Referred to an origin, O, the position vectors of two points A and B are a and b
respectively. a and b are not parallel.
(i)

The point C lies on AB produced such that AB:BC is 1:3. Find the position vector
of C.
[2]

(ii)

The point D lies on OB produced such that OB:OD is 1:k. Given also that CD is
2

perpendicular to AB, show that k

10

4 b 7a b 3 a
2

b ab

[3]

(iii)

Show that OADC cannot be a parallelogram.

[2]

(iv)

Find the area of triangle ABD in terms of k.

[3]

(i)

Describe completely, in geometrical terms, the locus of z given by


4
[3]
1 i and sketch it on an Argand diagram.

z 2 2i

The locus of w is given by arg( w k ki)


(ii)

where k is a real constant.


4

Find the set of values of k for which the locus of w intersects the locus of z at two
distinct points.
[3]

For the case where k = 4,


(iii)

sketch the locus of w on the same Argand diagram.

[1]

(iv)

find the range of values of |w z|.

[3]

2013 RVHS H2 Maths Prelim Paper 1

11

(a)

x
Sketch the graph of y sec 2 for x , showing the exact coordinates
2
of the points of intersection with the axes.
[2]

Hence find the exact value of

(b)

x
sec 2 dx .
2

(i)

Find

(ii)

Find the exact volume of revolution when the region bounded by the curve

dx using the substitution u x .

[3]

[3]

y 1 2 , the line y e2 and the x-axis is rotated radian about the


x
y-axis.
[3]

12

The curve C has parametric equations

x t 2 t 6, y t 3 4t , t 1 .
(i)

Sketch the curve C.

[1]

(ii)

Find

(iii)

Find the Maclaurin series for y up to and including the term in x2.

(iv)

Find the percentage error when the area of the region bounded by C, the x-axis
and the line x = 1 is approximated with the Maclaurin series found in part (iii).
[5]

d 2 y 2(3t 2 3t 4)
dy
and show that
.

dx
dx 2
(2t 1)3

End of Paper

2013 RVHS H2 Maths Prelim Paper 1

[4]
[3]

RVHS 2013 Yr 6 H2 MA Prelim Paper 1 (Solutions)


Question 1 [4 Marks]
Using the information, we form the following equations:
2 x 3y 1 z 1

2 1.5 x y z
2

x 1 2 y 1 z 1

which can be simplified to:


2 x 3 y z 00
3 x yz
2 x 2 y z 5
Using GC to solve the above system of equations, we
have
x y100,
5, z 35
x 10,
5, zy 35
Thus,
.

Each team has to play 100 3 5 1 =26 matches


Thus, Malan United lost 26 15 5 = 6 matches
while West Coast Rangers lost 26 11 12 = 3 matches.
So, Malan United has the most no. of loses.
Question 2 [5 Marks]

b(b 1)
(ax) 2 ...
2!
2
a b(b 1) 2
1 bax
x ...
2
Comparing coefficients,
3
3
ba a
2
2b
2
a b(b 1)
3

2
8

1 ax

1 b(ax)

3
3 b(b 1)

2
8
2b
3(b 1)

1
b
3(b 1) b
3
b
4
a 2
7

5 4 = 5 54

The series expansion is valid for


1
1
ax 1 2 x 1 x
2
2
However, the substitution x 2 falls outside the
validity range. Thus, it is not suitable.
Question 3 [7 Marks]
(i) Let P(n) be the statement:
n
, for all n Z
un
2
n 2
When n = 1,
1
1
LHS = u1 (given)
= RHS
2
9
1 2
P(1) is true.

Assume P(k) is true for some k Z ,


k
i.e. uk
2
k 2
We need to show that P(k+1) is true,
k 1
i.e. uk 1
2
k 1 2
When n k 1 ,
LHS uk 1
uk

k2 k 4
(k 3) 2 (k 2) 2

k 2

k2 k 4
(k 3) 2 (k 2) 2

k (k 3) 2 k 2 k 4

(k 3) 2 (k 2) 2
k (k 2 6k 9) k 2 k 4

(k 3) 2 (k 2) 2
k 3 6k 2 9k k 2 k 4

(k 3) 2 (k 2) 2
k 3 5k 2 8k 4

(k 3) 2 (k 2) 2
(k 2) 2 (k 1)
(k 3) 2 (k 2) 2
(k 1)
k 1

RHS
2
2
(k 3)
k 1 2

P(k+1) is true.

Since P(1) is true, and P(k) is true implies P(k+1) is true,


by Mathematical Induction, P(n) is true for all n Z .

(ii)

N
n2 n 4

un un1

2
2
n 1 ( n 3) ( n 2)
n 1
u1 u2
u2 u3
u3 u4
.
.
.
uN uN 1
u1 uN 1
1
N 1

9 N 3 2
N

n2 n 4
1
N 1

2
2
9 N 3 2
n 1 ( n 3) ( n 2)
N

As N ,

N 1

N 3

1
n2 n 4

2
2
9
n 1 ( n 3) ( n 2)

Question 4 [7 Marks]
(i)
y cos ln x 2

dy
1
sin ln x 2 2 (2 x)
dx
x
2
sin ln x 2
x

(ii)

e2

ln x
sin ln x 2 dx
x

1
2

e2

2
2
x sin ln x ln x dx

let u ln x and

du 1

dx x

e2
1
2

ln x cos ln x

1
2

dv
2
sin ln x 2
dx
x

and v cos ln x 2

e2

1
2
cos ln x dx

1
1
cos( )
2 2
2

e2

2
2
cos ln x dx

e2
1 1
sin ln x 2
1
2 2 2

1 1

sin
2 2 2

Question 5 [7 Marks]
2
2
(i)
Given 2 x y 3 x y 16 ,
Differentiate implicitly wrt x, we have
dy

dy
2 2 x y 2 6 x y 1 0
dx

dx
Then,
2 2 x y 6 x y ddyx 4 2 x y 6 x y
dy 8 x 4 y 6 x 6 y

dx 4 x 2 y 6 x 6 y
2 y 14 x

8 y 2x
7x y

x 4y
As the normal at the point P is parallel to the x-axis,
(ii)
the gradient of the normal is 0.
So, the gradient of the tangent tends towards .
dy 7 x y
I.e.

x 4y 0.
dx x 4 y
Thus, at the point P with horizontal normal, x 4 y .
(iii) When the curve cuts the y axis, x = 0.
Substituting into equation of curve:

0 y 2 3 0 y 2 16

4 y 2 16
Thus, y = 2 (negative y axis)
Thus, the point P is 0, 2 .
Next, at the point P, gradient of normal
1
8
=
4
2
7(0) ( 2)
0 4( 2)

Hence, the equation of the normal at the point P is then:


y 4 x 2

Question 6 [8 marks]
(a)(i)

y f ( x)

(ii)

y f ' ( x)

(b)(i)

C1 : x 2 y 2 6 x 8 y 16 0
( x 3) 2 9 ( y 4) 2 16 16 0
( x 3) 2 ( y 4) 2 32
So, a = 3, b = 4, c = 3
(ii) Scale parallel to x-axis with scale factor 3, followed by
translation of 3 units in the negative direction of x-axis,
followed by translation of 4 units in the positive
direction of y-axis.

---Or
Translation of 1 unit in the negative direction of x-axis,
followed by scaling parallel to x-axis, scale factor 3,
followed by translation of 4 units in the positive
direction of y-axis.

Question 7 [9 Marks]
w 3
i ... (1)
z 2
z * 2w 4 4i ... (2)

3
(1) w iz
2
Into (2):
3
z * 2 iz 4 4i
2
Let z x iy

x iy

3i x iy 4 4i

Comparing real and imaginary parts,


x 3 y 4 and y 3x 4
Solving,
x 2 and y 2.
z 2 2i
w 3 3i
i

e 3 (v w) z w
i

e 3v z w e 3w

e 3 1 w z

e 1 w e
1 e w e z

v e

i 3

i 3

i 3

i 3

i.e. a e

i 3

i 3

arg (w v) = angle VW makes with the positive real


direction, originating from V

tan 1 5

3
arg( w v) ( )

3
4
tan 1 5
3
tan 1 5

Question 8 [9 Marks]
The amounts supplied at the end of every month form an
(a)
AP with a = 200 and d = 20.
(i)
When n = 50 (ie end Feb 2017),
the total amount of raw materials from Company A
50
2(200) (50 1) 20 34500 units
2
(ii)
34500
Let Sn
.
2
n
2 200 n 1 20 17250
2
10n2 190n 17250 0
Using GC to solve the quadratic inequality:
We have n 33.1
So, it is when n = 34, i.e. end of Oct 2015, that
Company A would have first supplied more than half of
the total amount they could supply for the duration.
The amounts of raw materials supplied at the end of
(b)
each month form a GP.
(i)
With the given information, we have
T12 ar11 387.24 (1)

T24 ar 23 109.37 (2)


ar 23 109.37

ar11 387.24
r12 0.282434665
r 0.900001362 0.900 (3sf )

Then

(ii)

and a 1233.972205
Thus, the first supply of raw materials by Company B
was 1234 units (nearest whole no.)
We note that with no fixed duration, the total amount of
raw materials supplied = a + ar + ar2 + ar3 +
As r 1 , the sum to infinity S exists.
a
1233.972205

12339.89012
1 r 1 0.900001362
Thus, the theoretical total supply of raw materials from
Company B is 12340 units (to the nearest whole no.)
S

Question 9 [10 Marks]


(i)
By the ratio theorem,

(ii)

D lies on OB produced, thus


Since CD is perpendicular to AB,

1
k1

(iii)

(iv)

If OADC is a parallelogram, then


2a (4 k ) b ,
which implies that a is parallel to b.
But a is not parallel to b. Thus, OADC cannot be a
parallelogram.
Area of triangle ABD
1
= AB BD
2
1
= (b a) (k 1)b
2
k 1
=
(b b) (a b)
2
k 1
=
ab
2

Question 10 [10 Marks]


(i)

4
1 i is a circle centred
z 2 2i
at the point (2, 2) with radius
units.

The locus given by

Im

Re
(2, 2)

(ii)

The locus of w is a half-line from the point (k, k),


excluding the point itself, that makes an angle of
radians with the positive real axis.

(The diagram above illustrates possible loci for w for

10

(iii)

different values of k)
The locus of w will intersect twice when it is between
the two tangents of the circle excluding the case when k
= 0.
Thus, the set of values of k for which the two loci
intersect twice is
.
Im
locus of z
Re

(4, 4)

locus of w

(iv)

The largest value of |w z| occurs as


locus. Then

along the

The least value of |w z| occurs when w is at C and z is


at D.

Therefore, w z 2 2

11

Question 11 [11 Marks]


(a)
x
y sec 2
2
y

2
A , 0
3

2
B , 0
3

C 0, 1

C
Hence,

x
sec 2 dx
2

x
2 sec 2 dx

x
x 3
2 x 2 ln sec tan
2
2 0

2


2 ln sec tan
3
6
6

2
1
2
2 ln

3
3
3
2
2

2 ln 3 or
ln 3
3
3
1
Let u x
1 x dx
du
1
1

1
dx 2 x 2u

2u du
1 u
2

2
du
1 u
2u 2 ln 1 u c

(b)
(i)

2 x 2 ln 1 x c

12

(b)
(ii)

y 1 2
x

y = e2
y=1
x

y 1 2
x

y 1
x2

1
x2

1
1 y

Required volume
e2

x 2 dy
0

e2

1
dy, since 0 y e 2
1 y
e2

2 y 2 ln 1 y

0
2 e ln(1 e) units3

13

Question 12 [13 Marks]


(i)
y
C:

(4, 3)
(ii)

C : x t 2 t 6, y t 3 4t , t 1
dx
dy

2t 1 and
3t 2 4
dt
dt
dy dy dt

dx dt dx
3t 2 4

2t 1

d 2 y d dy

dx 2 dx dx

d dy dt

dt dx dx

(2t 1)(6t ) (3t 2 4)(2) 1

2t 1
(2t 1) 2

12t 2 6t 6t 2 8
(2t 1)3

2(3t 2 3t 4)
(shown)
(2t 1)3
When x 0, t 2 .
Thus, when x = 0:
dy 8 d 2 y 44
,

y 0,
dx 5 dx 2 125

(iii)

Hence, Maclaurin series of y

8
22 2
x
x
5
125

14

(iv)

C:

(4, 3)

Area of region bounded by C, the x-axis and the line x = 1


1

y dx
0

2.1926

(t 3 4t )(2t 1) dt

0.85740 units 2

Area approximated with Maclaurin series


1
22 2
644
8
x
x dx 0.85867 or
units2
5
125
750
0
Hence, the percentage error
0.85867 0.8574

100%
0.8574
0.148%

River Valley High School


Mathematics
H2 P2
Section A: Pure Mathematics [40 marks]
1

Find the general solution of the differential equation


dy
ky (1 y ) ,
dt

where 0 y 1 and k

. Give your answer in the form y f (t ) .

Find the particular solution given that y


solution.

[3]

1
when t 0 . Sketch the graph of the
4
[3]

The above differential equation is used to model the spread of certain disease among a
community of people, where y represents the proportion of people being infected with the
disease at time t. Explain why the differential equation may be suitable.
[2]
r

F
x

The diagram above shows a square piece of cardboard ABCD of side x cm, with a square
1
EFGH of side x cm removed from the centre. Two circular drops of water landed on
2
the cardboard, one centred at B and the other centred at D. Both droplets spread out, with
each radius r cm increasing at a rate of 0.5 cm s1.
(i)

Given that the drop of water which landed at B takes 10 seconds to reach F, show
that before the drop of water reaches F, the area of the cardboard which is dry,
1
2

S cm2, is given by S 150 r 2 .

[3]

(ii)

Upon the start of the spreading of the water droplets, determine the minimum
value of S during the first 4 seconds and the rate at which S is changing at the end
of the 4th second, giving your answers in terms of .
[4]

(iii)

Comment on the suitability of using the same method to find the rate at which S is
changing, when r 6 .
[1]

2013 RVHS H2 Maths Prelim Paper 2

The functions f and g are defined by

f :x

g: x

x2 4 x 5

ln x 10

for x , x k ,
for x , x 9 .

(a)

Determine the largest value of k for which f is a one-one function. With this value
[4]
of k, define the inverse function of f in similar form.

(b)

Given that k = 3,
(i)

show that the composite function gf exists and state its range,

[3]

(ii)

solve the inequality f ( x) 2 x 3 0 , leaving your answer in exact form.


[4]

The line l passes through the points A and B with coordinates 5, 1, 0 and 3, 2, 1
respectively. The plane 1 has equation r i 2 j k i k , where ,
Find

(i)

the acute angle between l and 1 ,

[4]

(ii)

the length of projection of AB on 1

[2]

Another plane 2 with equation 4 x 3 y z 12 intersects 1 at line l ' . Explain why


[2]
l ' cannot be equal to l. Find the equation of l ' .
A third plane 3 has equation 7 x ay 2 z b . Given that the three planes have no point
12
in common and that the distance of 3 from the origin is
, find the values of a and
62
b.
[5]

RIVER VALLEY HIGH SCHOOL

9740/02/2013

Section B: Statistics [60 marks]

Each of the two thousand students in Excel Secondary School owns a calculator
manufactured either by Company A, Company B or Company C. The number of students
using each type of calculator are summarised in the table below.
Calculator Company
Number of students

A
520

B
620

C
860

Researchers X and Y wish to carry out a survey to investigate the lifespan of the
calculators. Researcher X uses a random sample of 100 students.
(i)

Explain the meaning of the phrase a random sample in this context.

[1]

Researcher Y decides to use stratified sampling to select 100 students instead.

(ii)

Describe how a stratified sample can be obtained.

[2]

(iii)

State one advantage that stratified sampling would have compared to random
sampling in this context.
[1]

A committee of fourteen people consists of one group of four brothers, one group of five
sisters and five other people. The fourteen people are randomly arranged in a line.
(i)

Find the probability that the brothers are all separated or the sisters are all
together.
[4]

(ii)

The brothers are all separated. Find the probability that the sisters are not all next
to each other.
[3]

Call-to-Win is a recent telephone game that a caller calls in to test his luck on whether
he will win a prize at random. On average, one in every 1000 calls could get through.
However, the outcome of this game is not completely reliable. If a person could get
through, he has 1% chance of not receiving his prize, and if a person could not get
through when he calls, there is a 0.5% chance of still receiving a prize from the game
organiser.
(i)

Show that the probability that a person receives a prize is 0.005985. Hence, find
the expected number of people who receive a prize in a random sample of 1700
callers.
[2]

(ii)

For the events call gets through and receives a prize, determine whether they
are mutually exclusive and whether they are independent.
[4]

RIVER VALLEY HIGH SCHOOL

9740/02/2013

A factory produces different kinds of fruit juice and packs them into 1-litre cartons for
sale. The mass of vitamin C in a carton of apple juice has a normal distribution with mean
51 mg and standard deviation 3 mg. The mass of vitamin C in a carton of blueberry juice
has a normal distribution with mean mg and standard deviation mg.
(i)

The amount of vitamin C in a randomly chosen carton of blueberry juice is


equally likely to be less than 76 mg and more than 160 mg. Show that the value of
[1]
is 118.

(ii)

Let the variable B denote the mass, in mg, of the amount of vitamin C in a
2
randomly chosen carton of blueberry juice. Given that P( B 115) P( B 121) ,
5
show that the value of is 5.30, correct to 3 significant figures.
[3]

(iii)

Find the probability that of two randomly chosen cartons of apple juice, one has
more than 51 mg of vitamin C and the other has less than 51 mg of vitamin C. [2]

(iv)

A sample of five cartons of blueberry juice is chosen. Find the probability that the
carton with the least amount of vitamin C has a vitamin C content of more than
115 mg.
[2]

A publishing company claims that the mean time to write a book is at most 16 months. A
random sample of 15 book authors is selected and the time taken by each writer to write a
book, x months, is measured. The results are summarised by

x 247.5 , ( x 16)

10

25 .

(i)

A test at 5% significance level is carried out on the companys claim. Find the pvalue of the test and state the meaning of this p-value in the context of the
question. What is the conclusion of the test?
[7]

(ii)

Using the p-value obtained in part (i), state with a reason the conclusion, if a twotailed test is conducted at 5% level of significance.
[1]

Calls made by children to an emergency service occur randomly.


(i)

State two conditions needed for the number of calls made by children to the
emergency service in a random 1-hour period to be well modelled by a Poisson
distribution.
[2]

Assume that the number of calls made to the emergency service by children and adults in
a random 1-hour period have independent Poisson distributions with means 0.8 and 0.6
respectively.
(ii)

Use a suitable approximation to find the probability that in a random 20-hour


period, there are at least 5 more cases of emergency calls made by children than
adults.
[3]

(iii)

In a 10-hour period in which there are more than 4 calls to the emergency service,
find the probability that at least 4 of the calls are made by adults.
[3]

RIVER VALLEY HIGH SCHOOL

9740/02/2013

11

12

It is believed that the probability p of a randomly chosen person having presbyopia


(vision disorder due to aging) is related to the persons age x, in years. The table gives the
observed values of p for six different values of x.
x

25

28

30

35

40

45

0.00235

0.00648

0.01000

0.03500

0.10000

0.45600

(i)

Give a sketch of the scatter diagram for the above data and comment whether x
and p have a linear relationship.
[2]

(ii)

State, with a reason, which of the following would be an appropriate model to


represent the above data.
b
(A)
p a ,
x
(B)
p a be x ,
(C)
ln p a bx
where a and b are constants and b > 0.
[2]

(iii)

For the appropriate model, calculate the values of a and b, and find the product
moment correlation coefficient.
[2]

(iv)

Find an estimate of the probability of a 20 year-old person having presbyopia.


Comment on the reliability of the estimate.
[3]

A machine produces light bulbs in which defects occur randomly at an average rate of 1
per 250 produced. The light bulbs produced are then packed in boxes of 100.
(i)

Find the probability that there are more than 2 defective light bulbs in box.

[2]

(ii)

In a weekly routine check, 80 boxes of light bulbs are randomly selected. Find the
probability that the number of boxes which contain more than 2 defective light
bulbs is less than 3.
[2]

(iii)

In the monthly inspection, 1000 light bulbs are randomly selected to be checked
for defectiveness. By using a suitable approximation, estimate the probability that
there are at least 995 light bulbs that are not defective.
[3]

The weight of a light bulb is known to have mean 2.5 grams and standard deviation 0.15
grams. For a random sample of 150 light bulbs, estimate the probability that the mean
weight is between 2.48 grams and 2.52 grams.
Giving a reason, state whether it is necessary to make any assumptions about the
distribution of the weight of the light bulb.
[3]

End of Paper

RIVER VALLEY HIGH SCHOOL

9740/02/2013

RVHS 2013 Yr 6 H2 MA Prelim Paper 2 (Solutions)


Question 1 [8 Marks]
dy
ky (1 y )
dt
1
y(1 y) dy k dt

y 1 y dy k dt
y
ln
kt c, 0 y 1
1 y
y
Aekt , where A ec
1 y

Aekt
1 Aekt

When t 0, y
1

1 1
A
1
:

A
4 4 1 A
3

ekt

ekt
y 3
or y
1
3 ekt
1 ekt
3

y
1

(0, 1 4)
0

The higher the proportion of people infected with the


disease (y), the faster the disease will be spread by
these people;
The higher the proportion of people not infected with
the disease (1 y), the faster the disease can be
spread to these people.

Therefore, the rate of spread of the disease


proportional to the product of y and 1 y.

dy
is
dt

Question 2 [8 Marks]
(i)
Since the drop takes 10 seconds to move from B to F,
So distance BF = 0.5 10 5 cm
Similarly, distance DH = 5 cm
r
A
B

F
x

5
D

C
r

F
x

1 x
4
D 1
x
4

By Pythagoras Theorem,
2

1 1
DH x x
4 4
1
52 x 2
8

x 10 2
2

1 1
Area S x x r 2 (1)
2 2
3
1
x 2 r 2
4
2
2
3
1
10 2 r 2
4
2
1
S
150 r 2 (shown)
2
2

(ii)

We note that as time progresses, the radius increases


and thus S decreases over time. Thus, during the first 4
seconds, the minimum value of S occurs when t = 4s.
After 4 seconds, r = 0.5 4 = 2 cm.
Thus, during the first 4 seconds, the minimum value
1
2
for S = 150 2 = 150 2 = 2 75 cm2.
2

1
Next, S 150 r 2
2
dS
r.
dr
dS dS dr

dt dr dt
At the end of 4th second, r 2.
dr
Also,
0.5
dt
dS
So,
2 0.5 cm 2s-1
dt
Thus S is decreasing at a rate of cm2s-1 at the end of
the 4th second.
Not suitable, because when r = 6, the expression
By Chain Rule,

(iii)

1
S 150 r 2 does not hold, since 0 r BF 5
2

Question 3 [11 Marks]


(a)
Using GC, we sketch the curve y x 2 4 x 5 :

(-2, -4)
From x 2 4 x 5 x 2 9 ,
2

we note that the minimum point is at 2, 9


Thus, the largest value of k for which f is 1-1 is 2.
To find f1, we let
2
y x2 4 x 5 x 2 9

x 2

y9

x 2 y 9
x 2 y 9 (since x 2)
Also Df -1 Rf ,

(b)
(i)

Thus, f -1 : x 2 x 9 for x , x 9
We first note that for
f:x
x 2 4 x 5 for x , x 3,
Df , 3 Rf 8,

Also for g : x

ln x 10 for x , x 9

Dg 9,
Since Rf Dg , gf exists
To find the range of gf, we first note that:
Df ,3 Rf 8,
For the graph of g(x) ln x 10 ,

when x 8, g(x) ln 2.
Thus, Rgf ln 2,

f
g
In sum, ,3
8, )
ln 2, )

(b)
(ii)

f ( x) 2 x 3 0
x 2 4 x 5 2 x 3
Then

x 2 4 x 5 2 x 3 or x 2 4 x 5 2 x 3
x2 6x 2 0

or x 2 2 x 8 0

For solving x 2 6 x 2 0 , we have


6 62 4 1 (2)
6 62 4 1 (2)
or
x
2
2
ie, x 3 11 or x 3 11 (1)
x

For solving x 2 2 x 8 0 , we have

x 2 x 4 0
4 x 2 (2)
Note that 3 11 2 .
Combining (1) and (2) and taking into account that

Df , 3 ,
Df

3 11

3 11

So, the exact solution to the original inequality is

x 3 11 or 4 x 3 .

Question 4 [13 Marks]

(i)

1 1 2
1


A normal to 1 = 2 0 0 2 0
1 1 2
1


2

AB OB OA 3
1

acute angle between l and 1
sin

sin

sin 1

1

0 AB
1

1 2

0 3

1 1
2 0 1
2 14

34.538 34.5
(ii)

(to 3 s.f.)

Length of projection of AB on 1
( AB) cos 34.538
(2) 2 32 (1) 2 cos 34.538
3.08 units (to 3 s.f.)

Alternatively,
Length of projection of AB on 1

1

AB 0
1

2 1
3
1
1

3 0
1
2
2
1 1
3

19
or 3.08 units
2

Since the angle between l and 1 is not zero, l does not


lie in 1 . Hence, l ' (which lies in 1 ) is not equal to l.

1 : x z 0
2 : 4 x 3 y z 12

0
3


Using GC, l ' : r 4 5 ,
0
3


1
0


or l ' : r 4 5 ,
3
0


1
7

3 : r a b
2

7 3

3 // l ' a 5 0
2 3

21 5a 6 0
a3
7

3 : r 3 b
2

b
12

62
7 2 32 22
b
12

62
62
b 12

However, since l ' does not lie in 3 ,

0 7

4 3 b b 12
0 2

b 12

Question 5 [4 Marks]
(i)
A random sample here means the sample is obtained by
selecting 100 students from the 2000 students in such a
way that each of the 2000 students will have an equal
100
1
probability of
of being selected to do the

2000 20
survey.
(ii)
Calculator
A
B
C
Company
520
620
860
Number of
100
100
100
students to
2000
2000
2000
be surveyed 26
31
43

(iii)

26 students who use calculator A, 31 students who use


calculator B and 43 students who use calculator C will
be randomly selected to be surveyed.
Stratified sampling would ensure that each group
(stratum) in the population is proportionally represented,
while random sampling may have a chance of missing
out an important group completely, or may end up with
a certain group overly represented in the sample.

Question 6 [7 marks]
(i)
Let A denote the event that the brothers are all
separated.
Let B denote the event that all the sisters are together.
P A B P A P B P A B

(ii)

11
10! 4!
7
4 + 5!10! 5! 6! P4
P A B
14!
14!
14!
30
5
5
P A B +

91 1001 6006
2005
P A B
0.3338328338 0.334
6006
B ' denote the event that the sisters are not all together.
P B ' A
P B ' A
P A
P B ' A

P A P B
P A

30
5

P B ' A 91 6006
30
91
395
P B ' A
396

Question 7 [6 Marks]
(i)
0.001
0.999

Call gets
through

0.99

Receives prize

0.01

Not receive prize


Receives prize

0.005
Call did
not get
through 0.995

Not receive prize

P(receives a prize) = (0.001)(0.99) (0.999)(0.005)


= 0.005985 (shown)
Expected number of people receiving a prize
= 0.005985 1700 10.1745 (exact) or 10.2
(ii) P(call gets through) = 0.001
P(call gets through receives a prize)
= (0.001)(0.99) 0.00099 0
Call gets through and Receives a prize are not
mutually exclusive.
P(call gets through) P(receives a prize)
= (0.001)(0.005985)
= 0.000005985 P(call gets through receives a prize)
Call gets through and Receives a prize are not
independent.

10

Question 8 [8 Marks]
(i)
Let A and B be the mass, in milligrams, of a
randomly chosen carton of apple juice and
blueberry juice respectively.
A ~ N 51, 32
B ~ N , 2

(ii)

Given P( B 76 ) = P( B 160 )
By symmetry,
76 160

118 (shown)
2
2
P( B 115) P( B 121)
5
2
P( B 115) 1 P( B 121)
5
2
P( B 115) 1 P( B 115)
5
7
2
P( B 115)
5
5
2
P( B 115)
7
115 118 2

P Z

7
3 2

P Z

By InvNorm,
3 0.565948817

(iii)

(iv)

5.300832707 5.30 (shown)

Let A be the mass, in milligrams, of a randomly


chosen carton of apple juice.
A ~ N 51, 32

1 1
2P A 51 P A 51 2
2 2
1

P( B 115)5

0.71428575215
0.1859344813 0.186

OR
5

P( B 115)

5

7
0.1859344321 0.186

11

Question 9 [8 Marks]
(i)
x 247.5 16.5
Unbiased estimate of x
n
15
2
( x 16) 25 (Given) and

( x 16) x 16 247.5 15(16) 7.5

Unbiased estimate of the population variance,


2

(
16)

( x 16)2

s2

n 1
n

1
7.52

25

1.517857143
14
15

Test
Against

H 0 : 16
H1 : 16

1-tailed t-test at 5% level of significance


Reject H0 if p < 0.05
Under H 0 ,
Test statistic: T

X 16
~ t (14) approx.
1.517857143
15

From GC, p value = 0.0691576403 0.0692


Since p = 0.0691576403 > 0.05,
We do not reject H 0 and conclude that at 5% level of
significance, there is insufficient evidence that the mean
time to write a textbook is more than 16 months (i.e.
there is insufficient evidence that the companys claim is
not valid).

(ii)

The p-value of 0.0692


means that the smallest
significance level for which the claim that the mean time
spent on writing one textbook is (at most) 16 months
would be rejected, is 6.92%
For 2-tailed test,
p-value = 2(0.0691576403) = 0.1383152806 > 0.05.
So, H 0 would still not be rejected.
There is insufficient evidence that the mean time to write
a textbook is not equals to 16 months.

12

Question 10 [8 Marks]
(i)
Condition 1: The event that an emergency call is made
by a child is independent of one another.
Condition 2: The mean number of emergency calls
made by children is proportional to the duration of time
interval.
Condition 3: The mean number of calls in an 1-hour
period is constant over a period of time.
(ii)

Let A and B denote the number of emergency calls


made by children and adult in a 20-hour period.
A Po(16) .
Since A 16 10 is large, A N (16,16)
approximately.
B Po(12) .
Since B 12 10 is large, B N (12,12)
approximately.
Thus, A B N (4,28) approximately.
cc

P A B 5 P A B 4.5
0.4623594567
0.462 (3.s.f)
(iii)

Let X and Y denote the number of emergency calls


made by children and adult in a 10-hour period.
X Po(8) and Y Po(6)
X Y Po(14)
P Y 4 X+Y 5
P Y 4 X+Y 5
P X+Y 5

P Y 4 X+Y 5

P Y 4 P X 1 +P Y 5
P X+Y 5

P Y 4 X+Y 5 0.85028619
0.850 (3.s.f)

13

Question 11 [9 Marks]
(i) p

(45, 0.45600)

(25, 0.00235)

From the scatter diagram, x and p do not have a linear


relationship as x increases, p increases at an increasing
rate. (or diagram shows a curvilinear relationship.)
(ii) Model C is the appropriate model.
ln p a bx
p e abx
As x increases, p increases at an increasing rate. The
shape of the points follows the shape of an exponential
graph.
(iii) Using the correct model (C), by GC,
ln p 12.25629269 0.253201756 x
So, a 12.3 , b 0.253
r 0.998033722
= 0.998
Alternatively using Model (A),
1
p 0.7105973289 (19.76698157)
x
So, a = 0.7105973289, b = 19.76698157
r = 0.7514976242
= 0.751
Alternatively using Model (B),
p 0.1242084159 (9228856083)e x
So, a = 0.1242084159, b = 9228856083
r = 0.291980861
= 0.292
(iv) When x = 20,
ln p 12.25629269 0.253201756(20)
= 7.19225757
p 0.0007523886317
= 0.000752
Although r value is high suggesting a strong positive
linear correlation between x and p, x = 20 does not fall
in the data range, thus the estimate is not reliable.

14

Question 12 [10 Marks]


From the given information the probability that a
(i)
particular light bulb is defective is 1/250 = 0.004.
Let X be the number of defective light bulbs in a box of
100.
Then X B 100,0.004

(ii)

(iii)

Thus, P X 2 1 P( X 2)
= 0.0077547305
=0.00775 (3 sf)
Let Y be the number of boxes which contain more than 2
defective light bulbs among the 80 chosen boxes.
Then from part(i) we have Y B 80,0.0077547
As such, P Y 3 P( X 2)
= 0.975379952
= 0.975 (3 sf)
Let D be the number of defective light bulbs among the
1000 chosen ones.
Then D B 1000,0.004 .
Then we note that
n = 1000 (> 50) ,
p = 0.004 (<0.1)
and np = 4 < 5
We thus apply Poisson approximation ie D Po 4
And therfore
P( D 5) 0.7851303874 0.785 (3 sf)
Note: If we were to define D to be the number of nondefective light bulbs among the 1000 chosen ones, then
D B 1000,0.996 .
Although, we can check that np > 5, nq is not more than
5!!
Thus, we cannot apply normal approximation.
Let W be the weight (in gram) of a light bulb.
Since n =150 is large, we may apply Central Limit

0.152
approximately
Theorem to have W N 2.5,
150

Then P 2.48 W 2.52 = 0.8975295895


= 0.898 (3 sf)
It is not necessary to make an assumption on the
distribution of the weight of the light bulb as n is large
and Central Limit Theorem can be applied so that W
follows a normal distribution.

RAFFLES INSTITUTION
2013 Year 6 Preliminary Examination
Higher 2

MATHEMATICS

9740/01

Paper 1

17 September 2013

3 hours
Additional materials:

Answer Paper
List of Formulae (MF15)

READ THESE INSTRUCTIONS FIRST


Write your name and CT group on all the work you hand in.
Write in dark blue or black pen on both sides of the paper.
You may use a soft pencil for any diagrams or graphs.
Do not use staples, paper clips, highlighters, glue or correction fluid.
Answer all the questions.
Give non-exact numerical answers correct to 3 significant figures, or 1 decimal place in the
case of angles in degrees, unless a different level of accuracy is specified in the question.
You are expected to use a graphic calculator.
Unsupported answers from a graphic calculator are allowed unless a question specifically
states otherwise.
Where unsupported answers from a graphic calculator are not allowed in a question, you are
required to present the mathematical steps using mathematical notations and not calculator
commands.
You are reminded of the need for clear presentation in your answers.
The number of marks is given in brackets [ ] at the end of each question or part question.
At the end of the test, fasten all your work securely together.

This document consists of 4 printed pages.


RAFFLES INSTITUTION

RI 2013

Math Department

[Turn over

2
1

By considering the expansion of

1
1 x2

, or otherwise, show that

sin 1 x x

x3
ax5 ...
6

where a is a constant to be determined.

[5]

In the triangle PQR, angle PQR radians, angle PRQ radians and
6

QR = 3. Given that is sufficiently small, show that


PQ PR a sin sin b ,

6
for constants a and b to be determined.

[5]

is defined by u1 9 and
un1 un 2n 3 for n 1.

A sequence of positive integers u1 , u2 , u3 ,


(i)

Find u2 , u3 and u4 .

[1]

(ii)

By considering the value of un 5 , make a conjecture for a formula for un in terms of n.


Prove your conjecture by induction.
[5]

It is given that x and y satisfy the equation

y2
y 4 ln x 4 6 x 2 , y 0.
4

dy 2 xy ( x 2 3)

.
dx
2 y4 1

(i)

Show that

(ii)

Hence obtain the possible exact value(s) of

[3]
dy
when y 2 .
dx

[3]

OABC is a trapezium such that OA is parallel to CB, and CB : OA = k : 1 , where k is a


positive constant, and k 1 .
Given that OA = a, OB = b, and X and Y are the midpoints of OB and AC respectively, find
the following vectors in terms of k, a and b
(i)

OC ,

[1]

(ii)

OY .

[2]

Hence show that XY is parallel to OA.

[2]

It is given that OB and AC intersect at the point D. Find the ratio, in terms of k, between the
area of the triangle XYD and the area of the triangle BCD.
[2]

H2 MA 9740/ 2013 RI Year 6 Preliminary Examination/01

[Turn over

2
1


The line l has equation r = 1 1 , R and the plane has equation r .
3
1


(i)

2

1 7 .
1

Find the position vector of the point of intersection, A, of l and .

[3]

It is given that is the acute angle between l and .


(ii)

Find the exact value of sin .

[2]

(iii) The point B has coordinates (2, 1, 3). Hence or otherwise, find the shortest distance
from B to , giving your answer in the exact form.
[2]
7

An entomologist is investigating the change in population size N of a certain species of


insects at time t weeks. He suggests that N and t are related by the differential equation
dN
N kN 2 ,
dt
where k is a positive constant.
Show that (do not merely verify) the general solution to the differential equation is
1
,
N
k Aet
where A is an arbitrary constant.

[4]

Given that initially, there are 250 insects and after a very long time, the insect population is
expected to approach a limit of 10, 000. Find the time required for the insect population to
reach three times the initial population, giving your answer correct to the nearest number of
days.
[3]

1
Show that sin 4 A sin 2 A sin 2 2 A .
4
n
1
Given that Sn r sin 4 2r x .
r 0 4

(i)

[2]

1
sin 2 2n1 x .
n 1
4
(iii) Hence give a reason why S n converges and state the sum to infinity.

(ii)

By using the result in (i), prove that Sn sin 2 x

[3]
[2]

[Give all answers correct to the nearest dollar.]


Mr Tan decides to set up a scholarship fund for worthy students. On 1 January 2013, he
places this scholarship fund in a bank investment which guarantees an annual interest rate of
2.5%. This interest is added to the fund at the end of each year. The annual scholarship award
of $2000 is first awarded on 1 January 2014.
(i) To award the scholarship for year 2014, find the minimum amount of money $k that
Mr Tan needs for the fund. If the annual scholarship is to be given out for years 2014
and 2015, show that in addition to $k , Mr Tan will need at least a further $1904, correct
to the nearest dollar, for the fund.
[3]
(ii) Find the minimum amount Mr Tan needs for the scholarship fund if he wants the annual
scholarship to be given out for 10 consecutive years.
[3]
(iii) Find the minimum amount Mr Tan needs for the scholarship fund if he intends to keep
the scholarship going long into the future.
[2]
H2 MA 9740/ 2013 RI Year 6 Preliminary Examination/01

[Turn over

4
10

2
x 5
. Using an algebraic method, solve the inequality f x . [3]
2x 3
3
Hence find the exact range of values of x for which
2
(i) f ln x ,
[3]
3

It is given that f x

1
f x
2

(ii)

11

2
3.

[2]

Sketch, on separate diagrams, the graph of x 6 x 16


2

y 1

(i) k 2 ,
(ii) k 0 ,
making clear the main relevant features of each curve.

0 for

[3]
[4]

(iii) State the equation of one line of symmetry of the curve in part (i) and describe fully a
sequence of two transformations which would transform this curve onto the curve
[3]
2 x 2 y 2 50 .

12

(a)

Find the exact value of

x 2 ln x dx .

dx .
3 2 x x2
Hence, find the exact value of the constant a for which
2
a
9
dx
1 2 x dx.
0
12
3 2 x x2

(b)

Find

13

[3]

The curve C has parametric equations


x t 2 8,

y 2t 2 10t 16 ,

where t

[7]

dy
in terms of t. Hence find the coordinates of the minimum point on C, and state
dx
the coordinates of the point A on C whose tangent to the curve at A is a vertical line.
[You do not need to show that the stationary point is indeed a minimum point.]
[5]

(i)

Find

(ii)

Sketch the curve C.

[1]

It is given that the point P on the curve C has parameter p.


(iii) Show that the equation of the tangent at P is py 2 p 5 x 5 p 2 40 .

[2]

(iv) It is given further that the tangent at P passes through the origin. Find the possible exact
coordinates of P.
[3]
The set of points Q in an Argand diagram represents the complex number z that satisfies
z t 2 8 i 2t 2 10t 16 , t . By using the results obtained in (iv) or otherwise, find

the range of values of arg z , giving your answers correct to three decimal places.

[3]

END OF PAPER
H2 MA 9740/ 2013 RI Year 6 Preliminary Examination/01

[Turn over

ANNEX B
RI 2013 H2 Maths Preliminary Examination Paper 1
Qn/No Topic Set
Binomial Series
1
2

Maclaurin Series

Mathematical Induction

Answers
3
a .
40

a 2 3 and b 6
(i) u2 14, u3 21, u4 30
(ii) un n 1 5.
2

4
5

Differentiation and Applications 40 2


40 2
or
31
31
Vectors
(i) b ka
(ii)

1
1 k
b
a.
2
2

1 k
6

Vectors

Differential Equations

Method of Difference

AP & GP

10

Inequalities

: 4k 2

3
1
(i) 3
2
5
2
(ii)
3
(iii)

1
6

8 days
(iii) sin 2 x
(ii) $17504
(iii) $ 80000
(i)

x 9 or x
9

3
2

(ii) 0 x e or x e
(iii) x 1 or x 2

3
2

11

Curve Sketching

(i)

y
x 3
25

y 1

50

3,1 5
x

0
5 2

(i)

y k x 1 3 k

y k x 3 k 1

x 3

25

y 1

25k

(3,1)
O

12

Integration and Applications

13

Differentiation and Applications


(with Complex Numbers)

(ii) x 3 or y 1
8ln 2 7
(a)

3
9
x 1
(b) 9sin 1
c
2
1 2 2
a
2
(i) (8, 16)
y
(ii)
C

14.25,3.5
O

(iv) 16,32 20 2

0.228 arg z 1.311

2
Raffles Institution
2013 Year 6 Preliminary Examination Paper 2
H2 Mathematics 9740
Section A: Pure Mathematics [40 marks]
1

A graphic calculator is not to be used in answering this question.


Referred to the origin O , the points A and B have position vectors given by 7i 2 j 8k
and i 5j 8k respectively. The plane 1 has equation 2 x y 2 z 5 .
(i)

(ii)

The point C is the foot of perpendicular from A to 1 .


Find the position vector of C .
The plane 2 contains the line AB and is perpendicular to 1 .
Find the equation of 2 in scalar product form.

[3]

(iii) The line l is the common line of intersection between 1 and 2 .


Verify that l is parallel to the vector 2i 2 j k .
Without any further calculation, write down the cartesian equation of l .

(a)

[3]

[3]

The functions f and g are defined as follows:


2x 1
f :x
k, x ,
2
x 2x 3

g:x

x 5 2 ,

x 2k ,

where k is a constant.
(i)
(ii)
(b)

Find the range of values of k for which the function gf exists.


For k 5 , find the range of gf .

The function h is given by


h : x ax3 bx2 cx d ,
where a, b, c and d are real constants.

[3]
[2]

The graph of y h( x) passes through the points (1,1) and (2, 2) . Given that (2, 2) is
a maximum point, find three linear equations involving a, b, c and d .
It is given further that the point (4,14) lies on the graph of y h x , and that

h( x) is strictly decreasing for x 2 . Find the values of a, b, c and d .

[4]

3
3

The diagram shows the curve C with equation y

16
.
( x 2)2 4

It has a turning point (2, 4) .


y
y=4

(2, 4)
R

16
( x 2)2 4

0
The region R is bounded by C , the y axis and the line y 4 .
(i)
(ii)

By using the substitution y 4sin 2 , find the exact value of


Using the result in (i), find the exact area of R .

4
2

4 y
dy .
y

[5]
[3]

(iii) R is rotated through 2 radians about the x axis. Find the volume of the solid of
revolution formed.
[3]

(a)

(b)

If one of the roots of the equation z 3 2 z k 0 is i , find k .


Hence find the other two roots.
The complex number w has modulus r and argument .
1
(i) Write down
in the form x iy .
w

[4]

[1]

Given that w satisfies


500
3 w 40i ,
w
use a non-calculator method to find, in either order,

(ii)

the value of r ,

(iii) the complex number w in the form x iy .


[6]

4
Section B: Statistics [60 marks]

A polyclinic wishes to conduct a survey to find out its patients opinions of its service
quality.
(i)

Explain how 200 respondents can be chosen using simple random sampling.

(ii)

The following is known about the patients who have visited the polyclinic in the past
year:
Age ( x )
Percentage of patients

x 20
15%

20 x 35
20%

35 x 50
40%

[2]

x 50
25%

Describe how stratified sampling can be performed to obtain a random sample of 200
respondents.
[2]
(iii) State one disadvantage of the sampling method in (i) as compared to that in (ii).

[1]

Find the number of ways to arrange six married couples if


(i)

they stand in a line with at least one man not standing next to his wife,

[3]

(ii)

they stand in a circle with exactly five men standing next to their wives.

[3]

A box contains r identical red balls and w identical white balls, where r and w are
integers greater than 1. Balls are removed one at a time, at random and without
replacement, until only those of the same colour are left in the box.
Expressing your answers as single fractions in terms of r and w , find the probability that
(i)

a red ball is removed on the first draw, given that a red ball is removed on the second
draw,
[3]
(ii) a white ball is removed on the first draw, or a white ball is removed on the second
draw, or both,
[2]
(iii) exactly one white ball remains in the box when the removal stops.
[3]

5
8

The manufacturer of a particular type of battery claims that the lifetime is distributed with
mean 150 hours and standard deviation 16.877 hours. A random sample of n batteries,
where n is large, is taken and the lifetime, t hours, of each battery is measured.
It is found that

t 10 317 .

A test is carried out to determine whether the manufacturer has overstated the mean
lifetime of the batteries, and the p value is found to be 0.0975.
(i)

Write down appropriate hypotheses for the test, defining any symbols you use.
State, in context, the conclusion of the test if the level of significance is 8%.

(ii)

[3]

Show that n satisfies the equation


10317 21.871

150 0 ,
n
n
and solve it numerically.

[5]

(iii) State, with a reason, whether it is necessary to assume that the lifetime of the batteries
is normally distributed in order to carry out the calculation in (ii).
9

[1]

The data below shows the average height, measured in feet, of cherry trees from age 1 year
to age 11 years.
Age ( x )
Average
height ( y )
(i)

10

11

9.5

13

15

16.5

17.5

18.5

19

19.5

19.7

19.8

Calculate r , the value of the product moment correlation coefficient betweeen x and
y.

(ii)

State, with a reason, whether the value of r would be different if the average

height is measured in meters instead.

[2]

Give a sketch of the scatter diagram for the data.

[1]

(iii) It is desired to predict the average height of cheery trees beyond age 11 years.
Explain why neither a linear nor a quadratic model is likely to be appropriate.

[2]

It is suggested that the average height y can be modelled by the formula y a b ln x .


(iv) Find the equation of the regression line for the suggested model, and the product
moment correlation coefficient between ln x and y .
Estimate the average height of cherry trees age 5.5 years, and comment on the
reliability of your answer.

[4]

6
10

X and Y are independent, normally distributed random variables with the following
parameters:
Mean
Variance
100
25
X
90
36
Y

Calculate
(i) P X Y 5 ,

[3]

P X1 X 2 3Y 495 , where X1 and X 2 are two independent observations of X .


[3]
150 independent observations of Y are taken.
Use a suitable approximation to calculate the probability that at least 68 of these
observations are less than 90, giving 5 decimal places in your answer.
[4]
(ii)

11

Records had shown that 18% of the eggs from a particular farm were classified as small
while 8% of the eggs from the farm were classified as large. The rest were classified as
medium eggs, of which m% were substandard, m 20 .
(i)

Mrs Egg randomly selects 10 medium eggs from the farm.


Given that the probability that Mrs Egg found 2 substandard medium eggs is 0.3,
write down an equation involving m and hence find m .
[2]

A two-stage inspection scheme for accepting or rejecting a large batch of eggs from the
farm is as follows:
Stage 1: Randomly select and inspect a tray of 30 eggs.
Accept the batch if the tray contains at most 5 small eggs.
Proceed to stage 2 if the tray contains 6 or 7 small eggs.
Reject the batch otherwise.
Stage 2: Randomly select and inspect a further sample of 10 eggs.
Accept the batch if the further sample contains at least 1 large egg.
Reject the batch otherwise.
(ii)

Find the probability that the batch is eventually accepted.

[3]

Mr Egg randomly selects 60 eggs from the farm.


(iii) By using a suitable approximation, find the probability that the total number of
small eggs and medium eggs in his sample is more than 55.
[3]
A supermarket which opens from 7 am to 5pm daily sells eggs from the farm in trays of 30.
The average hourly demand of eggs in the morning (7 am to 12 noon) is 19 trays while in
the afternoon (12 noon to 5pm), the average hourly demand of eggs reduces to 9 trays. You
may assume that both the demand of eggs in the morning and that in the afternoon follow
Poisson distributions.
(iv) State an assumption needed for the daily demand of eggs to follow a Poisson
distribution.
[1]
(v)

Calculate the least number of trays of eggs the supermarket must stock up each day to
ensure that the probability of meeting the daily demand of eggs is more than 0.9. [4]

ANNEX B
RI 2013 H2 Maths Preliminary Examination Paper 2
Qn/No Topic Set
Vectors
1

Functions, System of Linear


Equations

Integration and Applications

Complex Numbers

Sampling Methods

Permutations and Combinations

Probability

Hypothesis Testing

Correlation and Regression

Answers
(i) i j 2k
(ii) r (i 2 j 2k ) 5
1 x y 1
(iii)

z2
2
2
1
(a) (i) k
2
(ii) 0, 1
(b) a 1 , b 4 , c 4 , d 2
(i) 2
(ii) (a) 2(4 )
(b) 35.9
i+i 5
i i 5
(a)
and
2
2
1
(b) (i) (cos i sin )
r
(ii) r 10
(iii) 6 8i
(i) 478955520
(ii) 92160
r 1
(i)
r w 1
2wr w2 w
(ii)
(r w)(r w 1)
wr
(iii)
(r w)(r w 1)
(ii) n 70
(i) r 0.917
(ii)

(iv) 16.5
10

Normal Distribution

11

Binomial and Poisson


Distributions

(i) 0.766
(ii) 0.0981
0.88966
(i) m 18.6
(ii) 0.710
(iii)0.476
(v) 155

CT Group : ________
Index no : ________

Candidate Name : ____________________________


PIONEER JUNIOR COLLEGE
JC 2 Preliminary Examination
MATHEMATICS
Higher 2

( 9740 / 1 )

Tuesday

17 Sept 2013

Additional material: Answer paper, List of Formulae MF15


TIME 3 hours
INSTRUCTIONS TO CANDIDATES
Do not open this booklet until you are told to do so.
Write your full name, index number and CT group on all the work you hand in.
Write in dark blue or black pen on both sides of the paper.
You may use a soft pencil for any diagrams or graphs.
Do not use staples, paper clips, highlighters, glue or correction fluid.
Answer all the questions.
Give non-exact numerical answers correct to 3 significant figures, or 1 decimal place in the case of angles in
degrees, unless a different level of accuracy is specified in the question.
You are expected to use a graphic calculator.
Unsupported answers from a graphic calculator are allowed unless a question specifically states otherwise.
Where unsupported answers from a graphic calculator are not allowed in a question, you are required to
present the mathematical steps using mathematical notations and not calculator commands.
You are reminded of the need for clear presentation in your answers.
The number of marks is given in brackets [ ] at the end of each question or part question.
Attach this question paper with your answers, and arrange your answers in numerical order.

For Examiners Use


Qn

Marks

Qn

10

Marks

Qn

Marks

11

Total

_________________________________________________________________________________
This question paper consists of 5 printed pages and 1 blank page.

PJC2013

[Turn over]

@PJC 2013
[Turn Over]

Express

r 1 r 3
in partial fractions and find

r 1 r 3 .

[4]

r 0

Hence find

N 3

r 1 r 3 .

[2]

r 4

It is given that y 2 e

dy
y3 2 .
dx

(i)

Show that 3 y 2

(ii)

Hence find the Maclaurins series for y , up to and including the term in x 3 .[4]

(iii)

1
x 3

[2]

Deduce that, for small x , 2 e

2
x 3

37
x
e 1 53 x 18
x

[2]

A function f is said to be self-inverse if f(x) f 1( x) for all x in the domain of f.


Functions f and g are defined by

f :x
g: x

1 ax
,
x , x a,
ax
ln(a x), x , x a ,

where a is a positive constant and a 1.


(i)
(ii)

Sketch the graph of f and write down its range.

[2]

Show that f is self-inverse. Hence, or otherwise, find f 2013 5 in terms of a.


[4]

(iii)

Show that the composite function gf does not exist.

[1]

(iv)

If the domain of f is restricted to the set {x : x k} , find the greatest value


of k for which gf exist.
[1]

@PJC 2013
[Turn Over]

In triangle PQR , angle PQR

radians and angle PRQ

4
4
x is sufficiently small for x and above to be ignored, show that
5

x
radians. Given that
2

PR
2 x2 5x4

1
.
QR
2
8 384

(a)

(b)

[7]

Find the exact area, in terms of a, of the region bounded by the curve y 2 4ax
and x 2 4ay , where a is constant.
[4]
8x2 9
The region R is bounded by the curve y
, the positive x-axis and
2
x

5
the lines y 2 x and y . Find, numerically, the volume of the solid
2
generated when R is rotated completely about the x-axis.
[4]

Do not use a graphic calculator in answering this question.


5

(i)


It is given that z 2 cos i sin . Solve the equation giving the roots
6
6

i
in the form re , where r 0 and .
[4]
6

(ii)

Show the roots clearly on an Argand diagram.

(iii)

The

complex

roots

represented

by

[2]
za and

zb

are

such

that

arg za arg zb 0 . Draw the locus of z za z zb on your


2
Argand diagram and hence find the exact cartesian equation of the locus. [4]

1
1
1

Using the fact that sin r sin r 2cos r sin , prove by


2
2
2

1
1

sin n sin
n
2
2

mathematical induction that cos r


for all positive
1
r 1
2sin
2
integers n.
[6]

Hence, find the exact value of cos

cos

2
20
...... cos
cos 7 .
3
3

[2]
@PJC 2013
[Turn Over]

The parametric equations of a curve are

x a 3 sec , y a 2 tan ,
where

and a is a positive constant.


2
2

(i)

Find the coordinates of the point on the curve at which the normal is parallel
to the line y + x =1.
[3]

(ii)

Find the equation of the normal to the curve at the point where ,
simplifying your answer.
[2]

(iii)

Hence find the coordinates of the points A and B where this normal meets the
x- and y-axes respectively.
[2]

(iv)

Find a cartesian equation of the locus of the mid-point of AB as varies.

[3]

Relative to the origin, the points A, B, C and D have position vectors 2i 3j 4k ,

3i 4 j 4k , 4i pj , and qi j k respectively, where p and q are constants. Given


that AC is perpendicular to both AB and DC , show that p =1 and q 6 .

[3]

The point E is on AC produced such that 2AC CE and l is the line through E
perpendicular to both AB and AE . Find

10

(i)

the coordinates of E and hence find the vector equation of l.

[4]

(ii)

the position vectors of the two points on l which are

243 units from A .

[4]

(a)

The complex number w has modulus 3 and argument

3
and w* denotes the
4

conjugate of w.

1 i
.
w*
Given that p n is purely imaginary, find the possible values of n.
Find the modulus and argument of p, where p

(b)

[2]
[3]

The complex number z satisfies the relations


z4
z 2 i 2 1 i 3 and arg
arg(1 i) .
i

(i)

Illustrate both of these relations on a single Argand diagram.

[4]

(ii)

Find the least and greatest possible values of arg(z).

[4]
@PJC 2013
[Turn Over]

11

Ms. Tan took up a car loan of $100000 from a bank in the beginning of a year and she
was offered the following bank loan payment packages:
Package A:
A monthly installment of $500 is to be paid in the beginning of the first month and
increases by $50 for every subsequent month. It is given that the bank charges a one time administrative fee of $12000 for Package A.
Package B:
A monthly installment of $x is to be paid at the beginning of each month and a bank
interest of 5% per annum will be charged on the outstanding loan amount at the end of
the year.
(i)

On which month would Ms. Tan pay up her loan completely under Package A?
[4]

(ii)

Show that the outstanding amount at the end of n th year under Package B is

1000001.05n 252 x 1 1.05

[3]

(iii)

Deduce the least amount $x corrected to 2 decimal places if Ms. Tan wishes to
pay up her loan completely within 7 years under Package B.
[2]

(iv)

Based on your calculations in (iii), determine which package will be cheaper


[2]
for Ms. Tan.

@PJC 2013
[Turn Over]

@PJC 2013
[Turn Over]

Pioneer Junior College


2013 JC2 H2 Maths Prelim Paper 1 (Solution)
Q1

A
B

r 1 r 3

r 1 r 3
2 A r 3 B r 1

r 3 : 2 B(3 1)
B 1
r 1: 2 A(1 3)
A 1

r 1 r 3

r 1 r 3

1
1

r 1 r 3

r 0

1 N
2

2 r 0 r 1 r 3

1 N 1
1

2 r 0 r 1 r 3
1 1 1
(
2 1 3
1 1

2 4
1 1

3 5
1 1

4 6
..........

1
1

N 1 N 1
1
1

N N 2
1
1

)
N 1 N 3
1 1
1
1
1

2 2 N 2 N 3
13
1
1

2 2 N 2 N 3
3
1
1

4 2 N 2 2 N 3

@PJC 2013
[Turn Over]

r 1 r 3 1 3 2 4 3 5 4 6 ....
r 0

1
1
1
1

N 2 N N 1 N 1 N N 2 N 1 N 3

N 3

r 1 r 3 31 4 2 5 3 6 4 ...
r 4

1
1
1

N N 2 N 1 N 1 N 2 N
1

N 1

r 0

r 1 r 3

3
1
1

4 2 N 1 2 N 2

Alternatively,
replace r with k 4,
1

N 3

r 1 r 3
r 4

N 1

k 0

k 4 1 k 4 3
1

N 1

k 0

k 3 k 1

1
3
1

4 2 N 1 2 2 N 1 3

3
1
1

4 2 N 1 2 N 2

(replace N with N 1 from previous part)

Q2
1

(i)

Let y 2 e x 3
y3 2 ex
dy
e x
dx
d
y
3y2
y 3 2 (shown)
dx
3y2

or

dy 1
2 e x 3 e x
dx 3
dy 1 e x
1 e x

2
dx 3
3 y2
x 3
2
e

3y2

dy
y 3 2 (shown)
dx

@PJC 2013
[Turn Over]

d2 y
dy
dy
6 y 3y2 2 3y2
dx
dx
dx
2
d2 y
dy
dy
2 y 2 y 0
dx
dx
dx

(ii)

2
2
d 3 y dy
d2 y
dy d y dy d y
4 2 2 y 3 y 2 0
dx dx dx dx
dx dx
dx
2

2
d 3 y dy
d2 y
dy d y
5 2 y 3 y 2 0
dx dx
dx dx
dx
When x 0 :
dy
1 d2 y
5 d3 y
37
y 1,
,

2
3
dx
3 dx
9 dx
27
1
5 2 37 3
y 1 x x
x ...
3
18
162
1
1
5
37 3
y 2 ex 3 1 x x2
x
3
18
162
2

dy 1
1 5
37
2 e x 3 e x x x 2
dx 3
3 9
54
2

37
5
37
1 5
2 e x 3 e x 3 x x 2 1 x x 2
54
3
18
3 9

(iii)

Q3
(i)

ya

1
a
1

xa

Rf =

\{a} or (- , a) (a, )

@PJC 2013
[Turn Over]

(ii)

1 ax
ax
ya yx 1 ax
y

x(a y ) 1 ay
1 ay
x
a y
1 ax
f 1 ( x)
f ( x), x , x a
ax
f 2 ( x) ff(x) ff 1 (x) x,

( x) f f

(x) f

f 3 ( x) f f 2 (x) f(x),
f4

(x) x,

By reiteration,
f 2013 (5) f(5)
(iii)

Rf =

1 5a
a 5

\{a} or (- , a) (a, )

D g (, a)
R f Dg
gf does not exist.
restrict the range of f to less than a.
k a

(iv)
Q4

x x

4 4 2 2 2

sin
4

PR

QPR
x
sin
2 2
QR


sin
PR
4

QR
x
sin
2 2

2
x
2 cos
2

@PJC 2013
[Turn Over]

2
x

2 1 2
2!

x

2

4!

2
x
x4
2 1

8 384

2 x2 x4

2
8 384

x 2 x 4 1(2) x 2 x 4
2

1 1


...
2
2! 8 384

8 384

2 x2 x4 x4
...
1
2
8 384 64

2 x2 5x4
1

2
8 384

a>0

Q5
(a)

Solving the equations


y 2 4ax and x 2 4ay
x4
4ax x( x3 64a 3 ) 0 x 0, 4a
2
16a
point of intersection of the curves is (4a, 4a).
4a

Required area 4ax dx


0

4a

4a

x2
dx
4a

a <0

4a
23
2x
1 x3

2 a

3
4a 3 0

16a 2
3

@PJC 2013
[Turn Over]

(b)

y = 2x

8x2 9
y

2
x

5
2

R
x

6
7

5
4

3
8

Volume of revolution required


= Volume of cone + Volume of cylinder Volume of solid rotated of region under the curve
2
2
6
8x2 9
1 5 5
5 6 5
(
) 37
dx
3 2 4
x2
7 4
2
8
12.0 (3 S.F.)

Q6
(i)


z 2 cos i sin
6
6




z 2 cos i sin
6
6

5
z 6 32 cos
6

z 32e

i 2 n
6

5 n
i
36 3

1
6

29
i

36

5
i sin

where n 2, 1, 0,1, 2,3

z 32 6 e
z 32 e

1
6

,32 e

17
i

36

1
6

,32 e

5
i
36

1
6

,32 e

7
i

36

1
6

,32 e

19
i

36

1
6

,32 e

31
i

36

(ii)
1

locus of z za z zb

z 32 6 e

19
i

36

Im(z)
1

z 32 6 e

z 32 6 e

31
i

36

7
i

36

Re(z)

1
6

zb 32 e
1

zb 32 6 e

29
i

36

Note : Roots are equally


spread and at a same
distance from the origin.

5
i
36

N
1
6

za 32 e

17
i

36

@PJC 2013
[Turn Over]

17 5

36 36 3
1
Z bON (issoc )
3 2 6

Z aOZb

(iii)

Acute locus makes with the real - axis

5 11

36 36

6
11
11
cartesian equation of locus: y tan
x or y tan
36
36

Q7

1
1

sin n sin
2
2

Let Pn be the statement that cos r


for n 1, 2,3......
1
r 1
2sin
2
n

When n 1 , LHS cos r cos


r 1

1
1
sin 1 sin
2
2
RHS
1
2sin
2
1
3
sin sin
2
2

1
2sin
2
1
2 cos sin
2

1
2sin
2
cos
LHS = RHS , P1 is true.
1
1

sin k sin
2
2

Assume that Pk is true, cos r


for k 1, 2,3......
1
r 1
2sin
2
Need to prove that Pk 1 is true. i.e.
k

3
1

sin k sin
2
2

cos r

1
r 1
2sin
2
k 1

k 1

LHS cos r
r 1

cos r cos k 1
r 1

@PJC 2013
[Turn Over]

1
1

sin k sin
2
2

cos k 1
1
2sin
2
1

1
1
1

sin k sin 2 cos k 1 sin

1
2
2
2
2sin
2
1

1
1
3
1

sin k sin sin k sin k

1
2
2
2
2

2sin
2
3
1

sin k sin
2
2

1
2sin
2
= RHS
Pk 1 is true.
Since P1 is true and Pk is true Pk 1 is true, by mathematical induction, Pn is true for all
positive integers n .
2
20
...... cos
cos 7
3
3
3
21

cos r
3
r 1
1
1

sin 21 sin
2 3
2 3

1
2sin
2 3
43

sin
sin
6
6


2sin
6
1 1

2 2
1
2
2
1

cos

cos

@PJC 2013
[Turn Over]

Q8
(i)

x a 3 sec , y a 2 tan

a 2 sec 2
2 1
dy

dx
a 3 sec tan 3 sin

1
2 1

3 sin
Normal // to line y x 1

Gradient of normal

3

sin

3

sin 1
2
2
, sec 3, tan 2
3
Hence the point is 3a, 2a

sin

(ii)
Equation of the required normal is
3
y a 2 tan
sin x a 3 sec
2

3
5a
y
sin x
tan ....... 1
2
2

(iii)

5a

Putting y 0 into eqn (1) A is


sec , 0
3

5a

Putting x 0 into eqn (1) B is 0,


tan
2

(iv)

Let the mid-point of AB be

5a

5a
sec ,
tan
2 2
2 3

x, y

12 x 2
5a
2
x
sec sec 25a 2
2 3
8 y2
5a
2
y
tan

tan

25a 2
2 2
@PJC 2013
[Turn Over]

10

Now using sec2 1 tan 2


12 x 2
8 y2

1
25a 2
25a 2
OR
12 x 2 8 y 2 = 25a 2

Q9

2

OA 3
4

3

OB 4
4

4 2 2

AC p 3 p 3
0 4 4

4

OC p
0

q

OD 1
1

3 2 1

AB 4 3 1
4 4 0

4 q 4 q

DC p 1 p 1

0 1 1

AC AB
2 1


p 3 1 0
4 0


2 p 3 0
p 1 0
p 1 (shown)
2 2
1



AC 1 3 2 2 1
4 4
2



4 q 4 q

DC 1 1 0
1 1

AC DC
1 4 q

1 0 0
2 1

4q02 0
q 6 0
q 6
q 6 (shown)
@PJC 2013
[Turn Over]

11

(i)
A

O
By ratio theorem,

2OA OE
1 2
3OC 2OA OE
OC

OE 3OC 2OA
4 2

3 1 2 3
0 4

12 4 8

3 6 3
0 8 8

E 8, 3,8
1 1 2
1


Direction vector of l: AB AC 1 1 2 2 1
0 2 2
1


8
1


Vector equation of l: r 3 1
8
1

(ii)

P1

Let position vectors of P1 and P2 = 3 , for some


8

P2

@PJC 2013
[Turn Over]

12

8 2


3 3 243
8 4

6 243
12

6 6 12 243
2
2
2
6 6 12 243
2
2
2
6 6 12 243 0
2
2
2 6 12 243 0
2

By G.C., 3 or 3

83 5


The position vectors of the 2 points are: 3 3 0 ,
8 3 11

8 3 11


3 3 6
8 3 5

Q10
(a)

1 i 1 i
2
*
*
w
3
w

3
1 i
arg( p) arg * arg(1 i) arg( w* )
4 4
w
n

2 i n2 2
n
n
p

i sin
e
cos

2
2
3
3
n
For p n to be purely imaginary, cos
0
2
n

3
5
7

,
,
,
,
2
2
2
2
2
n 1, 3, 5, 7, 2k 1, k
n

@PJC 2013
[Turn Over]

13

(b) (i)
z 2 i 2 1 i 3
z

2+i 2 2

A circle with centre


&

2,

2 & radius 2

z4
arg
arg (1 i)
i

arg( z 4) arg(i ) arg(1 i) arg( z 4)

3
4

A directed line originating from point (4, 0), making an

of

3
with the the initial line.
4

Note

Im(z)
B

Locus of P = z

M
A

When question states


illustrate both
relations, both loci
must be clearly shown
before indicating
clearly the final locus.

Re(z)
O

Consider OMN , MN ON 2
M is (2, 2)
& CM OM OC 2 2 2
Now consider rt

CM
MCA, MCA cos 1
CA
1

2

Least value arg(z )= =

& Greatest value of arg(z )

1
cos 1
4 2

1 2 2 2
1
cos
cos
2

2 1

2 1 rad 0.214 rad

1.357 rad

@PJC 2013
[Turn Over]

14

Q11
(i)
n
1000 n 1 50
2
To complete her loan under Package A:
Amount paid 100000 12000
n
1000 n 1 50 112000
2
n 2 19n 4480 0

Amount paid in n months =

By G.C.
n 77.10 (N.A as n 0)
or
n 58.10
Ms. Tan would finish paying up her loan on the 59th month.

(ii)

End of Year
1

Outstanding Amount
100000 12x 1.05

100000 12 x 1.05 12 x 1.05


100000 12 x 1.052 12 x 1.05

100000 12 x 1.052 12 x 1.05 12 x 1.05


100000 12 x 1.053 12 x 1.052 12 x 1.05

100000 12x 1.05n 12x 1.05n1 ... 12x 1.05

@PJC 2013
[Turn Over]

15

Outstanding amount at the end of month n


100000 12 x 1.05n 12 x 1.05n 1 12 x 1.05n 2 ... 12 x 1.05

100000 1.05n 12 x 1.05n 12 x 1.05n 1 12 x 1.05n 2 ... 12 x 1.05


100000 1.05n 12 x 1.05n 1.05n 1 1.05n 2 ... 1.05
100000 1.05n 12 x 1.05 1.052 ... 1.05n 1 1.05n

1.05 1 1.05 n
100000 1.05n 12 x

0.05

100000 1.05n 12 x 21 1 1.05

100000 1.05n 252 x 1 1.05


(iii)

(Shown)

To complete her loan under Package B in 7 years (i.e. n 7 ) :

100000 1.057 252 x 1 1.05

140710.0423 252 x 252 x(1.05) 7 0


252 x(1.05)7 252 x 140710.0423
102.58930 x 140710.0423
x 1371.5859
Minimum amount Ms. Tan has to pay per month for Package B is $1371.59
(iv)

Total repayment amount in Package A = $112000


Total repayment amount in Package B = (1371.59 7 12) $115213.56
Since total repayment amount in Package B > total repayment amount in
Package A,
Package A is cheaper for Ms. Tan.

@PJC 2013
[Turn Over]

Candidate Name : ____________________________

CT Group : ________
Index no : ________

PIONEER JUNIOR COLLEGE


JC 2 Preliminary Examination
MATHEMATICS
Higher 2

( 9740 / 2 )

Thursday

19 Sept 2013

Additional material: Answer paper, List of Formulae MF15


TIME 3 hours
INSTRUCTIONS TO CANDIDATES
Do not open this booklet until you are told to do so.
Write your full name, index number and CT group on all the work you hand in.
Write in dark blue or black pen on both sides of the paper.
You may use a soft pencil for any diagrams or graphs.
Do not use staples, paper clips, highlighters, glue or correction fluid.
Answer all the questions.
Give non-exact numerical answers correct to 3 significant figures, or 1 decimal place in the case of angles in
degrees, unless a different level of accuracy is specified in the question.
You are expected to use a graphic calculator.
Unsupported answers from a graphic calculator are allowed unless a question specifically states otherwise.
Where unsupported answers from a graphic calculator are not allowed in a question, you are required to
present the mathematical steps using mathematical notations and not calculator commands.
You are reminded of the need for clear presentation in your answers.
The number of marks is given in brackets [ ] at the end of each question or part question.
Attach this question paper with your answers, and arrange your answers in numerical order.

For Examiners Use


Qn

Marks

Qn

Marks

Qn

11

12

10

Marks

Total

_________________________________________________________________________________
This question paper consists of 6 printed pages and 1 blank page.

PJC2013

[Turn over]

@PJC 2013
[Turn Over]

Section A : Pure Mathematics [40 marks]


1

An energy drink manufacturing company produces 3 types of energy drink, namely


Super Plus, Super Power and Super Ultra. The amount of protein, fat and
carbohydrate in each 20 grams sachet of Super Plus, Super Power and Super Ultra is
given below.
Type of
Energy
Drink
Super Plus
Super Power
Super Ultra

Amount in grams for


each 20 grams Sachet
Protein
Fat
Carbohydrate
2.6
2.0
13.3
3.5
1.7
12.5
2.8
0.5
13.9

The company wants to produce a new type of energy drink called Ultra Power Plus by
mixing different amount of Super Plus, Super Power and Super Ultra. It is desired that
each 20 grams sachet of Ultra Power Plus should contain 3.0 grams of protein, 1.5
grams of fat and 13.0 grams of carbohydrate.
Determine the amount, in grams, of Super Plus, Super Power and Super Ultra that the
company should mix to produce one kilogram of Ultra Power Plus.
[5]
[ You may ignore the weight of the other ingredients which make up the rest of each
20 grams sachet. ]

1
to find the exact value of
x

4
3
2
3

dx .

Use the substitution y

In a loan repayment scheme offered by a finance company, the sum of money owed is
$M at time t months. Both M and t are taken to be continuous variables. The sum of
money owed increases, due to interest at a rate proportional to the sum of money
owed. Money is also repaid at a constant rate of $500 per month. When M = 10000,
interest charged will equal repayment. A man takes out an initial loan of $8000 under
such scheme on 1 March 2013.

x 9 x2 4

dM
1
M 10000 .
dt
20

[7]

(i)

Show that

[2]

(ii)

Find M in terms of t.

[4]

(iii)

Find the date when the man will clear all his loan.

[2]

@PJC 2013
[Turn Over]

The

equation

of

the

plane

p1

and

line

is

r 3i 2 j 5k 5 and

r 1 i 1 j k respectively, where is a parameter. The plane p2

contains l and the point A with position vector 6i 9 j 7k .


(i)

Find the equation of p2 .

[2]

(ii)

Find the coordinates of the point B, the foot of perpendicular from A to p1 .


[3]

(iii)

Show that l lies in p1 .

[1]

The equation of plane p3 is r 2i aj 3k b , where a and b are constants.


(iv)

Given that p1 , p2 and p3 have infinite number of common points, find a and
b.
[3]

(v)

What can be said about the values of a and b if p3 has no common point with

p1 and p2 ?

[2]

x 2 ax b
. The vertical asymptote of C is x 2 , and
cx
the coordinates of the turning points are 4, 2 and 0, 6 .

The curve C has equation y

(i)

Find the values of a, b and c.

[3]

(ii)

Sketch C, stating the equations of the asymptotes.

[2]

(iii)

By drawing an appropriate graph on the sketch of C, find the range of values


of k k 0 such that the equation x 4
roots.

(iv)

x 2 ax b

k 2 has no real
c x

[2]

When C undergoes a translation in the direction of y-axis by p units, C


intersects the line y 1 at one negative value of x. State the set of values for
p.
[2]

@PJC 2013
[Turn Over]

Section B : Statistics [60 marks]


6

Find the number of five-letter code-words that can be formed from the letters of the
[4]
word MATHEMATICS.

In Haha College, 70% of the students watch the show Jogging man and 60% of the
students watch the show Voice of me. 40% of those who do not watch the show Voice
of me watch the show Jogging man. Find the probability that a student chosen at
random from the college
(i)

watches both shows,

[3]

(ii)

watches exactly one show,

[2]

(iii)

watches the show Voice of me given that the student does not watch the show
Jogging man.
[2]

State, with a reason, whether the events watches Jogging man and watches Voice of
me are independent.
[1]
8

The table gives u, the trade-in value of a machine (in thousand dollars) after v years in
use.
v
u

1
83

2
76

3
73

4
70

5
65

6
64

It is thought that u and v follow a linear model


constants.

7
63

8
62

1
a bv , where a and b are
u

1
on v.
u

(i)

Find the equation of the regression line of

[2]

(ii)

Given that the trade-in value of the machine is $71000, use an appropriate
regression line to estimate the number of years the machine is used. State a
reason for your choice of regression line.
[3]

(iii)

Another corresponding pair of v and u, where v is 3.5 is added to the data set
1
and the new regression line of
on v is calculated to be
u
1
[3]
0.0119 0.000584v . Find the corresponding u.
u

@PJC 2013
[Turn Over]

In a certain college, the probability that a randomly selected graduating student will
enrol in an overseas university is 0.3.
From a randomly chosen graduating class of 20 students, find the probability that
more than 4 students will enrol in an overseas university.
[2]
Given that the graduating batch consists of 15 classes of 20 students each, find the
probability that there will be less than 13 classes in which at least 5 students in each
class will enrol in an overseas university.
[3]
In the graduating batch of 300 students, find, using a suitable approximation, the
largest value of n such that the probability of having more than n students enrolling in
an overseas university is more than 0.6.
[4]

10

A telecommunication company finds that the duration of calls made by its customers
to City A are normally distributed with mean 8 minutes and standard deviation 1.5
minutes. The duration of calls made by its customers to City B are normally
distributed with mean 10 minutes and standard deviation 1.8 minutes.
(i)

Find the probability that the total duration of three randomly selected calls to
City A differs from the total duration of two randomly selected calls to City B
by more than three minutes.
[3]

The company charges a rate of 22 cents per minute for every call to City A and a rate
of 30 cents per minute for every call to City B.
(ii)

Find the probability that the total cost of one call made to City A and one call
made to City B is at least $4.50.
[2]

(iii)

Three calls were made to City A. Find the probability that exactly one call
costs more than $1.80 and exactly one call costs less than $1.70.
[3]

(iv)

State an assumption needed for your calculations to be valid.

[1]

@PJC 2013
[Turn Over]

11

A company sells packets of flour. A manager claims that the average mass of flour in
a packet is at least 100 g.
(i)

To test this claim, a random sample of 10 packets of flour is checked and the
masses of flour per packet, x g, are summarised by
2

x 100 120 , x 100 4800 .

(ii)

(a)

Find unbiased estimates of the population mean and variance.

[2]

(b)

Test, at the 10% significance level, whether the managers claim is


valid.
[4]

(c)

Explain, in the context of the question, the meaning of at the 10%


significance level .
[1]

The packing process is improved and the new population standard deviation is
known to be 3g. A new random sample of 10 packets of flour is chosen. Find
the range of values of the average mass of the sample that will justify the
managers claim at 5% level of significance.
[3]

State any assumption(s) for your calculations in (i) and (ii) to be valid.
[1]
12

In a certain hospital, the number of newborn is recorded. On average, there are 5


newborn in a day.
(i)

Find the probability that there are at least 30 newborn in a period of a week
(7 days).
[2]

(ii)

In a period of a week, find the least value of r such that the probability that the
number of newborn is at least r is less than 0.2.
[2]

(iii)

Taking a year as 52 weeks, estimate the probability that the average number of
newborn per week in a year is between 34 and 37 inclusive.
[2]

(iv)

A day is considered as normal if the number of newborn that day differs


from its expected value by at most 3. In a month of 30 days, estimate the
probability that more than 25 days are normal.
[5]

@PJC 2013
[Turn Over]

@PJC 2013
[Turn Over]

Pioneer Junior College


2013 JC2 H2 Maths Prelim Paper 2 (Solution)
Q1
Let the amount of Super Plus, Super Power and Super Ultra in 20 grams of Ultra Power Plus
be x, y and z.
Protein :

2.6
3.5
2.8

x
y
z 3
20
20
20
0.13x 0.175 y 0.14 z 3

Eqn 1

Fat :

2
1.7
0.5
x
y
z 1.5
20
20
20
0.1x 0.085 y 0.025 z 1.5

Eqn 2

Carbohydrate :
13.3
12.5
13.9

x
y
z 13.0
20
20
20
0.665 x 0.625 y 0.695 z 13.0
Eqn 3
From GC
x 6.7419,
y 8.3089,
z 4.7821
To produce 1 kilograms of Ultra Power Plus
1000
Amount of Super Plus 6.7419
337.095 grams
20
1000
Amount of Super Power 8.3089
415.445 grams
20
1000
Amount of Super Ultra 4.7821
239.105 grams
20
Q2
4
3
2
3

dx

x 9 x2 4
3
5
4

3
2

1 1
y 9 y 4

1
2 dy
y

1
x

dy
1
2 y2
dx
x
4
3
x y
3
4
2
3
x y
3
2

@PJC 2013
[Turn Over]

1
2 dy
2
y

1
9

4
y

2
y
2
y

3
1
5
34
2 dy
y
2
2 1
y2 9 4 y

34

34
2

34
2

34
2

5
9 4 y2

dy

5
9

4 y2
4

5
2

3
2 y2
2

dy

Note

dy

Coefficient of y 2 must be one


before applying the formula in
MF15.

5
2 y 4
sin 1
3 3
2
2

5
1 5
sin 1 sin 1 1
2
2 2
5 5

2 6 2 2
5

Q3
(i)

dM
kM 500
dt
dM
0 when M 10000,
dt
1
k
20
dM
1
1
M 500 M 10000

dt
20
20

@PJC 2013
[Turn Over]

(ii)
1

M 10000

1
dt
20

dM

ln M 10000
M 10000 e

t
C
20

C
20

M 10000 e

e e
C

t
20

t
20
t

M 10000 eC e 20

M 10000 Ae

t
20

where A eC

t 0, M 8000 A 2000
t
M 10000 2000 e 20

(iii)

Loan is cleared when M 0,


20t
0 10000 2000 e

10000
5
2000
t 20 ln 5 32.189
t

e 20

The man will clear all his loan on 1 December, 2015.


Q4
(i)

6 1 7

a vector parallel to p2 is 9 1 10
7 0 7

7 1 17

10 1 14
7 1 3

17 17 1



r 14 14 1
3 3 0



@PJC 2013
[Turn Over]

17

r 14 3

3
(ii)

6
3


OB 9 2 , for a particular value of
7
5


6 3 3


9 2 2 5
7 5 5


18 9 18 4 35 25 5 0
2
6 3 0

OB 9 2 2 5
7 5 3

The coordinates of B is 0,5,3

(iii)
Method 1
1 3


LHS 1 2 3 3 2 2 5 5 = RHS
5


l is on p1 .
Method 2
1 3

1 2 3 2 5 0
1 5

l is parallel to p1

1 3

1 2 3 2 5
0 5

A point on l is on p1 .
Hence l lies on p1

@PJC 2013
[Turn Over]

(iv)
l is in both p1 and p2 . For p1 , p2 and p3 to have infinite number of common points, p3
must intersect p1 and p2 along l.
Method 1
Direction vector of l perpendicular to n3

1 2

1 a 0 a 5
1 3

Point on l is on p3
1

1
0

2

a b b 3
3

Method 2
l is on p3 , points on l is on p3

Put 0 and 2 into equation of l to obtain 1, 1,0 and 1,1, 2 , 2 points on l.

1 2

1 a b
0 3

2 a b
Eqn 1
1 2

1 a b
2 3

a 8 b
Eqn 2
Solving, a 5, b 3

2

a
3

p3

1, 1,0
l

1,1, 2

(v)
For p3 to have no common point with p1 and p2 , p3 must be parallel to l and does not contain
l. Direction vector of l perpendicular to n3
a 5 from (iv)
Point on l is not on p3
b 3 from (iv)

@PJC 2013
[Turn Over]

Q5
(i)
x 2 is a vertical asymptote, c 2
C passes through 4, 2 and 0, 6
2
4 a 4 b

2
2 4

4a b 12

Eqn 1

2
0 a 0 b

6
2 0

b 12
From Eqn 1, a 6

Eqn 2

(ii)
y

k
x

(iii)

x 4

x 2 ax b

k2
c x

x 4 y k 2
The appropriate graph to draw is a circle with radius k units, centred at 4, 0 .

For the equation to have no real roots, 0 k 2


@PJC 2013
[Turn Over]

(iv)
Translate C by 3 in the direction of y-axis, hence p 3
Or
Translate C by more than 5 units in the direction of y-axis, p 5
Q6
Case 1: all 5 letters are different,
number of code-words = 8C5 5! 6720
Case 2: 1 pair of repeated letters are used,
number of code-words = 3C1 7C3
Case 3: 2 pairs of repeated letters are used,

5!
6300
2!

5!
540
2!2!
Total number of 5-letter code-words = 6720 + 6300 + 540 = 13560
number of code-words = 3C2 6C1

Q7
Let event A be watched Jogging man
Let event B be watched Voice of me
Given P( A) 0.7 , P( B) 0.6 , P( A | B ') 0.4 ,

P( A B ')
0.4
P( B ')
P( A B ') 0.4 0.4 0.16
P( A B) P( A) P( A B ') 0.7 0.16 0.54

(i)

P( A | B ')

(ii)

P( A ' B) P( A B ')

P( B) P( A B) +P( A B ')

0.6 0.54 0.16


= 0.22
(iii)

P( B A ')
P( A ')
0.06

0.2
0.3
Since P( A) 0.7 P( A | B ') 0.4 , the two events are not independent
P( B | A ')

@PJC 2013
[Turn Over]

Q8
(i)
1
0.011899 0.00058357v
u
(ii)
Since u and v are dependent and independent variables, the line of dependent (
independent (v) should always
1
0.011899 0.00058357v
71
v 3.7451years 3.75years

be

used

regardless

of

estimating

1
) on
u
or v.

(iii)
New v 4.3889
Substitute v 4.3889 into the regression line
1
0.0119 0.000584 4.3889
u
1
0.014463
u
9
1
0.014463 9 0.13017

r 1 ur
8
1
1
0.13017
u9 r 1 ur

1
0.11620 0.13017
u9

Note :
8

u
r 1

can be obtained from the

gc, using the 1-variable stats


1
function on L3 .
u

x gives the value of

u
r 1

u9 71.582 71.6

Q9
Let X ~ number of students that will enroll in an overseas university in a class, of 20.

X B(20,0.3)
P( X 4) 1 P( X 4)

0.76249 0.762 (3sf)

Let Y ~number of classes that has at least 5 students will enroll in an overseas university out
of 15 classes.

B(15,0.76249)

P(Y 13) P(Y 12)

0.72847 0.728 (3sf)


@PJC 2013
[Turn Over]

Let W ~ number of students that will enroll in an overseas university in the graduating batch,
out of 300.

B(300,0.3)

Since n = 300 is large, np 300 0.3 90 5 , n(1 p) 300 0.7 210 5

N (90,63) approximately

P(W n) 0.6
1 P(W n) 0.6
P(W n) 0.4
P(W n 0.5) 0.4

c.c

n 0.5 87.989
n 87.489
Greatest n is 87.
Alternative
P(W n 0.5) 0.4 c.c
n = 86
P( X 86 0.5) 0.32962 0.4
n = 87
P( X 87 0.5) 0.37639 0.4
n = 88
P( X 88 0.5) 0.42505 0.4
Greatest n is 87.
Q10
(i)
Let X ~ duration of calls to City A, Y ~ duration of calls to City B

N (8,1.52 )

N (10,1.82 )

Let W X1 X 2 X 3 Y1 Y2

N (4,13.23)

P W 3 1 P W 3 1 P 3 W 3 0.63546 0.635 (3sf)

(ii)

22 X 30 Y ~ N (476, 4005)

P 22 X 30 Y 450 0.65940 0.659 (3sf)


(iii)
Let Q 22 X , then Q

N (176,1089)

Required probability = P Q 180 P Q 170 P 170 Q 180 x3! 0.13961 0.140

(iv)
The duration of any call is assumed to be independent of one another.
@PJC 2013
[Turn Over]

10

Q11
(i)(a)

120
100 88
10

120 ) 373.33 373


1
s (4800
9
10
2

(i)(b) Let X ~ mass of flour in each packet


H1 : 100
Test H0 : 100 vs
Level of significance = 10%
Since n is small and 2 is unknown, we use the t-test.
x "claimed value" 88 100
test statistic, t

1.9640
s
373.33
n
10
From GC, p value = 0.0405625 < 0.10
Since the p value is less than the level of significance, we reject H0 and conclude that
there is sufficient evidence, at 10% significance level, the managers claim is not
valid.
(i)(c) There is a 0.1 probability of wrongly concluding that the average mass of flour in a
packet is less than 100 when it is in fact at least 100 (or equal 100).
(ii)

To not reject Ho, standardized test statistic lies outside in the critical region.
Critical Region: z > -1.64485 at 5% level of significance
x 100
-1.64485
32
10
x 98.4

For both tests to be valid it is necessary to assume that the weight of flour follows normal
distribution.

@PJC 2013
[Turn Over]

11

Q12
(i)
Let X ~ the total number of newborn in 7 days.
X ~ Po(35)
P X 30 1 P X 29 0.82295 0.823 (3sf )
(ii)
P( X r ) 0.2
1 P( X r 1) 0.2

P( X r 1) 0.8
Using GC, Y1= poissoncdf(35, X-1)
r = 40
P( X 40 1) 0.78019 0.8
r = 41
P( X 41 1) 0.82494 0.8
r = 42
P( X 42 1) 0.86314 0.8
Least r = 41
(iii)
As n = 52 is large, by CLT,
35
X ~ N 35, approx
52

P 34 X 37 0.88117 0.881 (3sf)


(iv)
Let Y ~ the total number of newborn in 1 day.
Y ~ Po(5)
P 2 Y 8 P Y 8 P Y 1 0.89148
Let W ~ the number of not normal days, out of 30.

B(30,0.10852)

Since 30 is large, np = 3.2556 < 5


W ~ Po(3.2556) approx
P more than 25 days 'normal'

P less than 5 days not 'normal'


P W 5
= P W 4 0.77064 0.771

@PJC 2013
[Turn Over]

NANYANG JUNIOR COLLEGE


JC2 PRELIMINARY EXAMINATION
Higher 2
MATHEMATICS

9740/01
16th September 2013

Paper 1

3 Hours
Additional Materials:

Answer Paper
List of Formulae (MF15)

READ THESE INSTRUCTIONS FIRST


Write your name and class on all the work you hand in.
Write in dark blue or black pen on both sides of the paper.
You may use a soft pencil for any diagrams or graphs.
Do not use staples, paper clips, highlighters, glue or correction fluid.
Answer all the questions.
Give non-exact numerical answers correct to 3 significant figures, or 1 decimal place in the case of angles in
degrees, unless a different level of accuracy is specified in the question.
You are expected to use a graphic calculator.
Unsupported answers from a graphic calculator are allowed unless a question specifically states otherwise.
Where unsupported answers from a graphic calculator are not allowed in a question, you are required to
present the mathematical steps using mathematical notations and not calculator commands.
You are reminded of the need for clear presentation in your answers.
At the end of the examination, fasten all your work securely together.
The number of marks is given in brackets [ ] at the end of each question or part question.

This document consists of 6 printed pages.


NANYANG JUNIOR COLLEGE
Internal Examinations
NYJC 2013

[Turn Over

2
1

Prove by mathematical induction that


1

1
1

1 2 1 2 3

1
1 2 3

2n
.
n 1

Hence find an expression for


1
1
1

2 4 6 2 4 6 8 2 4 6 8 10

1
246

2n

[6]

A company manufactures containers in the shape of a right cone. Each container is made from a thin
flat sheet of metal in the shape of a sector of a circle, with radius a cm and angle radians (see
Diagram 1).
The two straight sides of each metal sector are then joined together, without overlap, to form a cone
with height h cm and radius r cm (see Diagram 2).

a
h

Diagram 1

Diagram 2

(i)

Express r in terms of a , and .

(ii)

Hence, show that the volume, V of a container is given by V 2

(iii)

Find, in exact form, the maximum volume of the container if the sector of the circle has a

[2]

radius of 2 cm as varies.

[2]

[4]

[Note: There is no need to show that the volume is a maximum.]

NYJC 2013 JC2 Preliminary Examination

a 6 4
4 2 2 .
4
576

9740/01

3
3

(i)

1 sin 2 2 x
Given that f ( x)
and that x is a sufficiently small angle in radians, show that
2cos x 1
f ( x)

1 2 x2
.
1 x2

[3]

Hence find a series expansion for f ( x), up to and including the term in x 2 . Find the range of
[3]

values of x for which this expansion is valid.

(ii)

State the equation of the tangent to the curve f ( x)

Sketch the graphs of y

1 sin 2 2 x
at the point where x = 0. [1]
2cos x 1

xb
x b
on a single diagram, where a and b are positive
and y
xa
b

constants and 1 a b , showing all asymptotes and axial intercepts clearly.

(i)

Using the diagram, solve

(ii)

Using (i), solve

x b x b

.
xa
b

[3]
[2]

ln x b
ln x
, x > 0.
1
ln x a
b

[3]

Relative to the origin O, the position vectors of two points A and B are a and b respectively. The
length of a is 2 units and the angle between a and b is

radians. It is also given that the vector a


6

and b 2a are perpendicular. Find


(i)

the exact length of projection of a onto b,

[2]

(ii)

the exact length of b.

[3]

The point P lies on OB such that the ratio OP : OB 3: 4 . Find


(iii)

[4]

the exact area of triangle APB.

NYJC 2013 JC2 Preliminary Examination

9740/01

[Turn Over

4
6

(a)

Find, in the form x iy , where x, y


satisfying both of the equations

(b)

The

real

numbers

, the possible values of the complex number z

z 3 4
i and zz* 5 .
*
5 5
z

and ,

where

r 0 and

[4]

are

such

that

r cos 2r 2 cos 2 3r 3 cos3 0 and r sin 2r 2 sin 2 3r 3 sin 3 0.

By writing z r (cos isin ) , and using (cos isin )n cos n i sin n , show that
z 2 z 2 3z 3 0 . Hence find the exact values of z, giving your answer in the form a ib ,

where a, b

(a)

. Deduce the value of r and the two possible values of tan .

[5]

A pendulum is swinging so that the distances of successive swings are in geometric


progression. The distance covered by the first swing (from left to right) is 7 cm, and the
distance covered by the subsequent swing (from right to left) is

7
of the distance of the
8

previous swing. Find


the distance covered by the nth swing,

(ii)

the least number of swings needed for the total distance to exceed 95% of the total
distance the pendulum covers before coming to rest.

(b)

[1]

(i)

[3]

The first term and common difference of an arithmetic series A are a and d respectively, and
the first term and common ratio of a geometric series G are b and r respectively.
The corresponding terms of series A and G are added to form the terms of a third series S. The
first and second terms of S are

5
and 4 respectively. The common ratio of G is twice the
2

first term of A, and the common difference of A is four times the first term of G. Find the
[3]

value(s) of a.

Given that a 3 , find the least number of terms in series S such that the sum of the series S
[3]

is greater than 200.

NYJC 2013 JC2 Preliminary Examination

9740/01

5
8

The plane p1 has vector equation r 4i 5j k s i k t 2i j 2k where s and t are real


parameters. The points A and B, which do not lie in p1 , have position vectors 3j k and i 2k
respectively. The perpendicular to the plane p1 from point A meets p1 at N.
(i)

Find the position vector of N.

[3]

(ii)

Find the exact length of projection of AB onto the plane p1 .

[2]

The line l passes through N and is contained in the plane p1 . Another plane p2 contains the line l
and is perpendicular to the line passing through B and N.
(iii)

Find the Cartesian equation of p2 .

[2]

(iv)

Write down a vector which is parallel to l .

[1]

The plane p3 has the equation r 2i j k where and are constants. What can be said
about the values of and if the planes p1 , p2 and p3 meet at a common point?

[2]

The functions h and g are defined by

(i)

h:x

2 x 2 x , x , 1 x 2 ,

g:x

1 (4 x 5)2 , x , x 0.

With the aid of a labelled diagram, show that h is a one-one function and find an expression
for h 1 ( x) .

[5]

(ii)

On the same diagram as in part (i), sketch the graph of h 1 .

[1]

(iii)

Hence, solve the inequality h( x) h 1 ( x) 0 .

[2]

(iv)

Show that the composite function gh 1 exists and state its range.

[2]

NYJC 2013 JC2 Preliminary Examination

9740/01

[Turn Over

6
10 The use of a graphing calculator is not allowed in this question.
(a)

By using integration by parts, find

(b)

Find

0 x tan

x dx .

1
d
(tan 2 ) . Hence use the substitution tan 1 x to evaluate x tan 1 x dx .
0
d

[4]
[6]

11 The curve C is defined by the equations


x sin ,

y 1 cos , for 2 2 .

(i)

Sketch C, showing all axial-intercepts and endpoints clearly.

(ii)

Using the fact that C is periodic with period 2 , or otherwise, find the exact area enclosed by
C, the lines x 2 , x 2 and the x-axis.

(iii)

[1]

Find the volume of the solid formed when R is rotated through 2 radians about the y-axis,
giving your answer to 1 decimal place.

(v)

[4]

C1 is the part of the curve C for 2 . The region R is bounded by C1 , the axes and
the line y = 2. State the area of R.

(iv)

[3]

[2]

Find the volume of the solid formed when R is rotated through 2 radians about the x-axis,
giving your answer to 2 decimal places.

[3]
[2]

END OF PAPER

NYJC 2013 Preliminary Examination

9740/01

2013 NYJC JC2 Prelim Exam 9740/1 Solutions


Qn
1

Let Pn be the proposition that 1

1
1

1 2 1 2 3

1
1 2 3

2n
for all n
n 1

When n 1 , LHS = 1
2
RHS=
1 =LHS
11
Thus P1 is true.
Assume Pk is true for some k
1
1
1
2k
1

1 2 1 2 3
1 2 3 k k 1
When n k 1 ,
LHS =
1
1
1
1

1
1 2 1 2 3
1 2 3 k 1 2 3 k k 1
2k
1

k 1 1 2 3 k k 1
2k
1

1
k 1
(k 2)
2
2k
2

k 1 (k 2)(k 1)
2k (k 2) 2

(k 2)(k 1)
2(k 1) 2
2(k 1)

RHS
(k 1)(k 2)
k 2
Thus if Pk is true, then Pk+1 is true.
Since P1 is true and if Pk is true implies Pk+1 is true, then by the Principle of Mathematical
Induction, Pn is true for all n .

2(i)

1
1
1
1


2 4 6 2 4 6 8 2 4 6 8 10
2 4 6 2n
1
1
1
1
1
(

)
2 1 2 3
1 2 3 n 2 2 4
n
1
1
n
2

n 1 2 2 4 n 1 3
Observe that the arc length of the sector in Diagram 1 is also the circumference of the circle in
Diagram 2.

Thus, we have a 2 r , i.e., r

a
.
2

Page 1 of 11

2013 NYJC JC2 Prelim Exam 9740/1 Solutions


Qn
2(ii)

1
V base area height
3
1
r 2 a 2 r 2
3
2
2
1 a 2 a

a

3 2
2
2

a 2 2 a
2
2

4
12 2
a 3 2
4 2 2 .

2
24
a 6 4
2
V
4 2 2
4
576

2(iii)

Differentiating V 2

V2

a 6 4
4 2 2 w.r.t. when a 2 , we have
4
576

(4 2 4 6 )

9
dV
1
2V
4 16 2 3 6 5
d 9
2 3
4 8 2 3 2
9
When

dV
0 , 3 8 2 3 2 0 , i.e., we have
d

0 (N.A.) or

Thus,

(i)

8 2
(reject ve).
3

8 2 1 2 8 2 8 4 2 16 3
8 2

.
and V

4
3 3 2
3 9 3
27
3

f ( x)

1 sin 2 2 x
2cos x 1
1 2x

x2
2 1 1
2!

Page 2 of 11

2013 NYJC JC2 Prelim Exam 9740/1 Solutions


Qn

1 4 x2
1 x2

1 4 x

1 x2
1 2 x2

1 x2

f ( x) 1 2 x 2 1 x 2

1 2 x 2 1 x 2

x2 1

1 2x2 x2
1 3x2
1 x 1

ii) From above, y = 1

x b
a b
1
,
xa
xa
xa
Asymptotes: x = a, y = 1
b
When x = 0, y .
a
When y = 0, x = b

xb
b
When x = 0, y = -1
When y = 0, x = b

xb
xa

b/a
1

0
y

xb
b

-1

Page 3 of 11

a+b

2013 NYJC JC2 Prelim Exam 9740/1 Solutions


Qn

For intersection, let


xb xb

xa
b
1
1
0
x b
xa b
bxa
x b
0
xa
x b or x a b
(i)

ii

x b x b

xa
b
Ans : x a or b x a b (1)

ln x
ln x b
1
ln x a
b

ln
x

ln x b

ln x a
b
ln x b ln x b

ln x a
b

x>0

From (1):
ln x a or b ln x a b
Ans :

0 x ea or eb x ea b

5
(i)

Length of projection of a onto b=

(ii)

a (b 2a) 0

ab

2 b cos

a b 2a a 0

ab 2 a

ab 8

a b cos

8
3
2

8
3

Page 4 of 11

2013 NYJC JC2 Prelim Exam 9740/1 Solutions


Qn
(iii)

3
OP b
4
Area APB=

1
AP AB
2

1 3
( b a) (b a)
2 4

1 3
ab ab
2 4
1
ab
8
=

1 8 1
1
sin
= a b sin = (2)
8 3 6
8
3

(a)

5 2
, z 3 4i
z
Let z x iy . z 2 x2 y 2 2ixy

Since z*

x2 y 2 i(2 xy) 3 4i

(b)

Equating real and imaginary parts, x2 y 2 3 and 2 xy 4


Solving , x 2, y 1
z 2 i or 2 i
By adding the two equations, and re-grouping, we have
r cos 2r 2 cos 2 3r 3 cos3 i(r sin 2r 2 sin 2 3r 3 sin 3 ) 0
r (cos i sin ) 2(r (cos i sin )) 2 3(r (cos i sin ))3 0
z 2 z 2 3z 3 0
z (3z 2 2 z 1) 0

Since z 0 , z
Therefore, z

2 22 4(3)(1)
2(3)

1
1 i 2
3

1 2 1
r z
3
9 9

tan 2

Page 5 of 11

2013 NYJC JC2 Prelim Exam 9740/1 Solutions


Qn
7

(a)

(b)

7
Tn 7( )n 1
8
7
(7)[1 ( ) n ]
(7)
8
(0.95)
7
7
1
1
8
8
7
[1 ( ) n ] 0.95
8
n 22.43
no. of complete swing is 23
Let first term and common difference of series A be a and d, and first term and common
ratio of series G be b and r.

ab 5

(1)

a d br 4
Sub r 2a, d 4b into (2)

(2)

a 4b 2ab 4
From (1), b 5 a
2

a 4( 5 a) 2a( 5 a) 4
2

a 4a 3 0
a 1 or a 3

3
( )[(2)n 1]
n
[2 (n 1)(6)] 2
200
2
(2) 1
n
1
[4 6n] [(2) n 1] 200
2
2
[A1]
least n 10
8(i)

1 2 1

A normal vector of p1 is 0 1 0 .
1 2 1

0
1


Equation of line AN: r 3 0 ,
1
1


Sub. equation of line AN into equation of plane p1 :

Page 6 of 11

2013 NYJC JC2 Prelim Exam 9740/1 Solutions


Qn
0
1 4 1 2


3 0 5 s 0 t 1
1
1 1 1 2


Solving, we have t 2, s 5 , 3
0 1 3

ON 3 3 0 3
1 1 2

8(ii)

Length of projection
1

AB 0
1

2
1 0 1

0 3 0
2 1 1
=
2
1 1

3 0
1 1

8(iii)

3

2
3

11 units
2

3 1 2

BN 3 0 3
2 2 4

2 3

Equation of p2 : r 3 3
4 2

2

3 23
4

Cartesian equation of p2 is 2 x 3 y 4 z 23 .
8(iv)

2 1 3

A vector parallel to l is 3 0 2 .
4 1 3

Page 7 of 11

2013 NYJC JC2 Prelim Exam 9740/1 Solutions


Qn
Observe that l is the line of intersection between p1 and p2 .
To meet at a common point, p3 must not be parallel to l

2


1

9(i)

3
9

2 0 2 and
3

y
(0,2)

(1,2)

yx

y h 1 ( x)

(2,1)

yk

y h( x)

(2,0)

As seen in the above diagram, any horizontal line y k , k

cuts the graph of h(x) at most

once. Therefore, h is one-one.


Let y 2 x 2 x

y 2 4 x(2 x)
y 2 4 4( x 1)2
( x 1) 2 1

y2
4

x 1 1

y2
4

Since 1 x 2 , x 1 1

h 1 ( x) 1 1

9(iii)

y2
.
4

x2
4

To find the x-coordinate of the point of intersection between y h( x) and y h 1 ( x) , we let


h( x) x .

Page 8 of 11

2013 NYJC JC2 Prelim Exam 9740/1 Solutions


Qn

2 x 2 x x
8x 4 x2 x2
x 8 5x 0
x

8
5

For h( x) h 1 ( x) 0 , we have 1 x
9(iv)

8
.
5

R h1 1, 2 0, Dg
Therefore, the composite function gh 1 exists.
y
1
O

x
y g( x) ,1 x 2

-8

R gh1 8 ,1 .

10

a)

0 x tan

x2

1 x2 1
x dx tan 1 x
dx
0 2 1 x 2
2
0
1 1 1
1
0 1
dx
2 4 2
1 x2
1
1
= x tan 1 x

0
8 2
1 1
= 1 =
8 2 4 4 2
=

Page 9 of 11

2013 NYJC JC2 Prelim Exam 9740/1 Solutions


Qn
d
(tan 2 ) 2 tan sec 2
d
dx
tan x sec 2
d

b)

04 tan sec

tan 2 4 4 tan 2

d

2
2

0
0
1 1
4 (sec2 1) d
4 2 2 0

11

1
tan 0 / 4
8 2
1 1
1
8 2 4 4 2
(i)

(2 , 2)

2
(ii)

(2 , 2)

By symmetry

Area 4 y dx
0

4 (1 cos )(1 cos ) d


0

4 (1 cos 2 ) d
0

4 sin 2 d
0

2 (1 cos 2 ) d
0

2 sin 2
2

0
2
(iii) The area is 4

units2.
Page 10 of 11

2013 NYJC JC2 Prelim Exam 9740/1 Solutions


Qn

(iv) We have
y
2

Volume x 2 dy
0

( sin ) 2 ( sin ) d

193.2

(v)

We have
y
2

Volume (22 )(2 )

y 2 dx

8 2 (1 cos ) 2 (1 cos ) d

8 2 sin 2 (1 cos ) d

74.02

Page 11 of 11

NANYANG JUNIOR COLLEGE


JC2 PRELIMINARY EXAMINATION
Higher 2
MATHEMATICS

9740/02
20th September 2013

Paper 2

3 Hours
Additional Materials:

Answer Paper
List of Formulae (MF15)

READ THESE INSTRUCTIONS FIRST


Write your name and class on all the work you hand in.
Write in dark blue or black pen on both sides of the paper.
You may use a soft pencil for any diagrams or graphs.
Do not use staples, paper clips, highlighters, glue or correction fluid.
Answer all the questions.
Give non-exact numerical answers correct to 3 significant figures, or 1 decimal place in the case of angles in
degrees, unless a different level of accuracy is specified in the question.
You are expected to use a graphic calculator.
Unsupported answers from a graphic calculator are allowed unless a question specifically states otherwise.
Where unsupported answers from a graphic calculator are not allowed in a question, you are required to
present the mathematical steps using mathematical notations and not calculator commands.
You are reminded of the need for clear presentation in your answers.
At the end of the examination, fasten all your work securely together.
The number of marks is given in brackets [ ] at the end of each question or part question.

This document consists of 6 printed pages.


NANYANG JUNIOR COLLEGE
Internal Examinations
NYJC 2013

[Turn Over

2
Section A: Pure Mathematics
1

(i)

Find the general solution of the differential equation x 2

(ii)

Given that y

d2 y
x 2 1.
dx 2

1
when x 1 , sketch, on a single diagram, three members of the family of
2
[5]

solution curves for x 0.


2

[3]

On an Argand diagram, the complex number a = 3+ 4i is represented by A and ia is represented by


B. On a single Argand diagram, sketch the following loci:
(i)

za 5,

[2]

(ii)

z ia 5 .

[2]

(iii) State a single transformation that will map the locus of (i) to the locus of (ii).

[1]

The point C represents the complex number a + ia on the Argand diagram.


(iv)

Explain why C lies on the loci of z a 5 and z ia 5 .

[1]

(v)

Find the exact area of the region satisfying both (i) and (ii).

[2]

(vi)

If z satisfies both (i) and (ii), find the maximum value of arg(z - ia + a) , leaving your answer
[2]

in exact form.

Show that

1
1
2n
- 2
= 4
.
n - n +1 n + n +1 n + n 2 +1

[2]

The population (in thousands) of a virus at the end of the nth day in an experiment is modeled by the
sequence un which satisfies the recurrence relation un+1 - un =

n
for n 1.
n + n2 +1
4

(i)

Give the meaning of the term un1 un in the context of the question.

[1]

(ii)

Express un in terms of u1 and n.

[4]

(iii) Show that, for large values of n, un is approximately equal to u1 +


for this value in the context of the question.
(iv)

Given that u1 =

1
and give an interpretation
2
[2]

1
, find the minimum number of days for the population of the virus to exceed
2
[1]

990.
NYJC 2013 JC2 Preliminary Examination

9740/02

3
4

The curve C has parametric equations


x t2 ,

(i)

y t2 t .

The point P on the curve has parameter p. Show that the equation of the tangent at P is

2 py 2 p 1 x p 2 .
(ii)

[3]

3
If the tangent at P passes through the point 4, , use a non-calculator method to find the
2

[2]

possible coordinates of P.
(iii)

The tangent at P meets the line y x at A and the y-axis at B. Show that the area of triangle
OAB is

(iv)

1 3
p , where O is the origin.
4

[3]

If the point P moves along the curve C such that its x-coordinate is increasing at a constant rate
of 2 units per second, find the rate of increase of the area of the area of triangle OAB when P is
at 16,12 .

[4]
Section B: Statistics

Alan wishes to choose 1 child at random from the 11 children in his music class. The children are
numbered from 2, 3, 4 and so on, up to 12. Alan then throws two fair dice, each numbered from 1 to
6 and choose the child whose number is the sum of the scores on the 2 dice.
(i)

Find the probability the child numbered 7 is chosen.

(ii)

Suppose Alan uses this method to choose 2 children. Explain why his method will not yield a

(iii)
6

[2]

random sample.

[1]

Describe how a simple random sample of 2 children can be obtained.

[2]

The number of chocolate chips found in a scone is denoted by X.


(i)

State, in context, a condition needed for the number of chocolate chips found in a scone to be
well modelled by a Poisson distribution.
[1]

Assuming that the number of chocolate chips in a scone has the distribution Po(6).
(ii)

A random sample of 50 scones is taken. By using a suitable approximation, find the probability
that the mean number of chocolate chips in each scone is greater than 5.5.
[2]

Raisins are also found in the scone. The number of raisins found in a scone, denoted by Y, is an
independent random variable with the distribution Po(6.5).
(iii)

A random sample of n scones is taken, where n is large. Find the least value of n in order to be
more than 85% sure that the mean number of chocolate chips is less than the mean number of
[3]

raisins found in n scones.

NYJC 2013 JC2 Preliminary Examination

9740/02

[Turn Over

4
7

In Country A, it is known that 35% of the adult population have some knowledge of a foreign
language. If 10 adults from this country are chosen at random, find the probability that at most three
of them will have some knowledge of a foreign language.

[2]

For one particular foreign language, only a small proportion r % of the adult population have some
knowledge of it. It is required to select n adults at random, where n is chosen so that the probability
of obtaining at least one adult having some knowledge of the language is approximately 0.99. Use a
suitable Poisson approximation to show that n
For the case when r

460.5
.
r

7
and n 920 , use a suitable approximation to find the probability that
8

exactly four adults in the sample will have some knowledge of the language.

(i)

[3]

[3]

A particular machine is designed to produce steel rods of length 50 cm. A random sample of 5
steel rods produced by that machine were measured and the length in cm were found to be:
50.5, 51.1, 49.9, 50.3, 50.8
Perform an appropriate test at the 5% level of significance to check if the machine is producing
steel rods of the desired length. State one assumption that is used in the test.

(ii)

[5]

Another sample of 60 steel rods was measured and the mean length x recorded. If the
population variance of the steel rods is 0.4602 cm2, find the range of values of x to conclude,
at the 1% level of significance, that the machines produced steel rods that are more than 50 cm.
[3]

Find the number of ways in which the letters of the word SYSTEMATIC can be arranged if
(i)

there are no restrictions,

[1]

(ii)

the two Ts must not be next to each other,

[2]

(iii)

there must be exactly 3 letters between the two Ts,

[2]

(iv)

the first letter is Y and the last letter is a consonant.

[3]

NYJC 2013 JC2 Preliminary Examination

9740/02

5
10 An athlete training for track event told his new manager that his time taken to complete a race may
be taken to be normally distributed. The manager remembered that the athlete told him the mean was
10 s but cannot remember whether the standard deviation mentioned was 5 s or 0.5 s.
(a)

Which do you think is the wrong standard deviation? Justify your answer.

(b)

Suppose the standard deviation is 2 s.

[2]

(i)

The probability that the athlete will complete a race in less than t s is 0.99. Find t.

[1]

(ii)

Find the probability that not more than 18 of 20 races will take less than t s.

[2]

(iii)

Find the probability that the mean time taken for 5 randomly chosen races will be more
[3]

than twice that of another randomly chosen race.

11 It is believed that the probability p of a randomly chosen pregnant woman giving birth to a Downs
Syndrome child is related to the womans age x, in years, by the relation p ab x , 25 x 45,
where a and b are constants. The table gives observed values of p for 5 different values of x.
25
0.000 67

x
p

30
0.001 25

35
0.003 33

40
0.010 00

45
0.033 30

It is given that the product moment correlation coefficient for the bivariate data (x, p) is 0.854,
correct to 3 significant figures. The scatter diagram for the data is shown below.
p (probability)
0.04

0.03
0.02

0.01
0
(i)
(ii)

25

30

35

40

45

From the given information, comment on whether a linear model is appropriate for the
bivariate data (x, p).
[1]
If ln p x is the regression equation of ln p on x, find the least squares estimates of

and .
(iii)
(iv)

x (womans age)

[1]

Draw a scatter diagram of ln p on x and calculate the product moment correlation coefficient.
Comment on its value.
[3]
Estimate the expected number of children with Downs Syndrome that will be born to
5000 randomly chosen pregnant woman of age 32. Comment on the reliability of the estimate.
[3]

NYJC 2013 JC2 Preliminary Examination

9740/02

[Turn Over

6
12 A machine-operated car wash offers four different options as follows:
Option
A
B
C
D

Description
Wash only
Wash and dry
Bubble-wash and dry
Steam-wash and dry

Duration
2 minutes
3 minutes
3 minutes
4 minutes

A driver makes his choice and then proceeds to the car wash. The probability of a driver choosing
5
Option A, Option B, Option C or Option D is given to be
, a , ar and ar 2 respectively, where a
12
and r are positive constants and r 1 .
At a particular instant, there are three cars queuing to use the car wash.
(i)

Show that the probability that at least one of the three drivers has chosen option C is given by
3
[1]
1 1 ar .

(ii)

By considering the sum of the probabilities for all the options, deduce that 0 a

For the rest of the question, take a

7
.
9

[3]

1
1
and r .
3
2

A fourth car, car X joins the queue at the instant when the first car enters the car wash. Assuming
that no time is lost between the time a car leaving the car wash and the next car starting the wash,
find the probability that
(iii)
(iv)

it will be at least 10 minutes before car X enters the car wash and at least one of the previous
three drivers chose Option C,
[2]
it will be at least 10 minutes before car X enters the car wash, given that at least one of the
previous three drivers chose Option C.
[2]

State a necessary assumption for your calculations in (iii) and (iv).

END OF PAPER

NYJC 2013 Preliminary Examination

9740/02

[1]

2013 NYJC JC2 Prelim Exam 9740/2 Solutions


Qn
1

(i)

x2

d2 y
x2 1
dx 2

d2 y x2 1
2
dx 2
x
1
1 2
x
1 x 2
dy
1
x A
dx
x
x2
y ln x Ax B
2

(ii)

x 1, y

1
2

1 1
0 A B
2 2
B A
x2
y ln x Ax A
2
x2
y ln x A( x 1)
2

A=0

A=1
A = 1

Page 1 of 10

2013 NYJC JC2 Prelim Exam 9740/2 Solutions


Qn
2

(i), (ii)
Im
Locus of (i)

Locus of (ii)

C
A

B
O

(iii)
(iv)
(v)

(vi)

Re

A 90 anti-clockwise rotation about the origin.


Since (a + ia) - a = (a + ia) - ia = 5, thus z = a + ia satisfies the equations z - a = 5 and

z - ia = 5. Thus C lies on the loci of z - a = 5 and z - ia = 5.


Since triangle OBC is an isosceles right angled triangle, thus
1
1

Reqd Area = 2 (5) 2 (5) 2


2
4

25
( 2)
2
Let D represents ia - a on the Argand diagram. By circle properties, ODC =

ia a 7 i , thus

max arg(z - ia + a) =

1
+ tan -1 .
4
7

OR
Since DC is parallel to OA,

4
max arg( z ia a) arg(a) tan 1
3
1
1
(n2 + n + 1) - (n2 - n + 1)
=
n2 - n + 1 n2 + n + 1 (n2 + 1- n)(n2 + 1+ n)

2n
(n + 1)2 - n2
2

2n
n + 2n 2 + 1- n2
2n
= 4
n + n2 + 1
=

(i)

The term gives the increase of the number of virus from day n to day n + 1.

Page 2 of 10

p
4

. Since

2013 NYJC JC2 Prelim Exam 9740/2 Solutions


Qn
(ii)

Summing the recurrence relation, we have

n 1

n 1

2r

ur 1 ur 2 r 4 r 2 1
r 1

r 1

1
1
1

2 r 1 r r 1 r r 1
n 1

Thus

1

u1 2
1

1
u2

3
un 1

(n 1) 2 (n 1) 1

1
1

un u1 1 2

2 n n 1
1
1
un u1
2
2 2(n n 1)

u2
u
3

u
n

(iii)

(iv)

1
1
0 for large value of n, thus un u1 + . This is the long term number of
2
2(n n 1)
virus for the experiment.
1
We have un 1
. Need un > 0.99 . Using GC, we have n 8 . Thus the minimum
2
2(n n 1)
Since

number of days is 8.

4(i)

2
(n 1) ( n 1) 1
1
3
1
7

x t2
dx
2t
dt
y t2 t
dy
2t 1
dt
d y 2t 1

dx
2t
At the point P p 2 , p 2 p ,
d y 2 p 1

dx
2p
2 p 1
y p2 p
x p2

2p
Page 3 of 10

2013 NYJC JC2 Prelim Exam 9740/2 Solutions


Qn

2 py 2 p3 2 p 2 2 p 1 x p 2

2 py 2 p3 2 p 2 2 p 1 x 2 p3 p 2
2 py 2 p 1 x p 2
(ii)

3
1
p
1
4
2 2p
2
3
2 p
4
2
p 2
2
p 5p 4 0

p 1 p 4 0
p 1 or

p4

Hence coordinates of P:
1, 0 or 16, 12
(iii)

When y x ,

2 px 2 p 1 x p 2

x p2

Hence A p 2 , p 2
When x 0 ,

p
2
p

Hence B 0,
2

Area of triangle AOB


(iv)

p , p
2

0, 2
B

1 2 p 1 3
p 2 4 p
2

x p2
dx
dp
2p
dt
dt
dp 1
dx
2

dt
dt
p
Area of triangle AOB , S
dS 3 2 d p
p
dt 4
dt
3
p
4

1 3
p
4

Page 4 of 10

2013 NYJC JC2 Prelim Exam 9740/2 Solutions


Qn

When P is at 16,12 , p 4 (from (ii))

dS
3 units per second
dt
Alternative method:

1 3
p
4
p x (reject p x since when P is at 16,12 , p 4 (from (ii))

Area of triangle AOB , S

x p2

1 32
Hence S x
4
dS dS dx

dt dx dt
3 1
x2 2
8
3 12
x
4
When P is at 16,12 ,
1
dS 3
16 2 3 units per second
dt 4
P(7) P(1, 6) P(4,3) P( 2,5)

2 2 2

36 36 36
1

6
Each child do not have an equal probability of getting selected, thus it is not a random
sample.

(i)

(ii)
(iii)
6

(i)
(ii)

(iii)

Number the children from 1 to 11, use a random number generator/table to generate 2
numbers, and that will be correspond to the 2 children to be chosen.
- Chocolate chips are scattered independently or randomly
-Average number of chocolate chips found in a scone is a constant

Let X be the number of walnuts in a scone


X Po(6)
6
By C.L.T X N(6, ) Approximately
50
P( X 5.5) 0.926
Let Y be number of raisins in a scone
Y Po(6.5)

Page 5 of 10

2013 NYJC JC2 Prelim Exam 9740/2 Solutions


Qn

ByC.L.T

N(6.5,

6.5
)Approx.,
n

12.5
) Approx.
n
P( X Y ) 0.85

6
N(6, )Approx.
n

N(0.5,

X Y

P( X Y 0) 0.85
0 0.5
P( Z
) 0.85
12.5
n

P( Z 0.141421 n ) 0.85
0.141421 n 1.03643338
n 7.3287
n 53.71
least n is 54
Alternative Solution:

ByC.L.T

N(6.5,

6.5
)Approx.,
n

12.5
) Approx.
n
P( X Y ) 0.85
X Y

6
N(6, )Approx.
n

N(0.5,

P( X Y 0) 0.85
Using GC,
n
P( X Y 0)
53
0.848
54
0.851
Thus least n = 54.
7

Let X denote the number of adults that have some knowledge of a foreign language out of the 10
adults, ie X B(10,0.35)
P( X 3 ) = 0.514
Let Y denote the number of adults that have some knowledge of a foreign language out of the n
r

nr
adults, ie Y B n,
. Thus Y Po
approx.
100
100
P (Y 1) 1 P(Y 0) =0.99
P(Y 0) 0.01 , ie

Page 6 of 10

2013 NYJC JC2 Prelim Exam 9740/2 Solutions


Qn
e

nr
100

0.01

nr
ln 0.01 4.605
100
460.5
n
r
7 nr 920(7)
r
7

5 and n 1
When n = 920 and r ,
920 1
5 . Therefore,
8 100
800
100
800
Y N (8.05,7.97956) approximately
P(Y = 4) P(3.5 Y 4.5) 0.0508

(i)

Ho: = 50
H1: 50
At 5% significance level, reject Ho if p < 0.05
Apply t-test, Data, o = 50, o,
From GC, p = 0.064986
Since p = 0.064986 > 0.05

Dont Reject Ho. Conclude at 5% significant level that there is insufficient evidence to suggest
that the new machine has produced steel rods that is not 50 cm
Assuming that the steel rods are normally distributed.
(ii)

Ho: = 50
H1: > 50

x o
At 1% significant level, reject Ho if P Z

x 50
PZ
0.99
0.460

60

x 50
2.32634
0.460
60

x 50.138 x 50.2

Page 7 of 10

0.01

2013 NYJC JC2 Prelim Exam 9740/2 Solutions


Qn
9(i)
9(ii)

10!
907200
2!2!
Method 1 (Complementation)
No of ways
No of ways the two Ts

without restriction are next to each other


9!
907200
2!
725760
Method 2 (Insertion method)
8! 9
725760
2! 2

9(iii)

X X X X X X X X
T

T
T

T
T

T
T

T
T

9(iv)

T
T
(Insertion method)
8!
6 120960
2!
Case 1: Last letter is S or T
8!
No of ways 2 40320
2!
Case 2: Last letter is M or C
8!
20160
No of ways 2
2!2!
Total number of ways 60480

Alternative method
Y________ _

10

6 8!
60480
No of ways
1 2!2!
(a) Let T s be the time taken to complete a race. I think 5 s is the wrong standard deviation.
Suppose T~N(10, 52).
Then P(T < 0) 0.0228 which is significantly large. However T is a non-negative quantity.
Page 8 of 10

2013 NYJC JC2 Prelim Exam 9740/2 Solutions


Qn

(b) (i) Now let T~N(10, 22). P(T < t) = 0.99 t 14.7
(ii) Let Y be the number of races out of 20 races that are less than t seconds. Y ~ B(20,0.99)
Required probability = P(Y 18)
= 0.0169

T T ... T5
2T . W ~ N(-10, 16.8)
(iv) Let W = 1 2
5
T T ... T5
2T ) = P(W > 0) 0.00735
P( 1 2
5
11

(i) Since r = 0.854, which is close to 1, it suggests a linear model is appropriate. However, the
scatter diagram shows the relationship between x and p to be non-linear.
(ii) ln p 12.465 0.19783x
where 12.5 and 0.198
(iii)

r = 0.99373 = 0.993

(3 s.f.)

Since r = 0.993 is very close to 1, it shows that there is a strong positive linear relationship
between ln p and x.
(iv) ln p 12.465 0.19783 32 6.1344
p e6.1344 0.0021669

The expected number of children with Down's Syndrome = 5000(0.0021669)


=10.835
= 10.8 (3 s.f.)
The estimated value is reliable since x = 32 lies within the data range of x.
12(i)

P 1 of 3 drivers choose C 1 P no one choose C 1 1 ar .


3

Page 9 of 10

2013 NYJC JC2 Prelim Exam 9740/2 Solutions


Qn
12(ii)

Taking the sum of the various probabilities, we have


a 1 r 3
1 r

5
a ar ar 2 1 , i.e.,
12

7
.
12

Simplifying, we have
12a 1 r 3 7 1 r 0

1 r 12a 1 r r 2 7 0

1 r 12ar 2 12ar 12a 7 0.

Since r 1 , 12ar 2 12ar 12a 7 0 , i.e.,

12a

4 12a 12a 7 0

144a 2 576a 2 336a 0


48a 7 9a 0.
7
Thus, 0 a .
9
12(iii) P 10 min before X enters 1 of 3 drivers choose C

P one car chose B one car chose C one car chose D


P two cars chose C one car chose D
P one car chose C two cars chose D
1 1 1 3! 1 1 1 3! 1 1 1
3!
3 6 12 2! 6 6 12 2! 6 12 12
11

.
288
12(iv)

P 10 min before X enters | 1 of 3 drivers choose C

P 10 min before X enters 1 of 3 drivers choose C


P 1 of 3 drivers choose C

11
288

3
1
1 1
6
33

.
364
We need to assume that the choice each driver makes is independent of each other.

Page 10 of 10

NATIONAL JUNIOR COLLEGE


PRELIMINARY EXAMINATIONS
Higher 2

MATHEMATICS

9740/01

Paper 1

02 September 2013
3 hours

Additional Materials: Answer Paper


List of Formulae (MF15)
Cover Page

0815 1115 hours

READ THESE INSTRUCTIONS FIRST


Write your name, registration number and subject class on all the work you hand in.
Write in dark blue or black pen on both sides of the paper.
You may use a soft pencil for any diagrams or graphs.
Do not use paper clips, highlighters, glue or correction fluid.

Answer all the questions.


Give non-exact numerical answers correct to 3 significant figures, or 1 decimal place in the case of
angles in degrees, unless a different level of accuracy is specified in the question.
You are expected to use a graphic calculator.
Unsupported answers from a graphic calculator are allowed unless a question specifically states
otherwise. Where unsupported answers from a graphic calculator are not allowed in a question,
you are required to present the mathematical steps using mathematical notations and not calculator
commands.
You are reminded of the need for clear presentation in your answers.
At the end of the examination, fasten all your work securely together.
The number of marks is given in the brackets [ ] at the end of each question or part question.
This document consists of 7 printed pages.

National Junior College


NJC 2013

9740/01/2013

[Turn over

Jacob wants to purchase 4 packets of cashew nuts, 6 packets of macadamia nuts and 7
packets of almonds. Based on the usual retail price, his total bill will be $57.05.
Supermarket A is having a sale and there is 30% off each packet of cashew nuts and a Buy
2 packets get 1 packet free promotion for macadamia nuts. After using a $10 voucher,
Jacobs bill is $33.05 if he made his purchase at Supermarket A.
Supermarket B is also having a sale and there is 20% off each packet of cashew nuts, a 2
packets for the price of 1 packet promotion for macadamia nuts, and 15 cents off each
packet of almonds. Jacobs bill at Supermarket B will exceed his final bill at Supermarket A
by $5.45.
Assume that the usual retail prices of nuts in both supermarkets are the same. Write down
and solve equations to find the usual retail price of each of the following: a packet of cashew
nuts, macadamia nuts and almonds.
[4]

Given that x is real, prove that 2 x 2 - x + 2 is always positive.


Solve the inequality

[1]

3x - 1
1
, leaving your answers in exact form.
2
2x - x + 2 x

[3]

Hence, without the use of a calculator, find the set of values of x for which

3 x -1
1
.
2
2x - x + 2 x

Express

[2]

n (1 - k ) - 1
in partial fractions and hence use the method of differences to find
n(n - 1)
1 n (1 - k ) - 1


n = 2 k n( n - 1)
N

in terms of constants N and k .

Explain why

n =2

NJC 2013

[3]

2n -1 ( n + 2 )
is a convergent series, and state its value.
3n [ n(1 - n )]

9740/01/2013

[3]

[Turn over

A convergent geometric progression has a positive first term a and common ratio r while an
arithmetic progression has a non-zero common difference d. The second and third term of
the geometric progression is the first and fourth term of the arithmetic progression
respectively. It is also given that the first term of the geometric progression is equal to the
sum of the first 4 odd-numbered terms in the arithmetic progression.
(i)

By expressing the first term of the arithmetic progression in terms of a and r, show
1
that r 2 = .
[4]
4

(ii)

It is given that the sum to infinity of the geometric progression is less than a.
(a)

Find the value of r, justifying your answer.

[2]

(b)

Explain if this implies that for all n, the sum of the first n terms of the
geometric progression must be less than a.

[1]

It is given that f ( x ) =
(i)

(ii)

x2 + x + a
, where a and b are constants.
x+b

Given that the graph of y = f ( x ) has a vertical asymptote x = -3 and passes


through the point ( 0, 2 ) , find the values of a and b .

[2]

On separate diagrams, sketch the graphs of


(a)
y = f ( x) ,

[3]

(b)

y2 = f ( x) .

[3]

(Your graphs should label clearly any axial intercepts and linear asymptotes.)

NJC 2013

9740/01/2013

[Turn over

6
C

River

10 m
A

q
100 m

In a fantasy land, a sheep at point A wishes to get to the greener grass at point B. In order to get
there, it has to swim across the still river at an angle q to get to a variable point C first before
walking to point B. The sheep can swim at a speed of 2 m s -1 in water and walk at a speed of

4 m s -1 on land.
(i)
(ii)

Find AC in terms of q .
[1]
Show that the total time taken, t seconds, for the sheep to get to point B from point A is
given by
5
t = 25 [3]
( cos q - 2 ) .
2sin q

(iii)

Using differentiation, find the shortest travelling time the sheep can take to get to point B
from point A.
[4]

Use the method of mathematical induction to prove that

r ( r + 1) = n + 1 for all positive


r =1

integers n.
Show that

[4]
x3 - x 2 + 1
1
.
= x+
x ( x - 1)
x ( x - 1)

[1]
r3 - r2 +1
.

r = 2 r ( r - 1)
20

Hence, use a non-calculator method to find

NJC 2013

9740/01/2013

[3]

[Turn over

The curve C has parametric equations


x = t 3 - t,

y = et ,

for t -1.

(i)

Sketch C, labelling all axial intercept(s).

[2]

(ii)

Find the equation of the tangent to the curve when t = 1 , leaving your answer in
terms of e.

(iii)

[3]

The normal to the curve C at point P ( p 3 - p,e p ) is parallel to the x-axis and
intersects the y-axis at point S ( 0, s ) , where s < 1. Find the exact value of p.

[2]

(iv)

Find the area of the region bounded by the curve C and the y-axis.

[3]

(a)

Find the general solution of the differential equation

( 4 - 2x)

d2 y
= 1,
dx 2

giving your answer in the form y = f ( x ) .


(b)

A water tank contains 500 litres of a solution with 15 kilograms of dissolved salt.
Pure water enters the tank at a rate of 20 litres per minute. The solution is drained
from the tank at the same rate. Let x kg be the amount of salt in the tank at time t
minutes.
(i)
(ii)
(iii)

NJC 2013

[3]

dx
x
=- .
dt
25
State an assumption required for this model to be more appropriate.
Find the amount salt in the tank after 30 minutes.
Show that

9740/01/2013

[1]
[1]
[4]

[Turn over

10

The function f is defined by

f:x

c
cx - d
, for x , x ,
d
dx - c

where c and d are fixed constants and c > d > 0 .


(i)

Sketch the graph of y = f ( x) , indicating the axial intercepts and the equations of any
asymptotes.

(ii)

[3]

Find f -1 ( x) and f 2 ( x) , stating clearly their domains. Hence, state the range of f 2 .
[5]

(iii)

On the same diagram in (i), sketch the graph of y = f 2 ( x) . Label your graph clearly.
[1]

11

(iv)

State the value of f 2013 (1) .

(a)

It is given that z = 3 + 2i is a root of the equation 2 z 3 + pz 2 + qz - 65 = 0 , where p and

[1]

q are real numbers.


(i)

Find the values of p and q.

[3]

(ii)

Without the use of a calculator, find the values of all the other roots of the

equation.
(b)

[3]

Solve the equation


z 4 = -1 + 3i .

Hence deduce the roots of the equation

[3]

3 + i w4 = -i .

[2]

[Express all answers in the form rei , where r > 0 and - < . Give r and in
exact form.]

NJC 2013

9740/01/2013

[Turn over

12

The equations of line l, planes 1 and 2 are given below.


3
l : r = g 2 , g

-4
1 : px + y + qz = 2 , where p and q are real constants.
1
0

2 : r = a 0 + b 1 , a , b


1
1

The

line

of

intersection

between

the

two

planes

1 and

has

equation

2 - 6l
r = 7l - 2 , l .

(i)

Find the values of p and q.

[3]

(ii)

Calculate the acute angle between the line l and plane 2 .

[3]

(iii)

Find the coordinates of the foot of the perpendicular from the point Q ( -3, -2, 4) to

(iv)

the plane 2 .

[3]

Hence or otherwise, find the vector equation of the reflection of the line l in 2 .

[4]

End of Paper

NJC 2013

9740/01/2013

NATIONAL JUNIOR COLLEGE


Preliminary Examinations
Higher 2
MATHEMATICS
Higher 2

9740/01
Paper 1

02 September 2013, Monday

0815 1115 hours

Candidate Name: ______________________

Registration No.:_____________

Subject Class: 2ma______/ 2IPma2_______

Subject Tutor: _______________


For office use

Question No.

over Page

Marks
Obtained

TOTAL
MARKS

10

10

10

11

11

12

13

INSTRUCTIONS TO CANDIDATES
Write your name, registration number, subject
class, subject tutors name and calculator model in
the spaces provided on the cover page and attached
it on top of your answer paper.
Circle the questions you have attempted and
arrange your answers in NUMERICAL ORDER.
Write your calculators model number(s) in the
box below.
Scientific Calculator Model:

Graphic Calculator Model:

Presentation
TOTAL
GRADE

NJC 2013

9740/01/2013

2
100

2013 SH2 H2 Math Prelim Paper 1 Solutions


1

Let x, y and z be the usual retail price of a packet of cashew nuts,


macadamia nuts and almonds respectively.
4 x + 6 y + 7 z = 57.05
4(0.7) x + 4 y + 7 z - 10 = 33.05 2.8 x + 4 y + 7 z = 43.05
4(0.8) x + 3 y + 7( z - 0.15) = 33.05 + 5.45 3.2 x + 3 y + 7 z = 39.55
By G.C.,
x = 3.5, y = 4.9, z = 1.95
Thus, the usual retail price of a packet of cashew nuts, macadamia nuts and
almonds is $3.50, $4.90 and $1.95 respectively.

1 15

2x - x + 2 = 2 x - +
4
8

15
( > 0)
8
2

OR
Discriminant = 1 16 = 15, and the coefficient of x 2 is positive. Thus,
2 x 2 - x + 2 is always positive.
OR
y = 2 x2 - x + 2

( 0.25,1.875)

3x - 1
1
, x0
2
2x - x + 2 x
3x - 1
1
2
- 0
2x - x + 2 x
3x 2 - x - 2 x 2 + x - 2

0
x (2x2 - x + 2)

x2 - 2
0 ( 2 x 2 - x + 2 > 0 )
x

x x- 2

)( x + 2 ) 0

- 2 x < 0 or x 2

Replace x by x .
3 x -1
1
, x0
2
2x - x + 2 x
\ - 2 x < 0 (N.A.) or x 2
\ x 2 or x - 2
3

n (1 - k ) - 1 1
k
= n(n - 1)
n n -1

k
1 n (1 - k ) - 1 N 1 1
=

n - 1
n = 2 k n ( n - 1)
n =2 k n
N

1
1
= n n -1
( n - 1) k
n = 2 nk
1
1
= 22k
k
1
1
+ 3- 2
3k
2k
1
1
+ 4- 3
4k
3k
+ ...
1
1
+
N -1
( N - 1) k
( N - 2 ) k N -2

1
1
N
Nk
( N - 1) k N -1

1
1
N
Nk
k

3
Let k = .
2
n
N
1 n (1 - k ) - 1


k n(n - 1)
n=2

- n - 1

2 2


3 n(n - 1)

2 ( n + 2)

3 2n(1 - n)

2n -1 ( n + 2 )
3n [ n(1 - n) ]

2n -1 ( n + 2 )
=
3n [ n(1 - n) ]

n=2
N

n=2
N

n=2
N

1
N

2
3

3
N
2
1
1
As N ,
0 and
0.
N
N
3

2
n=2

Thus

n=2

4(i)

2n -1 ( n + 2 )
2
=- .
n
3 [ n(1 - n) ]
3

First term of the arithmetic progression = ar


4
[ 2(ar ) + 3(2d )] = 4ar + 12d
2
a = 4 ( ar + 3d ) -- (Eq.1)
a=

ar 2 = ar + 3d
Take

-- (Eq.2)

Eq.2
, we have
Eq.1
ar 2
ar + 3d
=
a
4 ( ar + 3d )
r2 =

1
(shown)
4

Alternative Solution
In fact, you dont even have to make the observation that the first term of the
AP is ar. If you just let it be b, you will also easily arrive at the same solution:
a = 4 ( b + 3d ) -- (Eq.3)

ar 2 = b + 3d

-- (Eq.4)

Take

Eq.4
, we have
Eq.3
ar 2
b + 3d
=
a
4 ( b + 3d )
r2 =

4(ii)(a)

r2 =

1
(shown)
4

1
1
1
r=
or 4
2
2

Since S =

a
< a , we have
1- r

a
<a
1- r
1 - r > 1 ( a > 0 and 1 - r > 0)
r<0
1
Thus, r = 2
Alternative Solution 1
1
a
When r = , S =
= 2a < a
2
1
1-
2
2
1
a
When r = , S =
= a<a
2
1 3
1- -
2
1
Thus, r = - .
2
Alternative Solution 2
In order for sum to infinity to be less than a, there must be some negative terms
after the first. If all the terms had been positive, the sum of the terms will never
be less than a. Since the first term is positive, then r has to be negative in order
to create these negative terms. (If r had been positive, then all the terms will be
1
positive.) Hence, r = - .
2

4(ii)(b)

No, it does not imply that for all n, the sum of the first n terms of the GP must
be less than a.
a (1 - r n )
Counter Example: Since S n =
, when n =1, we have
1- r
a (1 - r1 )
S1 =
=a < a.
1- r
From the counter example above, we see that the statement is not true for n = 1,
hence not true for all n.

Note: The statement is actually true for all other values of apart from n = 1, but
that will need a separate proof to justify.
5(i)

x = -b = 3
b = -3

At ( 0, 2 ) ,
a
=2
b
a = 2 ( -3 ) = - 6
5ii(a)
For a = -6 and b = -3

y = x+4

2
-3

x=3

y=

5ii(b)

x2 + x - 6
x-3

For a = -6 and b = -3

-3

2
3

- 2

y2 =

x2 + x - 6
x-3

6(i)

AC sin q = 10
10
AC =
sin q

6(ii)

Time taken for sheep to cross the river =

10
5
=
2sin q sin q

distance along river travelled while crossing river


10
= AC cos q =
( cos q )
sin q

Distance CB = 100 -

10
( cos q )
sin q

10
( cos q )
sin q
time taken on land =
4
5
= 25 ( cos q )
2sin q
100 -

t = 25 6(iii)

5
5
5
= 25 ( cos q - 2 ) (Shown)
( cos q ) +
2sin q
sin q
2sin q

t = 25 - 2.5cot q + 5 cosecq

dt
= 2.5 cosec 2q - 5 cosecq cot q = 0
dq
cosecq - 2 cot q = 0
1 - 2 cos q = 0
1
p
cos q = q =
2
3

1.04

dt
dq
Sketch

OR from GC,

0.0418

d 2t
dq 2

q=
3

= 5.77 (>0)

Shortest travelling time:

p
3
0

1.05

0.0161

t = 25 -

5 3
5 3

cos - 2 = 25 - = 25 +
(or 29.3 )

3
2
2
3

2sin
3
5

Let Pn be the statement:

r ( r + 1) = n + 1 for all n

r =1

n = 1:
LHS =

1
1
=
1 (1 + 1) 2

RHS =

1
1
=
1+1 2

\ P1 is true.
Assume that Pk is true for some k + i.e.
k +1

To prove: Pk +1 is true i.e.

r ( r + 1) = k + 1
r =1

k +1

r ( r + 1) = k + 2
r =1

k +1

k
1
1
1
=
+

( k + 1)( k + 2 )
r =1 r ( r + 1)
r =1 r ( r + 1)

1
k
+
k + 1 ( k + 1)( k + 2 )

k ( k + 2) + 1
( k + 1)( k + 2 )

k 2 + 2k + 1
( k + 1)( k + 2 )

( k + 1)
=
( k + 1)( k + 2 )
2

k +1
k +2

Since P1 is true, and Pk is true Pk +1 is true , Pn is true for all n + .


x+

OR

x 2 ( x - 1) + 1 x 3 - x 2 + 1
1
=
=
x ( x - 1)
x ( x - 1)
x ( x - 1)

2
x 3 - x 2 + 1 x ( x - 1) + 1
=
x ( x - 1)
x ( x - 1)

x 2 ( x - 1)
1
1
+
= x+
x ( x - 1) x ( x - 1)
x ( x - 1)

r 3 - r 2 + 1 20
1
= r +

r ( r - 1)
r = 2 r ( r - 1)
r =2
20

20

20

1
r = 2 r ( r - 1)

= r +
r =2

19
19
1
( 2 + 20 ) +
2
s =1 ( s + 1)( s )

19
19 + 1
= 209.95
= 209 +

8(i)
e

x = t 3 - t,

y = et

e -1

8(ii)

dy
et
dy
dx
t
2
=
=e ;
= 3t - 1
dx 3t 2 - 1
dt
dt
dy
e
1
=
= e
dx t =1 3 - 1 2

t = 1 ( 0, e )
Equation of tangent: y - e =
8(iii)

e
e
x y = x+e
2
2

Normal is parallel to x axis means tangent is parallel to y axis so


3 p2 - 1 = 0 p = -

1
or
3

1
(N.A. since s < 1)
3

8(iv)

Required area
= -1 ( ln y ) - ln y dy +

e
1

( ln y )
e

- ln y dy

= 0.587 (3 s.f.)

OR
Required area
=

-1

(t

- t ) et dt +

(t
1

- t ) e t dt

= 0.587 (3 s.f.)
9(a)

(4 - 2x)

d2 y
=1
dx 2
3
-
2

d2 y
2 = (4 - 2x)
dx

1
-
2

dy ( 4 - 2 x )
=
+B
dx 1
- ( -2 )
2
1
-
2

dy

= (4 - 2x)
dx

y=

(4 - 2x)

9b(i)

1

2

1
( -2 )
2

y=-

+B

+ Bx + C

( 4 - 2 x ) + Bx + C

Since the in-flow and out-flow rates are the same, the volume of water in the
tank remains constant at 500 litres.
dx
= the rate of salt entering the tank - the rate of salt leaving the tank
dt
x
= 0( 20)
500
x
=(Shown)
25

9b(ii)

Assume that the concentration of the salt in the water is well-mixed and
uniform throughout the tank.

9b(iii)

x dx = - 25 dt
ln x = x = Ae

1
t +C
25

( x > 0)

1
t
25

t = 0, x = 15 A = 15
\ x = 15e

1
t
25

t = 30, x = 15e

1
( 30 )
25

\ x 4.52kg

10(i)

O
y = f ( x)

db
ca
0

Let y =

d
ca

y = f 2 ( x)

10(ii)

a
x= c
x =b
d

cx - d
. Then
dx - c
dxy - cy = cx - d
x ( dy - c ) = cy - d
cy - d
x=
dy - c

Thus, f -1 ( x ) =

a
y y= = c
bd

c c
cx - d

, Df -1 = R f = -, , .
d d
dx - c

f 2 ( x ) = ff ( x ) = ff -1 ( x ) = x .
c c

Domain of f 2 = Range of f 2 = -, ,
d d

c-d
= -1
d -c

10(iv)

f 2013 (1) =

11(a)(i)

Substituting z = 3 + 2i into 2 z 3 + pz 2 + qz - 65 = 0 ,
2 ( 3 + 2i ) + p ( 3 + 2i ) + q ( 3 + 2i ) - 65 = 0
3

Using GC,
( -18 + 92i ) + p ( 5 + 12i ) + q ( 3 + 2i ) - 65 = 0
( -18 + 5 p + 3q - 65) + ( 92 + 12 p + 2q ) i = 0

Comparing real and imaginary parts,


5 p + 3q = 83 .(1)
6 p + q = -46 (2)
Solving (1) and (2),
p = -17 and q = 56
11(a)
(ii)

Since the coefficients of the polynomial are all real, 3 - 2i is also a root.
2 z 3 - 17 z 2 + 56 z - 65 = [ z - (3 + 2i )][ z - (3 - 2i )] (2 z - k )
2 z 3 - 17 z 2 + 56 z - 65 = ( z 2 - 6 z + 13)(2 z - k )
Comparing constant term, k = 5.
The other roots are 3 - 2i and 2.5.
Alternative Solution to (a):
These two parts of the question can be done concurrently as follows:
Since the coefficients of the polynomial are all real, 3 - 2i is also a root.
2 z 3 + pz 2 + qz - 65 = [ z - (3 + 2i ) ][ z - (3 - 2i ) ] (2 z - k )
= ( z 2 - 6 z + 13)(2 z - k )
Comparing the constant term, -13k = -65 k = 5
The other roots are 3 - 2i and 2.5.
2 z 3 + pz 2 + qz - 65 = ( z 2 - 6 z + 13)(2 z - 5)
= 2 z 3 - 17 z 2 + 56 z - 65
Comparing coefficient of z and z2, we get p = -17 and q = 56 .

11(b)

z = -1 + 3i =
4

z=2

1
4

z=2

1
4

2p

+ 2 kp
i
3

,k
2e

1 2p

+ 2 kp
i
4 3
,
e

k = -2, -1, 0, 1

5p
p
p
2p
i 1 i 1 i
1 i

6
3
6

4
4
4
e
,2 e
,2 e
,2 e 3

3 + i w4 = -i
4

1
=
w

3 +i
-i

) = -1 +
5p

i
1
-1
w= = 2 4e
z

- 14

3i

,2 e

p
i
3

- 14

,2 e

p
i -
6

- 14

,2 e

2p
i -
3

12(i)

2
-6
p

Since r = -2 + l 7 , l lies on p 1 : r 1 = 2



0
1
q
-6 p
7 1 = 0 -6 p + 7 + q = 0

1 q
2 p
-2 1 = 2 2 p - 2 = 2

0 q
p = 2, q = 5

OR
-4 p
5 1 = 2 -4 p + 5 + q = 2

1 q
2 p
-2 1 = 2 2 p - 2 = 2

0 q
p = 2, q = 5

OR
1 0 -1
0 1 = -1

1 1 1

-1
so p 2 : r -1 = 0

1

p -1 1 + q
-6
1 -1 = - p - q = k 7


q 1 - p +1
1
1 + q = -6k , - p - q = 7 k , - p + 1 = k

2 p
-2 1 = 2 2 p - 2 = 2

0 q
p = 2, k = -1, q = 5
12(ii)

1 0 -1
0 1 = -1

1 1 1

-1
so p 2 : r -1 = 0

1

Required angle = sin -1

3 -1
2 -1

9
-4 1
= sin -1
87
3 -1
2 -1

-4 1

= 74.8 (1 d.p) or 1.31 (3 s.f)


12(iii)

-1
-3

Let OF = -2 + m -1 , where F is the foot of perpendicular from Q to 2 .




4
1

-3
- 1 -1
-2 + m -1 -1 = 0


4
1 1
3+ m + 2 + m + 4 + m = 0
m = -3
-3 -1 0
-2 - 3 -1 = 1

4 1 1
Coordinates of F is (0,1,1).
12(iv)

( -3, -2, 4) lies on line l .


(0, 0, 0) is the common intersection point of l and 2 .
Q ( -3, -2, 4)
(0, 0, 0)

F (0, 1, 1)

Q
1
OF = OQ + OQ '
2
0 -3 3

\ OQ ' = 2 1 - -2 = 4

1 4 -2

3
Equation of reflected line is r = t 4 , t .

-2

NATIONAL JUNIOR COLLEGE


PRELIMINARY EXAMINATIONS
Higher 2

MATHEMATICS

9740/02

Paper 2

17 September 2013
3 hours

Additional Materials: Answer Paper


List of Formulae (MF15)
Cover Page

1400 1700 hours

READ THESE INSTRUCTIONS FIRST


Write your name, registration number and subject class on all the work you hand in.
Write in dark blue or black pen on both sides of the paper.
You may use a soft pencil for any diagrams or graphs.
Do not use paper clips, highlighters, glue or correction fluid.

Answer all the questions.


Give non-exact numerical answers correct to 3 significant figures, or 1 decimal place in the case of
angles in degrees, unless a different level of accuracy is specified in the question.
You are expected to use a graphic calculator.
Unsupported answers from a graphic calculator are allowed unless a question specifically states
otherwise. Where unsupported answers from a graphic calculator are not allowed in a question,
you are required to present the mathematical steps using mathematical notations and not calculator
commands.
You are reminded of the need for clear presentation in your answers.
At the end of the examination, fasten all your work securely together.
The number of marks is given in the brackets [ ] at the end of each question or part question.
This document consists of 8 printed pages.

National Junior College


NJC 2013

9740/02/2013

[Turn over

Section A: Pure Mathematics [40 marks]

(i)

Using standard result given in the List of Formulae (MF15), find the Maclaurins
series for
, up to and including the term in
[2]

(ii)

Determine the set of values of x for which the expansion in part (i) is valid.

(iii)

Hence, or otherwise, obtain the series expansion of


the term in x 2 .

(a)

[3]

1 + 2x
up to and including
1 + x + x2
[2]

The complex number a has modulus r and argument q , where 0 < r < 1 and

0<q <

= 1 and arg( a ) + arg(b) = .


. The complex number b is such that
a
2

Let the points A, B, C and D represent the complex numbers a, b, a + b and

b
a*

respectively, where a * denotes the conjugate of a.


On a single clearly labelled Argand diagram, illustrate these four points.

(b)

[5]

The complex number z satisfies the relations z + 3 - 2i 13 and z z - 4i .


(i) Sketch, on an Argand diagram, the locus of points representing the complex
number z.
[3]
(ii) Find the range of values of z .

[2]

(iii) Find the largest value of arg( z + 10) , leaving your answer correct to 3 decimal
places.

NJC 2013

[3]

9740/02/2013

[Turn over

(i)

Use the substitution x = u + 1 to find

(ii)

Hence, find tan -1 ( x - 1) dx .

x
dx .
( x - 1) 2 + 1

[3]
[2]

.
2
R is rotated completely through 4 right angles about the x-axis to form a solid of revolution
of volume V. Show that
V = ( a + b ln 2 ) ,

The region R is bounded by the axes, the curve y = tan -1 ( x - 1) and the line y =

where a and b are exact constants to be determined.

[5]

A pyramid VOABC has a parallelogram base OABC and the vertex V is such that VC is
perpendicular to the base OABC. Relative to the origin O, the position vectors of the points
A and C are pi + 2 j - k and -4i - 2k respectively.
(a)

The pyramid has a base area of 6p.


(i) Show that p = 2.

[3]

(ii) Find the position vector of the point M on the line segment AB such that
AM : AB = 2 : 3.
[3]

(iii) The vector d is a unit vector in the direction of CV . Give a geometrical

interpretation of d AV .
[1]
(b)

NJC 2013

Let a, c and v be the position vectors of the points A, C and V respectively. Using the

fact that VC is perpendicular to both OA and OC , or otherwise, show that


[3]
v ( a + c ) = ( c a ) + q , where q is a real constant in terms of c .

9740/02/2013

[Turn over

4
Section B: Statistics [60 marks]

The Student Council of Nationally Junior College is organising a fun-fair to raise funds for
the CCAs. The College has 1500 students and the Council intends to survey a sample of 200
students to find out the kind of stalls they will like to have in the fun-fair.
The Student Councillors decide to conduct the survey by getting the first 100 male and 100
female students who enter the school on one particular day.
(i)
(ii)

Identify this sampling method and explain one disadvantage of this method used. [2]
Suggest how this method can be further improved.
[1]

The Student Councillors also considered an alternative approach to conduct the survey. As
the College has three different groups of CCAs: Sports, Aesthetics and Clubs, with the
number of students in each group as shown in the table below, the councillors decide to
randomly select 100 students from Sports, 60 students from Aesthetics and 40 students from
Clubs. It may be assumed that every student in the College has exactly one CCA.

(iii)

CCA Group

Number of Students

Sports

700

Aesthetics

500

Clubs

300

Explain, with numerical working, whether the Student Councillors are performing
stratified sampling.
[2]

Find the number of ways in which the letters of the word STATISTICS can be arranged if
(i)

the consonants (C, S, T) are not all together,

[3]

(ii)

all the vowels (A, I) cannot be next to one another.

[3]

How many ways can 3 letters be selected from the letters of the word STATISTICS (order
in which the letters are selected is not important)?
[3]

NJC 2013

9740/02/2013

[Turn over

In this question, you should state clearly the values of the parameters of any normal
distribution you use.
The Body Mass Index (BMI) is a measure of body fat based on height and weight that
applies to men and women. Typically, the BMI of eighteen year old well-built male and
female students are modelled as having normal distributions with means and standard
deviations as shown in the table.
Gender

(i)

Mean BMI

Standard deviation

Male

1.5

Female

25

1.2

Let M denote the BMI of an eighteen year old well-built male student. Given that
P ( M > 25 ) = P ( M < 31) ,
determine the value of m .

(ii)
(iii)
(iv)

[1]

Find the probability that the mean BMI of three randomly chosen male and five
randomly chosen female eighteen year old well-built students is more than 26.
[3]
Four eighteen-year-old well-built female students are randomly chosen. Find the
probability that exactly two of them have BMI less than 26.
[2]
State, with a reason, whether or not a normal model is likely to be appropriate for the
BMI reading of a combined group of male and female students in a school.
[2]

Find the number of ways to group 24 students into two teams of 12.

[2]

These 24 students enjoy either dancing or singing. Of these, 16 enjoy dancing and 13 enjoy
singing. A committee of 7 students is randomly selected from this group of students. Find
the probability that the committee consists of at least 4 students who enjoy both dancing and
singing.
[3]
Of the 14 girls in the group, 8 enjoy only dancing and 3 enjoy only singing. Given that 2
girls and 1 boy are randomly selected, what is the probability that the trio enjoy both
dancing and singing?
[3]
A student is randomly chosen. Let A be the event that a student is a girl and B be the event
that a student enjoys dancing.
Determine if A and B are independent.
[2]

NJC 2013

9740/02/2013

[Turn over

A new drug developed to reduce a smokers reliance on nicotine may reduce ones pulse
rate to dangerously low levels. To investigate the drugs effect on pulse rate, different
dosages of the drug were administered to six smokers, and the decrease in each smokers
pulse rate was recorded 30 minutes later.
Smoker

Drug dosage (x cm3)

2.0

1.5

3.0

2.5

4.0

3.5

Decrease in pulse rate


(y beats per minute)

14

18

16

21

20

(i)

Draw a scatter diagram to illustrate the data.

[1]

(ii)

Calculate the product moment correlation coefficient between x and y, and explain
why its value does not necessarily mean that the best model for the relationship
between x and y is y = a + bx.
[2]

(iii)

It is proposed that the decrease in pulse rate can be modelled by the formula

y = ln ( cx d ) , where c and d are constants. Explain why this is a better model.

[1]

(iv)

Calculate the least squares estimates of c and d for the model in (iii).

[2]

(v)

Use a regression line to give the best estimate that you can of the dosage of drug
administered when the decrease in pulse rate is 25 beats per minute. Comment on the
reliability of your answer.
[3]

NJC 2013

9740/02/2013

[Turn over

10

The table below shows the mean number of traffic accidents per km of each stretch of the
expressways: CTE, AYE and KPE, over a three-month period.

Mean number of traffic


accidents per km

(i)

CTE

AYE

KPE

10.58

5.70

3.00

State two conditions needed for the number of traffic accidents occurring in a
randomly chosen 1 km stretch of an expressway over a three-month period to be well
modelled by a Poisson distribution.
[2]

Assume that the number of traffic accidents occurring on each 1 km stretch of an


expressway over a three-month period follows a Poisson distribution.
(ii)

It is given that the probability that 35 traffic accidents occur on the whole stretch of
KPE (d km) over a three-month period is 0.0663. Write down an equation for d, and
solve it numerically given that d > 11.5, leaving your answer to 3 decimal places. [2]

(iii)

The Transport Authority records data on the number of traffic accidents occurring on
the whole stretch of CTE (15.5 km) over a three-month period. A random sample of
70 such recordings was collected. Find the probability that the mean number of
traffic accidents from the sample is no less than 165.
[2]

(iv)

In a particular three-month period, the number of traffic accidents occurring on a


randomly chosen 2 km stretch of CTE and AYE are denoted by the random variables
U and V respectively.
State the approximate distribution of V U, and hence find an approximate value for
[3]
P(0 < V - U 10) .

NJC 2013

9740/02/2013

[Turn over

11

A manufacturer claims that the rice from his factory is packed in bags with a mean mass of
10 kg each.
(i)

A random sample of 8 bags are examined and the mass, x kilograms, of the contents
are summarised by:

x = 76.6 and

Test, at 5% level of significance, whether the manufacturer is overstating the mean


mass of rice in a bag, stating an assumption that has to be made for the test to be
valid.
[6]
(ii)

Another random sample of 50 bags are now examined and the sample variance is
found to be 0.461. Calculate the set of values of x for the test to conclude, at 2%
level of significance, that the mean mass of rice in a bag is not 10 kg.
Explain why the assumption made in part (i) is not necessary in this context.

End of Paper

NJC 2013

9740/02/2013

[4]

NATIONAL JUNIOR COLLEGE


Preliminary Examinations
Higher 2
MATHEMATICS
Higher 2

9740/02
Paper 2

17 September 2013, Tuesday

1400 1700 hours

Candidate Name: ______________________

Registration No.:_____________

Subject Class: 2ma______/ 2IPma2_______

Subject Tutor: _______________


For office use

Question No.

over Page

Marks
Obtained

TOTAL
MARKS

13

10

10

10

10

11

10

INSTRUCTIONS TO CANDIDATES
Write your name, registration number, subject
class, subject tutors name and calculator model in
the spaces provided on the cover page and attached
it on top of your answer paper.
Circle the questions you have attempted and
arrange your answers in NUMERICAL ORDER.
Write your calculators model number(s) in the
box below.
Scientific Calculator Model:

Graphic Calculator Model:

Presentation
TOTAL
GRADE

NJC 2013

9740/02/2013

2
100

2013 SH2 H2 Math Prelim Paper 2 Solutions


1(i)

Using standard series, we have


y = ln (1 + x + x 2 )
= ( x + x2 ) -

2
3
1
1
x + x 2 ) + ( x + x 2 ) + ...
(
2
3
1
1
= ( x + x 2 ) - ( x 2 + 2 x 3 + x 4 ) + ( x 3 + ... ) + ...
2
3
1
1
= x + x 2 - x 2 - x 3 + x 3 + ...
2
3
1
2
= x + x 2 - x 3 + ...
2
3

1(ii)

-1 < x + x 2 1
x + x2 + 1 > 0

and

and

x + x2 - 1 0
-1 - 5
-1 + 5
x
2
2

-1 - 5
-1 + 5
(or - 1.62 x 0.618)
x
2
2

1(iii)
Observe that
Therefore

1 + 2x
d
1
2

= x + x 2 - x 3 + ... = 1 + x - 2 x 2 + ...
2
1+ x + x
dx
2
3

OR
-1
1 + 2x
= (1 + 2 x ) (1 + x + x 2 )
2
1+ x + x

( -1)( -2 ) x + x 2 2 + ...
= (1 + 2 x ) 1 - ( x + x 2 ) +
(
)
2!

= (1 + 2 x ) 1 - x - x 2 + x 2 + ...
= 1 - x + 2 x - 2 x 2 + ...
= 1 + x - 2 x 2 + ...

2(a)
Im

2r sin q C
B

-q

D
1

A
r

Re

b
b
= =1
*
a
a
b
arg * = arg(b) + arg( a ) =
a
2b(i)

Im

Locus of z

z = z - 4i

( -3,2 )
Q

P
10

z + 3 - 2i = 13
2b(ii)

2 z 2 13

2b(iii)

PQ = 72 + 22 = 53
Largest value of arg( z + 10)
13
2
= tan -1 + sin -1

7
53
= 0.796 (3 d.p.)

Re

3(i)

Let x = u + 1 . Then
x

( x - 1)

+1

dx =

dx
=1.
du
u +1
du
u2 +1
u
1
du + 2
du
u +1
u +1
2

1
ln ( u 2 + 1) + tan -1 ( u ) + c
2
1
= ln ( x - 1) 2 + 1 + tan -1 ( x - 1) + c
2
=

3(ii)

tan

-1

x
dx
1 + ( x - 1) 2
1
= ( x - 1) tan -1 ( x - 1) - ln ( x - 1) 2 + 1 + c
2

( x - 1) dx = x tan -1 ( x - 1) -


2
V =
(2) - 1
2

tan -1 ( x - 1)

) dx
2

2

1

2
-1
=
(2) - ( x - 1) tan ( x - 1) - ln ( ( x - 1) + 1)
2

1
2
1
2

= - (2 - 1) tan -1 (2 - 1) + ln ( 2 - 1) + 1 + 0 - 0
2
2

= - + ln ( 2 )
2 4 2

= + ln 2
4 2

1
1
\a = , b =
4
2

4a(i)

Area of parallelogram OABC = 6 p


p -4
2 0 = 6p

-1 -2

-4
2 p + 4 = 6 p

16 + ( 2 p + 4 ) + 64 = 36 p 2
2

80 + 4 p 2 + 16 p + 16 = 36 p 2
2 p2 - p - 6 = 0

( 2 p + 3)( p - 2 ) = 0
p=4a(ii)

3
(N.A.) or 2.
2

-4 2 -2

OB = 0 + 2 = 2
-2 -1 -3

-2 2
2 2 + 2
-3 -1

OM =
3
-2
1
= 6
3
-7

4a(iii)

d AV

is the length of projection of AV onto AC


Or is the perpendicular distance from A to CV
Or is the length of AC
4(b)


VC ^ OA (c - v ) a = 0
(c a) - ( v a) = 0 - - - (1)


VC ^ OC (c - v ) c = 0
(c c ) - ( v c ) = 0
2
c - ( v c) = 0
- - - (2)

(1) + (2):

( c a) - ( v a) + c - ( v c ) = 0
2
( v a) + ( v c ) = ( c a) + c
2
v ( a + c ) = ( c a) + c
2

5(i)

Quota sampling.
Disadvantages:

5(ii)

5(iii)

6(i)

6(ii)

6(iii)

Selection is biased as those who come to school later have no chance of


being selected and hence their views will not be considered.

It does not provide a fair representation of the distribution of the preference


of the stalls as too many students from one particular level may be selected.

Survey students over the entire day rather than getting the first 200 students.

Random selection of the 100 male and 100 female students.

Using stratified sampling (or any other method that resembles stratified
sampling i.e. proportionate and random sampling)

The method is not stratified sampling as the number of students surveyed in


each group is not proportional to the actual proportion of students in Sports,
Aesthetics and Clubs. The councillors should instead randomly select 93
students from Sports, 67 students from Aesthetics and 40 students from Clubs.

Number of ways =

Number of ways =

7! 8 3!
= 23520
3!3! 3 2!

Case I: All identical letters


Number of ways = 2
Case II: 2 identical letters
3 4
Number of ways = = 12
1 1

Case III: All distinct letters


5
Number of ways = = 10
3
Therefore total number of ways = 24
7(i)

m=

25 + 31
= 28
2

7(ii)

Let F denote the BMI of an eighteen year old well-built female student.
Let X =

M 1 + M 2 + M 3 + F1 + F2 + F3 + F4 + F5
13.95

~ N 26.125,
.
8
64

P ( X > 26 ) 0.60555
= 0.606 (3 s.f.)
7(iii)

F ~ N ( 25,1.22 )
Required probability
2
2 4
= P ( F > 26 ) P ( F < 26 )
2
= 0.156 (3 s.f.)

OR
Let Q be the random variable for the number of eighteen-year-old well-built
female students, out of 4, that have BMI less than 26.
Q ~ B ( 4,0.79767 )
P ( Q = 2 ) = 0.156 (3s.f.)
7(iv)

As the combined BMI have more than one maximum point, it does not follow
the characteristic of a normal distribution that it has only one maximum point
only. (Bimodal)
OR
The distribution does not follow a symmetrical bell-shaped curve.

Number of groupings

24
12
= = 1352078
2!
8
Singing

Dancing
11

Required probability

+
5 19 5 19
4 3 5 2 1
(or 0.0145)
= =
69

24
7

OR

5 4 3 2 19 18 17 7!
5 4 3 2 1 19 18 7!
1

+
=
24 23 22 21 20 19 18 4!2! 24 23 22 21 20 19 18 5!2! 69

8
Singing

Dancing
8G

3G

3G

Required probability

P ( 2G and 1B enjoy both )


P ( 2G and 1B are selected )


3 2
21
3
(or 0.00659)
= =
455
14 10
2 1

OR
8

3 2 2
3
=
14 13 10 455

P(A) =

, P(B) =

9(i)

y
(4.0, 21)

(1.5, 9)

9(ii)

From GC, r = 0.969 (3 s.f.)

9(iii)

Although r is close to 1, the scatter diagram shows a clear non-linear


relationship between x and y.
For y = ln ( cx d ) = ln( c ) + d ln( x ) , r = 0.992, which is closer to 1 and hence it

shows a stronger linear relationship between y and ln(x). Thus, y = ln ( cx d ) is


the better model.
OR

9(iv)

The scatter diagram shows that as x increases, y increases at a decreasing rate.


This trend better fits a logarithmic curve.
From GC, y = 4.84195 + 11.98564 ln( x ) .
\ ln( c ) = 4.84195 c = 127 (3 s.f.)
d = 12.0 ( 3s.f.)

9(v)

25 = 4.84195 + 11.98564 ln( x )


x = 5.38 cm3 ( 3s.f.)

10(i)

The estimation is unreliable as the value of y (=25) lies outside the data range
[9, 21].
1. Traffic accidents occur independently of one another on the 1 km stretch of
the expressway over a three-month period.
2. The mean number of traffic accidents per km stretch of the expressway
over a three-month period is a constant.
3. Traffic accidents occur randomly on the 1 km stretch of the expressway
over a three-month period.

10(ii)

Let K be the random variable for the number of traffic accidents occurring on
the whole stretch of KPE over a three-month period.
K ~ Po(3d )
P( K = 35) = 0.0663

e -3d (3d )35


= 0.0663
35!

y = 0.0663

y=

e -3d (3d )35


35!

12.007

From GC, d = 12.007 (3 d.p. , since d > 11.5)


10(iii)

Let C be the random variable for the number of traffic accidents occurring on
the whole stretch of CTE over a three-month period.
C ~ Po(163.99)
Since sample size = 70 is large ( > 30), by CLT,

C=

C1 + C2 + ... + C70
163.99

~ N 163.99,
approximately.
70
70

P(C 165) = 0.255 (3 s.f )


OR
C ~ Po(163.99)
C ~ N (163.99,163.99 ) approx

C=

C1 + C2 + ... + C70
163.99

~ N 163.99,
approx
70
70

P(C 165) = 0.255 (3 s.f )


OR
Let T = C1 + C2 + ... + C70 ~ Po (11479.3) .
P(C 165)
= P(T 70(165))
= 1 - P(T 11549)
= 0.256 (3 s.f )
OR
Let T = C1 + C2 + ... + C70 ~ Po (11479.3) .
T ~ N (11479.3,11479.3) approx

P(C 165)
= P(T 70(165))
= 1 - P(T 11549)
= 1 - P(T 11549.5)
= 0.256 (3 s.f )
10(iv)

U ~ Po(21.16)
Since l = 21.16 (>10), U ~ N(21.16, 21.16) approximately.
V ~ Po(11.4)
Since l = 11.4 (>10), V ~ N(11.4,11.4) approximately.
\V - U ~ N( -9.76,32.56) approximately.
P(0 < V - U 10)
= P(0.5 V - U 10.5)
= 0.0359 (3 s.f )

11(i)

Since

, then

81
2
s 2 = ( 736.5) - ( 9.575) = 0.43643 (5 d.p.)
78

Level of significance: 5% (lower-tailed)

Assuming that X follows a normal distribution and that

is true, we have

From GC, p-value = 0.0558 > 0.05


There is insufficient evidence at 5% level of significance to conclude that the
manufacturer has overstated the mean mass of rice in a bag.
11(ii)

s2 =

50
( 0.461) = 0.47041 (5 d.p.)
49

Level of significance: 2% (two-tailed)


Since sample size is large, then by Central Limit Theorem,
approximately,
i.e.

In order to reject

, we need to have

x - 10
< -2.40489 x < 9.76 or
s
50
x - 10
> 2.40489 x > 10.3
s
50
There is no need to assume that X is normally distributed as the sample size is
sufficiently large to use Central Limit Theorem to approximate
normal.
OR
We can also have

Then to reject

, we need to have

x - 10
< -2.32635 x < 9.77 or
s
50
x - 10
> 2.32635 x > 10.3
s
50

to be

MERIDIAN JUNIOR COLLEGE


JC2 Preliminary Examination
Higher 2

___________________________________________________________________

H2 Mathematics

9740/01

Paper 1

18 September 2013

3 Hours
Additional Materials:

Writing paper
List of Formulae (MF 15)

___________________________________________________________________
READ THESE INSTRUCTIONS FIRST
Write your name and civics group on all the work you hand in.
Write in dark blue or black pen on both sides of the paper.
You may use a soft pencil for any diagrams or graphs.
Do not use staples, paper clips, highlighters, glue or correction fluid.

Answer all the questions.


Give non-exact numerical answers correct to 3 significant figures, or 1 decimal place in the case of
angles in degrees, unless a different level of accuracy is specified in the question.
You are expected to use a graphic calculator.
Unsupported answers from a graphic calculator are allowed unless a question specifically states
otherwise.
Where unsupported answers from a graphic calculator are not allowed in a question, you are
required to present the mathematical steps using mathematical notations and not calculator
commands.
You are reminded of the need for clear presentation in your answers.

At the end of the examination, fasten all your work securely together.
The number of marks is given in brackets [ ] at the end of each question or part question.

___________________________________________________________________
This document consists of 7 printed pages and 1 blank page.

[Turn over

BLANK PAGE

MJC/2013 JC2 Preliminary Examination/9740/01

(i) Show that 4 x2 20 x 35 is always positive for all real values of x.

[1]

(ii) Hence, without using a calculator, solve the inequality

x2 2 x 3
0.
4 x3 20 x 2 35 x
(iii) Use your answer to part (ii) to solve the inequality

[3]

x2 2 x 3
4 x 20 x 2 35 x
3

0 . [2]

A factory wishes to dump 150 kg of a particular waste product into a local stream at
the beginning of each week. The flow of the water removes 100 p % of this
waste product from the stream each week. However, marine experts calculated that
if the amount of waste product present in the stream reaches 240 kg, there will be a
serious threat to aquatic life.
(i) Write down a recurrence relation for un , where un denotes the amount of
waste product present in the stream at the beginning of the nth week, where

, after the waste product has been dumped.

[1]

(ii) State, in the context of the question, the range of values of p such that the
sequence converges.

[1]

For parts (iii) and (iv), let p 40.

(iii) Express un in the form of a b k n , where a, b and k are constants to be


determined.

[3]

(iv) Determine, with justification, if the company should be allowed to dump this
waste indefinitely.

[2]

MJC/2013 JC2 Preliminary Examination/9740/01

[Turn over

Let f x

1
.
x 1

(i) Show that the series expansion of f x in descending powers of x, up to and


including the third term, is x

1
3

1 4 2 7
x 3 x 3.
3
9

[3]

Denote the answer in part (i) by h x .


(ii) By evaluating h 8 , find an approximation for

7 as a fraction in its lowest

form.

[2]

(iii) Given that x

, find the minimum value of x such that the value of

h x f x is less than 0.001.

[2]

(i) Prove by induction that, for every positive integer n,

12 22 32 42 ... 1
(ii) Hence for N , M

n 1

n2 1

n n 1
.
2

[4]

and M N , find

2 N 1 2 N 2 2 N 3 2 N 4
2

n 1

... 2M 1 2M

in terms of M and N.

MJC/2013 JC2 Preliminary Examination/9740/01

[3]

The diagram below shows a parallelepiped OBDCAPQR. Referred to the origin O,


the points A, B and C are such that OA a , OB b and OC c . The point E lies in
the plane OBDC such that it is directly below the point P.
[A parallelepiped is a 6-faced polyhedron whose faces are parallelograms lying in
pairs of parallel planes.]
Q

P
R

A
B

E
O

(i) State the normal of the plane OBDC. Hence, by considering the triangle BEP,
find the length of PE. Give your answers in terms of a, b and c .

[2]

(ii) Show that the volume of the above parallelepiped is given by a b c .

[1]

(iii) The vectors a, b and c are now given by


a 10i 3j , b 2i 3k and c pi j 5k ,

where p is a constant.
Given that the volume of the above parallelepiped is 18 units3, find the
possible value(s) of p.

[3]

A student claims that the following property holds:


a b c b a c

for any non-coplanar vectors a, b and c .


Do you agree? Justify your answer.

[1]

MJC/2013 JC2 Preliminary Examination/9740/01

[Turn over

The functions f and g are defined as follows:

f :x

x 2 ,

x , x 2,

g:x

ln 1 x ,

x , x 1,

where is a constant.
(i) Given that gf exists, find the largest value of .

[2]

In the rest of the question, use the value of found in part (i).
(ii) On the same diagram, sketch the graphs of y f x , y f 1 x and
y ff 1 x , showing clearly the relationship between the graphs.

(iii) Hence find the exact solution of f x f 1 x .

[3]
[3]

A tank contains 2 m3 of water initially. Water flows into the tank at a constant rate
3 1
of 4 m s and flows out at a rate which is proportional to the amount of water

V m3 in the tank. The rate of change of volume of water in the tank is equal to
3 1
1 m s at the instant when V = 6. By setting up and solving a differential equation,

show that V 8 6e0.5t .

[7]

(i) What will happen to the volume of water in the tank eventually?

[1]

(ii) Sketch a graph of V against t.

[2]

The curve C1 has equation y 3


(i) Express y in the form 3

x
.
x 2x 8
2

A
B

and show by differentiation that C1 has


xc xd

no stationary points.

[3]

(ii) Sketch C1, stating the axial intercepts and the equations of any asymptotes. [3]
(iii) Find the exact area bounded by the curve C1, the line x 2 and the axes.
(iv) The curve C2 has equation

x 5
32

[3]

y2
1 . Sketch C2 on the same diagram
22

as C1 and find the coordinates of any points of intersection between C1 and C2.
[2]
MJC/2013 JC2 Preliminary Examination/9740/01

(a) Find the exact value of p such that

1
p

1
dx
1 p2 x2

(b) The curve C has equation y

e2

ln x dx .

x
.
1 x2

(i)

Sketch the curve C.

(ii)

Use the substitution u x 2 to find

[1]

1 x dx , for n 0 .
n

(iii)

[5]

[3]

Hence find the exact volume of revolution formed when the region
between the curve and the positive x-axis is rotated completely about the
x-axis.

10

[2]

(a) A geometric series has common ratio r, and an arithmetic series has first term a
and common difference d, where a and d are non-zero. The first three terms of
the geometric series are equal to the seventh, fourth and second terms
respectively of the arithmetic series.
(i)

Find d in terms of a.

(ii)

Deduce that the geometric series is convergent and find, in terms of a,


the sum to infinity.

[3]

[5]

(b) A student wants to save $9900 for an overseas trip. On the first day of each
month, she puts $200 into a bank account which pays compound interest at a
rate of 2% per month on the last day of each month. After how many complete
months will the total in the account first exceed $9900?

[4]

MJC/2013 JC2 Preliminary Examination/9740/01

[Turn over

11

(a) Show that

1 i tan
i 2
e where .
2
2
1 i tan
11

1 i 3
i
Hence find the value of
, giving your answer in the form re
1 i 3
where r 0 and .

(b) (i)

[5]

Find all the fifth roots of unity, giving your answers in the form rei
where r 0 and .

[3]

(ii)

Show all the roots on an Argand diagram.

[2]

(iii)

Two roots from part (i) are represented by z1 and z2 such that

0 arg z1 arg z2 . Draw the locus w z1 w z2

on your

Argand diagram. Hence find the possible values of arg w given that
w 0.

MJC/2013 JC2 Preliminary Examination/9740/01

[4]

2013 MJC H2 MATH (9740) PAPER 1 JC 2 PRELIMINARY EXAM SUGGESTED SOLUTIONS


Qn
1
(i)

Solution

Inequalities
4 x 2 20 x 35

4 x 2 5 x 35
2

5 25
4 x 35
2
4

2 x 5 10 0,
2

Alternative Method
2
Discriminant 20 4 4 35

(ii)

160 0
Since the coefficient of x2 is positive and discriminant < 0, 4 x2 20 x 35 is always
positive for all real values of x.
x2 2 x 3
0
4 x3 20 x 2 35 x
x 3 x 1 0
x 4 x 2 20 x 35

Since 4 x 2 20 x 35 0 for all real values of x,

x 3 x 1 0
x

1
(iii)

1 x 0 or x 3
x2 2 x 3
3

4 x 20 x 2 35 x

+
3

Replace x by x .

1 x 0 (N.A since x 0) or x 3
x 3 x 3 or x 3.

Qn
2
(i)
(ii)
(iii)

Recurrence Relations
p
un
un1 150, n 2, u1 150.
100
p
0
1 0 p 100
100
u2 0.4 u1 150 150 0.4 1

Solution

u3 150 0.42 0.4 1

u4 150 0.43 0.42 0.4 1

un 150 0.4n1 0.4n2


un

150 1 0.4n

0.42 0.4 1

1 0.4
un 250 1 0.4n 250 250 0.4n
a 250, b 250, k 0.4
(iv)

As n ,0.4n 0. un 250.
Since as n , un 250 240 , it will pose a serious threat to aquatic life, hence the
company should not be allowed to dump this waste indefinitely.

Qn
3
(i)

Solution
Maclaurins Series and Binomial Theorem
1
f x 3
x 1

x 1
x

1
3

1
3

1
3

1
3

1
1
x

1
3

1 4
1 1 3 3 1 2


2!
x
3 x

1 1 2 2
1 x x
9
3

1 4 2 7
x 3 x 3 Shown
3
9
1
4

1
2 7
h x x 3 x 3 x 3
3
9
1
4

1
2 7
h 8 8 3 8 3 8 3
3
9
1
1
2

2 3 16 9 128
x

(ii)

1
3

301
576

1
1
3
8 1
7
576
3
7
301

f 8

(iii)

Using GC,
When x 4 , h x f x 0.0021546 0.001
When x 5 , h x f x 0.00097182 0.001
Therefore, the minimum value of x is 5.
Alternative Method
Using GC,
x 4.96 3 s.f.

y
y h x f x 0.001

Therefore, the minimum value of x is 5.

4.96

Qn
4
(i)

Solution

Mathematical Induction
Let Pn be the statement
n

r 1

r 1

r 2 1

n 1

n n 1
for all n
2

[Alternative: 12 22 32 42 ... 1

n 1

n2 1

n 1

When n = 1,
11
LHS = 1 12 1.
RHS = 1

11

P1 is true.
k

r 1

r 1

.]

r 2 1

k 1

k k 1
----- (*)
2

[Alternative: 12 22 32 42 ... 1
To prove Pk 1 is also true i.e.

k 1

k 1

r 1

r 1

[Alternative: 12 22 32 42 ... 1

r 2 1

k 2

LHS 1

r 1

k k 1
.]
2
k 2 k 1 k 2

k 2 1

k 1

When n k 1,
k 1

11 1
1 LHS.
2

Assume that Pk is true for some k


i.e.

n n 1
for all n
2

k 1

2
1

k 2

k 1 k 2 . ]
2

r2

r 1

r 1
k 11
2
1 r 2 1
k 1
r 1

k 1 k k 1
k 2
2
1
1 k 1 from (*)
2
k

1 k 1 k

k 1

1 k 1

k 1

1 2 k 1

k 2

1 k 1 k 2

k 2

RHS
Thus Pk is true Pk 1 is true.
Since P1 is true and Pk is true Pk 1 is true, by mathematical induction, Pn is true for

all n

(ii)

2 N 1 2 N 2
2

2M

r 2 N 1
2M

r 1

r 1

2 M 1

r 1

r2

... 2 M 1 2 M
2

M N
2N

r 2 1

r 1

r2

r 1

2M 2M 1

2
N 2 N 1 M 2M 1

2 N 1

2 N 2 N 1
2

Qn
5
(i)

(ii)

Vectors 1
Normal of plane OBDC = b c
a b c
PE
bc
Volume of the parallelepiped
= Base area height
= Area of OCDB PE
a b c
bc
bc

Solution

a b c
(iii)

a b c 18
10 2 p


3 0 1 18
0 3 5


10 3

3 3 p 10 18
0 2

30 9 p 30 18
p 2

Yes since
a b c b a c Volume of the given parallelpiped

Qn
6
(i)
(ii)

Solution

Functions
Rf , Dg (,1
Since Rf Dg , largest =1.

y f -1 ( x)

(1, 1)

(2, 1)
3

y ff -1 ( x)

1
3

(1, 2)

y = f(x)

(iii)

f x f 1 x
1 x 2 x
2

x2 5x 3 0
x

5 13
2
5 13

5 13
is rejected as x 2
2

Qn
7

Solution
DE (include sketching of graphs and interpretation)
dV
4 kV
dt
When V 6,
1 4 k (6)

dV
1,
dt

3 = k (6)
k 0.5
dV
4 0.5V
dt
1
4 0.5V dV 1 dt
1
ln 4 0.5V t c

0.5
ln 4 0.5V 0.5t 0.5c
4 0.5V e 0.5t 0.5c
4 0.5V e 0.5t 0.5c
4 0.5V Ae 0.5t

where A e 0.5c

When t 0, V 2
4 0.5(2) A
A3
4 0.5V 3e 0.5t
0.5V 3e 0.5t 4
V 8 6e 0.5t
(i)

As t , e0.5t 0, V 8 .
Therefore, the volume of water in the tank will increase and converge to 8 m3 .

(ii)

V
V=8

V 8 6e0.5t
t

Qn
8
(i)

Solution
Graphing and Finding Area under Graph
x
y 3 2
x 2x 8
2
1
3

3 x 4 3 x 2

dy
2
1

2
2
dx
3 x 4 3 x 2

(ii)

0
Therefore, C1 has no stationary points.
y
y 3

y 3

3
2.12

x 2

(iii)

Required area 3
0

2
1

3 x 4 3 x 2

3.78

x4

2
1
dx

3 x 4 3 x 2
2

2
1

3x ln x 4 ln x 2
3
3

0
2
1
1

6 ln 2 ln 4 ln 4 ln 2
3
3
3

1
1
1
or
6 ln 2 units 2
6 ln 2 ln 4 units 2
3
3
3

(iv)

y
y 3

2
1

3 x 4 3 x 2

2.12

y 3

3.78

2
x
5

x 5
32

x 2

y2
1
22

x4

Using G.C., the coordinates of the points of intersection are (3.50,1.73) and (3.86,
1.85).

Qn
9
(a)

Solution
Techniques and Application of Integration
1
p

1
dx
1 p2 x2

e2

ln x dx

e2 1
1
p
e2
1
tan
px

x
ln
x

1 x x dx
1

1
e2
tan 1 1 tan 1 0 e 2 ln e 2 x 1
p
1
2
2
2e e 1
p 4

e2 1

4p

4 e 2 1

(b)(i)

0.577, 0.570
x

y0

(b)(ii)

x0u 0
x n u n2
du
2x
dx

u x2

1 x2

n2
0

dx

1 u
2

1
du
2

1 n

1 u 2 du
2 0
2
1
1 n
1 u
0
2
1
1
1

2 1 n2
Volume of the revolution

(b)(iii)

x
1 x2

dx
lim
n
2 2

0 1 x

1
1
(1 0) units3
2
2

Qn
10
(a)(i)

Solution

APGP
Terms of the G.P.
a 6d , a 3d , a d ,

Since the G.P. has a constant common ratio r,


ad
a 3d

r
a 3d a 6d
a 3d a d a 6d
2

a 2 6ad 9d 2 a 2 7ad 6d 2
3d 2 ad 0

d 3d a 0
a
or d 0 (rejected since d 0)
3
ad
r
a 3d
a
a
3

a
a 3
3
4a

3

2a
2

d
(a)(ii)

2
1 , the geometric series is convergent.
3
a 6d
Sum to infinity =
2
1
3
a
a 6
3 9a

2
1
3

Since r

(b)

200(1.02)n 200(1.02)n 1

200(1.02) 9900

1 1.02
200(1.02)
9900
1 1.02
Using the G.C.,
n

When n 34, amount of money is 9798.9 9900


When n 35, amount of money is 10199 9900
Hence, the total will first exceed $9900 after 35 complete months.

OR

200(1.02)n 200(1.02)n 1

200(1.02) 9900

1 1.02n
200(1.02)
9900
1
1.02

Using the G.C.,


n 34.255 (5s.f.)
Hence, the total will first exceed $9900 after 35 complete months.

Qn
11
(a)

Solution

Complex 2 and 3
1 i tan
LHS
1 i tan
sin
1 i
cos

sin
1 i
cos
cos i sin

cos i sin
e i
i
e

i 2

RHS (shown)

11

1 i 3

1 i 3
Let

11

11
i 2 3
1 i 3

e
1

i
3

e
e

(b)(i
)

2
i

z5 1
z5 e

i 0 2 k

ze

(ii)

22
i

ze

, where k 2, 1, 0,1, 2

2 k
i

5
4
i

,e

2
i

i0

,e ,e

2
i

,e

4
i

Locus of w

z2 e

4
i

(iii)

z1 e

2
i

2
5

4
i

and z2 e

z1 e

2
i

z3 e

Im

z5 ei0
2
i

z4 e
4
i

Re

w z1 w z2
locus of w is perpendicular bisector of the line segment joining z1 and z2
2 3
arg w

5 5
5
2
Or arg w
5

MERIDIAN JUNIOR COLLEGE


3

JC2 Preliminary Examination


Higher 2 Paper 2

Section A: Pure Mathematics [40 marks]


1

A graphing calculator is not to be used in answering this question.


One root of the equation x4 ax3 5x2 x 10 0 , where a is real, is x 1 2i . Find the
value of a and the other roots.
[5]
Hence find the x-coordinates of the point(s) of intersection between the graphs of
[3]
y 5 x 2 2 x 4 and y x3 3x 1 .

2
y
y f x

y=2

(1, 6)

x=3

The diagram shows the graph of y f ( x) . The curve passes through the origin and the
point 2,0 and has a turning point at (1, 6). The equations of the asymptotes, also shown
in the diagram, are x 3 and y 2 .
Sketch, on separate clearly labelled diagrams, the graphs of
(i) 3 y f x ,

[3]

(ii) y 2 f ( x) ,

[3]

1
.
f ( x)

[3]

(iii) y

Indicate clearly any asymptotes, axial intercepts and turning points.

MJC/2013 JC2 Preliminary Examination 9740/02

[Turn over

(a) Each side of an equilateral triangle increases from an initial length of 9 cm at a steady
rate of 0.1 cm s 1 . Find the rate of increase of the area of the equilateral triangle after
30 seconds, giving your answer in exact form.
[4]
(b)

y
B
P 2,3

The diagram above shows a line with negative gradient that passes through the
point P 2,3 and meets the x-axis and y-axis at A and B respectively. It is also given
that OAB radians .

(i)

Show that the length AB 2sec 3cosec .

(ii)

Find, using differentiation, the value of which gives a minimum length of AB.
[5]

The line L has equation x 2

[1]

y4
z 2 , and the plane p1 has equation 3x y 2 z 5 .
2

(i) Find the position vector of the point of intersection between L and p1 .

[3]

(ii) Find the acute angle between L and p1 .

[2]

(iii) Hence, or otherwise, find the shortest distance from the point 2, 4, 2 to p1 .

[3]

The plane p2 has equation x 2 y 3z 3 .


(iv) Find, in scalar product form, a vector equation of the plane that contains the
point 1, 1, 2 and is perpendicular to both the x-y plane and p2 .
[2]
(v) Find the values of a and b for which the simultaneous equations

3x y 2 z 5
x 2 y 3z 3
x 5 y az b
have infinitely many solutions.
MJC/2013 JC2 Preliminary Examination 9740/02

[3]

Section B: Statistics [60 marks]


5

Merilian Secondary School has 1600 students. To celebrate its 10th anniversary, the school
decides to organise a carnival. A sample of 100 students is to be chosen to take part in a
survey. The purpose of the survey is to investigate students opinions about the types of
activities to be included in the carnival. Describe how the sample could be chosen using
systematic sampling.
[2]
Give a reason why a stratified sample might be preferable in this context.

[2]

Explain whether it would be realistic to use stratified sampling in this context.

[1]

A group of student representatives is to be chosen from three schools, R, S and T. The group
is to consist of 10 students and is chosen from a set of 15 students consisting of 3 from R,
4 from S and 8 from T. Find the probability that the group consists of
(i) students from S and T only,

[2]

(ii) at least 1 student from each school.

[3]

A scientist intends to investigate how the bacteria population y, in thousands, depends on the
nutrient concentration x, in grams per litre in a water body. He collected samples from
8 different locations and the results are listed in the table below.
x
y

0.101
4.90

0.197
5.93

0.310
7.44

0.400
8.90

0.488
10.6

0.597
13.2

0.711
16.6

0.798
19.7

(i) State, giving a reason, whether a regression line of x on y or y on x is more appropriate


to model the relationship between x and y.
[1]
(ii) Calculate the product moment correlation coefficient between x and y.

[1]

(iii) Draw a scatter diagram for the data, and explain why a linear model is not necessarily
the best model for the relationship between x and y.
[3]
(iv) Fit a model of the form ln y a bx to the data, and use it to estimate the bacteria
population when the nutrient concentration of a particular water body
is 1 gram per litre. Comment on the reliability of your estimation.
[3]

MJC/2013 JC2 Preliminary Examination 9740/02

[Turn over

(a) The

random

variable

has

and P Y 0 0.1296 . Find P Y 2 .

binomial

distribution

with

mean

1.6
[4]

(b) A car park has a large number of parking lots. 80 parking lots are observed and the
number of occupied parking lots is denoted by X. State, in context, two assumptions
needed for X to be well modelled by a binomial distribution.
[2]
The probability of a parking lot being occupied is 0.95. Using a suitable
approximation, find the probability that out of the 80 chosen parking lots, at least 90%
are occupied.
[4]

Jane and Peter play a game which consists of 2 stages. Each stage is won by either Jane or
Peter and the game is won by the player who wins both stages.
The game has the following rules.
Stage 1

Stage 2

die is tossed once. Jane wins this stage if the score on the die is a 1 or 6.
Otherwise, Peter wins.

he winner of the first stage will draw a card from a deck of 52 playing cards which
consists of an equal number of heart, spade, diamond and club suits.
f the card drawn is not a spade, the player who draws the card wins stage 2.
Otherwise, the other player wins.

Construct a probability tree showing this information.

[2]

Find the probability that


(i) no one wins the game,

[2]

(ii) Peter wins the first stage, given that Peter does not win the game.

[3]

The deck of 52 cards is shuffled. 4 cards are drawn at random with replacement. Find the
probability that 2 hearts, 1 diamond and 1 spade are drawn.
[3]

MJC/2013 JC2 Preliminary Examination 9740/02

T
I

10

(a) A manufacturer of instant powdered yeast claims that the product is packed in sachets
of 7 grams. A random sample of 8 sachets is taken and the masses are measured.
Denoting the mass of one sachet by x grams, the results are summarized by

x 55.2 ,

381.06 .

(i)

Find unbiased estimates of the population mean and variance.

[2]

(ii)

Test, at the 5% level of significance, whether the manufacturers claim is valid.


[4]

(iii)

Explain the meaning of p-value in the context of this question.

[1]

(b) A teacher in a school makes a claim about the mean height of girls in the school. It is
given that the height of girls in the school follows a normal distribution with known
variance.
Two students Kirstyn and Gwyneth decide to each conduct a hypothesis test, at
the 5% level of significance, to determine whether the teacher has understated the
mean height of girls in the school. Kirstyn obtained a sample with mean x and
concluded that there is sufficient evidence, at the 5% level of significance, that the
teacher has understated the mean height of girls in the school. Gwyneth took a
different sample and obtained the same sample mean as Kirstyn.
Determine, with a reason, whether Gwyneths test will yield the same conclusion if the
size of her sample is four times that of Kirstyns.
[3]
11

Ken makes X minutes of outgoing overseas calls and Y minutes of outgoing local calls in a
day. X and Y are independent random variables. The expected value of X is 2.4 and its
standard deviation is 1.5. The expected value of Y is 5 and its standard deviation is 2.5.
(i) Find the probability that, over a 60-day period, the number of minutes of outgoing
local calls made by Ken is more than twice the number of minutes of outgoing
overseas calls.
[4]
Giving a reason, state whether any assumption(s) about the distributions of X and Y are
needed for the calculation in part (i) to be valid.
[1]
Outgoing overseas calls cost $0.20 per minute and outgoing local calls cost $0.10 per
minute.
(ii) Find the probability that, over a 60-day period, the total cost of outgoing calls made by
Ken is greater than $60.
[3]
The number of incoming calls received by Ken in a day is a random variable with the
distribution Po(3).

MJC/2013 JC2 Preliminary Examination 9740/02

[Turn over

(iii) Use a suitable approximation to find the probability that, over a 60-day period, the
number of incoming calls received by Ken is at most 200.
[4]

MJC/2013 JC2 Preliminary Examination 9740/02

BLANK PAGE

MJC/2013 JC2 Preliminary Examination 9740/02

[Turn over

Qn
1

Solution
Complex Number I
Let f(x) = x4 ax3 5x2 x 10 .
Since the coefficients of f(x) are real, and 1 + 2i is a root of f(x) = 0, therefore 1 2i is
also a root.
f ( x) x 1 2i x 1 2i x 2 bx c

x 1 2i x 1 2i x 2 bx c

x 1

2i

bx c

x 2 2 x 5 x 2 bx c
Comparing coefficients of
constant: c = 2
x: 5b 2c = 1 b = 1
x3: 2 1 = a a = 3
f ( x) x 2 2 x 5 x 2 x 2 x 2 2 x 5 x 2 x 1
The other roots are 1 2i, 1 and 2.
5 x 2 2 x 4 x3 3x 1

10 x 4 x3 5 x 2 3x 1 0
1 3 5 1
10 0
x 4 x3 x 2 x
1
Replace x by ,
x
1
1
1 or
2
x
x
1
x 1 or x
2
Qn
2
(i)

Solution
Graphing Techniques 2 (transformation)
y
3y f x

2
3

MJC/2013 JC2 Prelim Suggested Solutions/H2 Math (9740)/Paper 2/Math Dept

Page 1 of 10

(ii)
y

y 2 f ( x)

y 2

x
y 2

x=3
(iii)

1
f ( x)
y

1
1,
6

x=0

Qn
3
(a)

Applications of Differentiation

1
2

x=2

Solution

3
x
Let the sides and area of the equilateral triangle be x cm and A cm 2 respectively.

dx
0.1
dt
dA
when x 12 cm
To find:
dt
dA dx dA

Chain Rule:
dt dt dx
Given:

MJC/2013 JC2 Prelim Suggested Solutions/H2 Math (9740)/Paper 2/Math Dept

Page 2 of 10

A
A

1
x x sin
2
3

Alternatively,
2

3x 2
x
h x
4
2
2

3 2
x
4

3x 2
1
A x
4
2

3 2
x

dA
3

x
dx
2
dA
3

12 6 3
dx
2
dA
3 3
Hence,
0.1 6 3
cm2s 1.
dt
5
AB BP PA
2
3
AB

cos sin
AB 2sec 3cosec shown
Let length AB be .
2sec 3cosec
d
2sec tan 3cosec cot
d
At x 12,

(bi)

(bii)

At stationary point(s),
d
0
d
2sec tan 3 cosec cot 0
Since is acute, using GC, 0.85277 0.853 3 s.f

d
d

0.85277

0.85277

0.85277

Hence length AB is minimum when 0.853 3 s.f .

Qn
4
(i)

Vectors 2 and 3

Solution

2
1


Vector equation of L is r 4 2 , . ----(1)
2
1


3

p1 : r 1 5 ---(2)
2

Let A the point of intersection between L and p1 .
Substitute (1) into (2)

MJC/2013 JC2 Prelim Suggested Solutions/H2 Math (9740)/Paper 2/Math Dept

Page 3 of 10

2 3


4 2 1 5
2 2


6 3 4 2 4 2 5
3 9
3

Substitute 3 into (1),


1

OA 10 .
1

(ii)

Let the acute angle between L and p1 be .

1 3

2 1
1 2

sin 1
6 14
3
sin 1
6 14

19.107 (3 d.p)

19.1 ( 1 d.p)
(iii)

Observe that the point B 2, 4, 2 lies on L.

2 1 3

AB 4 10 6
2 1 3

Method 1 (Hence)
AB 9 36 9 3 6

Let N be the foot of perpendicular from point B to p1 .

BN
AB
3
BN
3 6
6 14
9
BN
2.41 (3 s.f)
14

sin

p1

MJC/2013 JC2 Prelim Suggested Solutions/H2 Math (9740)/Paper 2/Math Dept

Page 4 of 10

Method 2 (Otherwise)

(iv)

3 3

6 1
3 2

BN
14
9
BN
2.41 (3 s.f)
14
0 1 2

n 0 2 1
1 3 0

Vector equation of the plane is


2 1

r 1 1
0 2

2

r 1 1.
0

(v)

2

1
0

Let 3x y 2 z 5, x 2 y 3z 3 and x 5 y az b be the equation of p1 , p2 and p3


respectively.
Let l be the line of intersection of p1 and p2 .
1
1

Using GC, the equation of the line l is r 2 1 for .


0
1


Given that the equations have infinitely many solutions the 3 planes intersect at a line
l lies on p3 .
1
Normal of p3 1 :
1

1 1

1 5 0 a 4 , and
1 a

Point 1, 2,0 lies on p3 :

1 1

2 5 b b 11
0 4

MJC/2013 JC2 Prelim Suggested Solutions/H2 Math (9740)/Paper 2/Math Dept

Page 5 of 10

Qn
5

Solution
Sampling Methods
Number all the students in the school from 1 to 1600. Compute the sampling interval, k,
1600
16 . Randomly select an integer from 1 to 16. Select every 16 th student
using k
100
thereafter until 100 members are obtained.
Stratified sampling which could draw random samples from the gender groups (or any
possible strata) might be preferable as it could ensure the representation of the opinions
from different gender groups across the entire population.
It is realistic to use stratified sampling as the school would have the exact number of
students in each gender group (or any possible strata).

Qn
6
(i)

Solution
Permutations and Combinations & Probability
4
12
C4 8C6 4C3 8C7 4C2 8C8

C10
Required Probability 15
or
15
C10
C10
2
91

(ii)

15

C10 12C10 11C10


Required Probability
15
C10

Qn
7
(i)
(ii)
(iii)

38
39

Solution

Correlation and Regression


A regression line of y on x is more appropriate as the bacteria population depends on the
concentration of nutrients in the water body.
r 0.98119 0.981 3 s.f.

y
19.7
4.90

x x
x x

0.101

0.798

Although r 0.981 suggests a strong linear correlation, the scatter diagram shows that as
x increases, y increases at an increasing rate. Therefore, a linear model is not
necessarily the best model for the relationship between x and y.
(iv)

By G.C.,
ln y 1.3869 1.9984 x
ln y 1.39 2.00 x (3 s.f.)

MJC/2013 JC2 Prelim Suggested Solutions/H2 Math (9740)/Paper 2/Math Dept

Page 6 of 10

When x 1,
y 29.527 29.5 3 s.f.
The bacteria population is 29500 (3 s.f.)
Since x 1 lies outside of the data range, the estimate is not reliable.

Qn
8
(a)

Solution
Binomial and Poisson Distributions (includes conditions and unknowns)
Let Y ~ B n, p
Given
np 1.6 -----(1)
(1 p)n 0.1296 -----(2)
1.6
From (1), p
, sub into (2)
n

1.6
1
0.1296
n

1.6
0.4
4
P Y 2 1 P Y 2 0.1792

Using GC, n 4 and p


(b)

1.

The probability of a parking lot being occupied is constant throughout the 80 parking
lots.

2.

Whether a parking lot is occupied is independent of whether any other parking lot is
occupied

Let W be the number of parking lots, out of 80, that are NOT occupied.
W ~ B 80,0.05
Since n 80 is large and np 80 0.05 4 5 ,

W ~ Po 4 approximately.
P W 8
0.979

3 s.f.

MJC/2013 JC2 Prelim Suggested Solutions/H2 Math (9740)/Paper 2/Math Dept

Page 7 of 10

Qn
9

Solution

Probability (Tree diagram)


Let J denotes the event that Jane wins the stage
3
J
4

1
3

2
3

1
4
1
4

3
4
(i)

(ii)

J
J

Required probability
11 21

3 4 3 4
1

4
Required probability
P Peter wins stage 1 and Peter does not win the game

P Peter does not win the game

P Peter wins stage 1 and loses stage 2


1 P Peter wins the game

21

34

23
1
34
1

3
Required Prob
4

13 4!

52 2!
3

64

MJC/2013 JC2 Prelim Suggested Solutions/H2 Math (9740)/Paper 2/Math Dept

Page 8 of 10

Qn
10
(a)(i)

(ii)

Solution

Hypothesis Testing

An unbiased estimate of the population mean is x

55.2
6.9
8

An unbiased estimate of the population variance is


2
55.2

1
2
s 381.06
0.025714 0.0257 (3 s.f.)
7
8
Let be the population mean mass of a sachet of yeast.
H0 : 7

H1 : 7

Assume X ~ N , 2

X
~ tn 1
S
n
Level of significance: 5%
Reject H 0 if p-value < 0.05

Test statistic: T

Under H 0 , using GC, p-value = 0.121

(iii)
(b)

Since p-value = 0.121 > 0.05, we do not reject H 0 and conclude that there is insufficient
evidence, at 5% level, that the mean mass is not equal to 7 grams. Therefore, the
manufacturers claim is valid at 5% level.
A p-value of 0.121 means there is a probability of 0.121 that the sample mean mass of a
sachet of instant powdered yeast is as extreme or more extreme than the observed value of
6.9 grams, assuming that the mean mass of a sachet of instant powdered yeast is 7 grams.
Let the size of Kirstyns sample be n.
H 0 : 0

H1 : 0

Given: X ~ N , 2
Test statistic: Z

2
X ~ N ,
n

n
Level of significance: 5%
Reject H 0 if z-value > 1.6449
x 0
Kirstyns z-value is z
.

n
x 0
Since H 0 is rejected, z
>1.6449

x 0

x 0

2
Gwyneths z-value is
2 z 2 1.6449 1.6449

4n
n

Hence, Gwyneths test should yield the same conclusion.

MJC/2013 JC2 Prelim Suggested Solutions/H2 Math (9740)/Paper 2/Math Dept

Page 9 of 10

Qn
11
(i)

(ii)

(iii)

Solution
Normal with Sampling
Let K = X1 + X2 + + X60 and C = Y1 + Y2 + + Y60.
Since n = 60 is large, by Central Limit Theorem,
K ~ N(144, 135) and C ~ N(300, 375) approximately.
C 2K ~ N(12, 915)
Required probability P C 2 K 0

0.654 (3s.f.)
It is not necessary. Since the sample size of 60 is large, by Central Limit Theorem, C and
K follow normal distributions approximately.
Let A = 0.2K ~ N(28.8, 5.4).
Let B = 0.1C ~ N(30, 3.75).
A + B ~ N(58.8, 9.15)
P A B 60 0.346 (3s.f.)
Method 1
Let M be the number of incoming calls received by Ken in a 60-day period.
M ~ Po(180)
Since 180 10 , M ~ N(180, 180) approximately.
Required probability P M 200
P M 200.5 after continuity correction
0.937 (3s.f.)

Method 2
Let W be the number of incoming calls received by Ken in a day.
W ~ Po(3)
Since n 60 is large, by Central Limit Theorem,
W1 W2 ... W60 ~ N(180, 180) approximately

Required probability P W1 W2 ... W60 200


0.932 (3s.f.)

MJC/2013 JC2 Prelim Suggested Solutions/H2 Math (9740)/Paper 2/Math Dept

Page 10 of 10

Class

Adm No

Candidate Name:

2013 Preliminary Examination II


Pre-University 3
MATHEMATICS

9740/01

Paper 1

16 September 2013

3 hours
Additional Materials:

Answer Paper
List of Formulae (MF 15)

READ THESE INSTRUCTIONS FIRST


Write your name and class on all the work you hand in.
Write in dark blue or black pen on both sides of the paper.
You may use a soft pencil for any diagrams or graphs.
Do not use paper clips, highlighters, glue or correction fluid.
Answer all the questions.
Give non-exact numerical answers correct to 3 significant figures, or 1 decimal place in the
case of angles in degrees, unless a different level of accuracy is specified in the question.
You are expected to use a graphic calculator.
Unsupported answers from a graphic calculator are allowed unless a question specifically
states otherwise.
Where unsupported answers from a graphic calculator are not allowed in a question, you are
required to present the mathematical steps using mathematical notations and not calculator
commands.
You are reminded of the need for clear presentation in your answers.
At the end of the examination, arrange your answers in NUMERICAL ORDER and fasten all
your work securely together.
The number of marks is given in brackets [ ] at the end of each question or part question.

This question paper consists of 5 printed pages.


[Turn over

Four friends, Ernest, Saleem, Chandru and Kumar went for tea in a cafe. The food and
the number of servings they each ordered, and the total expenditure per person are
shown in the following table:
Ernest
Saleem
Chandru
Kumar
Number of servings per person
5
1
3
2
3
1
5
1
4
2
4
2
95.00
30.00
k
40.00

Type of food
Cream puffs
Cupcakes
Pies
Total Expenditure ($)

[4]

Find k, the total expenditure for Chandru.


2

It is given that y 3

2 x

.
2

d 2 y 1 dy
(i) Show that
2 y ln 3 .
dx 2 y dx

[3]

(ii) Find Maclaurins series for y , up to and including the term in x 2 , leaving
the coefficients in exact form.

[3]

(iii) Hence estimate the value of

2 x

dx , giving your answer in exact form.

Explain how the accuracy of the estimation can be improved.


3

The complex number z satisfies the relations

z 3 3i 2 .

[3]

arg z 3 3i and
4
2

(i) Illustrate both of these relations on a single Argand diagram.

[4]

(ii) Find the greatest and least values of z 3i .

[3]

(iii) Find the least possible value of arg z in radians.

[3]

A curve C has parametric equations x 4e3t , y 3et where t 0 .


(i) Sketch the curve and write down the asymptote of C.

[2]

(ii) The region R is bounded by C , the x-axis, and the vertical lines x 4 and
x 32 . Calculate the exact area of R .

[4]

(iii) Find the volume of the solid of revolution formed when R is rotated
through 2 radians about the x-axis, giving your answer in an exact form.

[3]

1
(a) Given that cos , where is the angle between the vectors i j k
3
and 2i 2 j k , find the value of the constant .

[3]

(b) The planes p1 and p2 meet in a line l , and have the vector equations

r 4i 7 j 2k s i k t j k and r 4i 7 j 2k u 2i 3j v i k
respectively.

(i) Show that l is parallel to the vector 5i 6 j k .

[3]

(ii) Write down a vector equation for l .

[1]

(iii) The plane p3 has equation r 2i 3j 4k i 3j 2k 5i 8j k . Find


the position vector of the point A which is common to p1 , p2 and p3 .
6

The curve C has equation y

[3]

x2 2 x 3
.
x 1

(i) Find the range of values of y for which C does not lie within, in exact form.

[3]

(ii) Sketch C, showing clearly the axial intercept, asymptotes and stationary
points.

[3]

(iii) By adding an additional sketch for x , solve

3 x 1 cos x 1 x 2 2 x 2.
7

[3]

To improve on his standing broad jump, Glen does successive jumps on a running track
in a straight line, from the starting point towards the 15m mark. Due to fatigue, Glens
jumping distance will cover only 95% of the distance he covered in each previous jump.
As a result of wear and tear on Glens shoes after each jump, he will slide forward after
landing each time. The distance that Glen slides forward increases by 0.01m from the
previous slide. For the first set of jump and slide, Glen jumps a distance of 2m and
slides forward 0.01m on landing.
(i) Find the total distance that Glen will cover in his 3rd set of jump and slide.

[2]

(ii) Find the number of sets of jumps and slides Glen will need to cross the 15m
mark.

[3]

(iii) Glen is left with 10 weeks of training sessions and he believes that he will pass the
standing broad jump test in the 11th week which requires a single jump distance of
2.5m for passing. He finds that the distance he can cover for the first jump for
each training session improves by 2% every week from 2m in Week 1.
Comment on the validity of Glens belief.
[2]
[Turn Over
3

Find
1

(i)

1 cos 2 x dx ,

(ii)

(iii)

e2 x
x
e x 2 dx , using the substitution u e 2 .

[2]

x 2 2 x 3 dx ,

[3]

[4]

When a cake is removed from the oven, its temperature decreases at a rate proportional
to the positive difference between its temperature and the temperature of the room. The
temperature of the room is constant at 25C and T is the temperature of the cake t hours
after removing from the oven. The temperature of the cake at the instant when it is
removed from the oven is 50C .
(i) Show that T 25 25e kt , where k is a positive constant.

[5]

(ii) Sketch a solution curve for T where t 0 .

[2]

(iii) Pradip is a world renowned baker who is uncompromising on the freshness of the
cakes served by his bakery. On a particular morning at 10.00am, he found the
temperature of a cake at 40C , and 1 hour later, it was 31C . At what time, to
the nearest minute, was the cake removed from the oven?
[4]
State what happens to the temperature of the cake in the long run.
10

(i) Express

[1]

A
B
r 2 9r 19

in the form
where A and B are constants
r 5!
r 3 ! r 5 !

to be found. Hence show that

r 2 9r 19 6
n6

.

r 5! 5! n 5!
r 1
n

(ii) Prove your result in (i) by the method of mathematical induction.


(iii) Hence, find

r 2 7r 11
.

r 4 !
r 1

[5]
[5]

[4]

11

Referred to the origin O , the position vectors of the points A and B are a and b
respectively, and OAB is a triangle.
(i) Show that the area of the triangle is given by

1
ab .
2

[2]

1
a b.
2

[2]

(ii) Show that the maximum area of the triangle is

a1
b1

(iii) By considering a b where a a2 and b b2 , show that




a
b
3
3

a1b1 a2b2 a3b3

a12 a2 2 a32 b12 b2 2 b32

-End of paper-

[3]

Let x, y and z be the cost of cream puffs, cupcakes and pies respectively.
5 x 3 y 4 z 95

x y 2 z 30
2 x y 2 z 40
Using GC, x = 10, y = 5, z = 7.5.
k 3 10 5 5 4 7.5 85
2i

y3

2 x

ln y x 2 2 x ln 3
1 dy
2 x 2 ln 3
y dx
2

1 d 2 y 1 dy
2 ln 3
y dx 2 y 2 dx
2

2ii

2iii

d 2 y 1 dy
2 y ln 3
dx 2 y dx
when x 0, y 1,
dy
2 ln 3,
dx
d2y
2 ln 3 2 ln 3 1 .
dx 2
2 ln 3 1 2 ln 3 x 2
y 1 2 ln 3 x
2!
1 2 x ln 3 2 ln 3 1 x 2 ln 3 .

2 x

dx 11 2 x ln 3 2 ln 3 1 x 2 ln 3 dx

x3 ln 3
2
x x ln 3 2 ln 3 1

3 0

ln 3
1 ln 3
2 ln 3 1
3
The accuracy can be improved by including more terms in the expansion.

3i

o
2

x
4

3ii

greatest z 3i 32 62 2
3 52

least z 3i 32 62 2

3iii

3 5 2

2
18
0.49088
sin

arg z

4
0.295 rad

0.49088

4i
(4,3)

4ii

asymptote: y 0
32

Area y dx
4

ln 2

3e t 12e3t dt

36

ln 2

e 2t dt
ln 2

1
36 e 2t
2 0
54

4iii

x 4e 3t
1

x 3
et
4
y 3e t
x
3
4

1
3

32

Volume y 2 dx
4

32

3
x
3 dx

108
Alternatively,
32

Volume y 2 dx
4

ln 2

9e 2t 12e3t dt

108

ln 2

et dt

108 et
108

5a

cos

ab
a b

1 2

1 2
1 1

3
2 2 9
2 2 4

7
4

ln 2
0

5bi

5bii

5biii

1 0 1

n1 0 1 1
1 1 1

2 1 3

n 2 3 0 2
0 1 3

1 3 5

l is parallel to 1 2 6 . (shown)
1 3 1

4
5


l : r 7 6, .
2
1


1 5 19

n 3 3 8 11
2 1 7

19 2 19

p3 : r 11 3 11 23
7 4 7

4 5 19

7 6 11 23
2 7

1
1

OA 1
1

6i

x2 2 x 3
2
y
x 1
x 1
x 1
dy
2
1
0
2
dx
x 1
x 1 2
y 2 2
2 2 y 2 2

6ii

6iii

Asymptote: y x 1, x 1
Show 3cos x

x2 2 x 3
x 1

Sketch y 3cos x
x 0 or 0.335 (3sf)
7i
7ii

7iii

8i

T3 2 0.95 0.01 2 0.01


2

1.835m
2 1 0.95n n
Sn
2 0.01 n 1 0.01 15
1 0.95
2
Using GC, n 8.828.
Least n = 9.
9
10
T10 2 1.02 2.39 2.5 or T11 2 1.02 2.44 2.5
Glens belief is not valid.
1

1 cos 2 x dx 1 2 cos

x 1

dx

1
sec 2 x dx

2
1
tan x c
2

8ii

x 2 2 x 3 dx x 2 2 x 3 dx x 2 2 x 3 dx
0

1
1

x3 x 2 3x x 3 x 2 3x
3
0 3
1
4

8iii

u ex 2
du
ex u 2
dx

u 2 1
e2 x
e x 2 dx u u 2 du
2
1 du
u
u 2 ln u c
2

e x 2 2 ln e x 2 c

9i

dT
k T 25 , k 0
dt
1

T 25 dT k dt
ln T 25 kt c
when t 0, T 50 :
ln 25 c
T 25 e kt ln 25
9ii

25 25e kt (shown)
50
T=25

9iii

40 25 25e kt 0.6 e kt
31 25 25e k t 1 0.24 e k t 1
2.5 e k k ln 2.5
0.6 e t ln 2.5
ln 0.6
33.4 min
ln 2.5
The cake was removed at 9.26am (or 9.27am).
The cake will cool to 25 C .
t

10i

r 2 9r 19 A r 4 r 5 B
A 1, B 1

r 2 9r 19 n 1
1

r 5! r 1 r 3! r 5 !
r 1
n

1 1

4! 6!
1 1

5! 7!
1 1

6! 8!

1
1

n 2 ! n 4 !

10ii

1
1

n 3 ! n 5 !

1 1
1
1

4! 5! n 4 ! n 5 !

5 1 n 5 1

5! n 5 !

n6
6

(shown)
5! n 5 !
r 2 9r 19 6
n6
for all n

r 5! 5! n 5!
r 1
n

Let Pn be the statement


When n = 1, LHS =
RHS =

r 2 9r 19 1 9 19 29

6!
6!
r 5!
r 1
n

6
1 6
29

LHS
5! 1 5! 6!

Therefore, P1 is true.
Assume Pk true for some k
To show Pk+1 is true,

k 1

r 2 9r 19 6
k 6
,
.

r 5! 5! k 5!
r 1

r 2 9r 19

r 5!
r 1

6
k 7
.

5! k 6 !

r 2 9r 19
LHS=
r 5!
r 1
k 1

k 1 9 k 1 19
6
k 6

5! k 5 !
k 6 !
2

6 k 2 12k 36 k 2 2k 1 9k 9 19

5!
k 6 !

6
k 7

5! k 6 !

RHS
Pk true Pk+1 true.
10iii

Since P1 true and Pk true Pk+1 true, Pn true for all n


r j 1,

j 1 7 j 1 11 j 2 9 j 19 , j dummy variable

j 1 4 !
j 5 !
j 11
j 0

6
r 2 9r 19 19
n6
lim

r 5! 5! n 5! n 5!
r 0

11i

11ii

1
Area bh
2
1
OA OB OA
2
1
a b (shown)
2
Since sin 1,
1
Area a b sin
2
1
a b
2

25
5
0
5!
24

11iii

Since cos 2 1,
2

a b a b cos
2

a b
a1

a2
a
3

b1

2
2
2
2
2
2
b2 a1 a2 a3 b1 b2 b3
b
3

a1b1 a2b2 a3b3

a12 a2 2 a32 b12 b2 2 b32

Class

Adm No

Candidate Name:

2013 Preliminary Examination II


Pre-university 3
MATHEMATICS

9740/02

Paper 2

26 September 2013

3 hours
Additional Materials:

Answer Paper
List of Formulae (MF 15)

READ THESE INSTRUCTIONS FIRST


Write your name and class on all the work you hand in.
Write in dark blue or black pen on both sides of the paper.
You may use a soft pencil for any diagrams or graphs.
Do not use paper clips, highlighters, glue or correction fluid.
Answer all the questions.
Give non-exact numerical answers correct to 3 significant figures, or 1 decimal place in the
case of angles in degrees, unless a different level of accuracy is specified in the question.
You are expected to use a graphic calculator.
Unsupported answers from a graphic calculator are allowed unless a question specifically
states otherwise.
Where unsupported answers from a graphic calculator are not allowed in a question, you are
required to present the mathematical steps using mathematical notations and not calculator
commands.
You are reminded of the need for clear presentation in your answers.
At the end of the examination, arrange your answers in NUMERICAL ORDER and fasten all
your work securely together.
The number of marks is given in brackets [ ] at the end of each question or part question.

This question paper consists of 6 printed pages.

[Turn over

Section A: Pure Mathematics [40 marks]

Without the use of a calculator, solve the inequality


3x 2 4 x 7
2.
x2 x 6

[4]

Hence solve the inequality

3x 2 4 x 7
2.
x2 x 6

Given that ei is a root to the equation

z3
where 0

3 1 z 2 z 1 0

and , show that

( z 2 2 z cos 1)( z 1) 0 .

Find the values of and .

[4]
[2]

Using the results obtained above, solve z 9

[2]

The functions f and g are defined by


x2
f :x
,
1 2x
g : x x2 1,

3 1 z6 z3 1 0 .

[4]

x , a x b,

x , x 0 .

(i)

Find g 1 ( x) and write down the domain of g 1 .

[4]

(ii)

Solve g( x) g 1 ( x) , leaving your answer in exact form.

[2]

(iii) Expand f ( x) in ascending powers of x up to and including the term in x 3 . State


the minimum value of a and the maximum value of b for the expansion to be
valid.
(iv)

[3]

Given that the rate of change of x is 0.25 unit per second, estimate the rate of
change of f ( x) when x 0.4 unit.

Page 2 of 6

[2]

(i)

(ii)

6
, showing clearly any asymptotes and any points
x 2
where the curve crosses the axes.

Sketch the curve y

By sketching another curve on the graph in part (i), solve the inequality
6
ln x 2 1 .
2
x 2

(iii) Find

d 6

.
dx x 2 2

[2]

[3]

[2]

(iv) Hence find the minimum and maximum values of the gradient of the tangent to
6
the curve y 2
, leaving your answer in exact form.
x 2

[6]

Section B: Statistics [60 marks]


5

M Institute has a student population of 1500, of whom 375 students are from the Arts
stream, 600 from Business and 525 from Science. M Institute intends to obtain a
sample of 60 students for a survey. After assembly one morning, the teachers in
charge of the survey selected the first 20 students they see from each stream into the
sample.
(i) State the sampling method described in the question.
(ii) Describe an alternative method to obtain the sample.
(iii) State an advantage of the alternative method in part (ii) as compared to the
method in part (i).

[1]
[3]
[1]

Six identical boxes are arranged in 3 rows as shown in the following diagram.
Top Row
Middle Row
Bottom Row
Sandra is given 1 green, 2 blue and 3 red balls. The balls are identical except for their
colour. She is to put one ball in each box. Find the number of ways she can do this
when
(i) there is no restriction,
(ii) the balls in the bottom row are of different colours,
(iii) there are at least 2 red balls in the bottom row.
Page 3 of 6

[2]
[2]
[3]

A group of students take an examination in Science. A student who fails the


examination at the first attempt is allowed one further attempt. For a randomly chosen
student, the probability of passing the examination at the first attempt is p . If the
student fails the examination at the first attempt, the probability of passing at the
second attempt is 0.3 more than the probability of passing the examination at the first
attempt.
Show that the probability that a randomly chosen student passes the examination is
[1]
2
0.3 1.7 p p .
(a) Find the value of p such that the probability that a randomly chosen student passes [2]
the examination on the first attempt given that the student passes is 0.6.
Two students are randomly chosen.
(b) (i)
Find the probability that one passes the examination on the first
attempt and the other passes the examination on the second attempt,
leaving your answer in terms of p.
(ii)
Find the value of p such that the value of the probability in part (i) is
maximum.

[1]
[3]

The stress level and job performance of a companys employees are measured using
the stress index and job performance index that a human resource consultant
developed. The consultant is investigating the relationship between the stress level of
an employee and his job performance. The data of 8 employees is tabulated as
follows.

(i)

Stress level, x

2.5

3.13

3.75

4.38

6.25

7.5

8.75

Job Performance, y

4.20

3.75

3.56

3.40

3.23

3.09

3.03

2.97

Draw a scatter diagram for the data and calculate the product moment
correlation coefficient between y and x.

(ii) Explain whether a linear model is appropriate.


(iii) Observing the general trend of the points in the scatter diagram, the consultant
a
proposes that a model of the form y b where a and b are positive
x
constants is more appropriate. Evaluate the consultants proposal.
1
(iv) Calculate the least-squares regression line of y on .
x
(v) Using the least-squares regression line in part (iv), estimate the job
performance level of an employee when his stress level is 4. Comment on the
reliability of the estimate.
Page 4 of 6

[3]
[1]

[1]
[1]

[2]

(a)

The Traffic Authority claims that the average speed of vehicles along a road is
at most 70 km per hour. The speed, x km per hour, of 20 randomly selected
vehicles are recorded and summarised as follows:

x 70 71

x 70

1122 .

It is assumed that the speed of the vehicles is normally distributed.


(i)

Find the unbiased estimates of the mean and variance of the speed of
vehicles along the road.
(ii) Test, at 5% significance level, the Traffic Authoritys claim.
(b)

[2]
[4]

In XYZ School, the students mean running time in the 2.4 km run is claimed to
be 705 seconds. A programme to help the students perform better in the run is
introduced. After the programme, a sample of 100 students was taken and their
mean running time is 690 seconds. At a significance level of 7%, a z-test was
done to test if the students are performing better. It was concluded that there is
sufficient evidence that the students are performing better in the 2.4 km run.
(i)
(ii)

(iii)

10

In the test above, explain if any assumption about the students 2.4 km
running times is necessary.

[1]

If a two-tailed test had been conducted, is it necessarily true, necessarily


false or not necessarily true or false that the conclusion will remain the
same?
What can you do to increase the p-value in the test in part (b)?

[2]
[1]

A fruit stall sells 2 types of durians, XO and D24. For each type of durian, a
randomly chosen durian has a mass that is normally distributed with mean and
standard deviation, as well as unit price as shown in the following table.

(i)

Mean (kg)

Standard deviation (kg)

Price per kg

XO

2.5

0.8

$12

D24

2.0

0.5

$7

Find the probability that the total mass of 2 randomly chosen XO durians is

[3]

more than 3 times the mass of a randomly chosen D24 durian.


(ii)

Find the probability that the price of a randomly chosen XO durian differs from

[4]

the price of a randomly chosen D24 durian by less than $15.


(iii) A sample of n XO durians is randomly chosen. Find the least value of n such
that the probability that the mean mass is more than 2.25 kg exceeds 0.9.
Page 5 of 6

[4]

11

(a)

A special type of jackfruits can be of poor, mediocre and premium grades. The
percentage of jackfruits of poor, mediocre and premium grades are on average
20%, 65% and 15% respectively. A customer randomly selects 5 jackfruits. To
calculate the probability that 3 of the jackfruits are mediocre and 2 are
premium, the following calculation was carried out.
Let Y be the number of mediocre jackfruits
Y ~ B(5, 0.65)

P(Y 3) 5C3 (0.65)3 (1 0.65) 2 0.336


Explain, by giving two reasons, why the calculation is not valid.

(b)

[2]

A target board in an archery practice school is as shown in the following


diagram. The target board is a rectangle with a length of 100 cm and a width of
60 cm. It contains a shaded diamond-shaped region in the middle which has
sides of equal lengths.
15 cm
20 cm

20 cm

60 cm

15 cm
100 cm

(i)

By considering the area of the shaded region, show that the probability
that an archer will hit the shaded area is 0.15.
(ii) State two assumptions for the probability in part (b)(i) to be valid.
(iii) The archer shoots at the target board 100 times as practice. Using a
suitable approximation, find the probability that he hits the shaded area
more than 20 times.
(iv) The archer practiced daily for 8 weeks. Each day, he shoots at the target
board 100 times. Using a suitable approximation, find the probability that
he hits the shaded area at most 9 times in fewer than 4 days.

End of Paper

Page 6 of 6

[1]
[2]

[4]

[3]

PU3 H2 Mathematics Promo II Paper 2 Suggested Solutions


1(i)

3x 2 4 x 7
2
x2 x 6
x2 2 x 5
0
x2 x 6
( x 1) 2 4
0
x2 x 6
Since ( x 1) 2 4 is always positive,
1
0
2
x x6
x2 x 6 0
( x 2)( x 3) 0
x 2 or x 3

1(ii)

x is replaced by x
x 2 (rejected) or x 3
x 3 or x 3

z3

3 1 z 2 z 1 0

All coefficients in the above polynomial are real. By Conjugate


Root Theorem, ei and ei are roots to the equation.

z3

3 1 z2 z 1 0

( z ei )( z e i )( z c) 0
z 2 (ei e i ) z 1 ( z c) 0
z 2 2 z cos 1 ( z 1) 0 (shown)
Comparing z2 coefficient,

1 2 cos
cos

3
2

Comparing z coefficient,
2 cos 1

3 1

3 1

z9

3 1 z6 z3 1 0
i

( z 3 e 6 )( z 3 e
z3 e

or

2 k
i

z e 18
ze

2 k
i

18 3

)( z 3 1) 0

z3 e

or

z3 1

where k 1, 0,1 or
where k 1, 0,1 or

2 k
i

3(i)

ze
where k 1, 0,1
Let y g( x)

y x2 1
x y 1 (reject

y 1)

f 1 ( y ) y 1
f 1 ( x) x 1, x 1
3(ii)

Here, the solution of g( x) g 1 ( x) is the same as that for g( x) x.


g( x) x
x2 1 x
x2 x 1 0
x

3(iii)

1 5
1 5
or
(rejected)
2
2

f(x)
( x 2)(1 2 x) 1
( x 2)(1 2 x 4 x 2 8 x 3 ...)
20 x3 10 x 2 5 x 2 ...
For expansion to be valid,
Minimum a

3(iv)

1
1
and Maximum b
2
2

df ( x) df(x) dx
=
.
dt
dx dt
(60 x 2 20 x 5 ...).
At x 0.4,

dx
dt

df ( x)
60(0.4) 2 20(0.4) 5 .(0.25) 1.65 units per second
dt

4(i)

4(ii)

From graph,
1.95 x 1.95, x 1, x 1

4(iii)

d 6
12 x
2
2
dx x 2
( x 2)2

4(iv)

Let m be gradient of the tangent to the curve


12 x
m 2
( x 2)2

dm
( x 2 2) 2 (12) 2( x 2 2)(2 x)(12 x)

dx
( x 2 2) 4

12( x 2 2)(3x 2 2)
( x 2 2) 4

For maximum/minimum m,
dm
0
dx
12( x 2 2)(3 x 2 2)
0
( x 2 2) 4

2
3

Using First/Second Derivative Test:


2
m is maximum at x
3
m is minimum at x

m maximum

m minimum

2
3

2
12

3
2 2


2
3

12

2
3

2 2

27 2
16 3

27 2
16 3

5(i)

Quota Sampling

5(ii),
5(iii)

Alternative 1:
Simple Random Sampling:
Number each student on the student roster (or equivalent step) and
randomly select 60 students for the survey.
Advantage: Reduce biasness
Alternative 2:
Stratified Sampling:
Divide student population into distinct strata i.e. streams
Randomly select the students from each stream according to the
proportional size of the stream as follows:
Arts: 15 students, Business: 24 students, Science: 21 students
Advantage: Representative of the student population
Alternative 3:
Systematic Sampling:
Arrange the students in alphabetical order (or any form of
reasonable arrangement)
Randomly select a student into the sample.
Select every 25th student after the student above until the sample
size is reached.
Advantage: The sample could be more evenly spread across
classes in each stream (quota sampling likely to end up with many
students from the same class in this case).

6(i)

N1 =

6!
60
2!3!

6(ii)

6(iii)
7

N2 =

3!
(3!) 18
2!

3!
N3 =
2!
3R bottom

3!
3! 3!
(3!)
.
30
2!
2! 2!
2R+1B bottom 2R+1G bottom

Pass
p + 0.3

1-p

Pass

Fail
0.7 - p

Fail

P student passes exam p (1 p)( p 0.3) 0.3 1.7 p p 2 (shown)

7(a)

p
0.6
0.3 1.7 p p 2
0.6 p 2 0.02 p 0.18 0
p 0.565 or 0.531 (rejected)

7(b)(i)

Let the required probability be C


C p (1 p)( p 0.3) (1 p)( p 0.3) p
C 0.6 p 1.4 p 2 2 p 3

7(b)(ii)

dC
0.6 2.8 p 6 p 2
dp
d 2C
2.8 12 p
dp 2
For maximum / minimum p,
dC
0.6 2.8 p 6 p 2 0
dp
p 0.626 or 0.159 (rejected)
d 2C
2.8 12(0.626) 4.712 0
dp 2
C is maximum when p 0.626
At p 0.626,

8(i)

Job Performance, y

r 0.901

8(ii)

8(iii)

Stress Level, x

A linear model is not appropriate in this case as the general trend


of the points is not linear.
The correlation coefficient between y and

1
0.995 . r
x

1
is closer to 1 than r between y and x. The
x
model that the researcher proposed is more appropriate.
between y and

8(iv)

8(v)

9(a)(i)

1
y 4.2784 2.4293
x
1
y 4.28 2.43 (3 s.f.)
x

1
y 4.2784 2.4293 3.50
4
The estimate is reliable as the estimate was obtained using
1
interpolation and r between y and is close to 1
x
Unbiased estimate for population mean
=x

71
70 73.55 73.6
20

Unbiased estimate for population variance


=s 2

1
712
1122

45.7868 45.8
19
20

9(a)(ii)

H 0 : 70
H1 : 70
Assuming H 0 is true,
Using t -test, p value 0.0150 0.05
Reject H 0
There is sufficient evidence that 70
at a significance level of 5%

9(b)

(i) Not necessary as the sample size of 100 is large ( 50),


hence Central Limit Theorem applies.
(ii) p-value doubles but not necessarily true or false that the pvalue is still less than 0.07. Hence, not necessarily true or
false the conclusion will remain the same.
(iii) Decrease mean running time claimed

Let X be the mass of a XO durian


Let D be the mass of a D24 durian
X ~ N (2.5, 0.82 )
D ~ N (2, 0.52 )
10(i)

X 1 X 2 3D ~ N (1,3.53)
P( X 1 X 2 3D 0)
0.2973 0.297

10(ii)

12 X 7 D ~ N (16,104.41)
P(12 X 7 D 15)
P(15 12 X 7 D 15)
0.4598 0.460

10(iii)

0.82
X ~ N 2.5,

P X 2.25 0.9

2.25 2.5
PZ
0.9

0.82

0.25
n 1.28155

0.8
0.25
n 1.28155
0.8
n 16.82
Least value of n 17
11(a)

Any two of the following reasons:


1. There are more than 2 outcomes in the selection of
jackfruits.
2. The selection of jackfruits may not be independent.
3. The probability of selecting a jackfruit of the different grades
may not be constant.

11
(b)(i)

1
4 (30)(15)
2
P(hit shaded area)
0.15 (shown)
(100)(60)

11
(b)(ii)

Assumptions:
1. All the archers shots hit the target board.
2. There is equal chance for an archers shot to hit any point on
the target board.

11
(b)(iii)

Let X be the number of shots that hit the shaded area


X ~ B(100, 0.15)

np 15 5
n(1 p ) 85 5
X ~ N (15,12.75) approximately
c .c .

P( X 20) P( X 20.5) 0.06174 0.0617

11
(b)(iv)

Let Y be the number of days where archer hits the shaded area
at most 9 times out of 100 shots
Y ~ B(56, P( X 9))
Y ~ B(56, 0.055095)
n 50
np 3.0853 5
Y ~ Po(3.0853) approximately
P(Y 4) P(Y 3) 0.6281 0.628

JURONG JUNIOR COLLEGE


2

Higher 2, Paper 1

1 A curve C has equation x2 y 2 ax by c 0 . Given that the curve C passes through

1, 4 , 2, 5 and 3, 2 , find

a, b and c .

[4]

2 In the triangle PQR, angle PQR = radians, PQ = r and QR = 1, where r > 1. Given that is a
sufficiently small angle, show that
1

PR (r 1)2 r 2 2 .
Find the constants r and a such that PR

3 (i)

Show that

Hence find

r 1

(iii) Deduce that

[7]

1
r 1 r for all r
r 1 r
n

(ii)

1
a 2 .
4

r 1

[2]

1
.
r 1 r
1
2
r

[3]

n 1 1 .

4 Prove by mathematical induction that

[2]

2r 1 r (r 1) (n 1)
r 1

1
.
n 1

[5]

Hence find the least n such that


n

r 10

2r 1 r r 1 525.

[3]

5 Sketch on an Argand diagram the set of points representing all complex numbers z satisfying
all of the following inequalities:

z 1 i 2 ,

z 1 i z 1 i and

arg( z 1 i)

[4]

Hence find
(i)

the least possible exact value of z 3 2i ,

[2]

(ii)

the largest possible value of arg( z 3 2i) .

[2]

3
6 The diagram shows the graph of y a f ( x) , where a is a constant and f ( x) is a proper

1 1
rational function. The curve has a maximum point at , and it crosses the x-axis at the
2 2
points (0,0) and (1, 0). The lines x 1 , x 2 and y 2 are the asymptotes of the curve.
y

y a f(x)
2

x 1

y2

x2

(i)

State the value of a.

(ii)

Describe a transformation which would transform the above curve to the curve y f ( x).

[2]

(iii)

State the exact coordinates of the turning point of the curve y a f ( x).

[1]

1
.
a f ( x)

[2]

Determine the number of real roots of the equation f (| x |) 1 0.

[2]

(iv) State the equations of the asymptotes of the curve y

(v)

[1]

7 Vegetable farm A produces 1200 kg of vegetables in the first year. In each subsequent year, the
amount of vegetables produced is 8% less than the previous year.
(i)

If farming continues indefinitely, what is the theoretical total amount of vegetables


produced?

[1]

Vegetable farm B also produces 1200 kg of vegetables in the first year. In each subsequent
year, the amount of vegetables produced is 78 kg less than the previous year. For both farms A
and B, farming continues yearly up to and including the year where less than 400 kg of
vegetables are produced. Both the farms then stop production.
(ii)

In which year did farm A first produce less than 400 kg of vegetables?

(iii) Determine which farm produced more vegetables in total, justifying your answer.

[2]
[5]
[Turn over

4
8 (i)

Find a vector equation of the line passing through the points A and B with position
vectors 6 j k and 4i 2 j k respectively.

(ii)

[2]

Point C has the position vector 3i j 2k , where is a positive constant. Given that
the length of projection of AC on the line AB is 125 , find the value of .

(iii) Hence or otherwise, find the distance from the point C to the line AB.

[3]
[3]

9
l

2x

2x

The diagram shows a decoration in the shape of an octahedron, which is made by covering a
wire frame tightly with coloured paper.
The frame consists of twelve straight pieces of wire, joined at their ends to form two identical
right pyramids with a square base of side 2x cm and slant edges of l cm.
Given that the total length of the wire used is 200 cm, use differentiation to find the value of x
that makes the total surface area of the octahedron maximum.

[9]

10 The equation of a curve is x2 4 xy 2 y 2 k , where k is a constant.

dy
in terms of x and y.
dx

(i)

Find

(ii)

For the case where k 2, find the coordinates of each point on the curve at which the
tangent is parallel to the x-axis.

[2]

[3]

(iii) For the case where k 2, a point P(x, y) moves along the curve in such a way that its
x-coordinate is increasing at a constant rate of 2 units per second. Find the exact rate of
change of its y-coordinate at the instant when x = 4 and y = 7.
(iv) Show that for k > 0, every line parallel to the y-axis cuts the curve at two distinct points.

[2]
[3]

5
11 The functions f and g are defined by

f :x

x 2 2 x 3, x , x a ,

g:x

ln( x 1), x , x 1,

where a is a real number.


(i)

(ii)

State with a reason the largest possible value of a such that f 1 exists, and define f 1 in a
similar form.

[6]

Using the value of a found in (i), prove that the composite function gf exists.

[2]

(iii) Define gf in a similar form and find the range of gf.

12 (a)

(i) Given that a 1 , use the substitution u

(b)

(i) Show that

1
to show that
x

1
1
1
dx 1
du.
2
2
1 x
a 1 u

(ii) Hence show that tan 1 a tan 1

[3]

[4]

1
k , where k is a constant to be determined.
a

x3
1 1
dx ln 2 .
2
1 x
2 2

(ii) Without using a calculator, evaluate the integral

[3]

[2]

1
0

x 2 tan 1 x dx .

[3]

6
BLANK PAGE

2013 JJC H2 Maths Prelim P1 Solution


Qn
1

Solution
The curve C passes through 1, 4 :

1 4
2

a 1 b 4 c 0

a 4b c 17 --- (1)
The curve C passes through 2, 5 :

2 5
2

a 2 b 5 c 0

2a 5b c 29 --- (2)
The curve C passes through 3, 2 :

3 2
2

a 3 b 2 c 0

3a 2b c 13 --- (3)
a 2, b 2, c 23 .

Qn
2

Solution
R

1
Q

PR2 12 r 2 ? 2(1)(r )cos


1 r 2 ? 2r cos

2
2
2

PR 2 1 r 2 ? 2r 1
2

Since is a small angle, cos = 1

= 1 r 2 ? 2r r 2
(r 1) 2 r 2
1
2

PR (r 1) r . (Shown)
2

r
2
(r 1) 1

2
(r 1)

1 r

(r 1) 1
2 ...
2

2 (r 1)

r
r 1
2.
2(r 1)

1
a 2 , by comparison,
4
1
r
and
r 1
a
4
2( r 1)
5
5
and a .
r
4
2

Given PR

Qn
3 (i)

Solution
1
1
r 1 r

r 1 r
r 1 r
r 1 r

(ii)

r 1

r 1

(iii)

r 1 r (shown)
1
r 1 r

r 1 r

2
3

2 1
3
4

r 1

r 1 r
r 1 r

r 1 r

n 1 n

n 1 1

n
1
1

r 1 r r 1 r r
r 1
n
1
n 1 1

r 1 2 r
n
1

2 n 1 1 (deduced)
r
r 1

since

Qn
4

Solution

for all n

r 1

2r 1 r (r 1) (n 1)

Let Pn be the statement

1
n 1

1
1
1
1
LHS of P1 = 2r 1
2 1
3

r 1
r (r 1)
1(2)
2
1
1
RHS of P1 = (2) 2 3
2
2
P1 is true.

Assume that Pk is true for some k

We want to prove Pk 1

k 1

r 1

2r 1 r (r 1) (k 2)

LHS of Pk 1

r 1

k 1

r 1

, ie

1
.
k 1
is true, ie

2r 1 r (r 1) (k 1)
r 1

1
k 2
1

2r 1 r (r 1)
1

2r 1 r (r 1) + 2(k 1) 1 (k 1)(k 2)

1
1
+ 2(k 1) 1
k 1
(k 1)(k 2)
1
1
= k 2 2k 1 2(k 1) 1

(k 1)(k 2) k 1
1 (k 2)
= k 2 4k 4
(k 1)(k 2)
(k 1)
= (k 2)2
(k 1)(k 2)
1
RHS of Pk 1
= (k 2)2
k 2
Pk 1 is true

= (k 1)2

Since P1 is true and Pk is true Pk 1 is true, by Mathematical


Induction, Pn is true for all n
n

r 10

2r 1 r r 1 525
n

r 1

r 1

2r 1 r r 1 2r 1 r r 1 525

1
1
2
2

n 1 n 1 9 1 9 1 525

1
624.9
n 1
From G.C., n 24

n 1

Qn
5

Solution
P(3,
2)

C(-1, -1)
B

(i) Least value of z (3 2i) = PA


= PC CA
= 42 32 2
= 5 2
3

(ii) Largest value of arg( z (3 2i)) tan 1


4
2
= 2.21rad
Qn
6 (i)
(ii)
(iii)

(iv)

(v)

Solution
a=2
A translation of 2 units in negative y-direction.
1
1
2 ,

Asymptotes: x 0, x 1
1
y
2
f (| x |) 1 0
f (| x |) 2 1

f (| x |) a 1
From the diagram, the line y = 1 does not cut the
curve y a f ( x) for x 0, there is no solution for
f (| x |) a 1.
4

Number of real roots = 0.


Alternatives:
Consider graphs of y f (| x |) 2 and y 1.
Consider graphs of y f (| x |) 1 and x-axis.
Consider graphs of y f (| x |) and y 1.
Qn
7

Solution

1200
1 0.92
= 15 000

(i)

Theoretical total amount =

(ii)

Amt produced in nth year < 400


1200(0.92)n1 400
1
0.92n1

3
ln(1/ 3)
n 1
ln 0.92
n 1 13.176

14.176

n = 15, 16, 17
Farm A first produced less than 400kg of vegetables in the 15th
year.
(iii)

1200(1 0.9215 )
1 0.92
= 10 705
= 10 700 (3 s.f.)
Amt produced in nth year for farm B < 400
1200 (n 1)(78) 400
400 1200
n 1
78
400 1200
n

1
78
n
11.256
n
= 12, 13, 14
Farm B first produced less than 400kg of vegetables in the
12th year.
12
Total production of farm B =
(2(1200) (12 1)(78))
2
= 9 252
Since 10 705 > 9252, farm A produced more vegetables in
total.
Total production of farm A

Qn
8 (i)

(ii)

Solution

0
4


OA 6 and OB 2
1
1


4 0 4
1


AB 2 6 8 4 2
1 1 0
0


0
1


Vector equation of line through A and B: r 6 2
1
0


3 0 3

AC 6 6
2 1 1

3 1


6 2
1 0


125
2
1 22
3 2 12 625
15 2 25
2 40

or 2 10

(rejected, is positive)

= 20
3

AC 14 206
1

(iii)
Either Method 1 [3m]:

Required distance =
=

AC 125

206 125

=9

1

AC 2
0

1

2
0

Or Method 2 [3m]: Distance =

3 1

14 2
1 0

Qn
9

1
20

405
9
5

Solution

l
2x

2x
l

Let l be the length of the slant edge and A be the total surface
area of the octahedron.
Given total length of wire = 200 cm
8l 4(2 x) 200
l 25 x
h = height of the triangle

l 2 x2

(25 x)2 x 2

Area of the triangle

1
(2 x )h
2

x (25 x)2 x 2
x 625 50 x
A 8 x 625 50 x

Either Method 1:
1

dA
8 x (625 50 x ) 2 ( 50) 8 625 50 x
dx
2

200 x
8 625 50 x
625 50 x
200 x 8(625 50 x )
=
625 50 x
5000 600 x
=
625 50 x
dA
For stationary value of A,
0
dx
5000 600 x 0

25
1
8
3
3
Or Method 2:
A2 64 x2 (625 50 x) 40000 x 2 3200 x3
dA
2A
80000 x 9600 x 2
dx
dA
When
0, 80000 x 9600 x2 0
dx
25
1
x
8 (x 0)
3
3
Using lst derivative test,
x

dA
dA
0 and
0

dx x 25
dx x 25
3

Therefore A is maximum when x

25
.
3

Qn
Solution
10 (i) x 2 4 xy 2 y 2 k
dy
dy

2x 4 y x 4 y 0
dx
dx

dy
4 y 4x 4 y 2x
dx
dy 4 y 2 x

dx 4 y 4 x
2y x

2( y x)
(ii)
For k 2, x 2 4 xy 2 y 2 2

(1)
dy
0
For tangent parallel to the x-axis,
dx

2y x 0
x 2y
(2)
Substitute (2) into (1),
4 y 2 8 y 2 2 y 2 2

y2 1
y 1 or 1
x 2 or 2
Therefore the coordinates are (2, 1) and (2, 1) .
(iii)

(iv)

For k 2, x 2 4 xy 2 y 2 2.
dx
At x = 4 and y = 7,
2,
dt
dy dy dx

dt dx dt
2(7) 4

2
2(7 4)
10

(units/s)
3
For k > 0,
x 2 4 xy 2 y 2 k
(3)
Line // to y-axis: x a, a
(4)
2
2
Sub (4) into (3): a 4ay 2 y k
2 y 2 4ay a 2 k 0
Discriminant (4a)2 4(2)(a 2 k )
8a 2 8k 0 since a 2 0 and k 0 .
There are 2 distinct real solutions for y and thus 2
distinct points of intersection.

Qn
Solution
11 (i)
f ( x) x 2 2 x 3, x a

( x 1)2 2, x a
For f 1 to exist, greatest value of a 1
Let y ( x 1)2 2, y 2
Then ( x 1)2 y 2
x 1 y 2
since x 1 ,
x 1 y 2,
1

f :x

(ii)

R f 2,

y2

1 x 2, x , x 2 .

Dg 1,

R f Dg , gf exists

gf ( x) g f ( x) ,

(iii)

x 1

g x 2 2 x 3

ln ( x 2 2 x 3) 1 , x 1

ln x 2 2 x 2 , x 1

ln x 2 2 x 2 , x , x 1

gf : x

f
g
2,
0,
,1
R gf 0,

Qn
12
(a)

[or sketch the graph of gf]

Solution
1
(i) u
x
du
1
2 u 2
dx
x
1
When x 1, u 1, and x a, u .
a
1
a
1
1
1
1 1 x 2 dx 1a 1 2 u 2 du
1
u
1

1
du
1 u 1
1
1
1
du
2
a 1 u
a

(ii)

1
dx
1 x2

1
1
a

1
du
1 u2
1

tan 1 x
1

tan 1 u 1

tan 1 a tan 1 1

tan 1 1 tan 1

tan 1 a

tan 1 a tan 1

1
a

1
a

1
tan 1
4
a

.
2

10

(b)

(i)

1
0

x3
dx
1 x2

1
0

dx
x
2
1 x
1

x2 1

ln 1 x 2
2 2
0

1 1
ln 2
2 2

(ii)
1

3
1x
x3
1
1
0 x tan x dx 3 tan x 0 3 1 x 2 dx
0
1

1 1 x3

dx
12 3 0 1 x 2
11 1

ln 2
12 3 2 2

1 1

ln 2
12 6 6

11

JURONG JUNIOR COLLEGE


2

Higher 2 Paper 2

Section A: Pure Mathematics [40 marks]

A curve C has parametric equations

x 1 sin t sin t ,

y 1 sin t cos t ,

for

.
2
2
(i) Sketch C, giving the coordinates of any points of intersection with the x- and y-axes. [3]
t

(ii) Show that the area of the region enclosed by C and the x-axis, A is
2
2
2
cos t sin t cos t cos t 2sin t cos t dt sin t - sin t sin t cos t sin t dt ,

for some 0 ,
decimal places.

. Find the numerical value of A, giving your answer correct to 3


[4]

A researcher is investigating the spread of a certain disease in a town with a population of


3000 people. The researcher suggests that I, the number of people infected by the disease at
dI
time t days satisfies the differential equation
k I 3000 I , where k is a positive
dt
constant.
(i) Given that I 30 when t 0 , show that I

3000
.
1 99e3000 kt

[6]

(ii) It is further observed that I 240 when t 7, find the time it takes for 90% of the
population to be infected by the disease.
[3]
(iii) State, in the context of this question, one assumption needed to model the spread of the
disease in the town by the given differential equation.
[1]

Do not use a calculator in answering this question.


(i) The roots of the equation z 3 64i are z1 , z2 and z3 . Find z1 , z2 and z3 in the form
rei , where r 0 and .

[4]

(ii) Hence show that the roots w1 , w2 and w3 of the equation


(w 1)3 8i 0

lie on a circle with equation w 1 2 and are given by 1 2e

4 k 1
i

, k 1, 0, 1 .

[3]

(iii) Show w1 , w2 and w3 clearly by the points W1 , W2 and W3 respectively on an Argand


diagram and find the exact area of the triangle WW
[4]
1 2W3 .

3
4

The line l has equation

x 5 y 3 z 7
, and the plane p has equation 2 x y z 8 .

2
4
3

(i) Find the acute angle between l and p.

[3]

(ii) Find the point of intersection between l and p.

[3]

(iii) The plane q is perpendicular to p, and contains l. Find a cartesian equation of q.

[4]

(iv) A third plane s has equation r. 5i 10 j 8k 25 . Find the intersection of p, q and s.


[2]
Section B: Statistics [60 marks]
5

A school has 600 pupils. It is intended to obtain a sample of 12 pupils to attend a popular
course. All the 600 pupils are listed by class. A number, from 1 to 600, is allocated to each
name.
(i) Twelve random numbers, from 1 to 600, are obtained and the pupils whose names
correspond to those numbers are chosen. What is this type of sampling method called?
[1]
(ii) When the selection has been made, it is found that all of the numbers chosen are 500 or
less. A pupil in the school says, The sample obtained must be biased. Explain whether
the statement made is valid.
[1]
(iii) Describe, in this context, how a systematic sample can be obtained.

[1]

(iv) State, in this context, one advantage that systematic sampling has compared to the
sampling method described in part (i).
[1]

A machine is designed to make paper with mean thickness 56.8 micrometres. The thickness, T
micrometres, of a piece of paper made by the machine is normally distributed with standard
deviation 2.85 micrometres. A random sample of 300 pieces of the paper is taken and a test is
carried out, at the 5% significance level, to determine whether the machine is producing paper
of the desired thickness.
(i) State appropriate hypotheses for the test.

[1]

The sample mean thickness is denoted by t micrometres.


(ii) Use an algebraic method to calculate the range of values of t for which the null
hypothesis would be rejected.
[3]
(iii) State the conclusion of the test if t 56.9 .

[2]
[Turn over

4
7

The lifetime of a particular light bulb is normally distributed with mean 1100 hours and
standard deviation 38 hours.
(i) Find the probability that a randomly selected bulb has lifetime between 1100 hours and
1200 hours.
[1]
(ii) A random sample of 100 of these bulbs is selected. State the distribution of the number
of bulbs in the sample that have a lifetime between 1100 hours and 1200 hours. Use a
suitable approximation to find the probability that more than 40 bulbs have a lifetime
between 1100 hours and 1200 hours.
[4]
(iii) A company has a large number of these light bulbs installed. As soon as 1% of the bulbs
have come to the end of their lifetimes, it is company policy to replace all of the bulbs.
After how many hours, to the nearest integer, should the bulbs need to be replaced? [2]

The layout of a minivan consisting of eight passenger seats is as shown below.

driver

door

(i) To make the ride more comfortable, each of the eight passenger seats is to be attached
with either a red cushion or a blue cushion. Find the number of ways this can be done if
[2]
both colours are used.
Four boys and three girls board the minivan and occupy the passenger seats. Find the number
of possible seating arrangements
(ii) if there are no restrictions,

[1]

(iii) if no two boys sit next to each other when they are on the same side.

[4]

5
9

A bag contains 4 black balls and 6 white balls. Three balls are drawn from the bag at random,
one by one. If a black ball is drawn, it is replaced in the bag. If a white ball is drawn, it is not
replaced and 2 extra black balls are added to the bag.
(i) Show that the probability that all balls drawn are white is

1
.
11

[2]

(ii) Given that at least one of the balls drawn is black, find the probability that all balls
drawn are black.
[3]
A fourth ball is further drawn from the bag at random.
(iii) Find the probability that the third white ball appears on this fourth draw.

10

[3]

The number of flaws in a roll of microfleece fabric is known to follow a Poisson distribution
with mean . A roll of microfleece fabric is chosen at random. Given that the probability that
it contains no flaws is 0.9, find the value of , correct to 4 decimal places.
[2]
A random sample of n rolls of microfleece fabric is taken.
(a) If n 80 , find the probability that the mean number of flaws per roll is less than 0.1. [2]
(b) The probability that there is at most 1 roll containing no flaws is less than 0.02.
(i) Show that (0.1)n (1 9n) 0.02 .

[2]

(ii) Hence find the least value of n.

[2]

[Turn over

6
11

Flour is mixed with water to form glue. For the same amount of water used, the coefficient of
viscosity of glue produced (y N s m2 ) varies with the amount of flour added (x grams).
Measurements of y corresponding to six different values of x were made and the results are as
shown below.
x
y

10
5

20
21

30
30

40
38

50
46

60
50

(i) Draw a scatter diagram for the data.

[2]

(ii) Referring to the scatter diagram, comment on whether a linear model is appropriate. [2]
It is believed that y can be modelled by y a b ln x , where a and b are constants.
(iii) Calculate the least squares estimates of a and b.

[2]

(iv) Estimate the amount of flour required for the coefficient of viscosity of glue to be
[2]
70 N s m2 . Give your answer correct to 1 decimal place.
(v) Comment on the reliability of the estimate found in (iv).

12

[1]

In the vegetable section of a supermarket, leeks are sold loose and unprepared. The weight,
X grams , of an unprepared leek has a normal distribution with mean 260 grams and standard
deviation 24 grams.
(i) Two unprepared leeks are chosen. Find the probability that one of the leeks weighs less
than 300 grams and the other weighs more than 300 grams.
[2]
The supermarket also sells leeks prepared in packs of four. The prepared leeks have had 40%
of their weight removed, so that the weight, Y grams, of a prepared leek, is modelled by
Y 0.6 X .
(ii) Find the probability that the total weight of four prepared leeks in a pack is less than
[4]
600 grams .
Onions are also on sale in the vegetable section of the supermarket. The weight of an onion
has a normal distribution with mean 150 grams and standard deviation 18 grams.
(iii) Find the probability that the weight of an onion is within 10 grams of the weight of a
prepared leek.
[4]
(iv) State an assumption needed for your calculations in parts (i), (ii) and (iii).

[1]

1
JJC H2 Maths 2013 Prelim Paper 2 Solution
Qn
Solution
1
(i)
(0,1)

(ii)

(-2,0)
x 1 sin t sin t

sin(0,0)
t sin 2 t

y 1 sin t cos t cos t sin t cos t


dx
cos t 2sin t cos t
dt
dy
sin t cos 2 t sin 2 t
dt
Area of region bounded by C and x-axis, A
0

y dx x dy

1 sin t cos t cos t 2sin t cos t dt

1 sin t sin t sin t cos 2 t sin 2 t dt


0

cos t sin t cos t cos t 2sin t cos t dt

sin t - sin 2 t sin 2 t cos 2 t sin t dt


0

2.17810 0.17810
2.356

(i)

3 decimal places

dI
dt
1
dI
I 3000 I dt

k I 3000 I

k dt

3000 I 3000 3000 I dI

kt c

1
1
ln I
ln 3000 I
3000
3000

kt c

I 3000 I dI

ln

I
3000 I

I
3000 I
I
3000 I

3000kt 3000c

e3000 kt 3000 c

Ae3000 kt

3000 Ae3000 kt
1 Ae3000 kt
When t 0 , I 30 ,

3000 Ae3000 k 0
3000 k 0
1 Ae
30
1
A

2970 99
Therefore,
30

1
3000 e3000 kt
3000
99
I

1 99e3000 kt
1
1 e3000 kt
99
(ii)

When t 7, I 240 ,
3000
3000 k 7
1 99e
23
e21000 k
198
1
23
k
ln
21000 198
Therefore,
3000
I
1 23
240

ln

1 99e 7

t
198

When I 0.9 3000 2700 ,

1 23
ln
t
7 198

1
891

7
1
ln
22.1 days

23 891
ln
198
Assumption:
(1) Assume that the total population of the town is 3000
during the spread of the disease.
(2) Assume that a person infected by the disease will
remain infected by the disease.
(3) Assume that everyone in the town has an equal chance
of being infected by the disease.
t

(iii)

3
3

(i)

z 64 i
3

z 64e
3


i
2

4 k 1
i

z 4e

5
i
6

z1 4e

(ii)

4 e
3

i 2 k
2

4 e
3

4 k 1
i

, k .

, k 1, 0,1
1
i
6

, z2 4e

1
i

, z3 4e 2

(w 1)3 8 i

2(w 1)

64 i

From (i), 2( w 1) 4e

w 1 2e

4 k 1
i

4 k 1
i

, k 1, 0,1

, k 1, 0,1

w 1 2 (shown)
4 k 1
i

and w 1 2e
(iii)

, k 1, 0, 1 (shown)

W3

W1

Area of WW
1 2W3
2
1
3 (2) 2 sin

3
2
1 3
3 4

2 2
3 3

1
2
3

2
x

6
W2

4
Or
3

WW

2
2
cos

2 3
1 2

Perpendicular distance from W3 to WW


1 2
= 2 2sin

2 1 3

1
Area of the triangle WW
(2 3)(3) 3 3
1 2W3
2
(answer)

Or
Area of WW
1 2W3
1

(2 3)(2 3) sin
2
3
3
6

3 3
4

(i)
Equation of line l:

Equation of plane p:

5
2


r 3 4
7
3


2

r. 1 8
1

2 2

4 . 1
3 1

sin
2 2

4 1
3 1

sin

443

29 6
sin 13.1
(ii)

5
2 2


At point of intersection, 3 4 . 1 8 .


3 1
7
20 3 8
4
Position vector of point of

5 2 13

intersection 3 4 4 13 .
7 3 5

Point of intersection is (13, 13, 5) .
(iii)

2 2

A normal of plane q 4 1
3 1

1

8
10

5 1

3 . 8 5 24 70 41
7 10

Cartesian equation of plane q is x 8 y 10 z 41 .

(iv)

x 7 1.2 ,

By GC,

y 6 1.4 ,

z.

The

planes
intersect
6
7
5

r 6 7 , ..
5
0

at

the

line

(i)

Simple random sampling

(ii)

Not valid because every student has an equal chance of


being chosen as a member of the sample and one student
being chosen is independent of any other student then
being chosen.
Or:
Not valid because each possible sample of 12 students has
the same probability of being selected.

(iii)

600
50 . Select the first student randomly from the
12
first 50 students, then select every 50th student thereafter.

(iv)

The students selected are spread evenly over all the


classes.
Or:
Avoids clusters of students from a same class being

6
selected.
6

(i)

H 0 : 56.8

H1 : 56.8
(ii)

2.852
Under H 0 , T ~ N 56.8,
.
300

Test statistic: Z

T 56.8
2.852
300

N(0, 1) .

0.05 0.025

2
For H 0 to be rejected,
z 1.9600
t 56.8
1.9600
2

or

2.85
300

t 56.8 1.9600

2.85
300

2.852
300

t 56.5

z 1.9600
t 56.8
1.9600
2

t 56.8 1.9600

2.852
300

t 57.1

(iii)

Since t 56.9 does not fall within the critical region, we


do not reject H 0 at the 5% level of significance and
conclude there is insufficient evidence that the population
mean thickness of the paper is not 56.8 micrometres (or:
that the machine is not producing paper of the desired
thickness).

(i)

Let X h be the lifetime of a light bulb.


X ~ N(1100, 382 )
P(1100 X 1200) 0.49575

0.496 (3 s.f.)

(ii)

Let Y be the number of bulbs out of 100 that have a


lifetime between 1100 hours and 1200 hours.
Y ~ B(100, 0.49575)
Since np 100(0.49575) 49.575 5 and
nq 100(0.50425) 50.425 5 ,
npq 100(0.49575)(0.50425) 24.998
Y ~ N(49.575, 24.998) approximately.
C.C.
P(Y 40)

P(Y 40.5) 0.96524 0.965 (3 s.f.)

(iii)

7
Let k hours be the lifetime of bottom 1% of the bulbs.
P( X k ) 0.01
k 1011.60 (2 d.p.) 1012 (nearest integer)

The bulbs need to be replaced after 1012 hours.


8

(i)

No. of ways 28 2
254

(ii)

There are 8C7 ways of choosing 7 seats and 7! ways of


arranging the 7 persons among the chosen seats.
No. of possible arrangements 8C7 7! (or just 8! )
40320

(iii)

Two boys must be on each side and this can be done in


4
C2 ways.
On each side, the two boys can be separated in the
following 3 ways.
B

B
B

B
B

For each of the above 3 ways, there are 2! ways of


arranging the boys.
Lastly, arrange the 3 girls and 1 invisible person (empty
seat) among the leftover seats in 4! ways.
No. of possible arrangements 4C2 (3 2!)2 4!
5184

8
9

1st draw

3rd draw

2nd draw
4
10
4
10

B
6
10

6
11

4
10
6
10

6
11

6
10

(i)

(ii)

6 5 4

10 11 12

1
11

P(BBB)
1 P(WWW)
3

4

10

1
1
11
44

625
(iii)

P(3rd white ball appears on 4th draw)


P(WWBW) P(WBWW) P(BWWW)

6 5 8 4 6 6 5 4 4 6 5 4

10 11 12 12 10 11 11 12 10 10 11 12

0.14656 0.147 (3 s.f.)

5
11

6
11

5
11

8
12

4
12

P(all balls are black | at least one of the balls is black)


P(all balls are black)

P(at least one of the balls is black)

P(all balls are white)


P(WWW)

W
5
11

9
Let X be the number of flaws in a roll of microfleece
fabric.
X Po( )

10

P( X 0) 0.9

e 0.9
ln(0.9) 0.105361 0.1054 (4 d.p.)
(a)
Since

n 80 is

large,

by

CLT,

N ,

80

approximately.
P( X 0.1) 0.441 (3 s.f.)

(b)
(i)

Let Y be the number of rolls (out of n) containing no


flaws.
Y B(n, 0.9)
P(Y 1) 0.02

(0.1)n n(0.9)(0.1)n1 0.02


(0.1)n (1 9n) 0.02

(ii)

11

(i)

Using GC, n 3.17 .


Least value of n 4 .
y

(ii)

Since the data points seem to lie on a curve, a linear


model is not appropriate.

(iii)

Using GC, y 54.218 25.267ln x .


a 54.218 54.2 (3 s.f.)

and b 25.267 25.3 (3 s.f.).

(iv)

10
When y 70 , 70 54.218 25.267ln x .
x 136.485 136.5 (1 d.p.)

(v)

12

Since y 70 is outside the given range of data, this is


an extrapolation and the estimate may be unreliable.
Let X g be the weight of an unprepared leek.
X ~ N(260, 242 )

(i)

P( X 300) 0.95221
P( one weighs 300 g and the other weighs 300 g)
2(0.95221)(1 0.95221)
0.0910 (3 s.f.)

(ii)

Y 0.6 X
E(Y ) 0.6(260) 156

Var(Y ) 0.62 (242 ) 207.36


Y ~ N(156, 207.36)

E(Y1 Y2 Y3 Y4 ) 4(156) 624


Var(Y1 Y2 Y3 Y4 ) 4(207.36) 829.44
Y1 Y2 Y3 Y4

N(624, 829.44)

P(Y1 Y2 Y3 Y4 600) 0.20233 0.202 (3 s.f.)


(iii)

Let W g be the weight of an onion.


W ~ N(150, 182 )

E(W Y ) 150 156 6


Var(W Y ) 182 207.36 531.36
W Y

N( 6, 531.36)

P( W Y 10) P( 10 W Y 10)
0.32507 0.325 (3 s.f.)
(iv)

The distributions of the weights of all vegetables are


independent of one another.
Or:
The weight of each vegetable is independent of the
weight of any other vegetable.

INNOVA JUNIOR COLLEGE


JC 2 PRELIMINARY EXAMINATION 2
in preparation for General Certificate of Education Advanced Level

Higher 2
CANDIDATE
NAME
Civics Group

INDEX NUMBER

Mathematics

9740/01

Paper 1

19 Sep 2013
3 hours

Additional materials:

Answer Paper
Cover Page
List of Formulae (MF 15)

READ THESE INSTRUCTIONS FIRST


Do not open this booklet until you are told to do so.
Write your name, class and index number on all the work you hand in.
Write in dark blue or black pen on both sides of the paper.
You may use a soft pencil for any diagrams or graphs.
Do not use staples, paper clips, highlighters, glue or correction fluid.
Answer all the questions.
Give non-exact numerical answers correct to 3 significant figures, or 1 decimal place in the
case of angles in degrees, unless a different level of accuracy is specified in the question.
You are expected to use a graphic calculator.
Unsupported answers from a graphic calculator are allowed unless a question specifically
states otherwise.
Where unsupported answers from a graphic calculator are not allowed in a question, you are
required to present the mathematical steps using mathematical notations and not calculator
commands.
You are reminded of the need for clear presentation in your answers.
At the end of the examination, fasten all your work securely together.
The number of marks is given in brackets [ ] at the end of each question or part question.

This document consists of 5 printed pages and 1 blank page.


IJC/2013/JC2

Innova Junior College

9740/01/September 2013

[Turn over

2
1

A 2.4 km nature trail is divided into 3 sections, A, B and C. John took 25 min 58 s to
complete the trail by jogging while Mary took 13 min 36 s to complete the trail by
cycling. Their average speeds for each section of the trail are shown in the table below.
Section
A
B
C

Average jogging speed (m/s)


3.20
2.50
1.25

Average cycling speed (m/s)


5.00
4.00
2.50

Calculate the length, in metres, of each section of the trail.

[4]

The surface area of a sphere of radius r cm is decreasing at a rate of 2.5 cm2 s1 at an


instant when r 5 . Calculate the rate of decrease, at this instant, of the volume of the
sphere.
[5]
4
[The surface area of a sphere of radius r is 4 r 2 and the volume is r 3 .]
3

(i)

Find

(ii)

Hence find

1
d
1 e4 x 2 .

dx

2x

[1]

sin 1 e2 x dx .

[4]

y f ( x)
O

y b
xa

The diagram shows the sketch of y = f(x) with asymptotes x a and y b , passing
through the points c , 0 and 0 , d where a, b, c and d are positive constants.
(a)

State the root of the equation f ( x 4) 0 .

(b)

On separate diagrams, sketch the graphs of

[1]

(i)

y 2 f ( x) ,

[3]

(ii)

y f ( x) ,

[2]

stating the equations of any asymptotes and the coordinates of any points of
intersection with the axes, whenever possible. You should label the graphs clearly.

IJC/2013/JC2

9740/01/September 2013

[Turn over

3
5

(i)

Find the cartesian equation of the locus of a point P representing the complex
number z, where z 2 Imz . Sketch the locus on an Argand diagram.
2

(ii)

Hence find the greatest possible value of arg z 3 in exact form.

[4]
[2]

Let f ( x) 2e x1 cos2 ( x) .
(i)

Find the series expansion of f ( x) in ascending powers of x, up to and including


the term in x 3 .
[You may use standard results given in the List of Formulae (MF15).]

(ii)

(a)

Use your answer to part (i) to find a numerical value of the approximation
of

(b)

f ( x) dx .

[1]

Use your calculator to find an accurate value for


approximation in part (ii)(a) not very good?

[4]

f ( x) dx . Why is the

[2]

Referred to an origin O, the position vectors of two points A and B are a and b
respectively. State the geometrical interpretation of a b .

[1]

Given that a 2 and b 3 , find the value of a b a b .

[2]

Given further that the angle between a and b is

a b a b .

, find the exact value of


[4]

1 2x2 2
Find the expansion of
in ascending powers of x, up to and including the
4

term in x 2 .
(i)
(ii)

IJC/2013/JC2

[4]

Find the set of values of x for which the expansion is valid.


[2]
1
a
By putting x , show that 30 , where a and b are integers to be
4
b
determined.
[2]

9740/01/September 2013

[Turn over

4
9

Do not use a calculator in answering this question.


The complex number z is given by z k i 3 , where k is a non-zero real number.
(i)

Find z 4 in the form x iy .

(ii)

Given that z 4 is real, find the possible values of z, leaving your answers in exact
form.
[2]
For the value of z found in part (ii) for which k 0 , find the smallest positive

integer n such that arg( z n ) = . State the modulus of z n when n takes this
4

(iii)

[3]

value, leaving your answer in the form a b .

10

An increasing sequence u1 , u2 , u3 ,... is given by

u1
(i)

1
5

and

(ii)

5un
8un 1

for n 1.

5n 1
.
2 5n 1 3

[4]

Determine, with a reason, if the sequence converges.


2N

(iii)

un 1

Use the method of mathematical induction to prove that

un

Find

n N 1

11

[3]

5n
.
un

[2]
[3]

A curve C has parametric equations


x 1

(i)

1
1
, y t .
2
t
t

The point P on the curve has parameter p. Show that the equation of the tangent
at P is given by

p 2 p 2 1 x 2 py p 2 1 .
2

(ii)

[4]

This tangent passes through a fixed point X , Y . Give a brief argument to show
that there cannot be more than 4 tangents passing through X , Y .

[1]

(iii)

Sketch C, stating clearly the equation of any asymptote(s).

[2]

(iv)

Find the area of the region bounded by the curve and the lines x 2 and x 4 ,
giving your answer correct to 3 decimal places.
[3]

IJC/2013/JC2

9740/01/September 2013

[Turn over

5
12

The plane p 1 has equation 2 x 3 y z 8 and it meets the x- and z- axes at the points A
and B respectively. State the position vectors of A and B, relative to the origin O, and
z
show that a cartesian equation of the line AB is 4 x , y 0 .
[4]
2
The point C lies on AB such that

AC 1
. The plane p 2 passes through C and is
CB 3

parallel to the vector i 2 j . Given that p 2 is also perpendicular to p 1 , find

13

(i)

the cartesian equation of p 2 ,

[5]

(ii)

the perpendicular distance from A to p 2 .

[3]

It is given that

f ( x) (2 x 3) 2 2

for 2 x 1,
for 1 x 0,

and that f ( x) f ( x 2) for all real values of x.


(i)

5
5
Sketch the graph of y f ( x) for x .
2
2

(ii)

Find the exact value of

3
2

f ( x) dx .

[3]
[4]

The function h is defined by


h:x

(iii)

3
(2 x 3)2 2

for 2 x a .

Write down the greatest value of a such that h 1 exists.

[1]

Assume that a takes the value found in part (iii).


(iv)

Sketch, on a single diagram, the graphs of y h( x) and y h 1 ( x) .

(v)

Explain why the x-coordinate of the point of intersection of the curves in part (iv)
satisfies the equation

[2]

4 x3 12 x2 7 x 3 0 ,

and find the value of this x-coordinate, correct to 4 significant figures.

IJC/2013/JC2

9740/01/September 2013

[3]

[Turn over

BLANK PAGE

IJC/2013/JC2

9740/01/September 2013

[Turn over

Innova Junior College


H2 Mathematics Preliminary Examinations II
Suggested Marking Scheme
1

A 2.4 km nature trail is divided into 3 sections, A, B and C. John took 25 min 58 s to
complete the trail by jogging while Mary took 13 min 36 s to complete the trail by
cycling. Their average speeds for each section of the trail are shown in the table below.
Section
A
B
C

Average jogging speed (m/s)


3.20
2.50
1.25

Average cycling speed (m/s)


5.00
4.00
2.50

Calculate the length, in metres, of each section of the trail.


1

[4]

Let x m, y m and z m be the length of each section of


the trail.

x y z 2400 ------ (1)


x
y
z

1558 ------ (2)


3.20 2.50 1.25
x y
z

816 ------ (3)
5 4 2.50
Using GC, x 480 , y 320 and z 1600 .

The length of section A, B and C are 480 m, 320 m


and 1600 m respectively.

IJC/2013/JC2

9740/01/September 2013

[Turn over

The surface area of a sphere of radius r cm is decreasing at a rate of 2.5 cm 2 s1 at an


instant when r 5 . Calculate the rate of decrease, at this instant, of the volume of the
sphere.
[5]
4
[The surface area of a sphere of radius r is 4 r 2 and the volume is r 3 .]
3

dA
8 r
dr
4
dV
V r3
4 r 2
3
dr
A 4 r 2

When r = 5,
dr dr dA
1

2.5 0.0198943
dt dA dt 8 5
When r = 5,
dV dV dr
2

4 5 0.0198943 6.25
dt
dr dt
The rate of decrease of the volume of the sphere at the
instant when r = 5 is 6.25 cm3 s1.

IJC/2013/JC2

9740/01/September 2013

[Turn over

1
d
4x 2
1

.
dx

(i)

Find

(ii)

Hence find

2x

[1]

sin 1 e2 x dx .

3(i)

1
1
d
1
2e4 x
4x 2
4x 2
4x
1 e 2 1 e 4e
dx
1 e4 x

(ii)

[4]

e2 x sin 1 e2 x dx

1
e2 x sin 1 e2 x
2
1
e2 x sin 1 e2 x
2

2e2 x
1 e

4x

e4 x
1 e4 x

1
e2 x dx -----(*)
2
dx

1
1 2 x 1 2 x 1
4x 2
e sin e 1 e C
2
2

IJC/2013/JC2

9740/01/September 2013

[Turn over

y f ( x)
x

y b
xa

The diagram shows the sketch of y = f(x) with asymptotes x a and y b , passing
through the points c , 0 and 0 , d where a, b, c and d are positive constants.
(a)

State the root of the equation f ( x 4) 0 .

(b)

On separate diagrams, sketch the graphs of

y 2 f ( x) ,

(i)

[1]
[3]

y f ( x) ,
[2]
stating the equations of any asymptotes and the coordinates of any points of
intersection with the axes, whenever possible. You should label the graphs clearly.

(ii)

4(a) Root:
(b)(i) y 2 f ( x)

y
xa

y b

y b

(ii)

y f ( x)

y0

IJC/2013/JC2

xa

9740/01/September 2013

[Turn over

Find the cartesian equation of the locus of a point P representing the complex

(i)

number z, where z 2 Imz . Sketch the locus on an Argand diagram.


2

(i)

Hence find the greatest possible value of arg z 3 in exact form.

(ii)

[4]
[2]

z 2 Im z
2

x iy 2 y
x2 y 2 2 y
x2 y 2 2 y 0
2
x 2 y 1 1

Im(z)
2

(ii)

Re(z)

Im(z)
2
B
1 C

A
3

Re(z)

CAO
6
3

arg( z 3) 2
6 3
tan CAO

IJC/2013/JC2

9740/01/September 2013

[Turn over

Let f ( x) 2e x1 cos2 ( x) .
Find the series expansion of f ( x) in ascending powers of x, up to and including

(i)

the term in x 3 .
[You may use standard results given in the List of Formulae (MF15).]
(ii)

(a)

Use your answer to part (i) to find a numerical value of the approximation
of

(b)

f ( x) dx .

[1]

Use your calculator to find an accurate value for


approximation in part (ii)(a) not very good?

(i)

[4]

f ( x) dx . Why is the

[2]

f ( x)
2e x 1 cos 2 ( x)
e e x 1 cos 2 x

1
1
(2 x) 2

e 1 x x 2 x 3 ... 1 1
...
2
6
2

1
1

e 1 x x 2 x 3 ... 2 2 x 2 ...
2
6

e 2 2 x 2 2 x 2 x3 x 2 x3 ...
3

e 2 2 x x 2 x 3 ...
3

Alternatively,

f ( x) 2e e x cos2 ( x)

1
1 1

2e 1 x x 2 x3 1 x 2
2
6 2

1
1

2e 1 x x 2 x3 1 x 2
2
6

1
5

2e 1 x x 2 x3
2
6

(ii)
(a)
(b)

2
2

f ( x) dx

e 2 2 x x 2 x3 ... dx 7.25
3
2

f ( x) dx 10.9

Reason 1:
IJC/2013/JC2

9740/01/September 2013

[Turn over

The range of x-values of the limits in the integral are


not sufficiently small for the series expansion obtained
in (i) to provide a good approximation to f(x).
Reason 2:
The series expansion obtained in (i) does not contain
sufficient terms to provide a good approximation to
f(x).
Reason 3:
Using graphical method to show the difference
between the area of regions between each curve
and x-axis.. Thus the approximation is not good

IJC/2013/JC2

9740/01/September 2013

[Turn over

Referred to an origin O, the position vectors of two points A and B are a and b
respectively. State the geometrical interpretation of a b .

[1]

Given that a 2 and b 3 , find the value of a b a b .

[2]

Given further that the angle between a and b is

a b a b .
7

, find the exact value of


[4]

a b gives twice the area of triangle OAB.

a b a b a a b b a 2 b 2 4 9 5
a b a b a a a b b a b b
2 a b
2 a b sin

3
3
2 2 3
2
6 3
ALT: Apply geometrical interpretation of a b ,
So a b a b 2 a b

IJC/2013/JC2

(Award B2)

9740/01/September 2013

[Turn over

1 2x2 2
Find the expansion of
in ascending powers of x, up to and including the
4 x
term in x 2 .
[4]
(i) Find the set of values of x for which the expansion is valid.
[2]
1
a
(ii) By putting x , show that 30 , where a and b are integers to be
4
b
determined.
[2]
2

1 2x

4 x

1
2

1/2

x
1 2x
4 1
4
1 1

1 2 x 2 ...
2 2

1 3
1 x 2 2 x 2

...

2
2 4

1
1
3

1 x 2 ... 1 x
x 2 ...
2
8
128

1 1
3 2

1 x
x x 2 ...
2 8
128

1 1
131 2
1 x
x ...
2 8
128

2 1/2

1/2

(i)

2 x 2 1 and

x
1
4

1
1
x
and 4 x 4
2
2

Taking

(ii)

intersection, the
1
1
:
x
.
2
2

set

of

values

is

1
1 2
2
4 1 1 1 1 131 1



1
2
8
4
128

4
4
9 2243

30 4096
4096 12288
30 3

2243 2243

IJC/2013/JC2

9740/01/September 2013

[Turn over

Do not use a calculator in answering this question.


The complex number z is given by z k i 3 , where k is a non-zero real number.
(i)

Find z 4 in the form x iy .

(ii)

Given that z 4 is real, find the possible values of z, leaving your answers in exact
form.
[2]
For the value of z found in part (ii) for which k 0 , find the smallest positive

integer n such that arg( z n ) = . State the modulus of z n when n takes this
4

(iii)

[3]

value, leaving your answer in the form a b .


9(i)

z4 k i 3

k 4 4k 3 i 3 6 k 2 i 3 4 k i 3 i 3

k 4 18k 2 3 i 4 3k 3 12 3k
(ii)

z 4 real

[3]

4 3k 3 12 3k 0
4 3k k 2 3 0

k 0 (rej) or 3

z 3 i 3 or z 3 i 3
(iii)

arg 3 i 3

4
7 15
n arg 3 i 3 ,
,
,...
4 4 4
7 15
3
,
...
n ,
4 4 4
4
1 7 15
3
n , , ...
4 4 4
4
Least n 5

zn z 6

IJC/2013/JC2

36 6

9740/01/September 2013

[Turn over

10

An increasing sequence u1 , u2 , u3 ,... is given by

u1
(i)

1
5

and

un 1

Find

n N 1

(i)

5n 1
.
2 5n 1 3

[4]

Determine, with a reason, if the sequence converges.


2N

(iii)

for n 1.

Use the method of mathematical induction to prove that

un
(ii)

5un
8un 1

5n
.
un

Let Pn be the statement u n

[2]
[3]

5 n1
for n
2 5 n1 3

To prove P1 is true:
LHS u 1
RHS

1
5

50
1

0
2 5 3 5

Since LHS RHS, P1 is true.

Assume Pk is true for some k, i.e. u k

LHS

5 k 1
2 5 k 1 3

5u k
8u k 1

5 k 1

5
2 5 k 1 3

5 k 1
1
8
2 5 k 1 3

5k
8 5 k 1 2 5 k 1 3

5k
10 5 k 1 3

5k
RHS
2 5 k 3

(by Pk )

Pk is true Pk 1 is true.
Since P1 is true and Pk is true Pk 1 is true.
By Mathematical Induction, Pn is true for all n.
IJC/2013/JC2

9740/01/September 2013

[Turn over

(ii)

5 n 1

lim u n lim
n
n 2 5 n 1 3

1
1

lim
n 2 3 51 n

2
1
Since lim u n , which is a finite number, the sequence
n
2
converges.
OR
Using GC,

(iii)

2N

n N 1

, thus the sequence converges


2N

5n
2(5n ) 15

un n N 1

2(5N 1 )(5N 1)

15 N
5 1
1
(5N 1 )(5N 1) 15 N
2

IJC/2013/JC2

9740/01/September 2013

[Turn over

11

A curve C has parametric equations


x 1

1
1
, y t .
2
t
t

The point P on the curve has parameter p. Show that the equation of the tangent
at P is given by

(i)

p 2 p 2 1 x 2 py p 2 1 .
2

This tangent passes through a fixed point X , Y . Give a brief argument to show

(ii)

(i)

[4]

that there cannot be more than 4 tangents passing through X , Y .

[1]

(iii)

Sketch C, stating clearly the equation of any asymptote(s).

[2]

(iv)

Find the area of the region bounded by the curve and the lines x 2 and x 4 ,
giving your answer correct to 3 decimal places.
[3]

dx
2
dy
1
3 and
1 2
dt
t
dt
t

dy
t (t 2 1)

dx
2

At point P, gradient of tangent

p( p 2 1)
.
2

Equation of tangent:

1
p( p 2 1)
1
y p
x 1 2
p
2
p

p2 1
p( p 2 1)
p2 1

p
2
p2

p2 1
p( p 2 1)
( p 2 1)( p 2 1)

x
p
2
2p

2 py 2( p 2 1) p 2 ( p 2 1) x ( p 2 1)( p 2 1)
2 py p2 ( p2 1) x ( p2 1)( p2 1) 2( p2 1)
2 py p 2 ( p 2 1) x ( p 2 1) p 2 1 2

2 py p2 ( p2 1) x ( p 2 1)2
(ii)

Since the tangent passes through the fixed point

X ,Y

, we have 2 pY p 2 ( p 2 1) X ( p 2 1)2 ,

which is a polynomial equation of degree 4. Since this

IJC/2013/JC2

9740/01/September 2013

[Turn over

equation has at most 4 roots for p, therefore there


exists at most 4 tangents to the fixed point X , Y .
(iii)
y

x=1
(iv)

Area of the required region


2

y dx

1
3

1 2
t 3 dt
t t
1
4.262 (3 dec. pl.)
2

IJC/2013/JC2

9740/01/September 2013

[Turn over

12

The plane p 1 has equation 2 x 3 y z 8 and it meets the x- and z- axes at the points A
and B respectively. State the position vectors of A and B, relative to the origin O, and
z
show that a cartesian equation of the line AB is 4 x , y 0 .
[4]
2
The point C lies on AB such that

AC 1
. The plane p2 passes through C and is
CB 3

parallel to the vector i 2 j . Given that p2 is also perpendicular to p 1 , find


(i)

the cartesian equation of p2 ,

[5]

(ii)

the perpendicular distance from A to p2 .

[3]

At point A, y = 0 and z = 0, thus


Similarly, at point B, x = 0 and y = 0, thus
ALT: When x-axis meets

.
.

1 2

k 0 3 8 2k 8 k 4
0 1

Similarly, when z-axis meets
0 2

k 0 3 8
1 1

k 8

4 0


So OA 0 ; OB 0 .
0
8

4
1


AB 0 4 0
8
2

1

direction vector of line AB = 0 .
2

4
1


Thus vector equation of line AB is r 0 0 ,
0
2


.
x4 z
z
, y 0 , therefore 4 x , y 0 .
Thus
1
2
2

IJC/2013/JC2

9740/01/September 2013

[Turn over

(i)

12 3

AC 1
3 OA OB 1
0 0 .
OC
4
4
CB 3
8 2

is perpendicular to
i 2j

and parallel to the vector

1

n2 is perpendicular to both n1 and 2 .
0

2 1 2
2


3 2 1 n2 1 .
1 0 1
1


3 2

Since C lies on , 0 1 6 2 4
2 1

Cartesian ean of
is 2 x y z 4 .

(ii)

The perpendicular distance from A to


4 2

0 1 4
a n2 D2 0 1
2
4
6
or

3
n2
4 11
6
ALT: P(2,0,0) is a point on .
The perpendicular distance from A to
4 2 2

0 0 1
PA n2
0 0 1
4

n2
4 11
6

IJC/2013/JC2

9740/01/September 2013

[Turn over

13

It is given that

f ( x) (2 x 3) 2 2

for 2 x 1,
for 1 x 0,

and that f ( x) f ( x 2) for all real values of x.


(i)

5
5
Sketch the graph of y f ( x) for x .
2
2

(ii)

Find the exact value of

3
2

[3]

f ( x) dx .

[4]

The function h is defined by


h:x

(iii)

3
(2 x 3)2 2

for 2 x a .

Write down the greatest value of a such that h 1 exists.

[1]

Assume that a takes the value found in part (iii).


(iv)

Sketch, on a single diagram, the graphs of y h( x) and y h 1 ( x) .

(v)

Explain why the x-coordinate of the point of intersection of the curves in part (iv)
satisfies the equation

[2]

4 x3 12 x 7 x 3 0 ,

and find the value of this x-coordinate, correct to 4 significant figures.

IJC/2013/JC2

9740/01/September 2013

[3]

[Turn over

(i)

2.5

2.5
x

3
2

3
B1 correct graph for 2 x 1 and 1 x 0
B1 correct period with correct duplicates
B1 evidence of correct coordinates of end-points
(ii)

3
2

f ( x) dx

3
2

3
dx .(*)
(2 x 3) 2 2
1

3 3 1 2x 3 2
3
ln

2 2 2 2 2 x 3 2
2

(iii)

9
1 2
1 2

ln
ln

8 2
1 2
1 2

9
2 1
1 2

ln
ln

8 2
2 1
1 2

2 1
ln

4 2 1 2
9

Greatest value of a

IJC/2013/JC2

3
2

9740/01/September 2013

[Turn over

(iv)

3
2

(v)

Since the curves intersect at the line y = x, solving


h( x) h 1 ( x) is equivalent to solving h( x) x .

3
x
(2 x 3)2 2

3 x 4 x 2 12 x 9 2
4 x3 12 x2 7 x 3 0

Using GC, x 1.781 or


x 0.2808 (rej)

IJC/2013/JC2

x 1.5 (rej)

9740/01/September 2013

or

[Turn over

INNOVA JUNIOR COLLEGE


JC 2 PRELIMINARY EXAMINATION 2
in preparation for General Certificate of Education Advanced Level

Higher 2
CANDIDATE
NAME
Civics Group

INDEX NUMBER

Mathematics

9740/02

Paper 2

23 Sep 2013
3 hours

Additional materials:

Answer Paper
Cover Page
List of Formulae (MF 15)

READ THESE INSTRUCTIONS FIRST


Do not open this booklet until you are told to do so.
Write your name, class and index number on all the work you hand in.
Write in dark blue or black pen on both sides of the paper.
You may use a soft pencil for any diagrams or graphs.
Do not use staples, paper clips, highlighters, glue or correction fluid.
Answer all the questions.
Give non-exact numerical answers correct to 3 significant figures, or 1 decimal place in the
case of angles in degrees, unless a different level of accuracy is specified in the question.
You are expected to use a graphic calculator.
Unsupported answers from a graphic calculator are allowed unless a question specifically
states otherwise.
Where unsupported answers from a graphic calculator are not allowed in a question, you are
required to present the mathematical steps using mathematical notations and not calculator
commands.
You are reminded of the need for clear presentation in your answers.
At the end of the examination, fasten all your work securely together.
The number of marks is given in brackets [ ] at the end of each question or part question.

This document consists of 6 printed pages and 0 blank page.


Innova Junior College

IJC/2013/JC2

9740/02/September 2013

[Turn over

2
Section A: Pure Mathematics [40 marks]
1

The curve C has equation

2 y ax

2 ay 2 x ,

where a 0 . It is given that C has only one turning point.


(i)

2
Show that the turning point of C satisfies the equation 2 y ax .
a

[3]

(ii)

dy

2 a

2
d y
dx
. Determine whether the turning point is a
Show that

2
dx
4 y 2ax a
maximum or minimum.

(a)

[4]

A company predicts a yearly profit of $100 000 in the year 2013. The
company also predicts that the yearly profit will rise each year by 5%.
(i)

Show that the predicted profit in 2015 is $110 250.

[1]

(ii) Find the first year in which the yearly predicted profit exceeds $ 200 000.
[3]
(iii)

Find, in terms of m, the sum of the predicted yearly profits from the year
2015 till the year 2000 m , where m 15 .
[2]

(b)

An arithmetic progression has first term a and common difference d, where a


and d are non-zero. The second, seventh and ninth terms of the arithmetic
progression are consecutive terms of a geometric progression. Find, in terms of
d, the sum of the first 20 odd-numbered terms of the arithmetic progression. [4]

(a)

The curve C has equation y x 2 . The region R is bounded by C, the line


x 2 and the x-axis. The region S is bounded by C, the y-axis, and the line
y a , where a > 0. The volume of the solid formed when R is rotated

completely about the y-axis is denoted by VR. The volume of the solid formed
when S is rotated completely about the x-axis is denoted by VS. Given that
VR VS , find the exact value of a.
[6]
1
1
, the lines x , x
4
4
4
1 4 x2
and the x-axis. By using the substitution 2 x sin , find the exact value of the
volume of revolution formed when Q is rotated through 4 right angles about
the x-axis.
[5]

(b)

The region Q is bounded by the curve y

IJC/2013/JC2

9740/02/September 2013

By means of the substitution z (2 x 1)

(i)

2 x 1

dy
, express the differential equation
dx

d2 y
dy
2 1 as a differential equation involving z and x.
2
dx
dx

Find the solution of the differential equation 2 x 1

(ii)

y 2 and

[2]

d2 y
dy
2 1 given that
2
dx
dx

dy
2 when x 0.
dx

[5]

What can you say about the gradient of every solution curve as

(iii)

(a)

x ?

(b)

[1]

1
?
2

[1]

Sketch, on a single diagram, the graph of the solution found in part (ii), together
with 1 other member of the family of solution curves
[3]

(iv)

Section B: Statistics [60 marks]


5

Diabetes is present in 27% of the elderly population. In a health screening exercise, a


quick test for diabetes is used, but the test is not totally reliable. It has 0.98
probability of giving a positive result when an elderly has diabetes, and 0.04
probability of giving a positive result when an elderly does not have diabetes. A
randomly chosen elderly is given the test. Find the probability that
(i)

the result of the test is positive,

[2]

(ii)

the elderly has diabetes given that the result of the test is positive.

[2]

A family of 7 consisting of 4 adults and 3 children goes for dinner at a Japanese


restaurant. They choose to sit in a row of adjacent seats in front of the Sushi
Conveyor belt.
(i)

Find the number of possible seating arrangements if all the 3 children will sit
together between any 2 adults.
[2]

The family goes to a cafe for dessert after dinner and they sit at a round table.
(ii)

IJC/2013/JC2

Find the number of possible seating arrangements if at most two children can
be seated together.
[3]

9740/02/September 2013

[Turn over

4
7

One thousand five hundred employees of a large enterprise travel to work either by
car, by bus or by bicycle. Any given employee travels by the same method each day.
The numbers in each of the gender groups using each method of travel are
summarised in the table below.
Men
Women

Car
280
200

Bus
400
380

Bicycle
150
90

A researcher carries out a survey to investigate the length of journey times to work,
using a random sample of 100 employees.
(i)

Explain what is meant in this context by the term a random sample.

[2]

Another researcher decides to use stratified sampling with the data given in the table
above.

(ii)

Describe how a sample of 100 employees can be obtained.

(iii)

State one advantage that stratified sampling would have compared to random
sampling in this context.
[1]

(a)

The random variable X has the binomial distribution B(10, p) such that
P( X 2) 0.0325 . Write down an equation in terms of p and find the value of
p given that p 0.4 .

(b)

[3]

The random variable W has the binomial distribution B(25, 0.4) and the
independent random variable R has the binomial distribution B(30, 0.6). By
using suitable approximations, whose parameters should be stated, find
P R W 18 .

IJC/2013/JC2

[3]

[7]

9740/02/September 2013

5
9

John is training for National Shooting Championship and he practices shooting each
week. His percentage score, y % in week x, are as follows.

40

57

68

74

80

85

88

(i) Draw a scatter diagram to illustrate the data.

[2]

(ii) Calculate the value of the product moment correlation coefficient, and explain
why its value does not necessarily mean that a linear model is an appropriate
[2]
model for the relationship between x and y.
It is decided to fit a model of the form ln 95 y a bx .
(iii) Find, correct to 4 decimal places, the value of the product moment correlation
coefficient between ln 95 y and x and comment on its value.
[2]
(iv) Calculate the values of a and b correct to 4 decimal places. Use them to predict,
to the nearest whole number, the percentage score John will obtain for week 8.
Comment on the reliability of the estimate.
[4]
(v) Give an interpretation, in context, of the value 95 in the model
ln 95 y a bx .
[1]

IJC/2013/JC2

9740/02/September 2013

[Turn over

6
10

A website Koogle receives hits at a rate of 120 per hour. State two conditions
required for the number of hits obtained in a 1-minute interval to be well modelled
by a Poisson distribution.
[2]
Assume that the number of hits Koogle receives in a 1-minute interval follows a
Poisson distribution.
(i)

Find the probability that in a 2-minute interval, the number of hits received
by Koogle is less than 5.
[1]

Another website Noogle receives hits at a rate of 360 per hour. The number of hits
it receives in a 1-minute interval is an independent random variable that follows a
Poisson distribution.

11

(ii)

Find the probability that in a 1-minute interval, the total number of hits
received by both websites is more than 10.
[2]

(iii)

By using suitable approximations, find the probability that in a 30-minute


interval, the number of hits received by Noogle is at least 4 times as many
as that received by Koogle.
[6]

In a shopping mall, the number of hours that customers spend shopping in the mall
follows a normal distribution. The manager of the mall claims that the average
number of hours customers spend in the mall is at least 2 hours. To test this claim, a
random sample of 60 customers is taken and the time, x hours, spent is measured.
The results are summarized by

x 110.5 and x2 220.3 .


(i)
(ii)
(iii)

Find unbiased estimates of the population mean and variance.


Test at the 2% significance level whether the manager's claim is valid.
Explain, in the context of the question, the meaning of at the
significance level.

[3]
[4]
2%
[1]

After an upgrading project to improve the facilities in the mall, a new random
sample of 15 customers is taken. The data gives a sample mean of x with sample
standard deviation 0.51 hours. A test at the 2% significance level indicates that the
managers claim is valid with the improved facilities.
(iv)

IJC/2013/JC2

Using an algebraic method, calculate the range of values of x .

9740/02/September 2013

[5]

H2 Math Prelim 2 Paper 2 Mark Scheme

1.

Solution
(i)

2 y ax

2 ay 2 x
Differentiate with respect to x
dy

dy

2 2 y ax 2 a 2 a 2
dx

dx

d
y
d
y
2 y ax 2 a a 2 (*)
dx
dx

dy
0.
For stationary points,
dx
2 y ax a 2
2

2
a
Differentiate with respect to x again:
d 2 y dy
d2 y
dy

2 y ax 2 2 2 a 2 a a 2
dx
dx

dx dx
2y ax

(ii)

d2 y
d2 y
dy

a
2 a
2
2
d
x
d
x
d
x

2 y ax 2

d2 y
dy

4 y 2ax a 2 a
2
dx
dx

dy

2 a

2
d y
dx


2
dx
4 y 2ax a
At the turning point, 2y ax

2
dy
0.
(from (i)) and
a
dx

d2 y
a2
a3

0 since a 0
2
dx 2
4 a 4a
a

Therefore the turning point is a maximum turning point.


2.

(a)
(i)
(ii)

a 100 000 , r 1.05

Predicted profit in 2015 = u3 100000 1.05 =110250

100000 1.05

n1

200 000

1.05n1 2
ln 2
ln1.05
n 15.2 n 16
n 1

The 16th year is 2028


(iii)

Number of terms =
Sum =

110 250 1.05m14 1

1.05 1
2 205 000 1.05m14 1

Total profit = $ 2 205 000 1.05m14 1


(b)

u2 a d
u7 a 6d
u9 a 8d
a 8d a 6d

a 6d a d

a 8d a d a 6d 2

a2 9ad 8d 2 a2 12ad 36d 2


3a 28d 0
28
a d
3
Sum of first 20 odd-numbered terms of AP
20
2a 19(2d )
2
28

20 d 19d
3

3.

28

20 d 19d
3

580

d
3
(a)

y x2
a
R
-2

S
0

VR (2)2 (4)

VS (a)2

y dy 16 8 8

a5/2 x5
a5/2

x 4 dx

5/2
a

4 5/2
a
5
4 5/2
a 8
VR VS
5
a5/2 10
a 102/5

(b)

2 x sin

Volume of revolution of solid formed

2
2
2

1
4

1
4

dx

x2

dx

sin 2

1
cos d
4 1 sin 2
2

1 4 x2

sin 2 d

(1 cos 2 ) d

1
6
sin 2
8

(i)

(ii)

8 6 4
dy
z (2 x 1)
dx
dz
d2 y
dy
(2 x 1) 2 2
.(*)
dx
dx
dx
d2 y
dy
Given 2 x 1 2 2 1
dx
dx
2
d y
dy
2 x 1 2 2 2
dx
dx
dz
2
Substituting (*) ,
dx
dz
2
dx
1 dz 2 dx

4.

z 2x C

dy
2x C
dx
dy
2
When x = 0,
dx
dy 2 x 2
1

1
dx 2 x 1
(2 x 1)
dy
1
1
dx
(2 x 1)
1
y x ln 2 x 1 B
2
When x 0, y 2
(2 x 1)

2B

1
y x ln 2 x 1 2
2

(iii)

dy
1.
dx
1 dy

As x ,
2 dx
As x ,

(iv)
y

1
y x ln 2 x 1 2
2

2
x

1
y x ln 2 x 1 2
2

5.

(i)
0.27

0.73

0.98

+ve

0.02

-ve

0.04

+ve

with
diabetes

w/o
diabetes

1
2

0.96

-ve

P(the result is positive)

0.27 0.98 0.73 0.04


0.2938

(ii)

P(elderly has diabetes given that the result is positive)

P elderly has diabetes and the result is positive


P the result is positive

0.27 0.98
0.2938
0.9006126617 0.901
No. of ways = 4! 3C1 3!

6.

(i)

= 432

(ii)

Method 1:
No of ways = No restriction all 3 children seated together.
= 6! 4! 3!
= 576

Method 2:
No of ways = All children separated + 2 children together

3! 4 3 2 3! 3C2 (4 2!)(3)
7.

=144 + 432 = 576


(i)

(ii)

(iii)

8.

(a)

A random sample is a sample drawn from the population


of 1500 employees such that every employee has an
equal chance of being selected. The event that a
working adult is chosen or not is independent of the
event that any other adult is being chosen or not.
Using the given information, the strata used are the men
and women traveling by car, by bus and by bicycle.
strata

Car

Bus

Bicycle

Number
of men

280
100
1500
18.67 19

400
100
1500
26.67 27

150
100
1500
10

Number
of
women

200
100
1500
13.33 13

380
100
1500
25.33 25

90
100
1500
6

To obtain a sample of 100 working adults, draw random


sample of the required size from each stratum as shown
in the table.
Mention sample is more representative, with context, as
compared to random sample.
E.g.: One advantage is that using stratified sampling, the
proportion of employees in each stratum would be the
same as that among all 1500 employees, thus making the
sample more representative of the population than
random sampling, which may have omitted women who
travel by bicycle they only constituted 6% of the
population.
X ~ B(10, p), P( X 2) 0.0325
10 2
8
p (1 p) 0.0325
2

45 p 2 (1 p)8 0.0325 or equivalent
Method 1: Graphical
y 45 p2 (1 p)8 0.0325

y 45 p 2 (1 p)8
y 0.0325

or
Method 2: Using Eqn Solver

(b)

W ~ B(25, 0.4)
Since n = 25 is sufficiently large, np = 10 > 5, nq = 15 > 5,
W ~ N(10, 6) approximately.
R ~ B(30, 0.6)
Since n = 30 is sufficiently large, np = 18 > 5, nq = 12 > 5,
R ~ N(18, 7.2) approximately.
E(R W) = 18 10 = 8; Var(R W) = 7.2 + 6 = 13.2
R W ~ ( 8, 13.2 ) approximately.
Method 1

P R W 18
P R W 18 or R W 18
P R W 18.5 or R W 18.5 (by c.c.)

= 0.0019260924 0.00193
Method 2

P R W 18
1 P 18 R W 18
1 P 18.5 R W 18.5 (by c.c.)

=0.0019260924 0.00193
Note: As E(R W) 0, cannot use
P R W 18 2P R W 18 or

P R W 18 1 2P 0 R W 18 .

9.

(i)

(ii)

(iii)

(iv)

r = 0.961

A linear model is not suitable although the product


moment correlation coefficient is close to 1 as the points
on the scatter diagram follows the shape of a curve and it
is not possible for the percentage score to exceed 100 %
For P = 95,
ln 95 y a bx,
r = 0.9986 (4 d.p)
As compared to the previous r value of 0.961 which
indicates a strong positive linear correlation between x
and y, r = 0.9986 its absolute value is even closer to 1,
indicating a stronger linear correlation between the 2
variables.
Using GC,
ln 95 y 4.34055 0.33722 x
a = 4.3406 (4 d.p.)
b = 0.3372 (4 d.p.)
When x = 8,

ln 95 y 4.34055 0.33722(8)
95 y 5.16957
y 89.83
y 90

Percentage score = 90%

(v)

This estimate is not reliable as the value of x = 8 lies


outside the data range of values of x where the linear
relationship may not hold.
Since ln 95 y 4.34055 0.33722 x
OR y 95 e4.340550.33722 x

and y 95 as x .
The value of 95 is the maximum percentage score John
can achieve in the long run given his ability.

10.

Conditions:
(a) The hits occur independently / randomly
(b) The average number of hits in a 1-min interval remains
constant (or constant rate of hit against time).
(i)

Let the random variable K denote the no. of hits Koogle


receives in a n-minute interval.. Similarly, N no. of hits
Noogle receives in a n-minute interval. Thus K ~ Po(2n)
and N ~ Po(6n).
n = 2, K ~ Po(4).

P( K 5) P( K 4) 0.629 .

(ii)

n = 1, K + N ~ Po(8)

(iii)

n = 30, K ~ Po(60), E(K) = 60 > 10, K ~ N(60, 60)


approximately
N ~ Po(180). Similarly, N~N(180, 180) approximately.
E(N 4K ) = 180 240 = 60 ;
Var(N 4K ) = 180 + 1660 = 1140
N 4K ~ N(60, 1140)
P( N 4K ) P( N 4K 0) P( N 4K 0.5) (by c.c.)
0.0390

P( K N 10) 1 P( K N 10) 0.184

11. (i)

Unbiased estimates of the population mean

110.5
1.841666667 1.84 (3 s.f.)
60
1
110.52
s 2 220.3
0.2846751412 0.285 (3 s.f.)
59
60

(ii)

H0 :
H1 :
X is normal, n = 60 is large, use Z-test.
Under H0, test statistic Z

(iii)

(iv)

X 2
~ N (0,1)
s / 60

p-value = 0.0107623278
< 0.02
Reject H0 and conclude that at the 2% significance level, there
is sufficient evidence that the average time customers spend
in the mall is less than 2 hrs.
At the 2% significance level means there is 0.02 probability
that the test would indicate that the average number of
hours customers spend in the mall is less than 2 hrs when it
fact the managers claim is true (or it is more than 2 hrs).
H0 :
H1 :
X is normal, n = 15 is small,
is unknown, use T-test.

X 2
~ t14 .
S / 15
s 0.51
s2 1
2

0.51
n
n 14
14

Under H0, test statistic T

s2

15
0.512
14

Managers claim is valid H0 is not rejected


the test statistic lies outside the critical region.

x 2
2.263781249
0.51/ 14

x 1.691439296
x 1.69 (3 s. f .)

(v)

ln 95 y 4.34055 0.33722 x

y 95 e4.340550.33722 x
y = 95 is a horizontal asymptote
The value of 95 is the maximum percentage score John
can achieve given his ability.

HWA CHONG INSTITUTION


JC2 Preliminary Examinations
Higher 2

9740/01

MATHEMATICS
Paper 1

17 September 2013
3 hours

Additional Materials: Answer Paper


List of Formulae (MF15)

READ THESE INSTRUCTIONS FIRST


Write your name and CT group on all the work you hand in.
Write in dark blue or black pen on both sides of the paper.
You may use a soft pencil for any diagrams or graphs.
Do not use staples, paper clips, highlighters, glue or correction fluid.
Answer all the questions.
Give non-exact numerical answers correct to 3 significant figures, or 1 decimal place in the
case of angles in degrees, unless a different level of accuracy is specified in the question.
You are expected to use a graphic calculator.
Unsupported answers from a graphic calculator are allowed unless a question specifically
states otherwise.
Where unsupported answers from a graphic calculator are not allowed in a question, you are
required to present the mathematical steps using mathematical notations and not calculator
commands.
You are reminded of the need for clear presentation in your answers.
At the end of the examination, fasten all your work securely together.
The number of marks is given in brackets [ ] at the end of each question or part question.

This document consists of 6 printed pages.

Hwa Chong Institution 2013

9740 / 01 / JC2 Prelim 2013

[Turn Over

2
1

Given that a is a positive real number, solve the inequality


x2 x 2
0,
x 2 1 a x a
leaving your answer in terms of a .

[3]

A curve with equation y f x undergoes, in succession, the following


transformations.
Translation of 2 units in the negative x -direction,
1
II: Stretch with a scale factor of
parallel to the x -axis.
2
I:

The equation of the resulting curve is given by y ax3 bx 2 cx .


(i)

If the resulting curve has a stationary point at 1, 4 and meets the x -axis at

3, 0 , find the values of a , b


(ii)

and c .

[3]

Find f x .

[2]

There are two particles A and B with particle A at 13, 0 and particle B at

0, 9

with respect to origin O . A moves towards O at a constant speed of

u cms1 in the positive x -direction and B moves towards O at a constant speed of


4
u cm s 1 in the positive y -direction (see diagram).
3
y

A 13, 0

u cms1
4
u cm s 1
3

B 0, 9

Show that, after a time of t seconds, the distance between A and B is given by

25 2 2

u t 50ut 250 cm.


9

[2]

Find the coordinates of A and B at the instant when the two particles are closest. [4]

Hwa Chong Institution 2013

9740 / 01 / JC2 Prelim 2013

[Turn Over

3
4

A sequence of positive numbers x1, x2 , x3 ,


xn1

where n
(i)

is such that x1 1.5 and

n
2 n
xn 4
,
n 1
n

Find the least value of n such that xn 40 .

n 1 x

[1]

(ii)

By considering

n 1 n xn

, express xN in terms of N .

[5]

n 1

The points A and B lie on the circle with equation x 2 y 2 25 such that the angle

where O is the centre of the circle and is in radians. If is


6
sufficiently small, show that the line segment AB is given by

OAB is

AB 75 a b 2 2 ,

where a and b are constants to be determined.

[5]

Hence approximate AB as a quadratic polynomial in terms of .

[3]

The function f is defined by


f : x ln x a , x , a x a 2 ,
where a is a positive constant.
(i)

Show that f 1 exists.

[2]

(ii)

Find f 1 x and state the domain of f 1 .

[2]

(iii)

Find an expression for g x for each of the following cases.

(iv)

(a)

fg x x ,

[1]

(b)

gf x x 2 .

[1]

The function h is defined by


1
h:x
x 1 x 1 x 5 , x .
2
Find the range of hf .

Hwa Chong Institution 2013

9740 / 01 / JC2 Prelim 2013

[2]

[Turn Over

The curve C has equation y f x , where f x

3x 2 14 x 15
.
3x 1

(a)

Sketch C , stating the equations of any asymptotes, the coordinates of any


turning points and the points of intersection of C with the axes.
[4]

(b)

(i)

Find

(ii)

Let g x f x 2 . Find the area of the region bounded by the curve

3x

1
2

dx .

[2]

y g x , the x -axis, the lines x 1 and x 1 , giving your answer

in exact form.

[2]

A sequence un , where n 1, is defined as

un 3n 2 .
The sum S m , where m 2 , is defined as
Sm

1
.
um1um

3
.
10

(i)

Find the exact values of S 2 , S3 and show that S4

(ii)

By considering 3m 2 Sm , for m 2, 3, 4 , find a conjecture for S m in terms

[3]

of m , for m 2 .

[2]

(iii)

Prove your conjecture in part (ii) by mathematical induction.

[4]

(a)

Use the substitution y 1 2 x to find

(b)

(i)

n 1
sin d
n

where n

(ii)

x2 1

1 2 x

dx .

[4]

By writing sin n sin n1 sin , show that

1
1
1
+
+

u1u2
u2u3
u3u4

, n 2.

Hence find the exact value of

Hwa Chong Institution 2013

sin n2 d ,
[4]

sin 4 d .

[3]

9740 / 01 / JC2 Prelim 2013

[Turn Over

5
10

A group of medical researchers is investigating the spread of the H2C2 virus in a


province by keeping track of n , the population of infected people t weeks after the
virus is first discovered. Initially, 50 people are infected with the virus.
t

d2n
It is suspected that n and t are related by the differential equation 2 e 5 .
dt

(i)

Find the general solution of the differential equation.

(ii)

Explain why all solution curves of the differential equation are concave
upwards.
[1]

(iii)

Sketch on a single diagram, two distinct solution curves for the differential
equation to illustrate the following two cases:

(iv)

11

[3]

(a)

the population of infected people increases indefinitely,

(b)

the population of infected people stabilizes at a certain positive number.


[4]

Given that one week later, the population of infected people reduces to half of
its initial value, predict what will eventually happen to the population of
infected people.
[2]

Do not use a calculator in answering this question.


(i)

Given that 2 3i is a root of the equation


z 4 10 z 3 48z 2 122 z 143 0 ,
solve the equation, giving your answers in exact form.

(ii)

[4]

It is given that all the roots in part (i) satisfy the equation
wa b,
where w is a complex number, and a , b are non-zero real numbers.
Show that a 1 and find the exact value of b .

[4]

Sketch the locus of points representing w and all the roots obtained in part (i)
on a single Argand diagram.

[2]

Find the least value of arg w 5 6i , giving your answer in exact form. [2]

Hwa Chong Institution 2013

9740 / 01 / JC2 Prelim 2013

[Turn Over

6
12

The point A has coordinates 1, 2, 2 and the line l1 has Cartesian equation
x

(i)

y
z 1.
2

Find a Cartesian equation of the plane 1 which contains A and l1 .

The equation of the plane 2 is given by r j k d , where d

[3]

(ii)

Find the acute angle between 1 and 2 .

(iii)

The point B on 2 such that AB is perpendicular to 2 , find the position


vector of B , expressing your answer in terms of d .

[2]

[3]

Given that l2 is the line of intersection between 1 and 2 , and the shortest distance
from A to l2 is

2 6
.
3

(iv)

By using the answers in parts (ii) and (iii), deduce the value of d .

(v)

Given that the equation of the plane 3 is ax by 1 .

[3]

What can be said about the constants a and b if 1 , 2 and 3 have only
one point in common?

Hwa Chong Institution 2013

[3]

9740 / 01 / JC2 Prelim 2013

[Turn Over

2013 HCI H2 Mathematics Preliminary Paper 1 Solutions


Qn.
1

Solutions

x2 x 2
0
x 2 1 a x a
1
7
( x )2
2
4 0
( x a)( x 1)

Since numerator is always positive,


1
0
( x a )( x 1)
( x a )( x 1) 0
2

1 x a
Given y ax3 bx 2 cx

dy
3ax 2 2bx c
dx
At (1, 4), a + b + c = 4 --------(1)
3a + 2b + c = 0 ------(2)
At (3, 0), 27a + 9b + 3c = 0 9a + 3b + c = 0 -------(3)
Solving (1), (2) and (3),
a = 1, b = 6, c = 9.
1
x x

1 3 3 2 9
x x x
8
2
2
1
3
9
x x 2

y ( x 2)3 ( x 2) 2 ( x 2)
8
2
2
2
y x3 6 x 2 9 x
y

4
At time t s, distance between A and B, S (13 ut ) 2 (9 ut ) 2
3

169 26ut u 2t 2 81 24ut

Method 1:

dS 1 25 2 2

u t 50ut 250
dt 2 9

50 2
u t 50u
9

25 2 2
2
u t 50ut 250
9
When S is min.,

1
2

50 2

u t 50u
9

dS
50
0 u 2t 50u 0
dt
9

ut
50u 1 0
9

ut 9

Hence, S is min. when ut = 9.


Method 2:
25
S 2 u 2t 2 50ut 250
9
dS 50 2
2S
u t 50u
dt
9
dS 25 2
S
u t 25u
(1)
dt
9
When A and B are closest, S is minimum.

25 2 2
u t 50ut 250
9

16 2 2
ut
9

dS
25
0 u 2t 25u 0
dt
9
25
u 2t 25u
9
ut 9
Check that S is minimum:
Differentiate (1) again w.r.t. t,
2

d 2 S dS
25
S 2 u2
dt
9
dt

9 dS
d 2 S 25u 2
At t ,
0 2
0
u dt
dt
9S
Hence, S is min. when ut = 9.
x-coordinate of A = 13 + 9 = - 4;
4
y-coordinate of B 9 9 3
3
Coordinates of A are (4, 0) and B are (0, 3).
Method 3:
25 2 2
5

u t 50ut 250 u 2t 2 50ut 225 25


9
3

ut 15 25
3

Minimum occurs when


5
ut 15 0
3
ut 9
x-coordinate of A = 13 + 9 = - 4;
4
y-coordinate of B 9 9 3
3
Coordinates of A are (4, 0) and B are (0, 3).
4(i)

xn 1

n
2 n
x

n 1
n

n 1
2 ( n 1)
xn
xn 1 4

n
n 1

From GC, for xn 40 ,

least n = 21
(ii)

n 21

2 n
n
xn 1
xn 4

n 1
n
n 1 xn1 nxn 4n 2 n

n 1 xn1 n xn 4n 2 n
N

(n 1) x

n 1

n 1

2 x2
+ 3 x3
+ 4 x4

n xn

x1
2 x2
3 x3

N xn ( N 1) xN 1
+ ( N 1) xN 1 N xN
+

( N 1) xN 1 x1
( N 1) xN 1 1.5
N N 1

4n 2 4
N

n 1

1 1
1
2 2

1
1
2

1
2 N N 1 1
2

1
( N 1) xN 1 1.5 2 N N 1 1
2
N
1
1
1
xN 1
N
N

2
1


N 1
2
2
N

1
1 1
2N

2 N 1 2 N 1
N

1
xN 2 N 1
2

N 1

1 1

N 2N

x 2 y 2 25

2
AOB 2
2
6
3

Using cosine rule:

AB 2 52 52 2 5 5 cos
2
3

2
2

50 50 cos
cos 2 sin
sin 2
3
3

1 4 2

3
50 50 1
2

2 2
2

50 50 2 3
2

75 50 3 50 2
Hence

AB 75 50 3 50

1
2 2

a 50 3 and b 50

AB 75 50 3 50

1
2 2

2 3
2 2 2
75 1

3
3
1
2

1 1

12 3

2
2
2 2 2 3
75 1
2
2 ...
3
2! 3
3
2 3

12 3
2 1 4
75 2 1
2 2 ...
2 3

3 8 3

1
2

3
1
5 3 1
2 ...
3
2

6(i)

Method 1:
y

(a 2,ln 2)

a 1

Since every horizontal line cuts the graph at most once, f is one to one, therefore f 1 exists.
Method 2:
Since f is an increasing function within the domain, it is a one to one function and f 1 exists.
(ii)

Let y ln x a
x a ey
f 1 ( x) e x a,

x , x ln 2

Domain of f 1 Range of f = ( , ln2]

(iii)
(a)

fg x x
g( x) f 1 ( x) e x a

(b)

gf x x 2

g( x) gf f 1 ( x) e x a
(iv)

METHOD 1 (Using graph y = h(x))


1
h x x 1 x 1 x 5 , x
2
y
(3.43, 8.45)

ln 2
O

Rf : ( ,ln 2]
Rhf : ( ,8.45]
METHOD 2 (Using the composite graph y = hf(x))
y

(3.43, 8.45)

y hf x

x
(a 2, 1.48)

From graph, the range is given by (, 8.45].


7(a)

1
3

0,15
5
( ,0 )
3

y x 5

3, 0

(2.25, 0.169 )

2.92, 10.5

7(b) Method 1:
(i)
1
1
3x2 1 dx 3

1
3 tan 1
3
Method 2:

1
1
x

dx

3x C

3
tan 1
3

3x 1

dx

3x 1

(ii)

3x C

dx

1
tan 1
3

3x C

y g x

x
1

Note that g is symmetrical about the y-axis because


20
g x f x2 x2 5 2
3x 1

g x f x2 .

1
x3

20
1
2
2 x 5 2 dx 2 5 x 40
dx
2
0
0
3x 1
3x 1

3
0
1

28
1
40
tan 1
3
3

3x
0

28 40 1

tan
3 tan 1 0

3
3
28 40 40
28

3
3
3 3 3 3
u1 1 , u2 4 , u3 7 , u4 10

8(i)

1
1

(1)(4) 4
1
1
2
S3

(1)(4) (4)(7) 7
1
1
1
3
S4

(shown)
(1)(4) (4)(7) (7)(10) 10
1
For m 2, 3(2) 2 S2 = [4] = 1 = 2 1
4
2
For m 3, 3(3) 2 S3 = [7] = 2 = 3 1
7
3
For m 4, 3(4) 2 S4 = [10] = 3 = 4 1
10
S2

(ii)

for m 2 ,
3m 2 Sm m 1
Sm

(iii)

m 1
3m 2

Let Pm be the statement Sm

m 1
for all m
3m 2

When m = 2,
1
LHS = S 2
4
(2) 1 1
= LHS
RHS =
3(2) 2 4
P2 is true
Assume that Pk is true for some k

, k 2,

, m 2.

i.e. Sk

k 1
3k 2

To prove that Pk+1 is true, i.e. Sk 1

k
3k 1

LHS = Sk 1
1
1
1
1
=
+

u1u2
u2u3
uk 1uk
uk uk 1
1
k 1

+
3k 2 uk uk 1
1
k 1

+
3k 2 3k 2 3k 1

k 1 3k 1 + 1
3k 2 3k 1

3k 2 2k

3k 2 3k 1

3k 2 k
3k 2 3k 1

k
= RHS
3k 1
Pk+1 is true if Pk is true

Since P2 is true and Pk is true Pk+1 is true, by mathematical induction, Pm is true for all
m , m 2 .
9(a)

y 1 2x

x2 1
1 2x

dy
2
dx

dx
2

1 y

1 1

2
dy
y
2

5 2 y y2
8 y

dy

5
1
1
y 2 y 2 y 2 dy
8
4
8
5

1 2 1 2 5 2
y y y C --- *
20
6
4

5
3
1
1
1
5
1 2 x 2 1 2 x 2 1 2 x 2 C
20
6
4
u sin n 1

n
n 1
sin

sin

sin

2
2
u ' n 1 sin n 2 cos

9(b)
(i)

sin

d sin n 1 sin d
2

cos sin n 1 n 1 sin n 2 cos 2 d

n 1 sin n 2 cos 2 d

n 1 1 sin 2 sin n 2 d

n 1 sin n 2 sin n d

sin d n 1 sin n 2 sin n d

sin

d n 1 sin n 2 d n 1 sin n d
2

n sin n d n 1 sin n 2 d
2

sin

n 1
n2
d
sin

n 2

Method 1:
(ii)

v ' sin
v cos

sin 4 d

3 2
sin d
4 2

3 1
1d
4 2 2
3 1

4 2 2
3

16

Method 2:

sin

3 2
sin d
4 2
3
1 cos 2 d
8 2

3
sin 2

8
2

10
(i)


8
2
3

16
t

d2n
5

e
dt 2
t
t

dn
e 5 dt 5e 5 C
dt

t
5

n 25e Ct D
When t 0, n 50

50 25e0 C (0) D
D 25

(ii)

n 25e 5 Ct 25
t

d2n
5

e
0 for all values of t
dt 2

Solution curves are concave upwards.

(iii)

n 25e 5 Ct 25

When C 0

n 25e 5 25
As t , n 25

When C 1

n 25e 5 t 25
As t , n
C 1

n
50

C 0

25

t
(iv)

n 25e 5 Ct 25
t 1, n 25
25 25e

1
5

C 25

C 25e 20.468

t
5

n 25e 20.468t 25

n
50
2.03

The population of infected people will reduce to 0, i.e. there will no infected people after 2.03 weeks.
11

(i)

Since 2 3i is a root, its conjugate 2 3i is also a root.

z (2 3i) z (2 3i) ( z 2) 3i ( z 2) 3i
( z 2) 2 (3i) 2
z2 4z 4 9
z 2 4 z 13
By observation,

z 4 10 z 3 48z 2 122 z 143 ( z 2 4 z 13)( z 2 6 z 11)

OR
By long division,

z 2 6 z 11

z 2 4 z 13 z 4 10 z 3 48 z 2 122 z 143
z 4 4 z 3 13z 2
6 z 3 35 z 2
6 z 3 24 z 2 78 z
11z 2 44 z 143
11z 2 44 z 143
0

z 2 6 z 11 0
6 36 44
2
3 2 i

Hence the other roots are 2 3i , 3 2 i .


(ii) Since all 4 roots satisfy w a b , we must have
z1 a z2 a z3 a z4 a .

Take z1 a z2 a ,

(2 3i) a (3 2 i) a
(2 a) 3i (3 a) 2 i
(2 a ) 2 32 (3 a) 2 2
4 4a a 2 9 9 6a a 2 2

OR

2a 2
a 1
Im
3

z1 2 3i

z2 3 2 i

Re

z3 3 2 i

z4 2 3i

Distance from z1 to (a,0) and z2 to (a,0) must be the same.


Using Pythagoras Theorem,
(2 a) 2 32 (3 a) 2 ( 2) 2

a 2 4a 13 a 2 6a 11
2a 2
a 1
Subst a 1 into (2 3i) a b ,

b (2 3i) (1)
3 3i
32 32
18
3 2

A (5, 6)

Im

Locus of w

z1 2 3i

3 2
C (1, 0)

z2 3 2 i

1
O

5
z3 3 2 i

z4 2 3i
arg ( w 5 6i) arg w (5 6i)

Re

From the Argand diagram, AC 2 62 62 72


AC 72 6 2
sin

12.
(i)

3 2 1

6 2 2

11

Hence least value of arg ( w 5 6i)
12
2 4 6
0
1

Equation of l1 : r 0 2
1
1


l1
d
1 0 1

A
P
n1 2 0 2
2 1 1
1
1
= 4 0
1

1 0
equation: r 0 0
1 1

Plane 1

1

0 1
1

x z 1
12

The angle between two planes is same as the angle between two normal vectors.

(ii)
cos

1

0
1

0

1
1

2 2

600 or

1
2

12

Method 1:

(iii)

B is the foot of perpendicular from A to 2


A

l AB

1
0

: r 2 1
2
1

OB 2 for some
2

2
2

0

1 d
1

2 2 d

d
2

OB

d
2

2
Method 2:
Projected vector

AC n n
AB


n n
Where C is an arbitrary point on 2 : ( 0, d, 0)

1 0


d 2 1
2 1

AB
2

0

1
1

2

A
n

0
d
AB 1
2
1

d
d
OB OA 1 2
2
2
1
d
2

12
(iv)

600

l2

From right angle triangle ABC, AC


AB AC sin 600 =

2 6
3

2
3
2

2 6
, ACB 600
3

-------- (*)

1
d

Therefore AB OB OA 2
2 2

2
2

d
2

(d / 2)2 (d / 2)2 2

d2 d2

2
4
4
d2 4

d 2
Since d > 0 , d = 2
12
(v)

The direction vector of common line l2 between 1 and 2 is

1 0 1

d n1 n 2 0 1 1
1 1 1

In order for three planes have one common point, l2 must not be parallel to 3 .

1 a

1 b 0
1 0

l2

a b 0 or a b ,

a and b

and a b

HWA CHONG INSTITUTION


JC2 Preliminary Examinations
Higher 2

9740/02

MATHEMATICS
Paper 2

19 September 2013
3 hours

Additional Materials: Answer Paper


List of Formulae (MF15)

READ THESE INSTRUCTIONS FIRST


Write your name and CT group on all the work you hand in.
Write in dark blue or black pen on both sides of the paper.
You may use a soft pencil for any diagrams or graphs.
Do not use staples, paper clips, highlighters, glue or correction fluid.
Answer all the questions.
Give non-exact numerical answers correct to 3 significant figures, or 1 decimal place in the
case of angles in degrees, unless a different level of accuracy is specified in the question.
You are expected to use a graphic calculator.
Unsupported answers from a graphic calculator are allowed unless a question specifically
states otherwise.
Where unsupported answers from a graphic calculator are not allowed in a question, you are
required to present the mathematical steps using mathematical notations and not calculator
commands.
You are reminded of the need for clear presentation in your answers.
At the end of the examination, fasten all your work securely together.
The number of marks is given in brackets [ ] at the end of each question or part question.

This document consists of 6 printed pages.

Hwa Chong Institution 2013

9740 / 02 / JC2 Prelim 2013

[Turn Over

2
Section A: Pure Mathematics [40 marks]
1

(i)

Solve the equation

w5 1 i 3 0 ,
stating the roots clearly in the form rei , where r 0 and .
(ii)

Sketch on an Argand diagram, the set of points satisfying z 2 i z 2 . [3]

(iii)

A root w1 of the equation w5 1 i 3 0 is such that 0 arg ( w1 )


Find the value of z w1 such that it is a minimum.

[3]

(i)

.
[2]

Given that ln ky tan 1 kx , where k is a non-zero constant, show that

1 k x ddyx ky
2 2

. By further differentiation of this result, find the

Maclaurins series for y , in terms of k , up to and including the term in x3 . [5]


(ii)

Let k 1 . By using a suitable substitution for x , find, in terms of 3 , an


approximate value of

(a)

e6

[4]

Referred to the origin O , the points A and B are such that OA a and

OB b . The point C on OA is such that OC : CA 1: 2 , the point D on OB


is such that OD : DB 3: 4 and the point M on CD is such that 11CM 5CD .
(i)

Find OM in terms of a and b .

(ii)

By considering cross product, find the ratio of the area of triangle

OCM to the area of triangle OAB .


(b)

[3]

[3]

The points M and N have position vectors i j and i respectively. The



point P has position vector xi y j such that NM PM , PM PN and

PN NM are consecutive terms of an arithmetic progression. Show that the
locus of P is a circle, and find the radius and the coordinates of the centre of
this circle.

Hwa Chong Institution 2013

[5]

9740 / 02 / JC2 Prelim 2013

[Turn Over

3
4

A curve C has parametric equations


x tan ,

(a)

for

The point P on the curve has parameter , where

0 . The tangent
2
at P meets the y -axis at the point Q , show that the area of triangle OPQ is

given by
(b)

y 1 sec ,

1
tan sin .
2

[5]

(i)

Find a Cartesian equation of C , and sketch C , giving the coordinates


of any stationary points and the equations of any asymptotes.
[4]

(ii)

The region bounded by the curve, the lines x 3 , x 3 and the


x -axis is rotated through radians about the y -axis. Find the
volume of the solid obtained numerically.
[3]

Section B: Statistics [60 marks]


5

The manager of a popular fast-food restaurant wishes to sample 100 of the walk-in
customers on the quality of their food.
(i)

Describe how a quota sample can be obtained.

[2]

(ii)

Explain why stratified sampling is not an appropriate sampling method.

[1]

In a carton of apples, a sample of 8 apples is taken and examined for spoilt apples.
(i)

State, in context, an assumption for the number of spoilt apples in the sample
to be modelled by a binomial distribution.
[1]

The number of spoilt apples in a random sample of size 8 may be modelled by the
distribution B 8, p . If at least 2 apples in a sample are found to be spoilt, the carton
is rejected. It is known that the probability of a carton being rejected is 0.04 .

(ii)

Write down an equation satisfied by p and find the value of p .

(iii)

60 cartons of apples are loaded onto a lorry. Use a suitable approximation to


find the probability that more than 56 cartons loaded onto the lorry are not
rejected.
[3]

Hwa Chong Institution 2013

9740 / 02 / JC2 Prelim 2013

[3]

[Turn Over

4
7

The environment agency of a large city is tracking the number of dengue cases
reported across several months in year 2013 . The data recorded by its researcher is as
shown in the table below.
Month, t
Number of dengue cases, n

(i)

1
(Jan)

2
(Feb)

3
(Mar)

4
(Apr)

5
(May)

6
(Jun)

7
(Jul)

42

45

55

62

10

80

109

Draw a scatter diagram for the data. Identify one outlier and indicate it as P
on your diagram.
[2]

For the rest of the question, exclude P and use only the remaining six months data.
(ii)

Explain whether a linear model is appropriate.

(iii)

The following models are suggested for the data.


(B) n aebt ,

(A) n a bt 2 ,

[1]

(C) n a

where a and b are constants, and b 0 .

b
,
t

Without calculating the product moment correlation coefficient, state with a


reason which model is the most appropriate. Calculate the least squares
estimates for a and b using your selected model.
[3]
(iv)

It was discovered that there was an error in the transmission of the data. The
actual number of dengue cases n in each month was double that of the data
given above. By considering the relationship between n and n , write down
an appropriate regression model of n in terms of a , b and t .
[1]

Find the number of ways in which the eleven letters of the word ARRANGEMENT
can be arranged if
(i)

there are no restrictions,

(ii)

either the letter A is in the first position or the letter T is in the last position
but not both,
[3]

(iii)

between any two vowels there is exactly one consonant.

Hwa Chong Institution 2013

9740 / 02 / JC2 Prelim 2013

[1]

[3]

[Turn Over

5
9

A fish farmer breeds a new species of fish and investigates the life span, x years, of
this species of fish using a sample of 10 fishes. The results are summarized by

x 74.8

and

702.68 .

(i)

In a test at the 5% significance level, it is found that the mean life span is not
m years. Find the set of possible values of m and state an assumption made
for the test to be valid.
[6]

(ii)

Instead, the test is carried out at 2.5% significance level with the following
hypotheses.
Null hypothesis:

The mean life span of fish is 9 years.

Alternative hypothesis:

The mean life span of fish is less than 9 years.

Without carrying out any calculations, write down your conclusion of the test.
Justify your answer.
[1]

10

A bag contains 15 tokens that are indistinguishable apart from their colours. 2 of the
tokens are blue and the rest are either red or green. Participants are required to draw
the tokens randomly, one at a time, from the bag without replacement.
(i)

Given that the probability that a participant draws 2 red tokens on the first 2
1
draws is
, show that there are 3 red tokens in the bag.
[3]
35

(ii)

Find the probability that a participant draws a red or green token on the second
draw.
[2]

Events A and B are defined as follows.

A:

A participant draws his/her second red token on the third draw.

B:

A participant draws a blue token on the second draw.

(iii)

Find P A B .

[2]

(iv)

Determine if A and B are independent events.

[1]

Hwa Chong Institution 2013

9740 / 02 / JC2 Prelim 2013

[Turn Over

6
11

The time taken S (in minutes) for Tom to complete any statistics question has
distribution N 10, 2.42 .
(a)

(i)

By considering the normal distribution graph of S , explain why

P S 11 P S 9 .
(ii)

[1]

Tom attempts 4 statistics questions. Find the probability that 3 of the


questions take him less than 9 minutes each and 1 of the questions
takes him more than 11 minutes to complete.
[2]

The time taken C (in minutes) for him to complete any calculus question has
distribution N ,1.62 .
(b)

It is given that the probability that Tom completes a statistics question within
9 minutes is the same as the probability that he takes more than 7 minutes to
complete a calculus question. By using the method of standardization, find the
value of .
[3]

(c)

Instead, it is given that 6.4 . Find the probability that the time Tom takes
to complete 3 calculus questions is more than twice the time he takes to
complete a statistics question by less than a minute.
[3]

12

Ponding is observed to occur singly along Bukit Road.


(i)

State, in context, two assumptions needed for the number of ponding


occurrences along Bukit Road to be well modelled by a Poisson distribution.
[2]

On average, there are 4 ponding occurrences along Bukit Road in a year.


(ii)

In a particular year, the probability that at least n ponding occurrences are


observed is at most 0.05 . Find the least value of n .
[2]

(iii)

Find the probability that, along Bukit Road, there are at most 5 ponding
occurrences in a particular year given that there are at least 3 ponding
occurrences in the first 6 months of the same year.
[4]

On average, ponding is observed to occur singly along Orchid Road 3 times a year.
(iv)

Find the probability that over a period of 5 years, there are at least 10 more
ponding occurrences along Bukit Road than ponding occurrences along
Orchid Road.
[4]

Hwa Chong Institution 2013

9740 / 02 / JC2 Prelim 2013

[Turn Over

2013 HCI H2 Math Prelim Paper 2 Solutions


Qn.
1

Solutions

w5 1 3 i 0

(i)

w5 1 3 i
w5 (1) 2 ( 3) 2 2
3 2
arg ( w5 ) tan 1

1 3

w5 2e

1
5

2
3

w2 e
1
5

or

w2 e

2
i

15

1 2

i
2 k
5 3

1
5

,2 e

8
i

15

k 2, 1, 0, 1, 2
1
5

,2 e

14
i

15

1
5

,2 e

4
i

15

1
5

,2 e

2
i

Special note:
This is a standard question. Students who failed
to get the correct argument of 1 3 i will get
no credit at all for this part.
(ii)

z 2 iz 2
2

z (2) i z
i

z (2) i z 2i
z (2) z 2i

We shade the region where the distance between the point representing z and (-2,0) is the
distance between the point and (0,2).

Hwa Chong Institution 2013

Im
9740/02/Prelim/13

1
5

w1 2 e

(iii)

2
15

From diagram in (ii),


2 x
sin
1
4 15
25
1

23
1.0724023
60
minimum z w1 1.07
x 2 5 sin

2i)

ln ky tan 1 kx

1 dy
k

y dx 1 kx 2

1 k x ddyx ky
2

1 k x ddxy 2k
2

--- (shown)
2

dy
dy
k
dx
dx

2
d2 y
d2 y
2 d y
2 dy

2
k
x

2
k

k
dx 2
dx 2
dx
dx 2

1 k x ddxy 2k
2

Hwa Chong Institution 2013

9740/02/Prelim/13

1 k x ddxy 4k
2

x k

d2 y
dy
2k 2
0
2
dx
dx

When x 0 , ln ky tan 1 0 0 ky 1 ,

d2 y
d3 y
1 dy
,
1 , 2 k , 3 k 2
k dx
dx
dx

1
k
k2
x x 2 x3 ...
k
2
6

Note: y
(ii)

1 tan 1 kx
e
k

1 tan 1 kx 1
k 2 k2 3
y e
x x x ...
k
k
2
6
When k 1 , etan

ye

tan 1 x

tan 1 x

1 x

1 2 1 3
x x ...
2
6

e6

1
3

1
on both sides of
3
1
1
1 x x 2 x3 ... , we have
2
6

By substituting x

tan 1 x

ye

1
tan 1

1 1 1 1 1
e 1

3 2 3 6 3

1 1 1 1 7 17 1

3 6 18 3 6 18 3

3(a)

Since 11CM = 5CD,

(i)

CM : MD = 5 : 6.

By ratio theorem,

Hwa Chong Institution 2013

9740/02/Prelim/13

3
1

5 OB 6 OA
5OD 6OC
7
3
2 a+ 15 b
OM
=
=
11
11
11 77
2(a)
(ii)

Area of triangle of OCM =


=

1 1 2
15
a a+ b
2 3 11 77

1
2

1
OC OM
2

2 1 15
1 1 15
1
a b
a a+ a b =
77
2 3
77
3 11 3

1 15 1
5 1

= ab = ab
77 2

3 77 2
5
=
(area of triangle OAB)
77
Therefore the ratio of triangle OCM to area of triangle OAB is 5:77
3(b)

2
1 x
1 x
NM PM
PN

1 y ,
y
1 ,
NM PM 2 2 x 1 y 3 2 x y

PM PN x2 1 y y 2
PN NM 2 2 x y
Since the dot products formed consecutive terms of an AP,

( x2 1 y y 2 ) (3 2 x y) (2 2 x y) ( x2 1 y y 2 )

2 x2 2 y 2 4 x 3 0
( x 1)2 y 2
r

4
(a)

5
,
2

5
(shown)
2
Centre (1 , 0 )

dx
sec2
d
dy
y 1 sec
sec tan
d
dy sec tan sin

cos sin
dx
sec2
cos
x tan

Hwa Chong Institution 2013

9740/02/Prelim/13

5
Equation of the tangent is:
y (1 sec ) sin ( x tan )
y (sin ) x sin tan sec 1
y (sin ) x sec (1 sin 2 ) 1
y (sin ) x cos 1

y
R

Q
x

O
At Q, x 0, y cos 1

Area of triangle OPQ =

1
(OQ)( PR)
2

1
(1 cos )( tan ) (
2
(b)
(i)

x tan
y 1 sec sec y 1

1
tan (1 cos )

2
0 PR tan )
1
2
(tan sin )
2

Using trigonometric identity 1 + tan2 = sec2 to find the Cartesian equation, we have

1 x 2 ( y 1) 2
( y 1)2 x 2 1, y 1
To find equations of the asymptotes: ( y 1)2 x2 0
y

( y 1)2 x2 y 1 x .

y 1 x

y 1 x

(0, 2)
x

Hwa Chong Institution 2013

9740/02/Prelim/13

Only the upper half of the graph is sketched because


(ii)
3

y
R

2
O

When x 3, ( y 1)2 3 1 y 2 1 3 or 1.
Since y > 0, y = 3.
Required volume
= volume of cylinder volume generated by region R
3

= r 2 h x 2dy
2

3 3 ( y 1)
2

1 dy

23
24.1 unit 3
3

Hwa Chong Institution 2013

9740/02/Prelim/13

5 (i)

Quota sampling can be carried out as follows:

Manager will then pick customers at his own discretion


according to the age group and quota specified in the table
till all the quotas are met.
20 yrs
30

21-30
30

31-40
20

41
20

(ii)

In order to use stratified sampling, the manager will need to


know the composition of the customers (sampling frame)
according to strata, i.e. race, gender, age etc. However, the
manager may not have such information on walk-in
customers.
Or
The sampling frame in this situation is unknown since the
walk-in customers vary from day to day.

6(i)

The probability of obtaining a spoilt apple is constant.

Or
Obtaining a spoilt apple is independent of other spoilt
apples.

(ii)

Let A be the number of spoilt apples in a sample of 8.


A B 8, p

P( A 2) 0.04
1 P( A 1) 0.04

P( A 0) P( A 1) 0.96

(iii)

(1 p)8 8 p(1 p)7 0.96


From G.C. p 0.0410
Let C be the number of cartons that are not rejected out of
60. C B 60,0.96
n = 60 is large, np 57.6 5, n(1 p) 2.4 5
Let C be the number of cartons that are rejected out of 60.
C ' B 60,0.04 ---- (*)
Since n = 60 is large, np 2.4 5 ,
C ' Po 2.4 approximately ---- (**)
Required probability
= P(C 56) P(C ' 3) 0.779

Hwa Chong Institution 2013

9740/02/Prelim/13

8
7(i)

n
109

10
1

The outlier is that of month May, labelled as P on the


diagram.

Several students did


not have sketch with
sense of
proportionality of the
points. Without
proper scale used, the
regression line/curve
and the concavity is
not well shown.
Uneven width of
intervals will lead to
distortion to
perception of
regression line / curve.
The maximum and
minimum points are
not labeled.

(ii)

The scatter diagram shows a curvilinear relationship


between n and t. Thus a linear model is inappropriate.

Several students
carelessly obtained a
low r value using 7
data points to explain.

(iii)

Method 1:
Model A

Several students did


not comment of the
model A and C after
indicating the choice
of Model B.
Students should avoid
quoting specific
functions with
equations to explain.
Easier to draw general
graphs to explain
Avoid using term
sloping up / down.
Use term like concave
upward/downward.
Students are careless
in using 7 data points
to obtain the equation
of the regression line
wrongly, or do not
know how to get the
equation via
transformation.

Model B

Model C

Model B is most appropriate as it concaves upwards and n


increases as t increases like the scattered diagram drawn in
(i).
Method 2:
Model A has graphs which are concave downward.

Hwa Chong Institution 2013

9740/02/Prelim/13

9
Model B has graphs which are concave upward with
increasing n values as t increases.
Model C has graphs that concave upward for t > 0 and has
decreasing n values as t increases.
The scatter diagram for the data suggests Model B is most
appropriate.
n aebt

A number of students
did not find the values
of a and b. Either they
do not understand the
question or have
missed out that part.

ln n ln a bt
ln n 3.53539204 0.1539053017t
a e3.53539204 = 34.308462 = 34.3 (in 3 s.f.)
b 0.154 (in 3 s.f.)

(iii)

Students did not read


question carefully that
n has to be the
subject and has to be
expressed in terms of
a, b and t.
Some mistakes are
having answers
1
like n aebt or
2
answers with ln n as
the subject.

n aebt
n 2n
2aebt

8(i)

No. of ways =

11!
2494800
2!2!2!2!

(ii)

Method 1:

3 9! 3 9!
A in first and not T in last =

408240
2!2! 2!2!2!

Hwa Chong Institution 2013

9740/02/Prelim/13

Considerable number
of students obtained
11!
4989600
2!2!2!2!
.
Its likely a result of
misconception that
2!2!2!2! = 4(2!).
tudents who have
working like 11!
suggest that they yet to
revise conceptions of
counting with identical
objects.
Several students did
not specify what they
are computing, and
simply write out the
computation. Some

10
A not first and T in last =

3 9!
2 9!

181440
2!2!2! 2!2!2!2!

Total number of ways = 408240 181440 589680


Method 2:
No. of ways for A in first or T in last position
10!
10!
9!

2!2!2! 2!2!2!2! 2!2!2!


635040

No. of ways for A in first or T in last position but not


both
9!
635040
2!2!2!
589680

be 10!

10!
2!2!2! 2!2!2!2!

(iii)

There are 5 possible positions for the group of


A_A_E_E.
7!
4!
No. of ways 5

37800
2!2! 2!2!
Alternative:
Case 1 2 R + 0 N or 2 N + 0 R within vowels
4! 3
3! 5!
C1
2 6480
2!2!
2! 2!2!
Case 2 2 R + 1 N or 2 N + 1 R within vowels
4! 3!
5! 2 4320
2!2! 2!
Case 3 0 R + 0 N within vowels
4!
5!
3!
1080
2!2!
2!2!
Case 4 1 R + 0 N or 1 N + 0 R

Hwa Chong Institution 2013

9740/02/Prelim/13

practices are
discouraged as
markers are not sure if
students are working
(e.g. case of A in
first or case of A in
first and T not in
last).
Students may not have
understood the part on
question that states
not both, and unable
to work out the correct
computations. Several
interpreted solution to
only or 635040 as the
final answer.
Students need to know
that n A B is not
what the question
needs., and that its
not n A n B too.

Many students were


unable to see that the
arrangement of
consonants within the
group with vowels and
outside the group can
be done together to
7!
give
.
2!2!
For students who did
the listing of cases,
most of them did not
cover all the cases.
The cases are
o 2R+0N or 2N+0R
o 2R+1N or 2N+1R
o 0R+0N
o 1R+0N or 1N+0R
o 1R+1R
Another common error
for this group of

11
4! 3
5!
C2 3! 2 12960
2!2!
2!

Case 5 1 R + 1 N within vowels


4! 3
C1 3! 5! 12960
2!2!
Total = 37800

9(i)

H0 : m
H1 : m
Level of significance: 5%

Under H 0 , test statistic T

X m
s

t (9)

10

74.8
7.48
10
1
74.82
s 2 702.68
15.90844444
9
10
s = 3.988539136
x

Since we reject H 0 at 5% level of significance, we have

7.48 m
s

7.48 m
s

2.262157158

10

2.262157

10

or

7.48 m
s

2.262157

10

m 10.33 or m 4.627
m 10.4 or m 4.62 (to 3 sf)
m m 4.62 or m 10.4

Assumption is that the life span of the species of fish


follows a normal distribution.
Note:

Hwa Chong Institution 2013

9740/02/Prelim/13

students was that


many did not account
for the arrangement of
the vowel + consonant
group with the
remaining consonants
on the case by case
basis. Not encouraged
to do this method as it
is very long.
There was also a
significant number of
students did not take
into account that there
are identical letters in
the arrangement.
Poor presentation
o H0 and H1 were
statements made
about population
mean . It should
not contain
0 , 1, x or x .
o Many students had
the misconceptions
that x and

s2 2 .
o Test statistic T is
obtained by
standardizing X .
Thus, it is
inappropriate to
X
write T
s

10

o Many students
wrote
7.48 m
~ t (9)
15.9084
10
which is
inappropriate (it is a
value of the test
statistic so it cannot
have a distribution);

12
How to find critical values for t-test
Use invT function within GC

X m
s

10

is a

random variable
and thus, it can
follow a
distribution.
o The parameter for tdistribution is
degree of freedom
(which is n 1)
NOT mean and
variance! DO NOT
CONFUSE it with
normal distribution.
o GC keystrokes
should not be
written as part of
workings.
o Incorrect inequality
representation such
as 4.62 m 10.4
o Did not gave final
answer in set
notation
o Did not use more
accurate answer (>
3sf) for
intermediate
workings
Many students were
aware that it was a 2tail test but they only
gave 1 critical region!
There was also a
handful of students
who gave the region as
the one which H0 was
for not rejected.
A significant number
of students were
unable to find the
critical values due to
the following:
o used normal
distribution (GC

Hwa Chong Institution 2013

9740/02/Prelim/13

13

(ii)

Since m = 9 does not lie in the range of values in (i), we do


not reject H 0 for the 2-tail test at 5% level of significance.

Thus, for the 1-tail (left tail) test at 2.5% level of


significance, we also do not reject H 0 as test statistic also
lie outside the rejection region.
Conclusion is there is no significant evidence that the mean
life span of fish is less than 9 years at 2.5% level of
significance.

For two tail test at 5% level of significance:

2.5%

2.5%

test statistic lies here

For one tail (left tail) test at 2.5% level of significance:

Hwa Chong Institution 2013

9740/02/Prelim/13

keystroke invNorm)
instead of tdistribution (GC
keystroke invT)
o used probability
such as 0.5, 0.05,
0.25 or degree of
freedom 10
A handful of students
had a misconception
that CLT is necessary
to make the
assumption. CLT
cannot be applied here
as n = 10 is small!
The assumption made
should be about the
lifespan of the species
of fish X (which
should be defined first
before using) NOT
X or sample or
population.
Generally, the
reasoning given were
very lengthy and not
precise. It is not
necessary to tell the
marker the
difference/similarities
between a 2-tail test
and a lower-tail test.
Explanation should be
based the location of
m = 9 relative to the
critical region
determined in (i). It
should not be based on
calculations or pvalue.
The purpose of doing
hypothesis testing is to
disprove H0. Thus, it is
inappropriate to say
H0 is accepted; H1 is
rejected; H1 is not

14
accepted
2.5 %

test statistic also lies here


10(i)

Let n be the number of red balls.


P drawing two red balls in first two draws

n n 1

15 14
n2 n

210
2
n n 1

210
35
2
n n6 0

n 3 n 2 0
(ii)

n 3 or 2 (rej.)
Method 1:
P(red or green token on second draw)
= P(blue on first draw and red or green on second draw) +
P(non-blue on first draw and red or green on second draw)
2 13 13 12 13

or 0.867
15 14 15 14 15

Method 2:
P(red or green token on second draw)
2 1 13 2
1
= 0.867
15 14 15 14
(iii)

3 12 2
24
2
15 14 13
455
13 2 2 1
2
P B
15 14 15 14 15
3 2 2
2
P A B
15 14 13 455

P A

Hwa Chong Institution 2013

This part of the


questions was well
attempted.
Some students
mistook that the
denominator is always
15 (they neglected the
fact that the drawing
of tokens is without
replacement).

Some students
mistook
P A B
3 2 2

15 14 13
2

455
It should be

P A B P A P B P A B

Students are required


to show their steps
clearly, including the
solving of the
quadratic equation
n2 n 6 0 to obtain
n 3 or 2 (rej.)
No mark is awarded if
the student verify
3 2
1

15 14 35

9740/02/Prelim/13

15

248
or 0.182
1365

2
455
Quite a number of
students tried to list
down all possible
ways, which was a
tedious process, and
was prone to mistakes.
Students should note
and use the result
P A B
P A B

P A P B
P A B
(iv)

16
2

P A B
2275 455
A and B are not independent events.

Since P A P B

This part was poorly


attempted. Students
should check one of
the following
conditions to
determine
independence of 2
events:
1. P A P B P A B

2. P A | B P A
3. P B | A P B

Hwa Chong Institution 2013

9740/02/Prelim/13

Many students were


confused between
independence and
mutually exclusive.
They checked whether
P A B 0 which is
to determine 2 events
are mutually exclusive
or not.
Students are required
to substitute the
correct probability
values to gain the
credit for this part.

16
11
(a)(i)

From the graph, S 11 and S 9 are symmetrical about the


mean 10. Hence P S 11 P S 9 .

(Alternatively without sketch)


The distribution graph of S is symmetrical about the mean
S = 10, and 9 and 11 are both 1 unit away from the mean,
so the areas under graph from - to 9 is the same as 11 to
. Hence P S 11 P S 9 .
(ii)

Required probability
= P( S 9) P( S 11)
3

4!
0.0525
3!

(b)

S N 10, 2.42 , C N ,1.62

P(S 9) P(C 7)
1 P C 7 P S 9
7

P Z
0.66154 or 0.66155
1.6

7
0.41670
1.6
6.33

Hwa Chong Institution 2013

9740/02/Prelim/13

Some students did not


bother to sketch even
though the question
specifically asked to
consider the graph.
Note that if no sketch
is given, the
explanation has to be
very clear for a show
question, mentioning
graph symmetry,
equidistance and area.
Many students failed
to consider the
different permutations
of getting 3 (S < 9)
and 1 (S >11).
Some students
attempted to do a
binomial distribution,
but no. of questions
done within 9 minutes
is not independent of
no. of questions taking
more than 11 minutes.
A surprising no. of
students did not read
the question properly
and attempted the
entire part using C
only, i.e.
P(C 9) P(C 7)
Students are reminded
not to write G.C.
commands, but use
proper
notation such
6.33
7

as P Z
.
1.6

The answer could


have been checked by
keying in G.C.
P(C 7) with the
found.

17

C N 6.4,1.62

(c)

(C1 C2 C3 ) 2S N 0.8,30.72

Required probability = P(0 (C1 C2 C3 ) 2S 1)


0.0699

12(i)

Mean rate of ponding occurrences along Bukit Timah road


remains constant. (or average number of ponding
occurrences per unit time along Bukit Timah Road is
constant)

Hwa Chong Institution 2013

9740/02/Prelim/13

A small no. of
students changed
P(C 7) to
P(C 8) or something
similar, or even did
continuity correction.
You are required to
know the difference
between discrete and
continuous random
variables.
Many students did not
realize that
(C1 C2 C3 ) 2S had
to be more=than
0,
0.0699
while some ignored
the phrase by less
than a minute.
Those with incorrect
final answers would
have been awarded 1
working mark for
N 0.8,30.72 . It is
not sufficient to give
the mean and variance
without stating the
distribution type as
normal.
Some students did not
know the difference
between
(C1 C2 C3 ) and 3C.
Some students lost
marks because they
mixed up the mean
and/or variance of S
and C.

Do not accept:
- number of ponding
remains contant
- rate (without mean)
remains constant
Many missed out the

18
mean or rate for the
1st assumption.
Do not accept:
Ponding occurs independently (or randomly) along Bukit - probabilities are
Timah road.
independent
- number of ponding is
independent
- averages are independent
(ii)

Let Y be the number of ponding occurrences in a year.


Y Po 4

P(Y n 1) 0.95
From GC,
when n 8, P(Y 8 1) 0.94887 0.95
when n 9, P(Y 9 1) 0.97964 0.95
Thus, n 9

Students are able to


identify the correct
distribution, but a
small number did not
get the probability
equation (inequality)
correct.

Required probability
= P( X1 X 2 5 X 3)

ALeast
handful
n =of9 the
students took least
n = 8 instead of least
n 1 = 8.
Do not accept:
1.
P( X1 5) P( X 2 3)
because cases are not
correct.
2. Do not accept these
cases
P(Y 3)P( X 2 2),

P(Y n) 0.05

(iii)

=
=

P( X1 X 2 5 and X 3)
P( X 3)

P( X1 3)P( X 2 2) P( X1 4)P( X 2 1) P( X 1 5)P( X 2 0)

1 P( X 2)

= 0.506

P(Y 3)P( X 2 1),

Alternatively, the numerator can be written as

P( X1 3)P( X 2 2) P( X1 3)P( X 2 1) P( X 1 3)P( X 2 0)


+P( X 1 4)P( X 2 1) P( X 1 4)P( X 2 0) P( X 1 5)P( X 2 0)

Hwa Chong Institution 2013

9740/02/Prelim/13

...

Y Po 4
where
because Y (in a year)
and X2 (in half a year)
are not independent.

When listing out the


cases in the numerator
using the alternative
way, many students
missed out terms.
You are strongly
encouraged to use the

19
cumulative form in the
first solution.
(iv)

Let L be the number of ponding occurrences along Bukit


Road in 5 years and M be the number of ponding
occurrences along Orchid Road in 5 years.
L Po 20
M Po 15

15 10

20 10

L N 20, 20 approx M N 15,15 approx


L M N 5,35 approximately

Required probability
P( L M 10) P( L M 10)
c.c.

P( L M 9.5) 0.223

The most common


mistake made is
students thought that
L M follows a
Poisson distribution.
The beauty of this
question is to realize
that you need to
perform
approximation in order
to solve
P( L M 10). Hence,
the question need not
prompt you for an
approximation.

Hwa Chong Institution 2013

9740/02/Prelim/13

A handful of students
did not do continuity
correction here, or did
c.c. wrongly.

Name:

Index Number:

Class:

DUNMAN HIGH SCHOOL


Preliminary Examination
Year 6
MATHEMATICS (Higher 2)

9740/01

Paper 1

18 September 2013
3 hours
Answer Paper
List of Formulae (MF15)

Additional Materials:

READ THESE INSTRUCTIONS FIRST


Write your Name, Index Number and Class on all the work you hand in.
Write in dark blue or black pen on both sides of the paper.
You may use a soft pencil for any diagrams or graphs.
Do not use staples, paper clips, highlighters, glue or correction fluid.
Answer all the questions.
Give non-exact numerical answers correct to 3 significant figures, or 1 decimal place in the
case of angles in degrees, unless a different level of accuracy is specified in the question.
You are expected to use a graphic calculator.
Unsupported answers from a graphic calculator are allowed unless a question specifically
states otherwise.
Where unsupported answers from a graphic calculator are not allowed in a question, you are
required to present the mathematical steps using mathematical notations and not calculator
commands.
You are reminded of the need for clear presentation in your answers.
The number of marks is given in brackets [ ] at the end of each question or part question.
At the end of the examination, attach the question paper to the front of your answer
script.
The total number of marks for this paper is 100.

For teachers use:


Qn
Q1 Q2

Q3

Q4

Q5

Q6

Q7

Q8

Q9

11

Q10 Q11

Total

Score
Max
Score
DHS 2013

13

14

100

This question paper consists of 6 printed pages (including this cover page).

2
1

To prepare for an overseas trip, four friends went to a money changer together to obtain
some foreign currencies. The amount of foreign currencies they obtained and the total
amounts of Singapore Dollars they used for the conversion are shown in the table
below.
Euro
British Pound
Canadian Dollar
Total Amount in
Singapore Dollar

Eric
120
73
330

Benny
0
77
69

Charlie
48
290
63

Tania
P
50
100

750

235

723

332

The exchange rates for one Euro, one British Pound and one Canadian Dollar are $x, $y
and $z (in Singapore Dollars) respectively. Write down and solve equations to find the
exchange rates for the 3 foreign currencies in Singapore Dollars and compute P.
[4]

Without the use of a calculator, solve the inequality

1
1 x 1 x .
x

Hence, solve 1 e2 x e x e x 0 .

[3]
[2]

A sequence of integers u0 , u1, u2 , u3 , ... is defined by u0 1 and


un un1 4 2n 1 , for n 1, 2,3,... .

(i)

Find un for n = 1, 2, 3.

(ii)

By comparing the values of un with (2n)2 or otherwise, write down a conjecture


for un in terms of n, where n 0,1, 2,3,... .

(iii) Prove your conjecture using mathematical induction.

[2]

[2]
[4]

[Turn over
DHS 2013 Year 6 H2 Mathematics Preliminary Examination Paper 1

3
4

(i)

By considering the standard series expansions of e x and 1 x , find the


n

expansion of e

1 5 x

1
3

in ascending powers of x, up to and including the term in

2
x 2 . Express your answer in the form e a bx cx

(ii)

where a, b, c are

coefficients to be determined.

[4]

State the range of values of x for which the expansion above is valid.

[1]

(iii) It is given that and are constants, where 0 and 0 . In the series
expansions of e cos( x ) and e

1 5 x

1
3

in ascending powers of x, the constant

terms and the terms in x 2 are equal. Find the exact values of and .

[4]

Sketch on a single Argand diagram the set of points representing all complex numbers z
satisfying both the inequalities
1
3

0 arg( z 2 2i)

and

Im( z) Re( z) 4 0.

[5]

(i)

State the range of arg( z 2i).

[2]

(ii)

The locus of the complex number w is defined by w 2i k , where k is a real


positive constant. Find the range of values of k such that the loci of w and z
intersect.

[2]

(a)

Given that 10 OF 3 OE 7 OG, show that the points E, F and G


are collinear.

(b)

[2]

Relative to the origin O, the points A, B and C have position vectors given by

a 12i 2j 6k,

b 5i j 2k

and

c = (1 2 )i (5 2 ) j (2 )k

respectively. The point P lies on AB produced such that AP = 2AB.

(i)

Find OP .

[2]

(ii)

If AP is perpendicular to BC, find the value of .

[3]

[2]
(iii) Using the value of found in part (ii), find the area of triangle ACP.
[Turn over
DHS 2013 Year 6 H2 Mathematics Preliminary Examination Paper 1

4
7

During a promotion, a bank offers an interest-free loan for the first three months.
Thereafter, interest is charged at the end of each month on the outstanding amount at
1% per month. John borrows $30000 from the bank and pays back a fixed amount of $x
at the beginning of each month, starting from the first month.
(i)

Find in terms of x, the amount he owes after the third payment.

(ii)

Show that the amount he owes after the fifth payment is

1.012 30000 1.012 3x 1.01x x.

[2]
[2]

(iii) Find the amount he owes after the nth payment, simplifying your answer in terms
of n and x using the sum of a geometric progression formula.

[3]

(iv) If the loan is fully repaid after 60 payments, find the amount of each monthly
payment.

[2]

The resistance R of a newly-manufactured conductor is known to increase with the


temperature . Engineer A proposes the differential equation

d2 R
1

e0.01 to
2
d
10000

model the relationship between R and while Engineer B believes that the relationship
is better represented by
(i)

dR
R sec2 2 .
d

Find the general solutions of the two differential equations given above,
expressing R in terms of for both cases. You may assume that both R and are
non-negative quantities.

(ii)

[4]

Give a reason why the model proposed by Engineer B cannot represent the
relationship between R and well.

[1]

(iii) Using the model proposed by Engineer A, find the value of when R 200 ,
given that R 100 and

dR
0.1 when 0.
d

[4]

[Turn over
DHS 2013 Year 6 H2 Mathematics Preliminary Examination Paper 1

5
9

(a)

A graph with equation y f ( x) undergoes in succession, the following


transformations:
A: Reflection in the x-axis.
B: Translation of 1 unit in the negative x direction.
C: Scaling parallel to the x-axis by a factor of

1
.
2

The equation of the resulting curve is y e 2 x 1 . Determine f ( x).

[3]

(b) The diagram shows the graph of y g( x). The curve has a minimum point at

(0, 3), a maximum point at (2, 4), and cuts the x-axis at (1,0), (1, 0) and (3, 0).
y

y g( x)

(2, 4)

(0, 3)
On separate diagrams, sketch the graphs of
(i)

1
,
g( x)

[3]

(ii)

y 2 g( x).

[3]

The region R is bounded by the curve y 2 g( x) between x 1 and x 3. Given


that the volume of solid obtained when R is rotated through radians about the
x-axis is 12.6 units3 , find the area bounded by the curve y g( x) and the x-axis
between x 1 and x 3.

[2]

[Turn over
DHS 2013 Year 6 H2 Mathematics Preliminary Examination Paper 1

10

4
1
3
b

The lines l1 and l2 have equations r 1 1 , where b > 1, and r 0 1


1
1
2
1



respectively.
(i)

Given that the acute angle between l1 and l2 is 30o, find the value of b, giving your
answer correct to 2 decimal places.

[3]

For the rest of the question, use b = 3.


(ii)

Find the coordinates of the points A and B where l1 and l2 meet the xyplane
respectively.

[3]

(iii) The point C has position vector 2i + 7j + 3k. Show that the equation of plane p1
which passes through the points A, B, and C is given by x 2 y 5z 3.

[3]

(iv) Another plane p2 with equation x 2 y 3z 2 meets p1 in the line l3. Find the
vector equation of l3.
(v)

[2]

Explain whether the lines l2 and l3 are parallel, skew, coincident or intersecting,
justifying your answer clearly.

[2]

11
Q

The point Q(x, y) lies on the curve 4 x2 xy y 2 36, where y 0, as shown in the
diagram above. The curve cuts the x-axis at the points P and R.
(i)

Show that A, the area of triangle PQR, is given by A 3 y.

(ii)

Find

dy
in terms of x and y.
dx

[3]
[2]

(iii) Hence find the value of x for which A has a stationary value. Using the second
derivative test, determine the nature of this stationary value.

[6]

If x increases at a constant rate of 8 units/s, find the rate of change


of A when x = 0.

[3]
END OF PAPER

DHS 2013 Year 6 H2 Mathematics Preliminary Examination Paper 1

DHS 2013 Year 6 Prelims Paper 1 Solution


1

120 x 73 y 330 z 750


--- (1)
77y 69 z 235
--- (2)
--- (3)
48 x 290 y 63z 723

[For reference only]


120 73 330 750 1 0 0 1.6769

RREF 0
77 69 235 0 1 0 1.9479
48 290 63 723 0 0 1 1.2321

Answers to 3 sig fig:


x 1.68

y 1.95
z 1.23
The exchange rate in Singapore Dollars for Euro is $1.68, British Pound is $1.95 and
Canadian Dollar is $1.23.

P 1.6769 50 1.9479 100 1.2321 332


2

P = 66.43 (2d.p)
1
1 x 1 x
x
1
1 x 1 x 0
x
1 x x 2 1 x
0
x
1 x 1 x 2
0
x

1 x 1 x 0
2

x0

or

x 1

1 e2 x e x e x 0
1
e x 1 e x
x
e
Replace x by e x ,

e x 1 or e x 0 (reject)
x0

3(i)

3(ii)
3(iii)

entered as un1 un 4 2n 1

un

u0 1 (given)

u1 3

u2 15

u3 35

Conjecture : un (2n)2 1 , n
Let P(n) be the proposition

un (2n)2 1

for n

When n =0, LHS of P(0) : u0 1

(given)

RHS of P(0) = 02 1 1 P(0) is true.

Assume P(k) is true for some k

, i.e. uk (2k )2 1

When n= k+1
RHS of P(k+1) = uk 1 2(k 1) 1
2

LHS of P k 1 :

uk 1 uk 4 2k 1 (2k ) 2 1 4 2k 1
4k 2 8k 4 1

4 k 2 2k 1 1
2(k 1) 1 RHS
2

P(k) is true P(k+1) is true


Since P(0) is true and P(k) is true P(k+1) is true, by mathematical induction, P(n) is
true for all n

0.

1 5 x

1
3

1 2

1
3 3
2
1 5 x 5 x
3
2!

5 25
1 x x 2
3
9

ee

5 25 2
x x
3
9

x
5
25
3

2
e 1 x x
3
9
2!

25
5

e 1 x x 2
18
3

5
25
i.e. a 1, b , c
3
18
1
1
The expansion is valid for 5 x 1 i.e. x .
5
5
Method 1
Let f ( x) cos( x )

f '( x) sin( x )
f ''( x) 2 cos( x )
When x 0,
f (0) cos , f ''(0) 2 cos
Comparing coefficients,
cos 1 0 &
f ''(0)
25

2!
18
2

25
5


2
18
3
Method 2
Let f ( x) cos( x )

cos x cos sin x sin


2 x2
1

cos x sin

2 cos 2
cos sin x
x
2
Comparing coefficients,
2
25
5


cos 1 0 &
2
18
3
3

0 arg( z 2 2i) 3

0 arg( z (2 2i))

Im( z ) Re( z ) 4 0

3
4

Let z x iy
yx40
y x4
Let P( x, y) represent the complex number z.

Im
y x4
Locus of P
O
A
2

5(i)
5(ii)

3
4

(2, 2)

3
4
Consider right angle triangle OAB,
AB
sin 45
AB 2
2
range of values of k : k 2
0 arg( z (2i))

Re

6(a)

(a) 10 OF 3 OE 7 OG

3 OF 7 OF 3 OE 7 OG
3 OF 3 OE 7 OG 7 OF

6(b) (i)

3 EF 7 FG
Method 1

E , F & G are collinear

Using AP 2 AB ,


OP OA 2 OB OA

OP 2 OB OA

5 12 2

2 1 2 0
2 6 10

Method 2
Using mid-pt theorem,

1
OB OA OP
2

OP 2 OB OA

5 12 2

2 1 2 0
2 6 10

6(ii)

2 12 14
1 2 5 2 4

AP 0 2 2 , BC 5 2 1 2 4
10 6 16
2 2

AP BC 0 ( AP BC )
14 2 4

2 2 4 0 24 24 1
16

6(iii)

Method 1
2 4 2


Using =1, BC 2 4 6
1 1

14
1 1

Area of ACP AP BC 2
2
2

16
1
456 41 68.4 unit 2
2

2

6
1

Method 2
1 2 12 9


Using =1, AC 5 2 2 5
2 1 6 7


9 14
1 1

Area of ACP AC AP 5 2
2
2

7 16

94

1
2

46

2 18696 68.4 unit


88

Amount paid-up after the third payment = 3x


1

7(i)

7(ii)

Since 1st 3 months is interest free, amount owed after the third payment = 30000 3x
Amount owed after the 4th payment
1.01 30000 3x x

1.01 30000 1.01 3x x


Amount owed after the 5th payment
1.01 1.01 30000 1.01 3 x x x
7(iii)

1.012 30000 1.012 3x 1.01x x


After nth payment, amount owed
1.01n 3 30000 1.01n 3 3 x 1.01n 4 x 1.01n 5 x ... x

1.01n 3 30000 1.01n 3 3 x x 1.01n 4 1.01n 5 ... 1


1.01n 41 1
1.01n 3 30000 1.01n 3 3 x x

1.01 1
1.01n 3 30000 1.01n 3 3 x 100 x 1.01n 3 1

7(iv)

Since the loan is fully paid after 60 payments,


1.01603 30000 1.01603 3 x 100 x 1.015603 1 0

1.0157 3x 100 x 1.0157 1 1.0157 30000


x

8(i)

1.0157 30000

1.0157 (3) 100 1.0157 1

$648.13
Model A:
d2 R
1
e0.01

2
d
10000
dR
1 0.01
e

A
d 100
i.e. R e0.01 A B
Model B:
dR
R sec 2 2
d
1
1
dR 2sec 2 2 d
R
2
1
ln R tan 2 C '
2
1

8(ii)

8(iii)

tan 2

i.e. R C e 2
The reasons are:
(a) R is not increasing with (or R is only increasing with for disjoint intervals of ).
(b) R is not defined at some values of
(2k 1)
, k ).
(i.e. at
4
dR
dR
1 0.01
0.1 & 0 into

e
A,
Substitute
d
d 100
we have A = 0.09.
Substitute R 100 & 0 into R e0.01 A B ,
we have B = 99.
R e0.01 0.09 99
When R 200 , 415 (3 s.f.) [using GC]

9(a)

y e 2 x 1

Scaling parallel to the x axis by a factor of 2 x

y e x 1

1
x :
2

Translation of 1 unit in the positive x direction x


y e x

Reflection in the x axis y


y ex

x 1 :

y :

y f ( x) e x

9(b)(i)

y
y

1
g( x)
1
2,
4

0,
3

x 1

x 1

9(b)(ii)

x3

y
y 2 g( x)

2, 2

2, 2
3

Volume of solid y 2 dx
1
3

g( x) dx
1

Area under curve


12.6
Area under curve
4.0107 4.01 (3s.f)

10(i)

b 1

1 1
1 1
cos 30 o
3 b2 2

b 2
3
b2

2
3 b2 2
3 (b 2 2)

Alternative
0.86603

3 (b 2) 2(b 2)
2

9(b 2 2) 4(b 2) 2
5b 16b 2 0
2

b 3.07 (2 d.p.) ( b 1)

b 2
3 b 2
2

b2
3 (b 2 2)

1.50 (b 2 2) (b 2)

or 2.25(b 2 2) (b 2) 2
1.25b 2 4b 0.5 0
b 3.07 (2 d.p.) ( b 1)

10(ii)

Using b = 3
3
3 3 3

l1 : r 1 1 1
2
1 2

10(iii)

At xy plane, z 0 2
Point A 9,3, 0

4
1 4
At xy plane, z 0 1

l 2 : r 0 1
Point B 5,1, 0
1
1 1

Using points A(9, 3, 0), B(5, 1, 0) & C(2, 7, 3) which lie on p1,
5 9 4
2
2 5 3
1



AB 1 3 2 2 1 , BC 7 1 6 3 2
0 0 0
0
3 0 3
1



2 9 7


AC 7 3 4
3 0 3


2 1 1
7 2 3
1


Normal of p1 , n1 = 1 2 2 or 4 1 6 3 2
0 1 5
3 0 15
5


x 1 5 1

eqn of plane p1 : y 2 1 2 3
z 5 0 5

x 2 y 5 z 3 (shown)

10(iv)

1 0 4
1 2 5 3
2
rref

1 2 3 2 0 1 1 1

2 4
5
x 4z
2
1
1
y z
2
4
4
x 2.5

1

y 0.25
2
z 0
1


2.5
8


l3 : r 0.25 1 ,
0
2

10(v)

Since l2 and l3 have different direction vector, they are not parallel.
4 2.5 8
Equating l2 and l3 0.25
1 2

yields no consistent values for or


l2 and l3 do not intersect
skew lines.

10

11(i)
y
Q( x, y)

y
P

Given: 4 x2 xy y 2 36 where y 0. -------


At y 0, 4 x2 36 x 3, 3
ie, base length of triangle PR = 6

1
A (6) y 3 y -------
2
11(ii)

Diff wrt x
dy
dy
8 x x y 2 y 0 -------
dx
dx

x 2y

dy
8x y
dx

dy
8x y

dx
x 2y
11(iii)

Diff wrt x

dA
dy
3
dx
dx
For max A,
i.e.

dA
dy
0
0
dx
dx

8x y 0 or

y 8x -----

Substitute into :
4 x 2 x 8x 8x 36
2

60 x 2

3
3
x
,
5
5

3
(reject since y 8 x < 0 )
5

For 2nd derivative test, diff wrt x


11

8 x

At

d 2 y dy 2
d 2 y dy dy

2
y 2 0
dx 2 dx dx
dx dx

dy
0
dx

8 x 2y

d2 y
d2 y
8
8
8

2
2
dx
dx
x 2y
x 2(8 x) 15 x
2

d A
3 d y
At x
,
gives

0
0
5 dx 2
dx 2
ie. A is maximum at x

3
5

Method 1
At x = 0 , : 0 0 y 2 y
Using chain rule on : A 3 y

dy
dy
1
0

dx
dx
2

dA
dy dx
3
dt
dx dt
1
3 8 12 units 2 /s
2
2

i.e., the area of the triangle decreases at a rate of 12 units / s


Method 2
Diff wrt t
dx
dy
dx
dy
8x x y 2 y 0
dt
dt
dt
dt
At x 0 ,
dx
dy
dy
1 dx
y 2y 0
dt
dt
dt
2 dt

dy
1 dx
1

8 4
dt
2 dt
2

From A 3 y

dA
dy
3
dt
dt

dA
3(4) 12 units 2 /s
dt
2

i.e., the area of the triangle decreases at a rate of 12 units / s .

12

Name:

Index Number:

Class:

DUNMAN HIGH SCHOOL


Preliminary Examination
Year 6
MATHEMATICS (Higher 2)

9740/02

Paper 2

26 September 2013
3 hours
Answer Paper
List of Formulae (MF15)

Additional Materials:

READ THESE INSTRUCTIONS FIRST


Write your Name, Index Number and Class on all the work you hand in.
Write in dark blue or black pen on both sides of the paper.
You may use a soft pencil for any diagrams or graphs.
Do not use staples, paper clips, highlighters, glue or correction fluid.
Answer all the questions.
Give non-exact numerical answers correct to 3 significant figures, or 1 decimal place in the
case of angles in degrees, unless a different level of accuracy is specified in the question.
You are expected to use a graphic calculator.
Unsupported answers from a graphic calculator are allowed unless a question specifically
states otherwise.
Where unsupported answers from a graphic calculator are not allowed in a question, you are
required to present the mathematical steps using mathematical notations and not calculator
commands.
You are reminded of the need for clear presentation in your answers.
The number of marks is given in brackets [ ] at the end of each question or part question.
At the end of the examination, attach the question paper to the front of your answer
script.
The total number of marks for this paper is 100.

For teachers use:


Qn
Q1 Q2

Q3

Q4

Q5

Q6

Q7

Q8

Q9

12

12

Q10 Q11

Total

Score
Max
Score
DHS 2013

10

10

100

This question paper consists of 6 printed pages (including this cover page).

2
Section A : Pure Mathematics [40 marks]

x2

(a)

Find

x 1 x 2 dx .

(b)

(i)

Differentiate sin 1 x 2 with respect to x.

(ii)

Hence or otherwise, find a positive integral value of n such that

[3]

x sin 1 x 2 dx

[1]

1
.
4 2

[3]

The functions f and g are defined by

f :x

xe x ,

x 2 ,

g:x

x2 a ,

x 0 and a is a real constant.

(i)

Using differentiation, show that f concaves upwards.

[3]

(ii)

Sketch the graph of f and explain if f 1 exists.

[3]

(iii) State the minimum value of a such that the function fg exists.

[1]

(iv) Given further that a 0 , find the range of g 2 , giving your answer in terms of a.
[2]
3

A curve C has parametric equations


x t 2 1, y t 1, for t 1.

(i)

Sketch the curve C.

[2]

(ii)

Find the equation of the normal to the curve at the point P where t 2.

[3]

(iii) Without using a calculator, show that the area bounded by the x-axis, the curve
2

and the normal found in part (ii) is 2 t t 1 dt


1

1
. By using the substitution
16

u t 1 , find the exact value of this area.

[7]

[Turn over
DHS 2013 Year 6 H2 Mathematics Preliminary Examination Paper 2

3
4

The equation z 4 4 z 3 az 2 20 z 25 0, a , has a root ki, where k is a real


number.
(i)

Explain clearly why there is more than one possible value for k.

(ii)

Find the possible exact values of k and show that a = 10.

[1]

Hence find the roots of the above equation.

[7]

(iii) Deduce the roots of the following equations in the form x iy, where x and y are
real,
(a)

w4 4iw3 10w2 20iw 25 0,

(b)

25v4 20v3 10v2 4v 1 0.

[4]

Section B : Statistics [60 marks]


5

A lemonade dispensing machine dispenses S ml of syrup and W ml of water in each cup


of lemonade. S and W are independent random variables which follow the distributions

N(50,32 ) and N(140,52 ) respectively.


(i)

Find the probability that the volume of water exceeds three times the volume of
syrup by more than 5 ml in a randomly chosen cup of lemonade.

(ii)

[2]

The machine initially contains 10 litres of syrup and an unlimited supply of water.
If the probability that the machine is able to dispense n cups of lemonade exceeds
0.99, find the greatest value of n. (Answers obtained by trial and improvement
from a calculator will obtain no marks.)

[5]

Four chairs are placed on each of the longer sides of a rectangular table. These chairs
are fixed and numbered 1 to 8. Find the number of different ways that 4 boys and 4 girls
can be seated if
(i)

there are no restrictions,

[1]

(ii)

any two people of the same gender cannot sit next to each other on the same side
of the table.

[2]

Two extra chairs numbered 9 and 10 are added and fixed to each of the shorter sides of
the table. Find the number of different ways the 4 boys and 4 girls can be seated if
(iii) there are no restrictions,
(iv) two boys must sit at each of the shorter sides.

DHS 2013 Year 6 H2 Mathematics Preliminary Examination Paper 2

[2]
[3]
[Turn over

4
7

A box contains 5 red balls, 5 green balls and 5 blue balls. The balls are identical in size.
Balls of the same colour are each printed with a distinct number from 1, 2, 3, 4, 5. Two
balls are randomly drawn without replacement. Find the probability that
(i)

at least 1 red ball is drawn,

[2]

(ii)

the sum of the numbers on the balls drawn is at least 9,

[2]

(iii) the sum of the numbers on the balls drawn is at least 9 given that at least 1 red
ball is drawn.

[3]

State, with a reason, whether the events the sum of the numbers on the balls drawn is at
least 9 and at least 1 red ball is drawn are independent.

[1]

The soil at a nuclear plant is contaminated due to a nuclear leak. The radiation intensity
I at time t, in appropriate units, is measured at regular intervals from the time of leak.
The data shown below is taken at a particular spot.
t

0.2

0.4

0.6

0.8

1.0

1.2

3.22

1.63

1.55

0.54

0.36

0.22

(i)

Draw a scatter diagram for these values, labelling the axes clearly.

(ii)

One of the values of I appears to be incorrect. Indicate the corresponding point on


your diagram by labelling it P.

[1]
[1]

It is given that the radiation intensity I is related to time t by the formula I I 0e kt ,


where I 0 and k are constants.
(iii) Omitting P, calculate the least square estimates for I 0 and k using the regression
line of ln I on t.

[3]

(iv) Explain why it is reasonable to use the regression equation obtained in part (iii) to
estimate the value of t when I 1.55 . Obtain this value correct to one decimal
place.
(v)

[2]

It is later known that the data represented by P is correct. Suggest a possible


reason why this could happen.

[1]

[Turn over
DHS 2013 Year 6 H2 Mathematics Preliminary Examination Paper 2

5
9

The masses of mooncakes follow a normal distribution with mean 300 g when produced
under standard conditions. Recently, the machine producing the mooncakes was
replaced and customers claim that the mooncakes have become lighter. A random
sample of 8 mooncakes is obtained and their masses (in grams) are measured as
follows,
280, 285, 290, 295, 294, 297, 299, 302.
(i)

Calculate the unbiased estimates of the population mean and variance.

[2]

(ii)

Test the customers claim at the 2% significance level.

[4]

(iii) The population mean and standard deviation of the masses of mooncakes are now
given as 0 g and 6.4 g respectively. If the null hypothesis is not rejected at 2%
significance level, find the range of possible values of 0, using the same data
given above.

10

(a)

[3]

In a large company, the percentages of directors, managers and executives are


5%, 20% and 75% respectively. A random sample of 60 staff members is chosen
to take part in a survey to determine the productivity of staff in the company.
Describe how a stratified sample can be obtained and state an advantage of this
method of sampling compared to random sampling.

(b)

[3]

One question in the survey is as follows:


Do you use Facebook during office hours at least once a day? Yes or No?
It is known that on average, 7 out of 9 staff members answered yes to the above
question.
For the random sample of 60 staff members selected in (a),
(i)

find the standard deviation of the number of staff members who answer
yes,

(ii)

[1]

find the probability that the number of staff members who answer yes
exceeds the mean,

[3]

(iii) find the most probable number of staff members who answer yes,

[2]

(iv) state, with a reason, if the above answers are valid if the total number of
staff members in the company is small.

[1]
[Turn over

DHS 2013 Year 6 H2 Mathematics Preliminary Examination Paper 2

6
11

Marc, a building manager, is tasked to furbish an office building by changing the


carpets and window blinds. State two conditions needed for the number of flaws found
on a randomly chosen region of area 2 m2 on the carpet to be well modelled by a
Poisson distribution.

[2]

Assume that the number of flaws on 2 m2 of carpet has the distribution Po(0.5).
(i)
(ii)

Find the probability that there are exactly 2 flaws on 7 m2 of carpet.

[2]
2

Each floor (with identical layout) of the office building requires 200 m of carpet.
Use a suitable approximation to find the probability that there are between 40 and
50 flaws on the carpet of a randomly chosen floor. State the parameters of the
distribution that you use.

[3]

Marc chooses Superior Quality roller blinds for the windows. The number of flaws on
the roller blinds of a level, R, has the distribution Po(0.3).
(iii) Find the probability that the average number of flaws on the roller blinds per level
of the 50 level building is less than 0.4. State any approximation made.

END OF PAPER

DHS 2013 Year 6 H2 Mathematics Preliminary Examination Paper 2

[3]

DHS 2013 Year 6 Prelim Paper 2 Solution


Section A : Pure Mathematics [40 marks]
1(a)

x2

x 1 x 2 dx
1
4
dx

x 1 x 2
x ln x 1 4 ln x 2 c
1

1(b)(i)
1(b)(ii)

d
2x
sin 1 x 2
dx
1 x4

x sin 1 x 2 dx

2
n x
x2

2x
dx
sin 1 x 2
2
0 0 2 1 x4
n

x2

1
sin 1 x 2 2 1 x 4
4
2
0

n2
1
1 1
sin 1 n 2
1 n4
2
2
2 4 2
From GC or observation, n = 1 (reject n 1 since n

2(i)

f ( x) xe x
f ( x) xe x e x
f ( x) xe x e x e x e x x 2
Since x 2, x 2 0 & e x 0
f ( x) 0 for all x 2
f concaves upwards.

2(ii)

y f ( x)

(2, 0.271)

x
y 0.3

(1, 0.368)

Since the line y 0.3 cuts the graph of f at 2 points, f is not one-one, f 1 does not
exists.
2(iii)

For fg to exist, R g Df

Since R g a, and Df (2, )

y g( x)

min a 2
O

a
2(iv)

Since R g a, new Dg

R g 2 a 2 a,

At y 0, t 1

3(i)

x 12 1 2 x-intercept (2, 0)
y

x t 2 1, y t 1, for t 1

3(ii)

O
dx
2t ,
dt

dy
1
dy
1

dt 2 t 1
dx 4t t 1
1
2
At t = 2, gradient of normal
4(2) 2 1 8, x 2 1 5, y 2 1 1
dy
dx
y

8(
x
5) y 8x 41
Equation of normal :

3(iii)

y 8x 41

P
A
O
Required area

41
8

= Area A Area B y dx + 1 bh or
2
5

41
8

8 x 41

dx

41
d
x
1
41

t 1 dt 5 1 or 4 x 2 41x 8
5
2 8

dt

1
(shown)
16
1
dt
u 2 1 u
du +
16
du
1
u 3 u 2u du +
16
1
u 4 u 2 du +
16

x 5, t 5 1 2
x 2, t 2 1 1

2 t t 1 dt +
1

Since t 1,

u t 1 u2 t 1 t u 2 1

t 1: u 1 1 0

t 2 : u 2 1 1

u5 u3
1 32 1
527
47
4

2
unit 2
5
3
16
15
16
240
240

dt
2u
du

4(i)

4(ii)

Since the coefficients of the equation are all real, by Conjugate Root Theorem, all
complex roots must occur in conjugate pairs,
where k q, q is a positive real number.
Given that z = ki is a root, we have

ki

4 ki a ki 20 ki 25 0
3

ak 2 25 i 4k 3 20k 0

comp real and imag parts : 4k 3 20k 0 & k 4 ak 2 25 0


k 5 (rej. k = 0)
Subst. k 5 into k 4 ak 2 25 0, we have

25 0

a 10

z 4 4 z 3 10 z 2 20 z 25 z 5i z 5i z 2 bz 5
z 4 4 z 3 10 z 2 20 z 25 z 2 5 z 2 bz 5

By comparison of coefficient of z 3 ,
b 4

z 4 4 z 3 10 z 2 20 z 25 z 2 5 z 2 4 z 5
z 5i, 2 i
4(iii)(a)

w4 4iw3 10w2 20iw 25 0


w4 4 iw 10 iw 20 iw 25 0
3

i.e. z iw
w iz
w 5, 1 2i

4(iii)(b)

25v 4 20v3 10v 2 4v 1 0


4

Alternative

1
1
1
1
4 10 20 25 0
v
v
v
v
1
i.e. z
v
1
v
z
i 1
v
, 2 i
5 5

25v 4 20v3 10v 2 4v 1 0


625v 4 500v 3 250v 2 100v 25 0

5v

4 5v 10 5v 4 5v 25 0
3

i.e. z 5v
v

i 1
, 2 i
5 5

Section B : Statistics [60 marks]


5(i)

S ~ N(50,32 )

W ~ N(140,52 )

&

W 3S ~ N 140 3 50,52 32 32 W 3S ~ N 10,106


P(W 3S 5) 0.072568 0.0726

5(ii)

Let T S1 S 2 ... S n ~ N(50n,32 n)

P T 10000 0.99

10000 50n

P Z
0.99
3 n

10000 50n
From GC,
2.3263
3 n
Solving using GC, 0 n 198.04, n

6(i)

6(ii)

6(iii)

greatest n 198.
Number of ways
8!
=40320
Number of ways 4 = ways to arrange the boys and girls on each side
4!4!4
4!4! = ways the groups of 4 boys & 4 girls can
arrange themselves
2304
Number of ways
10
10!
C8 = ways to choose any 8 out of the 10 seats
10
10
C8 8! or P8 or
to sit the 4 boys and 4 girls
2!
8!
=
ways
to arrange the boys and girls
1814400
10!
arrange 10 objects in a row with 2 identical
2!
objects (empty seats)

6(iv)

Number of ways
4C2 2! 8C6 6!

or 4 P2 8 P6

241920

C2 2! = ways to choose any 2 out of the 4 boys to sit on the 2


seats and ways to arrange the 2 boys

C6 6! = ways to choose any 6 out of the 8 seats to sit the

remaining 2 boys/4 girls and ways to arrange the 2


boys/4 girls

8! = ways to arrange 6 people and 2 empty chairs


2

7(i)

10 9 4

15 14 7
10
C
4
or 1 15 2
C2 7

P(at least 1 red ball is drawn) 1

5 10
5 4 4
2
15 14
15 14 7
3 3
3 2
4
P(sum of numbers at least 9) 2
15 14
15 14 35
3
3
3
C1 C1
C
4
or 15
15 2
C2
C2 35
P(sum of numbers at least 9|at least 1 red ball is drawn)
P(sum of numbers at least 9 & at least 1 red ball is drawn)
=
P(at least 1 red ball is drawn)
P(4(R),5(RGB))+P(4(GB),5(R))+P(5(R),5(GB))
=
P(at least 1 red ball is drawn)
1 3
2 1
1 2
2 2 2
7
15 14
15 14
15 14

4
60
7
3
2
2
C1
C1
C1

15
15
15
C2
C2
C2
7

4
60
7
Since P(sum of numbers at least 9|at least 1 red ball is drawn)
P(sum of numbers at least 9), the 2 events are not independent.
I
or

7(ii)

7(iii)

8 (i)
& (ii)

t
8(iii)

I I 0 e kt
ln

8(iv)

I
kt
I0

ln I kt ln I 0
From GC, ln I 2.6549t 1.6127
k 2.6549 2.65 (3sf)
I 0 e1.6127 5.0164 5.02 (3sf)
The radiation intensity I is dependent on time t. Thus obtaining an estimate from the
6

8(v)
9(i)

regression line t on ln I will be meaningless.


ln1.55 2.6549t 1.6127 t 0.4 (1 d.p.)
The spike in radiation intensity can be due to a second/additional nuclear leak.
From GC,
unbiased estimate of population mean ,
x 292.75
unbiased estimate of population variance 2 ,
s 2 7.36302 54.214

9(ii)

Let X represent the mass (in g) of a randomly chosen mooncake with population
mean .
To test H0: 300
against H1: 300
One-tail test at 2% significance level.
Since sample size n = 8 is small, with X normal ( 2 unknown)
X 300
under H0, T =
~ t (7).
7.3630
8
Using t-test, p-value = 0.013551 = 0.0136 (3 s.f.) (from GC).

9(iii)

Since p-value = 0.0136 < 0.02, we reject H0 and conclude that there is sufficient
evidence at 2% significance level that the customers claim is true.
Since 6.4 is known & population of mass of mooncakes is normal, a Z-test is
used.
To test H0: 0
against H1: 0
One-tail test at 2% significance level.

6.42
under H0, X ~ N 0 ,

Given that H0 is not rejected p value %


P( X 292.75) 0.02

292.75 0

P Z
0.02
6.4 / 8

292.75 0
2.0537
z value
6.4 / 8
292.75 0 4.6470
0 297.40
0 297 (3 s.f.)

10(a)

Obtain a staff list and group the staff into the following 3 mutually exclusive strata:
directors, mangers & executives.
No of directors
5% 60 3

No of managers
20% 60 12

No of executives
75% 60 45

Randomly select the number of staff from each stratum according to the numbers
above to obtain a stratified sample.
The advantage of stratified sampling is that the sample obtained is representative of
the various strata (types of staff) by proportion.
10(b)(i) Let X be the number of staff members who answer yes out of 60 staff members.
7

X ~ B 60,
9

7 2 280
Var( X ) 60
9 9 27

Standard deviation

280
3.22 (3s.f)
27

7
2
46
9
3
2

P X 46 1 P X 46 1 0.46787 0.53213 0.532 (3s.f)


3

10(b)
(ii)

E( X ) 60

10(b)
(iii)

From GC,
P( X 46) 0.11846
P( X 47) 0.12350
P( X 48) 0.11707
the most probable number is 47.
The answers are not valid as the probability of a staff member answering yes will
not be approximately constant if the total number of staff members in the company
is small.

10(b)
(iv)

11

11(i)

11(ii)

1) Flaws are randomly distributed on the carpet.


2) The mean number of flaws found in any randomly chosen region of area 2 m2 of
carpet is constant.
Let X be the number of flaws found in a randomly chosen area of 7 m2 of carpet.
7

X ~ Po 0.5 X ~ Po 1.75
2

P X 2 0.266 (3sf)
Let Y be the number of flaws found in a randomly chosen area of 200 m2 of carpet
(on each floor)
200

0.5 50 10
2
Y ~ N 50,50 approximately

P 40 Y 50 P 40.5 Y 49.5 (after continuity correction)


0.382 (3sf)

11(iii)

Method 1
R ~ Po 0.3 ,
Since n 50 is large, by Central Limit Theorem,
0.3

R ~ N 0.3,
approximately
50

P R 0.4 0.902

Approximation used: The average number of flaws on the roller blinds per level, R
follows a normal distribution by Central Limit Theorem as the sample size n 50 is
large.
Method 2
R ~ Po 0.3
Let Q be the number of flaws on the roller blinds of 50 levels of the building.
Q ~ Po 15
Since 15 10, Q ~ N 15,15 approximately.
1
15 15
Q~ N , 2
50
50 50
P Q 0.4 0.902 (3s.f.)

Method 3
R ~ Po 0.3
Let Q be the number of flaws on the roller blinds of 50 levels of the building.
Q ~ Po 15
Since 15 10, Q ~ N 15,15 approximately.

P Q 20 P Q 19.5 (after continuity correction)


0.877 (3s.f.)
9

CATHOLIC JUNIOR COLLEGE


General Certificate of Education Advanced Level
Higher 2
JC2 Preliminary Examination

MATHEMATICS

9740/01

Paper 1

27 Aug 2013
3 hours

Additional Materials: List of Formulae (MF15)


Graph Paper
READ THESE INSTRUCTIONS FIRST
Write your name and class on all the work you hand in.
Write in dark blue or black pen on both sides of the paper.
You may use a soft pencil for any diagrams or graphs.
Do not use staples, paper clips, highlighters, glue or correction fluid.
Answer all the questions.
Give non-exact numerical answers correct to 3 significant figures, or 1 decimal place in the case of
angles in degrees, unless a different level of accuracy is specified in the question.
You are expected to use a graphic calculator.
Unsupported answers from a graphic calculator are allowed unless a question specifically states
otherwise.
Where unsupported answers from a graphic calculator are not allowed in a question, you are required
to present the mathematical steps using mathematical notations and not calculator commands.
You are reminded of the need for clear presentation in your answers.
At the end of the examination, arrange your answers in NUMERICAL ORDER.
Place this cover sheet in front and fasten all your work securely together.
The number of marks is given in brackets [ ] at the end of each question or part question.
Name: ___________________________

Question

Class: ________________

10

11

12

12

16

Total

Marks
Total

100

This document consists of 5 printed pages.


Catholic Junior College

9740/01/Prelim/2013

[Turn over

2
A curve has

1.

c
, where a, b and c are constants. Given that it passes through the point
x3
5
1

[4]
1,
and has a turning point at 2, , find the values of a, b and c.
4
16

Find the value of


k such that

equation y ax b

2.

3.

2c

x c dx , where c is a positive constant.

sequence is given by u r r (3r 2) for r 1 . Prove by mathematical induction


of the first n terms is

4.

x c dx k

[4]

The rth term of a


that the sum

n
(n 1)(2n 1) for all positive integers n.
2

[5]
Using an

algebraic method, solve the inequality


Hence solve the inequality

x4
1.
3 2 x x2

[5]

x2 4
1.
x4 2 x2 3

[2]

5.

The diagram shows the graph of y 3 e x x2 . The root of the equation 3 e x x2 0 is .


(i)
Fin
d the value of correct to 3 decimal places.
[1]
A sequence of real numbers x1 , x2 , x3 , satisfies the recurrence relation
2
xn1 ln 3 xn for n 1.

(ii)
ve algebraically that, if the sequence converges, it converges to .
(iii) By considering e xn1 e xn , prove that

9740/01/Prelim/2013

Pro
[2]

[Turn over

3
xn1 xn if xn ,

[3]

xn1 xn if xn .

Use

(iv)
a calculator to determine the behaviour of the sequence for each of the cases x1 0
and x1 3 . State briefly how the results in (iii) relate to the behaviours determined.
6.
w that
(a)

1
A
B
C

, where A, B and C are constants to be found.


r (r 1)(r 2) r (r 1) (r 2)

Use your results to find

r (r 1)(r 2) .

[3]
Sho
[2]
[2]

r 1

(b)

Hence find

1
and

r 2 r ( r 1)( r 2)

r (r

(ii)

r 3

(i)
[2]

1
.
1)

[3]

7.

(a)

(0, 1)

x = 1

x=1

The diagram above shows the graph of y f ( x) . The curve has asymptotes x = 1, x = 1
and y = 0. It has a minimum point at (0, 1). On separate diagrams, sketch the graph of
1

(i)
[2]
y f x 1 ,
2

(ii) y f '( x) ,
[2]
showing clearly the equations of any asymptotes, coordinates of any points of intersection
with the axes and coordinates of any stationary points.
(b)

The graphs y g( x) and y 2 g( x) are shown below.


y

y
(2, 2)
y=1

O (1, 0) (3, 0)

y=1
x

(1, 0) (3, 0)

y = 1

[Turn over

9740/01/Prelim/2013

y g( x)

y 2 g( x)

Sketch the graph of y g( x) , showing clearly the equations of any asymptotes, coordinates
of any points of intersection with the axes and coordinates of any stationary points.
[3]
2

d2 y
dy
(sin y) 0 .
2
dx
dx
Hence write down the first two non-zero terms in the Maclaurin series for y .

8.

(i)

(ii)

Given that y sin 1 x , show that (cos y)

[5]

Given that the first two non-zero terms in the Maclaurin series for y are equal to the first
x
two non-zero terms in the series expansion of
, where a and b are constants, find
a bx 2
a and b .
[4]
(a) Referred to the origin O, the points A and B have position vectors given respectively by

9.

OA a and OB b . R is the point that divides AB internally in the ratio : .


2

r a a a b
It is given that OR = r and
.

r b b 2 a b

r a a
.
r b b
Hence, show that the line OR bisects the angle AOB.

Deduce that, when : a : b ,

(b)

[2]

1
2
2



Planes 1 , 2 , 3 have equations r . 2 = 1, r . 1 = 7 and r . a = 5 respectively.
3
9
b



(i) When a = 1 and b = 3, find a vector equation of the line of intersection of these
three planes.
[2]
(ii) Given instead that the three planes have no point in common, what can be said
about the values of a and b ?
[3]

(a)

10.

[2]

Use the substitution x 2sec to find

1
2

x2 4

dx .

[5]
(b)

The region bounded by the curve y

1
1 2 x2

, the x-axis, the lines x

9740/01/Prelim/2013

3
and
2

[Turn over

1
is rotated completely about the x-axis to form a solid of revolution of volume V.
2
Find the exact value of V, giving your answer in the form k 2 .
[4]
x

9740/01/Prelim/2013

[Turn over

11.

The curve C has

parametric equations
x t2 2 ,

(i)

y t 3 where t

Sketch the curve C .

[1]

The tangent to the curve at point P where t 2 is denoted by l .


(ii) Find the cartesian equation of l .
(iii) The tangent l meets C again at point Q . Use a non-calculator method to show that the
coordinates of Q is 3, 1 .

[4]

(iv) Find the exact area enclosed by the curve C and l .


12.

(a)

Given that

argument of z is

1 i
z

[4]

3 i

, find the exact value of the modulus of z and show that the

11
.
12

[4]

By first expressing z in the form x iy , where x and y are real numbers, find the exact
11
real and imaginary parts of z and hence find the exact value of tan
.
12
(b)

On a single Argand diagram, sketch the following loci given by


z 4 3i 5 ,
(i)
(ii)

[3]

z i z 7i .

[4]

[2]
[2]

Hence, or otherwise, find the greatest and least possible values of arg (z 6), giving your
answers in radians correct to 3 decimal places.
[4]

THE END

9740/01/Prelim/2013

[Turn over

CATHOLIC JUNIOR COLLEGE


H2 MATHEMATICS
JC2 PRELIMINARY EXAMINATION PAPER I 2013

Solutions
1
c
y ax b
x3
dy
c
a
2
dx
x 3

--------- (*)

Substituting the x- and y-coordinates into eqn (*),


1
5
5

------ (1)
1,
: a b c
4
16
16
1
1
1

------ (2)
2, : 2a b c
5
4
4

Since there is a turning point at 2, ,


4

dy
x 2 would be a solution to
0.
dx
1
a c 0 ------ (3)
25
1
3
25
Solving, a
(or 0.25), b
(or 1.5), c
4
2
4

x c dx c xdx
c

x2
cx
2 c

( c ) 2
c ( c )
2

2c

1
c2 c2
2
3 2
c
2

2c

x c dx c xdx x cdx
0

2c

x x2
cx cx
2 0 2

c
2
2
c2

c 4c
c2
2c 2 c 2
2 2
2

c2

x c dx k

3
x c dx c 2 kc 2
2
3
k
2

2c

(or 6.25)

CATHOLIC JUNIOR COLLEGE


H2 MATHEMATICS
JC2 PRELIMINARY EXAMINATION PAPER I 2013

Alternative:
y

y xc
2c
c
A1 A2
c

A3
c

x c dx k

2c

2c

x c dx

Area A1 = k (Area A2 + Area A3)


1
1
1

c(2c c) k c(c) c(c)


2
2
2

1
3c 2 kc 2

2
3
k
2

Let Pn be the statement

r (3r 2) 2 (n 1)(2n 1) for n

r 1

When n = 1: L.H.S. = 1(3 2) = 1


1
R.H.S. =
(2)(1) = 1
2
P1 is true and forms the basis for induction.
Assume Pk is true for some k
R.T.P Pk 1 is true, i.e.
L.H.S.

k 1

, i.e.

r (3r 2)
r 1

r (3r 2) 2 (k 1)(2k 1)
r 1

(k 1)
(k 2)(2k 1)
2

k 1

r (3r 2)
r 1

r (3r 2) (k 1) 3(k 1) 2
r 1

Pk 1

k
(k 1)(2k 1) ( k 1)(3k 1)
2
(k 1)

[k (2k 1) 2(3k 1)]


2
(k 1)

(2k 2 k 6k 2)
2
(k 1)

(2k 2 5k 2)
2
(k 1)

(k 2)(2k 1) = R.H.S.
2
is true

Since P1 is true, and Pk is true Pk 1 is true hence by MI, Pn is true for n

CATHOLIC JUNIOR COLLEGE


H2 MATHEMATICS
JC2 PRELIMINARY EXAMINATION PAPER I 2013

x4
1 ----------- (*)
3 2 x x2
x4
1 0
3 2 x x2
x4
3 2 x x2

0
3 2 x x2 3 2 x x2
x2 x 1
0
3 2 x x2
x2 x 1
0
x2 2 x 3
2

1 3

Since x 2 x 1 x 0 for any real x


2 4

2
(OR coefficient of x 1 0 and discriminant, b2 4ac (1)2 4(1)(1) 3 0 imply that
x2 x 1 0 for any real x)
1
2
0
x 2x 3
1
0
( x 1)( x 3)
+

x 1 or x 3
By replacing x with x 2 in (*), we have:

x2 4
1
3 2 x2 x4
x2 4
1 which is what we need to solve.
x4 2 x2 3
From earlier part, x2 1 or x2 3(N.A., since x 2 0 for
all real x)
x2 1
x 1 or x 1
(i)
1.873
(ii)

If sequence converges, as n , xn l and xn1 l


2
xn 1 ln 3 xn

2
l ln(3 l )

el 3 l 2
3 el l 2 0
l (proved)
(iii)

Consider e x e x eln[3( x )2] e x 3 xn e x


n1

CATHOLIC JUNIOR COLLEGE


H2 MATHEMATICS
JC2 PRELIMINARY EXAMINATION PAPER I 2013

If xn , y > 0
3 e xn xn 2 0
e xn1 e xn
xn 1 xn (proved)

If xn , y < 0
3 e xn xn 2 0

e xn1 e xn
xn 1 xn (proved)

(iv)

When x1 0, the sequence increases and converges to 1.873 (i.e. ).


When x1 3, the sequence decreases and converges to 1.873 (i.e. ).
x1 0 satisfies x1 , thus
xn1 xn i.e. the sequence increases and converges to .
x1 3 satisfies x1 , thus
x x i.e. the sequence decreases and converges to .
n 1

1
A
B
C

r (r 1)(r 2) r (r 1) (r 2)
1 A(r 1)(r 2) Br (r 2) Cr (r 1)
1
2
Substituting r 1, B 1
1
Substituting r 2, C
2
Substituting r 0,

(a)

1 11

r (r 1)(r 2) 2 (1) 2 2 3
r 1

11 1 11

2 2 3 2 4
11 1 11

23 4 25
.......

1 1 1 11


2 n 2 n 1 2 n

1 1 1 1 1

2 n 1 n 2 n 1
11 1 1 1

2 n n 1 2 n 2

1 1 1 1 1

4 2 n 1 2 n 2

CATHOLIC JUNIOR COLLEGE


H2 MATHEMATICS
JC2 PRELIMINARY EXAMINATION PAPER I 2013

(b) (i)

r (r 1)(r 2)
r 2

1
1

1(1 1)(1 2)
r 1 r ( r 1)( r 2)
1 1

4 6
1

12
(ii)

1
1

2
r 3 r (r 1)
r 3 ( r 1) r ( r 1)

Let r = s + 1

1
1

s 13 ( s )( s 1)( s 2)
s 2 ( s )( s 1)( s 2)
1

12
1

(a) (i) y f x 1
2

(2, 1)

x=0

x=4

(ii)
y

x = 1

x=1

CATHOLIC JUNIOR COLLEGE


H2 MATHEMATICS
JC2 PRELIMINARY EXAMINATION PAPER I 2013

(b)
y

y g( x)

y=1
O (1, 0)

(3, 0)

(2, 2)

(i)
y sin 1 x

sin y x
Differentiate with respect to x :
dy
1
cos y
dx
Differentiate with respect to x :
d 2 y dy
dy
cos y 2 sin y 0
dx
dx
dx
2

d2 y
dy
cos y 2 sin y 0
dx
dx
Differentiate with respect to x :
d3 y d 2 y
dy
cos y 3 2 sin y
dx
dx
dx
2

dy d 2 y dy
dy
sin y 2 2 cos y 0
dx
dx dx dx
3

d3 y
dy d 2 y
dy
cos y 0
cos y 3 3 sin y
2
dx
dx dx
dx
When x 0
y sin 1 0 0
dy
dy
cos 0 1 1
dx
dx
2
d y
d2 y
2
cos 0 2 sin 0 1 0 2 0
dx
dx
3
d y
d3 y
3
cos 0 3 3 sin 0 1 0 cos 0 1 0 3 1
dx
dx
2
3
3
x x
x
Hence y 0 1 x 0 1 x
6
2! 3!

CATHOLIC JUNIOR COLLEGE


H2 MATHEMATICS
JC2 PRELIMINARY EXAMINATION PAPER I 2013

(ii)

1
x
x a bx 2
2
a bx

1 b
x 1 x 2
a a

1 b 2
x 1 x
a a
1
b
x 2 x3
a
a
1
Comparing a 1, b
6

r a a a b
(a)

r b b 2 a b
Given : a : b

r a

r b

ab

b ab

a
2
a ab
b
a

b
a

b ab

a b a b
a b ab

deduced

From the above result,


b r a a r b
Let 1 and 2 be the angle between a and r , b and r respectively. Then
b r a cos 1 a r b cos 2
cos 1 cos 2

1 2
Since cos is a 1-1 function from 0 .
the line OR bisects angle AOB (shown).

CATHOLIC JUNIOR COLLEGE


H2 MATHEMATICS
JC2 PRELIMINARY EXAMINATION PAPER I 2013

(b) (i)
x + 2y + 3z = 1 (1)
2x + y 3z = 5 (2)
2x y 9z = 7 (3)

3
3
Solving, line of intersection : r = 1 + 3 ,
0
1
(ii) If the three planes dont intersect, then the plane 3 does not intersect the line of
intersection. Thus the plane is parallel to the line and thus the normal is perpendicular to the
direction vector of the line.
2 3

a 3 0 6 3a b 0 3a b 6
b 1

Since (3,1, 0) cannot lie on 3 .
3

1
0

2

a 5 6 a 5 a 1
b

Thus b 3a 6,
10

\ 1 .

(a)

dx
2sec tan
d
1
1
x2 x2 4 dx 4sec2 4(sec2 1) 2sec tan d
1
tan d

2sec 4 tan 2
1

tan d
2sec 2 tan
x 2sec

1
d
4sec

1
cos d
4
1
sin C
4
x2 4

C
4x

Note: x 2sec cos

2
x

CATHOLIC JUNIOR COLLEGE


H2 MATHEMATICS
JC2 PRELIMINARY EXAMINATION PAPER I 2013

x2 4

1
2
3

(b) V
1
2
3

1
dx
1 2 x2

1
2
3

dx
2
1 2x

1
x2
2

dx

x
1

tan 1
2 1
1

2
2

2 tan 1
2

2x

1
2

3
2

1
2
3

2 tan 1 1 2 tan 1 3

2


2 2
2 4
3

11

2 2 1 1

2
4 3
2 2

24
(i)

(ii)

dy dy dt
1 3
3t 2 t
dx dt dx
2t 2

dy
3
dx
Equation of tangent: y 8 3 x 6
y 3x 10

When t 2 , x 6 , y 8 ,

CATHOLIC JUNIOR COLLEGE


H2 MATHEMATICS
JC2 PRELIMINARY EXAMINATION PAPER I 2013

(iii)Let t q at the point of intersection Q.

Then the point of intersection is q 2 2, q3 .


Since Q lies on line l , q3 3 q 2 2 10

q3 3q 2 4 0
Since q 2 is a solution to the cubic equation,
q3 3q 2 4 q 2 q 2 aq 2

Comparing the coefficient of q 2 ,


2q 2 aq 2 3q 2
a 1
q 3 3q 2 4 0

q 2 q2 q 2 0
q 2 q 2 q 1 0
q 2 rejected since that is point P
Q 3, 1

or q 1

(iv)

6,8
3, 1
8

Area Area of trapezium x dy


1

8
1
3 6 8 1 1 x dy
2

Consider

x dy t 2 2 3t 2 dt
2

3t 4 6t 2 dt
1

t 5 2t 3
5
1
3
3

32 2 23 1 2 1
5
5

4
37 units 2
5
1
4
7
Area 40 37 2 units 2
2
5
10

CATHOLIC JUNIOR COLLEGE


H2 MATHEMATICS
JC2 PRELIMINARY EXAMINATION PAPER I 2013

12

(a)

1 i

2
3 i

1 i
arg

3arg 1 i arg( 3 i)

3 i

11
3
4 6 12

1 i
z

3 i

1 i

3 i

2e

11
i

12

11
11

2 cos
i sin

12
12

2(1 i ) 2(1 i)
3 i

3 i
3 i
3 i

(1 3) ( 3 1)i

2
2
(1 3) ( 3 1)i
11
11

2 cos
i sin

2
2
12
12

11
(1 3)

12
2
11 ( 3 1)

2 sin
12
2
11
3 1
tan

32
12
3 1
2 cos

(b)

(i)
A

(4,3)

(ii)
3

y=3

6
7

3
Min arg(z 6) = tan 1 tan 1 1
3

(i)
Re

CATHOLIC JUNIOR COLLEGE


H2 MATHEMATICS
JC2 PRELIMINARY EXAMINATION PAPER I 2013

0.785

3
Max arg(z 6) = tan 1
7
2.737
Alternatively, we can locate the points of intersections then proceed to obtain the
required argument:
Equation of circle: ( x 4) 2 ( y 3) 2 25 (1)
Equation of perpendicular bisector: y = 3
(2)
2
Substitute (2) into (1): ( x 4) 25
x 4 5
x = 9 or 1
Points of intersection are (9, 3) and (1, 3).

CATHOLIC JUNIOR COLLEGE


General Certificate of Education Advanced Level
Higher 2
JC2 Preliminary Examination

MATHEMATICS

9740/02

Paper 2

4 Sep 2013
3 hours

Additional Materials: List of Formulae (MF15)


Graph Paper
READ THESE INSTRUCTIONS FIRST
Write your name and class on all the work you hand in.
Write in dark blue or black pen on both sides of the paper.
You may use a soft pencil for any diagrams or graphs.
Do not use staples, paper clips, highlighters, glue or correction fluid.
Answer all the questions.
Give non-exact numerical answers correct to 3 significant figures, or 1 decimal place in the case of
angles in degrees, unless a different level of accuracy is specified in the question.
You are expected to use a graphic calculator.
Unsupported answers from a graphic calculator are allowed unless a question specifically states
otherwise.
Where unsupported answers from a graphic calculator are not allowed in a question, you are required to
present the mathematical steps using mathematical notations and not calculator commands.
You are reminded of the need for clear presentation in your answers.
At the end of the examination, arrange your answers in NUMERICAL ORDER.
Place this cover sheet in front and fasten all your work securely together.
The number of marks is given in brackets [ ] at the end of each question or part question.
Name: ___________________________

Question

Class: ________________

10

11

12

10

12

10

10

Total

Marks
Total

100

This document consists of 5 printed pages.


Catholic Junior College

9740/02/Prelim/2013

[Turn over

2
Section A: Pure Mathematics [40 marks]
1.

(i)

A certain machine A is pumping a liquid into an empty container of total volume 850 m 3.
The first pumping action fills the container with a volume of 4 m 3. Each subsequent
pumping action fills the container with 0.5 m 3 more than the previous pumping action.
Pumping continues until the container is completely filled with the liquid. Find the volume
of liquid that overflows from the container at the final pumping action.
[3]

(ii)

Suppose that, after the 50th pumping action by machine A, a different machine B is used
instead to fill the remaining volume of the container. Using machine B, the first pumping
action fills the container with a volume of 5 m 3. Each subsequent pumping action fills the
5
container with the amount filled in the previous pumping action.
6
(a)
(b)

2.

Find the total volume of the container filled after the 10th pumping action by
machine B.
Explain whether the container would be completely filled eventually.

[3]
[2]

The line l passes through the point A, whose position vector is i + j, and is parallel to the vector
4i 3j + k and the plane p has equation 3x + y = 26.
The plane p is parallel to the line l.
(i)
(ii)
(iii)
(iv)

3.

Show that = 4.
Find the position vector of the foot of the perpendicular, N, from A to the plane p.
Hence, find the coordinates of the point B which is the mirror image of A in p.
Find the area of triangle OAB, where O is the origin.

[2]
[4]
[2]
[2]

Functions f and g are defined by

f :x
g: x

( x 2)2 1,
ln( x2 1),

x , x 2
x

Find f 1 ( x) and state the domain of f 1 .


Sketch the graphs of y f ( x) and y f 1 ( x) on the same diagram. Your sketch should
indicate the position of the graphs in relation to the origin.
(iii) Write down the equation of the line in which the graph of y f ( x) must be reflected in
order to obtain the graph of y f 1 ( x) , and hence find the exact solution of the equation
f ( x) f 1 ( x) .
(iv) Only one of the composite functions fg and gf exists. Give a definition (including the
domain) of the composite function that exists, and explain why the other does not exist.

(i)
(ii)

9740/02/Prelim/2013

[2]
[3]

[3]
[4]

[Turn over

3
4.

A population of fish is modelled by the differential equation


dP
0.02 P 100 P ,
dt
where P is the size of the population at time t (in weeks).
(i)

Find the range of values of P for which the population is strictly increasing.

[2]

Initially, P = 20.

100
.
1 4e2t
(iii) What happens to P for large values of t? Sketch the graph of P against t.
(ii)

Show that P

[5]
[3]

Section B: Statistics [60 marks]


5.

6.

The Student Council wants to collect feedback for the Student Leaders Appreciation Ceremony
from 800 JC1 and 700 JC2 students.
(i) Explain how a systematic sample of size 50 might be obtained.
[2]
(ii) Give one disadvantage of using systematic sampling in this context.
[1]
(iii) Suggest a method of obtaining a more representative sample and describe how it may be
carried out.
[2]
3
7
11
For events A and B, it is given that P ( B) , P ( A B) and P ( A ' B) .
5
8
36
Find
(i)
[2]
P ( A) ,
(ii) P ( B ' | A) .
[2]
A third event C, has P (C )
(iii) Find P ( A ' C ) .

7.

4
and that A and C are independent.
7
[2]

10 students from 2T39 and 2T40 are to be seated in a round table of 10 during Graduation Night
this year. The number of boys and girls who are attending Graduation Night from these two
classes are shown below.

2T39
2T40

Boys
4
1

Girls
2
3

Find the number of ways the students can be seated if


(i)
e are no restrictions.
(ii)
boys and girls must alternate.

ther
[1]
the
[2]

The 10 seats are now numbered.

9740/02/Prelim/2013

[Turn over

4
(iii)

Fin
d the number of ways these students can be seated if students from the same class must
sit together.

[2]
The

(iv)

8.

boy from 2T40 is absent. Find the number of ways the remaining students can be
seated in the 10 seats if students from the same class must sit together.
[2]
A school bought Upads for JC1 students to use in Project Work lessons. On average, 3% of the
Upads are faulty. Faults occur independently and randomly.
(i)
(ii)

In a sample of 25 Upads, find the probability that more than 2 Upads are faulty.
[2]
Three samples of 25 Upads are taken. Find the probability that there are more than two
faulty Upads in one sample and exactly one faulty Upad in each of the other two samples. [2]

The school bought a batch of 80 Upads for JC1 students.


(iii) Use a suitable approximation to find the probability that there are at least 77 Upads that
are not faulty in this batch.
9.

[3]

A factory manufactures light bulbs. The lifetime of a light bulb is denoted by X and, over a long
time, it is known that the mean lifetime of a light bulb is 330.4 hours. After the factory replaces
all its machines with new ones, 15 light bulbs are randomly selected for testing. The results are
summarised below:

x 330.4 26.3 x 330.4


(i)

159.49

Fin
[2]
Tes
t at 1% level of significance whether the mean lifetime of a light bulb has increased. What
assumptions, if any, should be made about the population, in order to carry out this test? [4]
d unbiased estimates of the population mean and variance.

(ii)

It is now known that the standard deviation of X is 4.1 hours. 50 light bulbs are selected for
testing at % level of significance, with the same null and alternative hypotheses as (ii). The
sample mean is 331.5.
(iii)
still necessary to make the assumption(s) in (ii)? Explain your reasoning.
(iv)

10.

d the least value of , correct to 3 significant figures, such that the null hypothesis can
rejected.

Is it
[1]
Fin
be
[1]

In a junior college in Singapore, the number of students who are on crutches per month is
recorded. It is known that in the first half of the year, the number of JC2 students on crutches per
month follows a Poisson distribution with mean 4 and the number of JC1 students who are on
crutches per month follows a Poisson distribution with mean .
(i)

Find the probability that between 3 and 7 JC2 students are on crutches in a randomly
chosen month in this period.

[2]

9740/02/Prelim/2013

[Turn over

5
Write down an equation involving if the probability that the total number of JC1
and JC2 students on crutches in a month being exactly 2 is 0.0027. Hence, find the value
[3]
of , correct to the nearest integer. State an assumption you have made.
(iii) Using the integer value of found in (ii), find the probability that exactly one JC1
student is on crutches in a month, given that the total number of students on crutches in
that particular month is exactly 4.
[3]
(iv) Explain why the Poisson distribution may not be appropriate in modelling the number of
students who are on crutches per month over a period of one year.
[1]
(ii)

9740/02/Prelim/2013

[Turn over

6
11.

A scientist is studying the growth of a bacteria colony in a petri dish. He places bacteria at one
spot of the dish and he measures the area A the bacteria cover on day t. His results are recorded
below:
Time, t (days)
Area, A (cm2)
(i)

0
1.00

1
5.00

2
27.10

3
41.39

4
46.95

5
49.95

6
51.07

7
52.05

Dra
[1]
The
scientist would like to predict the future growth of the bacteria. Explain why, in this context, a
linear model is not likely to be appropriate.
[1]
w a scatter diagram to illustrate the data.

(ii)

It is proposed to fit a model of the form ln( D A) a bt , where D is a suitable constant. The
product moment correlation coefficient between t and ln( D A) is denoted by r. The following
table gives values of r for some possible values of D.
D
r
(iii)

(iv)

(v)

12.

53

54
0.99132

55
0.98767

culate the value of r for D = 53, giving your answer correct to 5 decimal places.
Hence, explain which value of D is the most appropriate.
h the value of D in (iii), calculate the values of a and b correct to 5 decimal places,
use them to predict the area covered by the bacteria after 20 days.
e an interpretation, in context, of the value of D.

Cal
[2]
Wit
and
[3]
Giv
[1]

The waiting times, in minutes, for orders of Chili Crab and Pepper Crab at a popular hawker stall
are modelled as having independent normal distributions with means and standard deviations as
shown in the following table:
Mean Waiting Time
Chili Crab
Pepper Crab
(i)

10

Standard Deviation
3
2

If the probability that one randomly chosen order of Pepper Crab takes a longer waiting
time than one randomly chosen order of Chili Crab is 0.290, show that = 12.0, correct
to 3 significant figures.
[3]

Customers who drive park their cars at a nearby car park at $0.05 per minute. The walking time
between the car park and the hawker stall is negligible. The time taken by a randomly chosen
customer to finish his meal is normally distributed with mean 20 minutes and standard deviation
4 minutes.
(ii)

Find the probability that a randomly chosen customer who parked at the car park, bought
one order of Pepper Crab, finished his meal and immediately drove off, spent more than
$2 on car park charges.
[3]

9740/02/Prelim/2013

[Turn over

7
(iii) A random sample of n customers who each make one order of Pepper Crab is taken.
Given that the probability that their mean waiting time exceeds 10.5 minutes is at most
0.0385, find the smallest possible value of n. Explain briefly if any assumption is needed
for the above calculation.
[4]
THE END

9740/02/Prelim/2013

[Turn over

CATHOLIC JUNIOR COLLEGE


H2 MATHEMATICS
JC2 PRELIMINARY EXAMINATION PAPER 2 2013

Solutions
1
(i)
Total volume: 850 m3
Volumes (in m3) filled: 4, 4.5, 5, 5.5, 6,
Assume that the container is completely filled at the nth pumping action.
For the AP above,
n
Sn 2(4) (n 1)(0.5) 850
2
2
n 15n 3400 0 ----- (*)
Considering n2 15n 3400 0,

15 152 4(1)(3400)
we have n
,
2(1)
i.e., n 51.2899 or n 66.2899 (or use G.C.)
Hence from (*),
(n 51.2899)(n 66.2899) 0
n 66.2899 (NA, since n 0) or n 51.2899 at the 52th pumping action, the
container would be completely filled, with some liquid overflowing.
52
S52 2(4) 51(0.5)
2
871
volume of liquid that overflows 871 850 m3 21 m3

Alternative
n
Sn 2(4) (n 1)(0.5) 850
2
Using G.C.,
n
n
2(4) (n 1)(0.5)
2
51
841.5
52
871
53
901
volume of liquid that overflows
871 850 m3
21 m3

(ii)
For machine A,
50
S50 2(4) 49(0.5)
2
812.5
5
Volumes (in m ) filled by B: 5, 5 ,
6
3

5
5 , ...
6

CATHOLIC JUNIOR COLLEGE


H2 MATHEMATICS
JC2 PRELIMINARY EXAMINATION PAPER 2 2013

For the GP above,


5 10
5 1
6

S10
5
1
6
156941

25.1548 m3
6239
total volume filled after 10th pumping action by B
(812.5 25.1548) m3
837.6548 m3
838 m3

(b)
(For the GP above, since | r |
Sum to infinity

5
1, sum to infinity exists.)
6

5
1
6
30

Volume of container to be filled by machine B


850 812.5 m3 37.5 m3
Since the theoretical maximum volume that B alone can fill is only 30 m3, which is less than
37.5 m3,
the container would never be completely filled.
Alternative:
Since the container can only be filled to a theoretical maximum total volume of 812.5 + 30 =
842.5 m3 , which is less than 850 m3, the container would never be completely filled.
2

(i)

1
4

l: r == 1 + 3 , (1)
0
1
3
p: r = 26
(2)
0
If l is parallel to p, then normal of p l
3 4
i.e. . 3 = 0
0 1
12 3 = 0
=4
(ii)
1
3

1
Let ON be + 4 for a particular .
0
0

CATHOLIC JUNIOR COLLEGE


H2 MATHEMATICS
JC2 PRELIMINARY EXAMINATION PAPER 2 2013

1
3 3

Then [ 1 + 4 ].4 = 26
0 0
0
3 + 9+ 4 + 16 = 26
= 1
1
3 2

ON = 1 + 14 = 5
0 0
0
(iii)
Using Ratio Theorem,

2 OA
+ OB
5 =

2
0
2 1 5

OB = 25 1 = 9
0 0 0

2,5,0
B

Coordinates of B: (5, 9, 0)
(iv)
Area of triangle OAB
1 5
0
1

1 1
=
| OA OB | 1 9 0 7
2
2 2

0 0
14

(i)

y ( x 2)2 1

( x 2) 2 y 1
x 2 y 1
x 2 y 1 (N.A.) or 2 y 1

Df 1 = R f 1,
f 1 : x

2 x 1, x , x 1

(ii)
y
y = f (x)
y=x

(1, 2)

3
O

(iii)

3
(2, 1)

x
y = f 1(x)

CATHOLIC JUNIOR COLLEGE


H2 MATHEMATICS
JC2 PRELIMINARY EXAMINATION PAPER 2 2013

y=x

f ( x) f

( x) f ( x) x
( x 2) 2 1 x
x2 5x 3 0
5 25 4(1)(3)
2
5 13
5 13
or

2
2
5 13 5 13
rejected

2
2

(iv)
For fg to exist, R g Df
R g = 0,
Df , 2

Since R g Df , fg does not exist.


For gf to exists, R f Dg
R f = 1,

Dg ,

Since R f Dg , gf exists.

gf(x) g ( x 2)2 1

2
ln ( x 2)2 1 1

ln x4 8x3 22 x2 24 x 10

Dgf Df , 2

gf : x

2
ln ( x 2)2 1 1 , x , x 2

(i)
For increasing population,
0.02 P 100 P 0

dP
0
dt

0 P 100
(ii)
dP
0.02 P 100 P
dt
1
P 100 P dP 0.02dt

1 1
1

dP 0.02t C

100 P 100 P
ln P ln 100 P 2t C '

ln 100 P ln P 2t C ''

x <2

CATHOLIC JUNIOR COLLEGE


H2 MATHEMATICS
JC2 PRELIMINARY EXAMINATION PAPER 2 2013

ln

100 P
2t C ''
P
100 P
e2t C ''
P
100 P
eC ''e2t
P
100 P
Ae2t
P
100
P
1 Ae2t

100
1 A
1 A 5

Given P = 20 when t = 0, 20

A 4
100
(shown)
P
1 4e2t
(iii)
P is an increasing function and for large values of t, P 100 .
P
P 100

20
t

(i)
Obtain the list of all 1500 students (i.e. sampling frame)
Arrange them in some order (e.g. alphabetical, NRIC, class, level, etc)
Select a random starting point.

Subsequently select students at regular intervals of


50 is obtained.

1500
30 students until the sample of
50

(ii)
Systematic sampling of every 30th student arranged in by class may result in the smaller
classes being skipped over altogether.
OR
Any reasonable answer.
(iii)
Stratified sampling.
Divide the students into non-overlapping strata (e.g. JC1 and JC2).
Select the number of students to pick from each stratum according to proportion.
JC1
JC2
700
800
50 23.33 23
50 26.67 27
1500
1500
Select required number of students randomly from each stratum.

CATHOLIC JUNIOR COLLEGE


H2 MATHEMATICS
JC2 PRELIMINARY EXAMINATION PAPER 2 2013

(i)
A

P ( A ' B) P ( A B) P ( A)
P ( A) P ( A B) P ( A ' B)
7 11 41
P ( A)
8 36 72
(ii)
P ( B ' A)
P ( B ' | A)
P ( A)
A

P ( B ' A) P ( A B) P ( B)
7 3 11
P ( B ' A)
8 5 40
11
P ( B ' A)
99
P ( B ' | A)
40
41
P ( A)
205
72
(iii)
If A and C are independent,
P ( A C) P ( A) P (C)
P ( A ' C) P ( A ') P (C)

41 4
P ( A ' C ) 1
72 7
31 4 31

72 7 126
(i)
No of ways for 10 students to sit in a round table = (10 1)! =9! =362880
(ii)
Arrange the boys then slot in the girls.
No. of ways of arranging the 5 boys in a circle
= (5 1)! = 4! = 24
No. of ways of slotting the 5 girls into the 5 positions = 5C5 = 1
No. of ways of arranging the 5 girls = 5! = 120
Total no. of ways = 24 1120 2880
(iii)

CATHOLIC JUNIOR COLLEGE


H2 MATHEMATICS
JC2 PRELIMINARY EXAMINATION PAPER 2 2013

Group the students from 2T39 and 2T40.


Obtain one group of 6 and one group of 4.
No. of ways of arranging the 2 groups in a circle = (2 1)! = 1! = 1
No. of ways of arranging the group of 4 = 4! = 24
No. of ways of arranging the group of 6 = 6! = 720
Total no. of ways = 1 24 720 10 172800
(iv)
Group the students from 2T39 and 2T40. Obtain one group of 6 and one group of 3 with 1
empty chair.
No. of ways of arranging the 3 groups in a circle = (3 1)! = 2! = 2
No. of ways of arranging the group of 3 = 3! = 6
No. of ways of arranging the group of 6 = 6! = 720
Total no. of ways = 2 6 720 10 86400

(i)
Let X be the r. v. denoting the number of faulty Upads out of 25.
X ~ B(25, 0.03)
P( X 2) 1 P( X 2)
= 0.0379596464
= 0.0380
(ii)
2
Required probability P( X 2) P( X 1) 3
= 0.0148
(iii)
Let Y be the r. v. denoting the number of faulty Upads out of 80.
Y ~ B(80, 0.03)
Since n = 80 is large, np = 2.4 < 5,
Y ~ P0(2.4) approximately.
P(Y 3) 0.779
(i)

x 330.4

330.4
15
332.153
2

x 330.4
1
2
2

s
x 330.4

15 1
15

8.09838

(ii)
To test
H0: = 330.4
H1: > 330.4 at 1% level of significance
Value of test statistic, t

332.153 330.4
8.09838 / 15

Degree of freedom = 14
p-value = 0.01586 > 0.01
We do not reject H0 as there is insufficient evidence at 1% level of significance to conclude
that the mean has increased.
Assume that the lifetime of a light bulb is normally distributed.
(iii)

CATHOLIC JUNIOR COLLEGE


H2 MATHEMATICS
JC2 PRELIMINARY EXAMINATION PAPER 2 2013

10

No, as n is large, Central Limit Theorem may be applied.


(iv)
p-value = 0.028906
least to reject H0 = 2.90 (3s.f.)
(i)
Let C be the random variable denoting the no. of JC2 students on crutches in a month.
C ~ Po (4)
P(3 C 7)

P(4 C 6)
P(C 6) P(C 3)
0.45586 0.456

(ii)
Let J be the random variable denoting the no. of JC1 students on crutches in a month.
J ~ Po()
C J ~ Po(4 )
Given that P(C J 2) 0.0027
(4 ) 2
0.0027
2!
(4 ) 2 0.0054e(4 )
e (4 )

Using G.C. to solve the equation (4 )2 0.0054e(4 ) , we have 6 .


We assume that the number of JC1 and JC2 students who are on crutches follow independent
Poisson Distributions.
(iii)
P(J = 1C+ J = 4)
P(C 3) P( J 1)
P(C J 4)
0.1536

0.154

(iv)
The average number of students who are on crutches may vary from month to month, for
example, on an average there will be more students who suffer leg injuries during intensive
training just before sports competitions compared to months when the competition is over.
11

(i)

(ii)
A linear model is not likely to be appropriate as the area covered would then increase
continuously, eventually to an infinite area.
(iii)

CATHOLIC JUNIOR COLLEGE


H2 MATHEMATICS
JC2 PRELIMINARY EXAMINATION PAPER 2 2013

D = 53: r = 0.99349

12

Since D = 53 gives a value of r closest to 1, it is the most appropriate.


(iv)
a = 4.26272; b = 0.60899
Equation of regression line is
ln(D A) = 4.26272 0.60899t
When t = 20, A = 52.99964 cm2
(v)
D is the maximum area of the petri dish.
(i)
Let X and Y be random variables denoting the waiting times, in minutes, for one random
order of Chili Crab and one random order of Pepper Crab respectively.
Then X ~ N( , 32) and Y ~ N(10 , 22)
Y X ~ N(10 , 32 + 22)
Y X ~ N(10 , 13)
P(Y X > 0) = 0.290
0 (10 )
P( Z >
) = 0.290
13
10

= 0.5533847152
13
= 11.99525697 12.0 (to 3 s.f.) (shown)
(ii)
Carpark charges $0.05 per minute
Let T be the random variable denoting the carpark charges of one random customer in cents.
T ~ N( 5 10 + 5 20 , 52 22 + 52 42 )
T ~ N( 150, 500)
P( T > $2) = P( T > 200 cents)
= 0.0126736174 0.0127
(iii)
Since Y ~ N(10 , 22)
22

So, Y ~ N(10, )
n

Given P(Y > 10.5) 0.0385


Method 1: Using G.C. table
n
P(Y > 10.5)
50
0.03855
51
0.0371
52
0.03571
Hence, least value of n is 51.

CATHOLIC JUNIOR COLLEGE


H2 MATHEMATICS
JC2 PRELIMINARY EXAMINATION PAPER 2 2013

Method 2: Using standardization

P(Y > 10.5) 0.0385


10.5 10
P(Z >
) 0.0385
2
n
0.5 n

1.768364425
2

n 50.03380381
Hence, least value of n is 51.
No assumption needed since Y is normally distributed.

ANDERSON JUNIOR COLLEGE


2013 Preliminary Examinations
H2 MATHEMATICS (JC2)
PAPER 1

Duration : 3 hr

Answer ALL questions.


1.

It is given that y

1
, where a is a positive constant. If x is
5 2cos 2 (ax)

sufficiently small such that the series expansion for


powers up to and including the term in x2 is

2.

1
in ascending
5 2 cos 2 (ax)

1 18 2

x , find the value of a .


7 49

[4]

By means of the substitution 2u cos x , show that


1
2
0

4u 2 1 4u 2 du

1 2 2
sin 2 x dx ,
8 0

and evaluate this definite integral exactly.


1

Hence or otherwise, evaluate 2


0

3.

1 4u 2

[3]

du exactly.
3

[2]

The function f is a strictly increasing function such that y f ( x) for x 0. The


coordinates of certain points on the curve of y f ( x) are as follows:

1
3

7
2
7
2

11

14

14

19

24

26

(i) State the value of ff (6) and the value of f 1 (8) .


Another function g is defined by g: x

tan 2x for 0 x

[2]

(ii) Explain why the composite function fg exists.

[1]

(iii) Find the range of fg. Hence find the set of values of x such that the composite
function fg satisfies the inequality fg x 1 . Leave your answer in exact form.

Page 2 of 7

[3]

4.

Given that y sin 1 (2 x) , show that


2

dy
(1 4 x 2 ) 4 .
dx

[1]

(i) By further differentiation of this result, find the Maclaurins series of y up to and
including the term in x3 .

[4]

(ii) Region R is bounded by the curve y sin 1 (2 x) , the x-axis, the lines x

1
and
2

1
. Using your answer to part (i), write down a definite integral that will give an
2

approximate value for the volume of the solid generated when R is rotated through
2 radians about the x-axis.

[1]

By referring to the graph of y sin 1 (2 x) and the Maclaurins series of y in part (i),
determine whether this approximate value is an under-estimation or over-estimation
to the actual volume of the solid generated.
5.

[2]

A trough is 8 metres long and its cross sectional ends are in the shape of an isosceles
triangle whose width is 5 metres and height is 2 metres. It is held in the position as
shown in the figure by stands of the same height. At time t seconds, the height of the
water in the trough is h metres and the width of the water surface is w metres. The
trough is initially empty and water is being pumped in at a constant rate of 5 m3/s.

8
2

(i) Show that w

5
h . Hence find the volume of the water in the trough in terms of h.
2

(ii) Find the rate of change of h at t = 2.

[2]
[3]

From t = 2, the rate at which water is being pumped into the trough is changed to
2h m3/s.
(iii) Find the total time taken for the trough to be completely filled.

Page 3 of 7

[3]

6.

(a) The first term of an increasing arithmetic progression is 1. Sn is the sum of the first
n terms of the arithmetic progression and S5, S10 and S20 form a geometric
progression. Show that the common difference of the arithmetic progression is 2.

[3]

Without the use of a graphing calculator, find the least value of n such that

2Sn
1.
Sn 1 100

[3]

(b) Mr Wee borrowed $3400 from an unlicensed money lender on 1st June 2013. The
amount he owes the lender at the beginning of each month is twice the outstanding
amount at the end of the previous month.
From the month of August 2013, Mr Wee decided to repay $7000 in the middle of
each month. Find, in terms of n, the outstanding amount owed at the end of the nth
month.
Hence find the earliest month when he has fully repaid his loan.
7.

[5]

The diagram shows the graph of y f x . The curve has a minimum point at (-1, 0)
and a stationary point of inflexion at (1, -1). The asymptotes are y 4 , y 0 , x 0 and

x 2.

Sketch , on separate clearly labelled diagrams, the graphs of


(i) y 2 f x ,

[3]

(ii) y f x .

[3]

In each case, indicate clearly the coordinates of any points where the curve crosses the
axes, turning points and equations of asymptotes whenever possible.
Find the number of points of intersection between the curves y = f 1 x and y f ' x . [3]

Page 4 of 7

8.

a

The plane p with equation r b 15 contains point Q with coordinates (0,5,0).
4

Plane p makes an acute angle of with the x-axis.
(i)

Find b.

[1]

(ii)

Write down an equation relating , a and b.

[1]

a

It is known that the angle between b and the positive x-axis direction is obtuse and
4

45o .
(iii) Show that a 5 .

[2]

Plane p, the x-z plane and the x-y plane all meet at a common point W.
[2]

(iv) Find the position vector of W.


The point M with coordinates (1,0,5) is a point on plane p. The line l is the
intersection line between p and the x-z plane.
(v)

Explain why M is on l . Hence or otherwise, find the shortest distance of Q to


the line l .

9.

[3]

(a) The sequence of real numbers u1, u2 , u3 , . . . is defined by


un 1

n2
un and u1 2a, where a is a positive real constant.
n2

(i) Prove by mathematical induction that un


(ii)

a(n 1)n
for n
(n 1)!

Show that un1 un for n 3 . Hence show that

16

u
r 3

84a .

p(q)n1
r 2

d
x

ln

where p and q are constants to be


2 2

x 1
r 2
n(n 1)
determined.
12
r 2 2

dx .
Hence find the exact value of
2
2
x 1
r 6

(b) Show that

[4]
[5]

Page 5 of 7

[6]

10. (a) The general solution of the differential equation

dy
1
2 y 2 x 2 is y 2
,
x 4x C
dx

where C is an arbitrary constant.


(i) When C = 4, show that there is no stationary point for this particular solution.

[2]

(ii) Sketch, on separate diagrams, the graph of the solution in part (i), and 2 other
typical members of the family of solution curves.
[You need not work out the values of the axial intercepts and the equations of the
asymptotes (if any) in your diagrams.]

[3]

(b)A certain circular-shaped leaf has radius r cm that is proportional to the amount of
water, w, it contains during a period of its growth at time t. The leaf absorbs water
from the plant at a rate equals to 8 times the radius of the leaf and loses water by
evaporation at a rate equals to

1
times the area of the leaf. During the period of

growth, it may be assumed that the shape of the leaf will be the same.
Given that

dr
6 when r = 2, show that the growth of the leaf can be represented by
dt

the differential equation

dr 1
8r r 2
dt 2

[3]

Given that, when t = 0, r = 4, find r in terms of t. What happens to the radius of the
leaf for large values of t.

[4]

11. (a) The complex numbers s and w satisfy the equations

s w 6i

sw 10 .

and

Given that Re(s) > 0, solve the equations for s and w , giving all answers in the
form x iy , where x and y are real.

[4]

Hence find the solution to the following equations

u v 6

and

uv 10 .

Give your answers for u and v in the form x iy , where x and y are real.

Page 6 of 7

[2]

(b) Find, in the form z rei , the three roots z1 , z2 and z3 of the equation z 3 3 3i
where arg( z1 ) arg( z2 ) arg( z3 ) . Give your answers in exact form.

[3]

The points Z1, Z2 and Z3 represent z1 , z2 and z3 respectively. Find the area of the
triangle formed by Z1 Z2 Z3.

[2]

The constant c is a complex number such that the points representing cz1 , cz2 and

cz3 forms another equilateral triangle which is congruent to triangle Z1 Z2 Z3, and
one of its vertices lies on the positive real axis. Find a suitable value for the
complex constant c in the exponential form.

END OF PAPER

Page 7 of 7

[2]

ANDERSON JUNIOR COLLEGE


2013 Preliminary Examinations
H2 MATHEMATICS (JC2)
PAPER 1 (Solutions)

1
For small , cos 1 2
2
1
y
5 2 cos 2 ( ax)
1

or
5 1 cos(2ax)
1
6 1 2a 2 x 2
1
=
7 2a 2 x 2

1
1 2a 2 2
y

x
1
7
2a 2 2 7

(7) 1
x
7

1
5 2 cos 2 ( ax)
1
a2 x2
5 2 1

1
5 2 1 a 2 x 2

1
7 2a 2 x 2

1 2a 2 2
x
1
7
7

1 2a 2 2

x
7 49
1 18 2

x
7 49

2a2 18

2u cos x
1
2
0

a3

du
sin x

dx
2

4u 2 1 4u 2 du

0
sin x
cos 2 x 1 cos 2 x
dx
2

2
1 0
cos 2 x sin 2 x dx
2 2

1 2 2
sin 2 x dx
8 0
1
2 1 cos 4 x dx
16 0

1 sin 4 x 2
x
16
4 0

32

1
2
0

1 4u 2

du
3

1
3 2
1

u 1 4u 2 2 2 u 1 4u 2 2 8u du

0 0 2

1
2
0

3 4u 1 4u
2

1
2 2

du


3
32
3

32

Alternatively,

1 4u
1
2
0

1 4u 2 du

2 1 4u 2 du 2 4u 2
0

1 4u 2 du

1 0 2

sin x dx

2 2
32

1 2

1 cos 2 x dx

4 0
32

1 sin 2 x 2
x

4
2 0 32

3
32

(i)

From the tables,

(ii)

Since Rg = [0, 1] Df = [0, ), fg exists.

(iii)

ff (6) = f(14) = 26
f 1 (8) = 4


Dfg = Dg = 0, .
8
Taking Rg as new Df , Rfg = [2, 0.5]

(f is increasing)

fg x 1

f (tan 2 x) 1

But Rfg = [2, 0.5],

1< f (tan 2 x) 0.5


1

tan 2 x 1
3

2x

12

y sin 1(2 x)
diff. w.r.t. x
dy
2

dx
1 2x

dy
2
dx
2
dy
(1 4 x 2 ) 4
dx
1 4 x2

(i)
2
dy d y
dy
2(1 4 x 2 ) 2 8 x 0
dx dx
dx
2

2
dy
dy
2 d y
2(1

4
x
)
2 8x 0

dx
dx
dx

dy
0,
dx
d2 y
dy
2(1 4 x 2 ) 2 8 x 0
dx
dx

Since

d3 y
d 2 y dy
d2 y
2(1 4 x ) 3 16 x 2 8 8 x 2 0
dx
dx dx
dx
2

Sub x = 0

dy
2
d2 y

2,
0,
2
2
dx
d
x
1 0
f 0 0 f ' 0 2 f '' 0 0 f ''' 0 8

y sin 1 0 = 0 ,

f ( x) f (0) f '(0) x

f x 2x

4 3
x ...
3

f ''(0) 2 f '''(0) 3
x
x
2!
3!

d3 y
dx3

(ii)
2

1
4

Volume 21 2 x x3 dx

3
2

The approximated volume is an under-estimation of the actual volume. This can be seen
from the above diagram, the region under the graph of y sin 1 (2 x) is larger than the
4
region under the graph of y 2 x x3 .
3
(i)
Using similar triangles ABC and DEC,
w h
5
w h
5 2
2

Volume of the water in the tank = Base area length


1
V = wh 8
2
5
V = 4hw = 4h h =10 h2
2
dV dV dh

(ii)
dt
dh dt
dV
dh
20h
---- (1)
dt
dt
At t = 2 seconds, V = 2(5) = 10 m3
To find h,

10 = 10 h2
h =1m

dh
5
dt
dh 1

m/s
dt 4

20

(iii) from (1),

2h 20h

dh
dt

dh 1

m/s
dt 10
Since the rate of change for h is a constant,
time taken for h from 1 to 2 m = 10 seconds
Therefore the time taken for the trough to be completely filled is 12 seconds.
First term =1, common difference = d
6
(a) S5 , S10 , S20 form a GP
S
S
10 20
S5 S10
2

20
10
5
2 9d 2 4d . 2 19d
2
2
2

2 9d 2 4d . 2 19d
2

5d 2 10d 0
d 2 or d = 0 (rejected as AP is increasing)
2Sn
1
S n 1 100
2n 2

n 1

100

2n 2 (n 1) 2 100

n 1

100

n 2 2n 99

n 1

100

Since n2 2n 99 0 as discriminant < 0 and coefficient of n2 is +ve.


2
n 1 100 0

(n 1 10)(n 1 10) 0
(n 11)(n 9) 0
n9
least n is 10.

6
(b)

nth
month

Outstanding amt owed at


the start of the month (in
hundreds)

Outstanding amt owed at the end of the nth


month (in hundreds)

34

34

(34)2

(34)2

(34)22

34(2)2-70

2 34(2)2 70

34(2)3 70(2) 70

2 34(2)3 70(2) 70

34(2)4 70(2)2 70(2) 70

34(2)n1 70(2)n3 .......... 70(2) 70

Total amount of money owed at the end of nth month


= 100( 34(2)n1 70(2)n3 .......... 70(2) 70 )

70((2) n 2 1)
100 34(2) n 1

2 1

n1
100 70 2
To be free from debt,
70 2n1 0
2n 140
ln140
n
7.129
ln 2
Least n = 8
Earliest month to be free from debt = Jan 2014

From graph, there are 2 points of intersection. Thus, there are 2 solutions.

(i) The points Q (0,5, 0) is on plane p


0 a

5 b 15 5b 15 b 3
0 4

a 1

3 0
4 0
a

(ii) sin
i.e. sin
a 1
a 2 25

3 0
4 0

a

(iii) The angle between b and
4

a
1

45
2
a 2 25
a 2 25 2 a

1
a 1


0 is obtuse b 0 0 a 0
0
4 0

Since a a 2 ,
2

a 2 25

Since a < 0, a = 5
(iv) Equation of plane p :
Equation of x-z plane :
Equation of x-y plane :

5x 3 y 4 z 15
y0
z0

Solving simultaneously

3

x 3 , y 0 , z 0
i.e. OW 0
0

(v) M (1,0,5) is on plane p (given) and
is also on the x-z plane ( y-coordinates of M 0 )
M is on their line of intersection l .
Points M and W are on l , and the shortest distance of Q to the line l

3 4

5 0
0 5
WQ WM

=

WM
4 2 52

25

15
20
1250

units or
41
4 2 52

MQ WM
WM

9
a(n 1)n
P
be
the
proposition
that
u

for n .
(i)
Let
n
n
(a)
(n 1)!
For n = 1, LHS of P1 = 2a
a(1 1)1 2a
=
2a = LHS of P1
RHS of P1 =
(1 1)!
1
Thus, P1 is true.
Assume that Pk is true for some k
a(k 1)k
i.e. uk
(k 1)!
To prove that Pk 1 is true:

k2
uk
k2
(k 2) a (k 1)k

k2
(k 1)!
a (k 2)(k 1)
=
k (k 1)!
a (k 2)(k 1)
=
k!
Thus Pk 1 is true.
uk 1

Since P1 is true, and Pk true Pk 1 is true. By Mathematical Induction,

Pn is true for all n


(ii) Consider

n2
un un
n2
n2
un ( 2 1)
n
n 2 n2
)
un (
n2
(n 1)(n 2)
un
0
n2

un 1 un

since for n 3 and a 0,

(n 2) 0 and un

un1 un for n 3
Alternatively,
un 1 un
a (n 2)(n 1) a (n 1)(n)

n!
(n 1)!
a (n 1)

n 2 n2

n!
a (n 1) 2 (n 2)

n!
Since n 3 and a 0 , (n 1)2 0 and (n 2) 0
un1 un 0 for n 3

16

u
r 3

u3 u4 u5 ... u16
u3 u3 u3 ... u3

since ur 1 ur for r 3

14u3
84a
16

ur 84a
r 3

9
(b
)

since u3 6a

a (n 1)(n)
0
(n 1)!

r 2

dx
x 1
2

r
n
x 1
ln

x 1 2
r 2
n
r 1

ln
ln 3
r 1

r 2
n
r 1 n
ln
ln 3
r 1 r 2
r 2
n

ln r 1 ln r 1 ( n 1) ln 3
r 2

ln(3)
ln(1)

ln(4)
ln(2)
ln(3)
ln(5)

(n 1) ln(3)

ln(n 2) ln( n)

ln(n 1) ln(n 1)
ln 2 ln( n) ln( n 1) ln 3n 1
ln

12

2(3) n 1
n(n 1)


r 6

r 2

m 2 12

dx
x 1
2

m 26

dx
x 1
2

14
m 2

2 dx
2 x 1

m 8
14
m 2
7 m 2

2 dx 2 dx
2 x 1
2 x 1
m2

m2

2(3)13
2(3)6
ln
ln
14(15)
7(8)

10
(a)

4(3)6
ln
5
(i)

dy
2x 4

2
2
dx
x 4x 4
2( x 2)
( x 2) 4
2

( x 2)3
dy
2
0 , there are no stationary point when C = 4.
0,
Since
3
dx
( x 2)

(ii)

When C = 4

10
(b
)

When C > 4

When C < 4

Let w be the amount of water present in the leaf at any time t.


dr
dw
where m is a positive constant
m
dt
dt
dw
1
Since
8r r 2 where m is a positive constant.
dt

dr
m 8r r 2
dt
dr
6.
When r = 2 ,
dt
6 m(16 4)

1
2

dr 1
8r r 2
dt 2

1
1
dr dt
2
8r
1
1
r 4 2 42 dr 2 t C

1 r 44
1
ln
t C
8 r 44
2

[M1]

1 r 8
1
ln
t C
8
2
r
r 8
ln
4 t 8C
r
r 8
e 4t 8C Be 4 t where B = e8C
r
8
r
1 Be 4 t
When t = 0, r = 4
8
4
1 Be0
B 1
8
r
1 e4t

11
(a)

As t , e4t 0, r 8
The radius of the circular shaped leaf will grow to a radius of 8 cm for large values of t.
s w 6i The real part of s and w are the same.
Let s a bi and w a ci

b c 6 --- (1)

a bi a ci 10
a 2 bc a(b c)i 10

a 2 bc 10 and b c 0
a 1, b 3 and c 3
Since a > 0,
s 1 3i, w 1 3i
Alternatively,

Subst w

10
into s w 6i ,
s

10
6i
s
s 2 10 6i s
s

s 2 6i s 10 0
6i 36 4(10)
1 3i
2
Since Re(s) > 0, s 1 3i and w 1 3i
s

Let u is and v iw and we would arrive at the original pair of given equations.

u 3 i and v 3 i
Alternatively,
s iv, w iu
v 3 i and u 3 i
11
(b
)

3 3i 12 and arg(3 3i)


5

z 3 12e 6
1

i 2 k i
5

z 12 6 e 18
1
6

z1 12 e

Z1OZ2 ,

5
6

2 k i
3

7
i
18

, k 0, 1
1

, z2 12 6 e 18

17

and z3 12 6 e 18

1
6

Z2OZ3 and Z1OZ3 are congruent triangles with z1 z2 z3 12 and

Z1OZ 2 Z 2OZ3 Z1OZ3

2
3

1
1
1
2
6
12
12
6 sin
2
3
1
3 3

12 3
4

Area of triangle Z1Z 2 Z3 3

Let c ei
i

cz2 e e

5 i
18

i
18

Since cz2 is a positive real number, arg(cz2 ) 0

5
5
0
18
18
ce

5 i
18

Simliarly, we can also consider cz2 or cz3


The corresponding values for c would be e

17 i
18

and e

Any one of the above 3 values of c is acceptable.

7 i
18

respectively.

ANDERSON JUNIOR COLLEGE


2013 Preliminary Examinations
H2 MATHEMATICS (JC2)
PAPER 2

Duration : 3 hr

Answer ALL questions.


Section A: Pure Mathematics [40 marks]
1.

The function f is defined by

f: x

2 x 5

for x , x

5
.
2

(i)

Sketch the graph of y = f (x) and show that f 1 exists.

[3]

(ii)

Find f 1 ( x) and write down the domain of f 1 .

[3]

The solutions to the equation f (x) = f 1 (x) are x = a and x = 2 where a < 2.
(iii) Find, in terms of a, the area of the region bounded by the curves y = f (x) and
y = f 1 (x).

[3]

2.

B
C
A

The points A, B and C lie on a circle with center O and diameter AC. It is given that

OA a and OB b .
(i)

Find BC in terms of a and b. Hence show that AB is perpendicular to BC.

(ii)

Show that the area of triangle ABC can be written as k a b where k is a

[4]

constant to be found. Hence find, in terms of a , the maximum area of


triangle ABC.
(iii)

Given that

[4]

AOB 30o , find OF where F is the foot of perpendicular of B to

AC. Hence find OB ' where B ' is the reflection of B along the line AC.

Page 2 of 7

[3]

3.

The complex number z satisfies the following conditions:


z 3 4i 5 ,
(a)

z 3 4i z 4 3i .

(b)

On a single Argand diagram, sketch the locus of the points representing z.

[4]

Find

4.

(i)

the range of values for arg

(ii)

the least value of iz 7 i .

z 10 .

[4]
[2]

A curve C has parametric equations

x 1 tan 1 t
t

y 1 tan 1 t where t
t

, t 0.

The curve has an oblique asymptote y = ax.

dy
in terms of t and show that a = 1.
dx

(i)

Find

(ii)

Sketch C, showing clearly all the asymptote(s), axial intercepts and end points.

[3]
[3]

Hence find the range of values of k such that the line y kx does not intersect C. [1]
(iii) Find the equation of the normal to C at the point where t =1. Hence find the
value of the parameter t at the point where the normal intersects C again.

[3]

Section B: Statistics [60 marks]


5.

At the final football match of the World Cup 2014 in Brazil, the organisers wants to
sample 2% of the spectators in the stadium to find out their opinions of the catering
facilities.
(i)

Give a reason why it would be difficult to use a stratified sample.

[1]

(ii)

Explain how a systematic sample could be carried out.

[1]

Page 3 of 7

6.

The Winners Club consists of 2 married couples and 8 singles. The club is to select
a delegation of 4 members to participate in an overseas conference.
(i)

Find the probability that the delegation contains exactly one married couple.

[2]

The 4 selected delegates from the Winners Club are invited for a photo shoot with
8 delegates from other clubs after the conference. They are required to stand in two
rows of 6.
(ii)

Find the probability that at least 3 delegates from the Winners Club stand in the
same row.

7.

[3]

(a) In an experiment involving a large group of students, each student is asked to bend
a wire of length 15 cm into 2 straight segments to form an L-shaped figure with
lengths X and Y, such that Y is as close to thrice the length of X as possible. This is
to be done without the assistance of any measuring tools.

Y
X
It is found that X follows a normal distribution with mean cm and standard
deviation cm. It is known that P(X < 5) = 0.579 and P(Y < 10.5) = 0.788.
(i)

Find and .

[3]

(ii)

Find P(X < 5 and Y < 10.5).

[2]

(b) At a durian plantation, a large consignment of grade A durians was mixed with
an equal number of grade B durians. As the durians are indistinguishable, the
workers were unable to separate them.
It is known that the mass of a grade A durian is normally distributed with mean
800g and standard deviation 40g and the mass of a grade B durian is normally
distributed with mean 1000g and standard deviation 65g.
(i) Find the probability that two randomly chosen durians from the consignment
each has a mass of more than 880g.

Page 4 of 7

[3]

The workers decided to pack the durians into baskets of 50. It is known that the
mean and standard deviation for the mass of a durian from the mixed consignment
are 900 g and 78 g respectively. The mass of an empty basket is 500 g.
(ii) Find the probability that a randomly chosen basket filled with durians has a
mass that differ from 44.5 kg by more than 500 g.

8.

[4]

In an online statistics quiz, students are given two problems to solve, the second
of which is harder than the first. The probability of any student solving the first
problem correctly is
is

3
and the probability of solving the second problem correctly
4

7
. If a student solves the first problem correctly, the probability of solving the
12

second problem correctly is

3
.
5

A student is chosen at random.


(i)

Find the probability that the student solves both problems correctly.

[1]

(ii)

Find the probability that the student solves at least one problem correctly.

[2]

(iii) Given that the student solves the second problem wrongly, find the probability
that the first problem is solved correctly.

[3]

A student makes multiple attempts until he solves both problems correctly in the same
attempt. For each new attempt, the student has to solve a new pair of problems.
Find the least value of n if the probability that he needs to make at most n attempts
until he solves both problems correctly exceeds 0.999.

9.

[3]

Mr Gru claims that the average height of his minions is k cm. A random sample of
100 minions is chosen and the height, X cm of each minion is measured. It is found
that

x 50 96,

and

x 50

1879.

(i)

Find unbiased estimates of the mean and variance of X.

[2]

(ii)

What do you understand by the term unbiased estimate?

[1]

Page 5 of 7

Based on this sample, a hypothesis test is carried out and it was found that there is
significant evidence at 5% significance level that Mr Gru had underestimated the
average height of his minions.
[3]

(iii) Find an inequality satisfied by k.


(iv) Is there a need to make the assumption that the height of a randomly chosen
minion is normally distributed? Justify your answer.

[1]

Based on another sample of 10 minions, a t-test is conducted and it is found that there
is insufficient evidence at 5% significance level that the mean height of the minions
differs from 0 cm. State, with reasons, if the following statements are necessarily
true, necessarily false, or neither necessarily true nor necessarily false.
(I)

There is insufficient evidence at 2.5% significance level to conclude that the


minions are taller than expected.

(II)

There is insufficient evidence at 5% significance level to conclude that the mean


height differs from 0 cm if the researcher had carried out a z-test instead.

10.

[3]

A study comparing the number of hotspots detected in Indonesia each day and the
average 24-hour PSI reading in Singapore was conducted. The results over 7 days were
given below:

Number of hotspots (x)


Average 24-hour PSI reading (y)

50
45

97
98

128
160

150
205

170
263

189
341

215
400

(i) Draw a scatter diagram and find the value of the product moment correlation
coefficient between x and y. Hence comment on the suitability of the linear model.

[3]

(ii) Can we conclude that an increase in the number of hotspots will lead to an increase
in the PSI reading? Explain your answer.

[1]

(iii) A few alternative models, in the form of w ax b , were proposed to describe the
relation between x and y. State, with a reason, which model is the most appropriate.
Model A: w ln y
Model B: w y
Model C: w y 2

Page 6 of 7

[2]

It was decided that the data for an additional day should be included in the study. After
inserting the data for the additional day, it was found that the equation of the regression
line of w on x is w 0.082742 x 2.1735 and the equation of the regression line of x on w
is x 12.004w 25.106 based on the most appropriate model in part (iii).
(iv) Find the product moment correlation coefficient between w and x.

[1]

(v) Find the average 24-hour PSI reading on this additional day, correct to the nearest
integer.

[3]

In estimating the number of hotspots when the average 24-hour PSI reading was m,
where 100 < m < 200, Tarzan used the regression line of w on x for the estimation and
Jane used the regression line of x on w for the estimation. It was found that both of
them had approximately the same answer. Explain why this was observed.

[1]

11. Over a period of 120 days, the number of serious accidents in a day along a certain
expressway are recorded as follows:
No. of serious
accidents in a day

7 or more

No. of days

21

36

32

18

Find the mean and variance of the data and hence suggest why a Poisson model may be
valid.

[2]

The number of minor accidents in a day along this expressway is a random variable with
the distribution Po(3). The number of serious accidents in a day along the same expressway
is a random variable with the distribution Po(1.76). It can be assumed that the accidents
occur independently.
(i)

In a randomly chosen week, find the probability that there are 9 serious accidents
from Monday to Friday and 7 of them occur from Monday to Wednesday.

(ii)

[2]

In a given day, at least 2 accidents have occurred. Find the probability that all are
minor accidents.

[3]

(iii) Find, by using suitable approximations, the probability that the number of days with
exactly 2 serious accidents exceeds the number of days with exactly 2 minor
accidents in a randomly chosen 50-day period.

[3]

(iv) Explain why a Poisson distribution may not be a good model for the number of
serious accidents in a year.

[1]
END OF PAPER
Page 7 of 7

ANDERSON JUNIOR COLLEGE


2013 Preliminary Examinations
H2 MATHEMATICS (JC2)
PAPER 2 (Solutions)

Qn
1

(5/2, 4)

(i)Since any horizontal line y = k , where k


is any real value, cuts y = f (x) at most once,
f is one-one f 1 exists.
y = f (x) curve (must indicate end pt)

2 x 5
x=

,x

5
2

(ii)
2

2 x 5 1

4
y

5 1 4
5

1 since x .
2 2 y
2

5 1 4

1
2 2 x
(0, 4]

f 1 ( x)
Df 1

(iii) Required Area =2 area of region bounded by y = x and y = f (x)


2
2
4
2 [ x f ( x)] dx 2 x
dx
a
a
2 x 5 2 1

2 x
a

x 2.5

x2 1
= 2 1 tan 1
2 2

dx
2

x 2.5
1
2 a

a2
= 2 2 2 tan 1( 1) 2 tan 1 (2a 5)
2

= 4 4 tan 1(2 a 5) a 2

(i) OC a
BC a b
AB b a
AB BC

b a a b
b a b b a a+a b
2

=a b
Since |a|=|b| (radius of the circle),
AB BC 0
Therefore AB is perpendicular to BC.
Alternatively, to show that AB is perpendicular to BC.
2

AB BC

b a a b
2

b a a+b

b a (b a) a+b a+b
b b 2a b a a a a 2a b b b
2

2 b 2 a
4a

2a

AC

since b a (radius)
2

This satisfies Pythagoras theorem and therefore AB is perpendicular to BC,

1
AB AC
2
1
= (b a) 2a
2
= (b a) a | b a a a |

(ii) Area of triangle ABC =

Since a a = 0 ,
= ba
= ab
There are many alternative methods for this part:
Method 2:

1
2
1

2
1

2
a

Area of triangle ABC

AC b sin where is

AOB .

AC b sin
2a b sin
b sin

ab
Method 3:
Area of triangle ABC = Area of triangle OAB + Area of triangle OBC
1
1
a b a b
2
2
1
1
ab ab
2
2
ab
Method 4:
Area of triangle ABC = Area of triangle OAB + Area of triangle OBC
Since Area of triangle OAB = Area of triangle OBC as they shared the same height and
base i.e. OA OC .

Area of triangle ABC = 2 a b a b


2

Since a b a b sin and 0 sin 1 for 0 ,


Max area of triangle ABC = a b | a |2

(iii) OF b a a

3
b
2
3
3 a
3
OF
b a
b
a
2
2
2 a
OB OB '
OF
2
OB ' 2OF OB 3a b

Since b a b cos30o

since |a|=|b|

5
Locus
of z
10
(i)
Cartesian equation of relevant part of the circle: y 4 25 ( x 3)2
Cartesian equation of perpendicular bisector: y 7 x
From the GC, points of intersections are (0,0) and (1,7) .
Max argument of arg z 10 = 0.5 (max argument of arg ( z 10) )
=

radians.
2

Min argument of arg z 10 = 0.5 (min argument of arg ( z 10) )

7
1.2874 1.29 radians.
11

= 0.5 tan 1
Range 1.29 arg z 10

2
(ii) iz 7 i | i | z 7i 1 | z (1 7i) |
Least value of iz 7 i = Least value of | z (1 7i) | = 0

dy
dx 1 1
12 1 2
and
2
2
d
t
dt
t 1 t
t 1 t
dy dy dx

2t 2 1
dx dt dt
Since y = ax is an oblique asymptote, as x, the gradient of the curve approaches
a.
dy
But x t0
2t 2 1 1 , a = 1 (proved)
dx
(i)

(ii)

2 2

y=x

1.72

1.72

2 , 2

As t , x 0 and y 0 .
2
2
As t , x 0 and y 0 .
2
2

2 2

1.72

y = kx

From the diagram,


For y = kx not to intersect the curve,
k 1 or k 1

y=x

1.72

2 , 2
(iii)

Eqn of normal at t = 1,

y 1
4 1
3
x 1
4
3 y 1 x 1
4
4
To find the points of intersection,

3 1 tan 1 t 1 1 tan 1 t 1
t
4
t
4
4 2 tan 1 t 4 0

t
2
5

Using GC, t = - 8.41


(i) It is difficult to obtain the profile of all the spectators in the stadium to form the
sampling frame for a stratified sampling method to be carried out.
(ii) Sample every 1 out 50 spectators that exit the stadium after the match to take the
survey.

(i) P( exactly one married couple)

C1 ( 2C1 8C1 8C2 )


12
C4

88
8

0.178
495 45

(i) Number of ways with no restriction = 12!


At least 3 in the same row :
Case 1: one delegates in one row and 3 in the other row
4C3 8C3 6! 6! 2 232243200
Case 2: all 4 delegates in one row
4C4 8C2 6! 6! 2 29030400
Required probability

232243200 290030400
12!
6
0.54545... 0.545
11

7
(a)

(i)

P(X < 5) = 0.57925 = 0.579


5
0.19934 ---- (1)

P(Y 10.5) P(15 X 10.5) P( X 4.5) 0.788


4.5
0.79950 ------(2)

Using (1) and (2), 4.9002 4.90

0.50058 0.501

(iii) P(X < 5 and Y < 10.5) = P(X < 5 and X > 4.5)
= P(4.5 < X < 5)
= P( X 5) P( X 4.5) 0.579 (1 0.788)
= 0.367
(b)

Let the random variables A and B denote the mass of a grade A durian and grade B durian
respectively.
A ~ N(800, 402)
B ~ N(1000, 552)
(i) Prob
= 2(0.5)2 P(A > 880) P(B > 880) + [(0.5)P(A > 880)]2 + [(0.5)P(B > 880)]2 =0.254
(i) By CLT, since the number of durians in each basket is large (=50),
Let D be the random variable denoting the mass of a durian randomly chosen from
a population with mean 900g and standard deviation 78g. Since n is large, by CLT

D1 + D2 + . . . + D50 ~ N(50 900, 50 782) approximately.

Let T = 500 + D1 + D2 + . . . + D50


P(| T 44500 | 500)

P(| D1 D2 . . . D50 44000 | 500)


1 P(| D1 D2 . . . D50 44000 | 500)
1 P(43500 D1 D2 . . . D50 44500)
0.821
8

Let A be the event that student get first problem correct.


Let B be the event the student gets second problem correct.
3 3 9
(i)
P(both problems correct) = P A B =
4 5 20
(ii)
P(at least one problem correct) = P A B

3 7 9 53

0.883
4 12 20 60

Required probability = P A | B '

(iii)

Last part:
P(at most n attempts) >0.999
2

P A B '
P B '

P A B P B
1 P B

53 7
3

60 12 10 0.72
7
5
1
12
12

9 11 9 11 9
11

.......
20 20 20 20 20
20
n
9 11
1
20 20

0.999
11
1
20
11
0.001
20
n

n1

9
0.999
20

ln 0.001
11.555
11
ln
20
Least n = 12
n

(i)

Unbiased estimate of the population mean,

x 50 50 96 50 49.04
100

100

Unbiased estimate of the population variance,


s2

(ii)
(iii)

1
(96)2
1879

18.048 18.0 (3 s.f.)


99
100

The expected value of the unbiased estimate is the population parameter.

H0 : k
H1 : k

Since is unknown, but sample size 100 is large,


under H 0 , test statistic Z

X
~ N (0,1)
s/ n

From GC, zcritical 1.6449


Since H 0 was rejected, zcalc zcritical 1.6449
49.04 k
1.6449
18.048 /10
k 48.3
(t- test can also be used)
(iv) No assumptions are necessary since sample size 100 is large, by CLT, mean height
of minions in a randomly chosen sample is normally distributed.
(v)
Statement I
Insufficient evidence at 5% significance level for a two-tail test implies H 0 was not
rejected.
p-value of the two tail test > 0.05
For a right-tail test,

[A1]

p-value of the right tail test = (p-value of the two tail test) > 0.025
Since p-value of the right tail test > significance level,

H 0 will not be rejected.


Statement is necessarily true.
Statement II
Under t-test, since H 0 was not rejected, we know that
2.2621 tcalc 2.2621

Under z-test, for H 0 not to be rejected,


1.9599 zcalc 1.9599

which is not necessarily true since zcalc tcalc


necessarily true nor false.
10

(i)

x o
s/ n

. Therefore, statement is neither

y
400

45

x
50

215

As the points lie close to a straight line and r = 0.97940 is close to 1, a linear model is
valid.
(ii) No. The model may suggest that there is a correlation between the number of
hotspots and the average 24-hour PSI reading, but does not imply causation. The
increase in the average 24-hour PSI reading could have been due to other factors such
as wind direction.
(iii)Model B. The product moment correlation coefficient for Model A, B and C are
0.9923, 0.9972 and 0.9139 respectively, thus model B has a r which is closest to 1
implying a strong positive linear correlation.
(iv) r (0.082742)(12.004) 0.99661
(v) w 0.082742 x 2.1735 --- (1)
x 12.004w 25.106 --- (2)
Solving equations (1) and (2) simultaneously using GC,

We get w 14.217 and x 145.556


45 98 160 205 263 341 400 a
w 14.217
8
a 15.478
a 240 (to the nearest integer)

11

(vi) As the r value is very close to 1, both regression lines are almost identical to each
other.
Using GC
:
mean of data 1.75833 1.76 ( 3 s.f.)
variance of data 1.322852 1.75 ( 3 s.f. )
The data support the expectation that the distribution can be approximated by a Poisson
distribution as the mean of the data is approximately equal to the variance of the data.
Let X n be the random variable denoting the number of serious accidents in a
n-day period .
X n ~ Po(1.76n)
Then

X 2 ~ Po(3.52) & X 3 ~ Po(5.28)

(i)

Required probability
P( X 3 7) P( X 2 2)
(0.115593)(0.183374)
0.021197
0.0212 ( 3 s.f. )

(ii) Let Y be the random variable denoting the number of minor accidents in a 1 day
period.

X1 ~ Po(1.76) , Y ~ Po(3) , X1 Y ~ Po(4.76)


P( X 1 0 | X 1 Y 2)

P( X 1 0) P(Y 2)
P( X 1 Y 2)

P( X 1 0) 1 P(Y 1)
1 P( X 1 Y 1)

0.145
(iii) Let V denote the r.v No. of days, out of 50 days, with 2 serious accidents

[M1]
[M1]
[A1]

Let W denote the r.v No. of days, out of 50 days, with 2 minor accidents
Then V ~ B(50, P( X1 2)) i.e. B(50,0.26646)
with np 13.323 , nq 36.677, npq 9.7730
W ~ B(50, P(Y 2)) i.e. B(50,0.22404)
with np 11.202 , nq 38.798 and npq 8.6924

Since in both cases np 5 and nq 5


V ~ N 13.323,9.7730 and W ~ N (11.202,8.6924) approximately
V W ~ N (2.121,18.465)

Required probability P(V W 0)


continuity correction

P(V W 0.5)

0.647

(3 s.f)

(iv) As the traffic conditions may differ over a year, more accidents may occur during the
rainy season of the year.

ANGLO-CHINESE JUNIOR COLLEGE


MATHEMATICS DEPARTMENT
MATHEMATICS
Higher 2

9740 / 01

Paper 1

28 August 2013

JC 2 PRELIMINARY EXAMINATION
Time allowed: 3 hours
Additional Materials: List of Formulae (MF15)
READ THESE INSTRUCTIONS FIRST
Write your Index number, Form Class, graphic and/or scientific calculator model/s on the cover page.
Write your Index number and full name on all the work you hand in.
Write in dark blue or black pen on your answer scripts.
You may use a soft pencil for any diagrams or graphs.
Do not use paper clips, highlighters, glue or correction fluid.
Answer all the questions.
Give non-exact numerical answers correct to 3 significant figures, or 1 decimal place in the case of
angles in degrees, unless a different level of accuracy is specified in the question.
You are expected to use a graphic calculator.
Unsupported answers from a graphic calculator are allowed unless a question specifically states
otherwise.
Where unsupported answers from a graphic calculator are not allowed in the question, you are
required to present the mathematical steps using mathematical notations and not calculator commands.
You are reminded of the need for clear presentation in your answers.
The number of marks is given in brackets [ ] at the end of each question or part question.
At the end of the examination, fasten all your work securely together.

This document consists of 6 printed pages.

[Turn Over

ANGLO-CHINESE JUNIOR COLLEGE


MATHEMATICS DEPARTMENT
JC2 Preliminary Examination 2013

/ 100

MATHEMATICS 9740
Higher 2
Paper 1

Index No:
Form Class: ___________
Name: _________________________
Calculator model: _____________________

Arrange your answers in the same numerical order.


Place this cover sheet on top of them and tie them together with the string provided.

Question No.

Marks

/5

/6

/5

/5

/5

/8

/10

/9

/6

10

/8

11

/9

12

/10

13

/14

Anglo-Chinese Junior College


H2 Mathematics 9740: 2013 JC 2 Preliminary Examination Paper 1
Page 2 of 6

Do not use a graphic calculator in answering this question.


By considering the cases x < 0 and x 0, find the range of values of x that satisfy the
inequality
(10 x)(10 |x|) > 11,
giving your answers in exact form.
[5]

The diagram shows the graph of y = f(x). The curve crosses the axes at the points
1
3, 0 , 1, 0 and 0, . The minimum point on the curve has coordinates
2
1
1

2, and the asymptotes are x = 2 and y .


2
2

y=

1
0,
2

1
2

(3, 0)

(1, 0) O

x=2

Sketch, on separate diagrams, the curves with equation


[3]
(a) y2 = f(x),
1
(b) y
,
[3]
f x
stating clearly the exact coordinates of any points of intersection with the axes and
turning points and the equations of any asymptotes.

The complex number z satisfies the relations z 25 15 and z 25 z 35 20 i .


(i) Illustrate both of these relations on a single Argand diagram, indicating clearly the
intersection of the two loci.
[3]
(ii) Find the greatest value of arg z 25 .
[2]

In the triangle PQR, PQ = 3, QR=

2 and angle PQR=

a sufficiently small angle, show that

radians. Given that is

PR 5 6 3 2 2 a b c 2 ,
for constants a, b and c to be determined.

[5]
[Turn Over

Anglo-Chinese Junior College


H2 Mathematics 9740: 2013 JC 2 Preliminary Examination Paper 1
Page 3 of 6

A curve C is defined by the parametric equations


x 4cos 2, y sin 1 , where 0 .
The region S is bounded by the curve C, the line x = 2, the x-axis and the y-axis.
Find the volume generated when S is rotated completely about the y-axis.
[5]

By expressing

A
B
1
in the form
, where A and B are real constants, find,
r r 2
r r 2

in terms of n,
n

r r 2 .

[4]

r 3

Give a reason why the series is convergent and state the sum to infinity.
Hence deduce the value of
1
1
1
.

3 5 4 6 5 7
7

[2]
[2]

The function f is defined as follows.


f :x

(i)
(ii)

1
(2 x 1)(2 x 3), x , x .
2

Find the range of f and define f 1 in a similar form.

[4]

Given that f ( x) f( x) at x = a, find the exact value of a.

[3]

e x b, x 0 , where b is an integer. Determine


The function g is defined by g : x
the largest value of b such that fg exists. With this value of b, find the range of fg. [3]

x2 2 x
.
ex
Find the range of values of x for which the curve y = f(x) is concave upwards. [3]
Hence sketch the graph of y = f(x), indicating clearly the equations of any
asymptotes and the coordinates of any stationary points and any intersections with
the axes.
[3]

(a)

It is given that f(x) =

(b)

The diagram shows a circular cylinder with a ten-centimetre long rod passing
through the centre C of the cylinder and joining one rim to the other rim. The
curved side of the cylinder is perpendicular to the horizontal and the radius of
each horizontal circular face is r cm. The cylinder is made of thin cardboard and is
closed at both ends.

(i) Show that the area, A square centimetres, of the cardboard used is given by
A = 2r2 + 4r (25 r2).
[1]
(ii) By sketching an appropriate graph or otherwise, find, as r varies, the value of
r that gives the greatest possible value of A.
[2]
Anglo-Chinese Junior College
H2 Mathematics 9740: 2013 JC 2 Preliminary Examination Paper 1
Page 4 of 6

10

Keith makes a deposit of $10 000 into his bank account at the beginning of every year.
The account earns him an interest of k% per annum, paid out at the end of every year.
Every year, after the interest is paid out, Keith withdraws 25% of what is remaining in
the account, and continues to deposit another $10 000 at the beginning of the following
year.
(i) Write down the amount of money remaining in his account after his withdrawal at
the end of the first year, leaving your answer in terms of k.
[1]
(ii) Show that the amount of money remaining in his account at the end of n years
after withdrawal is
n
k 100 3k 300
[3]
30000
1
.
100 3k 400
(iii) If Keith decides that he wants to maintain the account for 20 years, find the
minimum value of k so that he has $32 000 in the account by the end of that
period, after withdrawal.
[2]

(i)

By using the substitution u

x , find

Hence, using integration by parts, show that

(ii)

2 x

dx 2e

2 x

e 2 x
dx.
2 x

[2]

2 x 1 C .
2

[3]

1
. The region R is
4
2 . Using the result in
[3]

The diagram below shows the graph of y ln 4 x 2, x


bounded by the curve, the y-axis and the lines y
(i), find the area of R in exact form.

2, y

, 2
4

11

In a yogurt-making process, Jane adds 10 g of yogurt at 12 00 hrs into 500 g of warm


milk to turn the milk into yogurt. After t hours, the amount of yogurt in the mixture is
x g. In a simple model, it is assumed that the rate of growth of yogurt is proportional to
500
the product of x and 500 x . Show that x
, where k is a constant.
[6]
1 49e500 kt
98
It is given that initially, the rate of growth of yogurt is
ln 2 g per hr. Jane wishes to
5
place the yogurt formed into the refrigerator once there is 450 g of yogurt in the
mixture. At what time should she place the yogurt in the refrigerator?
[3]
[Turn Over
Anglo-Chinese Junior College
H2 Mathematics 9740: 2013 JC 2 Preliminary Examination Paper 1
Page 5 of 6

12

Given that y = (sin1x)2, show that

dy
1 x 4 y
dx
2
dy
2 d y
and 1 x
[3]
x 2.
2
dx
dx
By further differentiation of these results, find the Maclaurin series of y up to including
the term in x4.
[3]

The series found above is to be used to estimate a value for 2. Explain with the aid of a
1
sketch why using x in the series gives a better approximation than x 1 , and hence
2
find an approximation to 2, giving your answer in fraction form.
[4]

13

The equations of the plane , and the lines l1 and l2 are given by
: tx 2 y z 13 ,

l1 : r 2i j 3k 2t i t 2 1 j 2k ,

l2 : x 4 y, z 5 ,
where t and are real constants.
(a) Given that the shortest distance from the point P with coordinates 3,3, 5 to
is 6, find the possible values of t.
[4]
(b) Show that l1 is parallel to , and find a condition on t such that l1 is not on . [3]
For the case where t 2 ,
(i) show that l2 lies on ,
[1]
(ii) given that A is a point on l1 and B is a point on l2 , find the position vectors
of A and B such that AB is perpendicular to both l1 and l2 ,
[4]
(iii) find the vector equation of the line of reflection of l1 in .
[2]

Anglo-Chinese Junior College


H2 Mathematics 9740: 2013 JC 2 Preliminary Examination Paper 1
Page 6 of 6

Qn
1

Anglo-Chinese Junior College


H2 Mathematics 9740
2013 JC 2 Preliminary Exam P1 Solution & Marking Scheme
Solution
Given (10 x)(10 |x|) > 11
When x < 0, |x| = x
(10 x)(10 + x) > 11
100 x2 > 11
89 x2 > 0
(89 x)(89 + x) > 0

89 < x < 89
89 < x < 0

When x 0, |x| = x
(10 x)(10 x) > 11
100 20x + x2 > 11
89 20x + x2 > 0
20 400 356
x=
2
10 11
x < 10 11
or x > 10 + 11

Then 0 x < 10 11
or x > 10 + 11

Combining answers,
89 < x < 10 11 or x > 10 + 11
Alternatively,
(10 x)(10 |x|) > 11
When x < 0, |x| = x (10 x)(10 + x) > 11

100 x2 > 11
Sketch y = 100 x2 for x < 0
When x 0, |x| = x (10 x)(10 x) > 11

(10 x)2 > 11


Sketch y = (10 x)2 for x 0
y
(0, 100)

y = 11
(10, 0)

(10, 0)

At intersections,
100 x2 = 11 for x < 0 and (10 x)2 > 11 for x 0
20 400 356
10 11
x = 89 and x =
2
x = 89 (as x < 0) and x = 10 11
From graph, 89 < x < 10 11 or x > 10 + 11
1

2(a)

y=

(3, 0)
y=

1
2

(0,

1
)
2

(1, 0)

1
2

(0,

x=2

1
)
2

2(b)

maximum value of arg(z-25) = +

AC 102 52 125
10
tan
5
125
cos
15
1.84 (3 sig fig)
4

PR 2 32

2 3

2 cos 4

11 6 2 cos cos sin sin


4
4

1
1

11 6 2
cos
sin
2
2

11 6cos 6sin
2
11 6 1 6
2

5 6 3 2
1

PR 5 6 3 2 2 (shown)

PR 5 6 3

1
2 2

6
3 2
5 1 2
5
5
2
1

16
3 2 16
3 2
2
5 1 ...
5 8 5
5
2 5

3 5
3 5 2

5
25

3 5
3 5
, c
5
25
x 4cos 2, y sin 1

a 5, b

y
1
1

3
2

S
2

3
, y 1
2
3
2
x

2,cos

0,
y

1
When

When x 0, cos

y sin 1

dy
cos
d
2

Volume of revolution 2 1

3
2

x 2 dy

4cos 2 2 cos d

6.24 units (3 s.f)


6

1
1 1
1

r r 2 2 r r 2
1 1
1

.
2r 2 r
1
1
A , B .
2
2
n
1
1 n 1
1

2 r 3 r 2 r
r 3 r r 2

1 1 1 1 1 1
1
2 3 2 4 3 5

1 1
1 1
1
1

n 4 n 2 n 3 n 1 n 2 n

13
1
1

.
2 2 n 1 n

as n ,

Therefore

13
1
1
3
1
1
0 . Hence
.
n 1 n
2 2 n 1 n
4

r r 2 4 .
r 3

1
1
1

3 5 4 6 5 7

1
r 5 r r 2

1
1
1

3 1 4 2
r 3 r r 2

7(i)

R f 4,

3
1
1

4 3 1 4 2

7
.
24

Let y (2 x 1)(2 x 3) 2 x 1 4
2

2 x 1

y f ( x)

y4

2x 1 y 4
1
Since x , 2 x 1 y 4
2
1 1
x
y4
2 2

x
O
1

, 4
2

1 1

x 4, x , x 4
2 2

f 1 : x

7(ii)

Since y f (x) and y f 1 (x) meet at y x ,


consider f (a) = a
(2a 1)(2a 3) a
4a2 3a 3 0
3 9 4(4)(3)
a
2(4)
3 57
8
1
3 57
Since a , a
2
8

y
b+1

y = g(x)

y=b
x

R g (b, b 1], Df ,
2

For fg to exist, R g Df b 1
Since b is integer, largest b 2

1
3
i.e. b
2
2

0, 2, 1 3,5
R fg 3,5
8(a)

x2 2 x
ex

dy
2 x 2 e x x 2 2 x e x = (x2 + 4x 2)ex
dx
d2 y
2 x 4 e x x 2 4 x 2 e x
2
dx
x 2 6 x 6 e x > 0

x < 1.27 or x > 4.73


OR x < 3 3 or x > 3 + 3
dy
Put
0, (x2 + 4x 2)ex= 0
dx
x 2 2 = 0.586, 3.41
22 2
When x 2 2, y
0.461 .
e 2 2
22 2
0.159
When x 2 2, y
e 2 2

x2 2 x
y
ex
(3.41, 0.159)

(0.586,-0.461)
asymptote: y = 0

Let height be h.
Then 4r2 + h2 = 102 h = (100 4r2)
Area = 2r2 + 2rh = 2r2 + 2r (100 4r2)
A = 2r2 + 2r 4 (25 r2)
A = 2r2 + 4r (25 r2)

8(b)

(i)

8(b)

(ii)

Graph of A = 2r2 + 4r (25 r2)

Maximum point at r = 4.25 cm (3 sf)


dA
Alternatively, graph of y =
is
dr

dA
= 0 when r = 4.25 (3 sf)
dr
dA
Gradient of y =
is negative at r = 4.25 so value of A at r = 4.25 is maximum or
dr
greatest possible.

Using GC, y =

Alternatively, use first derivative test:


r
4.25 4.25 4.25+
dA/dr
+ve
0
ve

Alternatively, using differentiation,


2r
dA
= 4 r 4 25 r 2 4 r
=0
2
dr
2 25 r
Using GC, r = 4.25 cm (3 sf)
First derivative test:
r
dA/dr

9(i)
9(ii)

4.25+
ve

4.25 4.25
+ve
0

Note that those who solve the equation algebraically by squaring will have two positive
answers for r. Students should explain/justify why they choose r = 4.25 cm as the final
answer.
k
3
10000 1

100 4

k 3
k 3

end of 2nd year: 10000 1


10000 1

100 4

100 4
2

k 3
k 3

10000 1
10000 1

100 4
100 4

end of nth year:

10000 1

k 3
k

10000 1

100 4
100

n 1

3

4

n 1

k 3

10000 1

100 4

n
n
k 3
k 3

1
1
1



100 4 100 4
10000
k 3

1 1

100 4
n
n
k
k 3

3 1
1


100 100 4
10000
k

4 3 1

100

n
100 k 3k 300
1



100 100
30000
100 3k
100
n

k 100
3k 300
30000
1


. (shown)
100 3k 400

9(ii)

10(i)

When n 20 , find minimum value of k such that


n
k 100 3k 300
30000
1
32000
100 3k 400
Using the G.C., k 1.6948
Hence, least value of k is 1.70.
eu
e 2 x
u 2 x
2 x d x = u 2u du
1
du 1

= 2eu c
dx 2
1
2e 2x c
=
2u

x2

dx

(where u 2 x ,

e x 2
x2
dx
x2

x 2 2e
2 x 2 2e

2e
10(ii)

dv
e 2 x

)
dx
2 x

x 2

x 2

x 2

2e 2
2e c
x2

1
dx
2 x

x 2

2 x 1 C (shown)
2

y ln 4 x 2, x

1
4

ln 4 x y 2
1
Since x , 4 x 1, ln 4 x 0
4
Thus ln 4 x y 2
4x e
1
x e
4

y 2

y2

Area of R =

x dy

1 2 y2
e
dy
4 2
1
2 e2 (2 1) e0 (0 1)
4
1
e2 1
2

11

dx
kx(500 x)
dt
1

dx k dt
where k is a constant
x(500 x)
1 1
1

dx k dt

500 x (500 x)
1

ln x ln 500 x kt C
500
x
ln
500kt B
500 x
x

e500 kt B
500 x
x

e500 kt B
500 x
x

Ae500 kt , where A ec
500 x
x Ae 500kt 500 x
1 Ae500kt x 500 Aekt

500 Ae kt
1 Ae 500 kt

10

When t = 0, x 10 ,

500 A
1
A
49
1 A

500 500 kt
e
Thus x 49
1
1 e 500 kt
49
500e 500 kt

49 e 500 kt
500
(shown) , where k is a constant

1 49e 500 kt

When x 10,

dx 98
ln 2 ,
dt
5

98
ln 2 k (10)(500 10)
5
ln 2
k
250

When x 450
500
450

1 49e
1 49e
e

ln 4 t

ln 2
500 250 t

ln 4 t

10
9

1
9 49

1
t 4.39
9 49
Jane should place the yogurt in the refrigerator at 16 24 hr.
dy 2sin 1 x
1 2

y = (sin x)
dx
1 x2
ln 4 t ln

12

dy

dx

4 sin 1 x
1 x

4y
1 x2

dy
1 x 4 y (shown)
dx

Differentiating again,

2
dy
dy d y
dy
1 x 2 2 2x 4
dx
dx dx
dx
dy
Divide both sides by 2
to arrive at
dx
2
dy
2 d y
1 x
x 2
2
dx
dx

Differentiating again, 1 x 2

ddxy 3x ddxy ddyx 0


3

and 1 x 2
Sub x = 0:

d4 y
d3 y
d2 y

5
x

4
0
dx 4
dx3
dx 2

d3 y
d2 y
d4 y
dy
= 0, 2 = 2, 3 = 0, 4 = 8
dx
dx
dx
dx
Maclaurin Series is
2
8
1
y = x 2 x 4 ... = x 2 x 4 ...
2!
4!
3
y = 0,

y = (sin-1x)2

y = x2 + x4/3
1

x
1

0.5

0.5

1/2

1.5

1
1
Any approximation using y x 2 x 4 is better for x than for x = 1 as the graphs of
3
2
1
y x 2 x 4 and
3
1
1
y = (sin1x)2 are closer together at x than at x = 1. Hence x gives a better
2
2
approximation to 2 than x 1 .
4

1 1 1
1 1
1
1
Using x , (sin1 )2
4 3 2
4 48
2
2

13(a)

13


48
6
13
39
3
or 9
2 36
48
4
4

Let M be a point on plane .


3
0
PM 3
OM 0 ,

18
13

10

3 t

3 . 2
18 1

t2 5

6 t 2 5 3t 24
36(t 2 5) 9t 2 144t 576

13(b)

27t 2 144t 396 0


22
t
or t 2
3
2t t
2
2
2
t 1 . 2 2t 2t 2 2 0
2 1


Hence l1 is parallel to .

Since l1 is not on ,
2 t

1 . 2 13
3 1

2t 2 3 13
t 6, t

when t 2 ,
4
1


l2 : r 0 1
5
0

1 2

1 . 2 2 2 0 l2 is parallel to .
0 1

4 2

0 . 2 8 5 13 l2 is on .
5 1

11

13(b)(i)

2 4
4

OA 1 3 , OB

3 2
5

2 4

AB 3 1
2 2

1
4


AB. 1 0,
AB. 3 0
0
2


1 2 7 ,
1=-7 +29
22
5
, =9
9
2
14
9
9

2
22

OA , OB
3
9

37
5
9

13(b)(ii)

AA ' 2 AB
16 2 10
9 9 3

16 2 38

OA ' 2 AB OA 2


9 3 9

8 37 53
9 9 9
10
3
4

38
Line of reflection of l1 in : r 3
9
2


53

12

ANGLO-CHINESE JUNIOR COLLEGE


MATHEMATICS DEPARTMENT
MATHEMATICS
Higher 2

9740 / 02
17 September 2013

Paper 2
JC 2 PRELIMINARY EXAMINATION

Time allowed: 3 hours


Additional Materials: List of Formulae (MF15)
READ THESE INSTRUCTIONS FIRST
Write your Index number, Form Class, graphic and/or scientific calculator model/s on the cover page.
Write your Index number and full name on all the work you hand in.
Write in dark blue or black pen on your answer scripts.
You may use a soft pencil for any diagrams or graphs.
Do not use paper clips, highlighters, glue or correction fluid.
Answer all the questions.
Give non-exact numerical answers correct to 3 significant figures, or 1 decimal place in the case of
angles in degrees, unless a different level of accuracy is specified in the question.
You are expected to use a graphic calculator.
Unsupported answers from a graphic calculator are allowed unless a question specifically states
otherwise.
Where unsupported answers from a graphic calculator are not allowed in the question, you are
required to present the mathematical steps using mathematical notations and not calculator commands.
You are reminded of the need for clear presentation in your answers.
The number of marks is given in brackets [ ] at the end of each question or part question.
At the end of the examination, fasten all your work securely together.

This document consists of 6 printed pages.

[Turn Over

ANGLO-CHINESE JUNIOR COLLEGE


MATHEMATICS DEPARTMENT
JC2 Preliminary Examination 2013

/ 100

MATHEMATICS 9740
Higher 2
Paper 2

Index No:
Form Class: ___________
Name: _________________________
Calculator model: _____________________

Arrange your answers in the same numerical order.


Place this cover sheet on top of them and tie them together with the string provided.

Question No.

Marks

/8

/9

/10

/13

/3

/4

/6

/5

/9

10

/11

11

/11

12

/11

Anglo-Chinese Junior College


H2 Mathematics 9740: 2013 JC 2 Preliminary Examination Paper 2
Page 2 of 6

Section A: Pure Mathematics [40 marks]


1

Referred to the origin O, the points A and B are such that OA a and OB b , where
a 2 and b is a unit vector. The midpoint of OA is M, and the point L on AB is such
that AL:LB=1:2.
(i) Give the geometrical meanings of
(a) |b.a|,
[1]
(b) |b a|.
[1]
(ii) Find OL in terms of a and b. Hence find the area of triangle OAL in terms of a and
b.
[3]
(iii) Given that LM is perpendicular to AB, show that
a.b = k
where k is a constant to be determined.
[3]
The polynomial P z 2 z 4 aiz 3 2 z ai , a
(i)
(ii)

, has factor 2 z i .

Find the exact value of a.


[2]
i
Solve the equation P z 0 , leaving your answers in the form re , where

r 0 and .
(iii) One of the roots, z1 , is such that 0 arg z1

[4]

. The locus of points


2
representing z, where arg(z z1 ) k , passes through the origin. Find the exact
value of k, and the cartesian equation of this locus.
[3]

A curve C has parametric equations


2
2
x 2t ,
y 2t ,
t
t
where t < 0.
dy
(a) Find
in terms of t and hence find the exact value of t for which the tangent to
dx
the curve at t is parallel to the y-axis.
[4]
(b) Find the value of t for which the distance from the point (1, 0) to the curve is the
shortest possible.
[2]
Find a Cartesian equation of C.
[2]
The curve C is transformed by a translation of 2 units in the negative x-direction,
1
followed by a stretch with scale factor
parallel to the y-axis. Write down a Cartesian
2
equation of C after the transformations.
[2]

[Turn Over

Anglo-Chinese Junior College


H2 Mathematics 9740: 2013 JC 2 Preliminary Examination Paper 2
Page 3 of 6

(a)

Prove by mathematical induction that, for every positive integer n,


n
4n1 n 1 2
4r r 2

3 n 2 3
r 1 r 1 r 2

4r r 1
.
Deduce
r 0 r 2 r 3
1
It is given that f ( x)
.
x 1
(i) On separate diagrams, sketch the graphs of y f x and y f x .
n

(b)

[4]

[3]

[2]

(ii) Without the use of graphing calculator, find the exact value of the constant a,
where 0 a 1, for which

1
2

f ( x ) dx

a
0

f (x) dx .

[4]

Section B: Statistics [60 marks]


5

The mean number of rotten eggs per tray is found to be 0.25 and the variance 0.2475.
Estimate the probability that, in 100 trays of eggs, an average of at most 0.15 is rotten.
[3]

For a school learning journey, one thousand students were given three venues to choose
to visit. The number of students in each of the two year-groups and their choices are
summarised in the table below.
Marina Barrage
Year 1
Year 2

240
160

Gardens by the
Bay
200
180

National
library
100
120

The National Education committee wishes to investigate if the learning outcomes were
achieved by interviewing a random sample of 50 students.
(i) Explain what is meant in this context by the term a random sample.
[2]
[2]
(ii) Describe how a stratified sample can be obtained.
7

Ten chefs, six males and four females, qualify for the final phase of a Top Chef
competition consisting of nine Elimination Challenges. One chef is eliminated after
every challenge and the last chef remaining is the winner of the competition.
(i) In how many ways can the eliminations be done?
[1]
(ii) For the first challenge, the ten chefs are divided into two groups of five to work in
two different restaurants. In how many ways can the two groups be formed such
that each group has at least one female chef?
[2]
(iii) After the first challenge, a particular male chef was eliminated. At a photography
session before the next challenge, nine chairs are arranged in two rows: four in
front and five at the back. In how many ways can the remaining chefs be arranged
so that the male chefs and the female chefs must alternate?
[3]

Anglo-Chinese Junior College


H2 Mathematics 9740: 2013 JC 2 Preliminary Examination Paper 2
Page 4 of 6

2n marbles, identical except for their colour, are such that n are red and the rest are
white. Box A contains 3 of the red marbles and n 3 of the white marbles. The
remaining marbles are placed in box B. A marble is taken at random from box A and put
into box B. A marble is then taken at random from box B.
Find, in terms of n, the probability that
(i) the marble taken from box B is white,
[2]
(ii) the marble taken from box A is red given that the marble taken from box B is
white.
[3]

A pastry chef bakes two types of cakes, the falafel cake and the polenta cake. The
weights of the falafel cake and the polenta cake are modelled by independent normal
distributions with means 2.5 kg and 3.6 kg, and standard deviations 0.1 kg and 0.2 kg
respectively.
(i)
(ii)

Find the probability that the mean weight of three randomly chosen falafel cakes
is more than 2.65 kg.
[2]
The probability that the total weight of n polenta cakes is less than 71 kg is 0.132.
Find n.
[4]

A party organizer plans to serve the cakes in sets as described below.


Regular Set: one falafel cake and one polenta cake.
Special Set: one falafel cake and two polenta cakes.
(iii) Find the probability that the total weight of two randomly chosen Special Sets is
more than thrice the weight of a Regular Set.
[3]

10

The number of apple strudels sold by a bakery in a day has the following probability
distribution.
No. of apple strudel
<10
10 15
16 20
>20
sold in a day
Probability
0.05
0.45
0.40
0.10
(i)

Use a suitable approximation to find the probability that, in 60 days, there are at
most 5 days with fewer than 10 apple strudels sold.
[2]

A week of 7 days is called strudel week if there are at least 5 days with more than 15
apple strudels sold. If the probability that a particular week is a strudel week is p,
29
show that p
.
[2]
128
Find the probability that, in 4 consecutive weeks, the second strudel week occurs
on the fourth week.
[2]
(iii) The probability that there are at least k strudel weeks in a period of 256 weeks is
at least 0.9. Using a suitable approximation, find the maximum value of k.
[5]
(ii)

[Turn Over
Anglo-Chinese Junior College
H2 Mathematics 9740: 2013 JC 2 Preliminary Examination Paper 2
Page 5 of 6

11

A company supplies sugar in small packets. The mass of sugar in 1 packet is denoted by
X grams. During a routine check, the companys data analyst takes a random sample of
12 packets. The masses of the 12 packets are summarised by
2
x x 8.91.

x 115.2 ,

Calculate unbiased estimates of the mean and variance of X.

[2]

The mean mass in a packet is stated to be 9 grams.


(a) On the basis of the above sample, the data analyst wishes to determine whether
the mean mass in a packet has been understated.
(i) Stating a necessary assumption, carry out a test at the 5% level of
significance.
[5]
(ii) Explain what is meant by 5% level of significance in the context of the
question.
[1]
(b)

12

Given that the population variance of X is 0.72, and that the assumption made in
(a)(i) is still valid, a second test is carried out. For the alternative hypothesis 9
, the null hypothesis is not rejected if 8.6 x k .
Find, in either order,
(i) the value of k,
[1]
[2]
(ii) the significance level of the test.

The human resource department of a company is tasked to determine if there is a


relationship between the age and cholesterol level of their staff. The following table
shows the information on age, x years, and cholesterol level, y milligrams per decilitre
of blood, mg/dL, for a random sample of 10 members of staff.
Age, x
(years)
Cholesterol
level, y
(mg/dL)
(i)

38

64

39

34

25

46

37

26

55

31

205

225

206

192

150

213

190

165

220

181

Draw a scatter diagram for the data.


[1]
(a) Explain why, in this context, a quadratic model would probably not be
appropriate for long term predictions.
[1]
(b) Explain why the diagram shows that the data may be consistent with the
model of the form y = a + b lnx.
[1]

One of the values of y is recorded wrongly. Indicate the corresponding point on your
diagram by labelling it P.
[1]
Omitting P, use the data for the remaining workers to answer the following questions.
(ii) Calculate the product moment correlation coefficient between ln x and y.
[1]
[2]
(iii) Calculate the least squares estimates of a and b, for the model y = a + b lnx.
(iv) Assume that the value of x at P is correct. Estimate the value of y for this value of
x.
[2]
(v) If the cholesterol levels in the above table were recorded in micrograms per litre
of blood instead, state whether your answer in (ii) will be different.
[1]
(vi) The manager wishes to estimate the cholesterol level for an employee who is
22 years old. Comment on the reliability of the estimate.
[1]
~End of Paper~
Anglo-Chinese Junior College
H2 Mathematics 9740: 2013 JC 2 Preliminary Examination Paper 2
Page 6 of 6

Anglo-Chinese Junior College


2013 Preliminary Examination H2 Mathematics 9740
Paper 2
Qn
1(i)

(ii)

(iii)

2013 H2 Mathematics Paper 2 Solution

(a) the length of projection of OA onto OB .


(b) the perpendicular distance from the point A to the line OB.
OR
the area of parallelogram with adjacent sides OA and OB.
2a b
OL
3
1 2a + b 1
1
Area of triangle OAL = a ?
a +a a
=
2 3 6
6
1
1
LM a b
6
3
1
1
Since LM AB, a b b a 0
3
6
a.b a.a b.b b.a

0
6
6
3
3
a.b 22 1 b.a


0 a.b 2 k 2
6
6 3 3

2(i)

(ii)

i
i
i
2 ai 2 ai 0
2
2
2
1 a
i ai 0
8 8
a 1
METHOD 1
2 z 4 iz 3 2 z i 2 z i z 3 Az 2 Bz 1

By comparing coef of z 3 , i 2A i A 0
By comparing coef of z 2 , 0 2B i A B 0

2 z 4 iz 3 2 z i 2 z i z 3 1
To solve 2 z 4 iz 3 2 z i 0,

2 z i z 3 1 0

i 1 i
z = e 2 or z 3 1= ei
2 2
z =e
ze

2 k
i

3,

3,

, k 0, 1
ei

METHOD 2

2 z 4 iz 3 2 z i 2 z i z 3 1 0

2z i 0
z=

or

i
2

z 1 0
z 1 z z 1 0
z 1 0 or z z 1 0
3

z =-1 or z =

(1) 12 4 1 i 3

2
2

i
i
1 i
z e 2 , e 3 , e 3 , ei
2

(iii)

z1

sub z 0
i

arg(0 z1 ) arg(e 3 )
i

arg(1) arg(e 3 )

2
k
3


y tan x 3x
3

dx
2
2
x 2t
1 2
t
dt
t
dy
2
2
y 2t
1 2
t
dt
t
2
2
dy t 2
dy 2 t
or
2

dx t 2
dx 2 t 2

Cartesian Equation:

3(a)

Tangent parallel to y-axis 2 t2 = 0


Since t < 0, we have t = 2

Alternatively, in Cartesian form,

x 2 2 y 2 2 8

dy
0
dx
dy x 2 t 2 / t
t2 2

2
dx y 2 t 2 / t t 2
Tangent parallel to y-axis y 2 = 0
2
y = 2 2 2t
t
2
t = 2 t = 2 (as t < 0)
2 x 2 2 y 2

3(b)

Distance from (1, 0) to any point on curve = L


L=

2
2

2 t 1 2 t 0
t
t

2
2

1 t 2 t
t
t

dL
From GC,
0 t = 0.9378999 = 0.938 (3 sf)
dt
d2 L
and 2 8.31 0 shortest
dt
4
x y4
t
x y 2t
x y 4 x y 8
=

Or x2 4 x y 2 4 y 8
2

Or x 2 y 2 8 etc.
Translate 2 units in negative x-direction

x y 2 x y 2 8
2

Or x 2 4 x 2 y 2 4 y 8
2

Or x 2 y 2 8 etc.
Stretch with factor 1/2 parallel to y-axis

x 2 y 2 x 2 y 2 8
2

Or x 2 4 x 2 4 y 2 8 y 8
2

Or x 2 2 y 2 8 etc.
4(a)

4 n 1 2
4r r 2

.
3 n 2 3
r 1 r 1 r 2
n

Let P n be the statement


To show P 1 is true:

n 1

LHS of P 1

4
2
.
1 11 2 3

RHS of P 1

42 0 2 2
.
3 1 2 3 3

Since LHS of P 1 = RHS of P 1 , therefore P 1 is true.


Assume that P k is true for some positive integer k, i.e.,

4k 1 k 1 2
4r r 2

is true.

3 k 2 3
r 1 r 1 r 2
k

To show P k 1 is true:
4r r 2
r 1 r 1 r 2

k 1

LHS of P k 1

4k 1 k 1
4r r 2

k 2 k 3
r 1 r 1 r 2
2

4k 1 k 1 2 4k 1 k 1
by assumption


3 k 2 3 k 2 k 3
2

2
4k 1 k 1 k 1 2

k 2 3
k 3 3
2
4k 1 k 1 k 3 3 k 1 2

3 k 3
k 2
3

2
2
4k 1 k 2k 3 3 k 2k 1 2

3 k 3
k 2
3
4k 1 4k 2 8k 2

k 2 3 k 3 3
4 k 2 k 2 2k 2

k 2 3 k 3 3
4k 2 k
2

= RHS of P k 1 .
3 k 3 3

Since P 1 is true and P k is true implies P k 1 is true, by the principle of


Mathematical Induction, P n is true for all positive integer values of n.
Let r 1 k :

k 1 n
4r r 1
4k 1 k 2

r 0 r 2 r 3
k 1 0 k 1 k 2
2

4k 1 k 2
k 1 k 1 k 2
n 1

(b)(i)

1 n 1
4k k 2

4 k 1 k 1 k 2

1 4n 2 n
2

4 3 n 3 3

4n 1 n
1
.
3 n 3 6

yfx

1
x 1

y=0

-1

-1

x = -1
y f x

x=1

x 1

1
y=0

(b)(ii)

1
2

1
2

1
2
0

f ( x ) dx

a
0

x=1

f (x) dx

a
1
1
dx
dx
0
x 1
x -1
a
1
1
dx
dx
0
x 1
x -1
1

2 ln x 1 02 ln x 1 0

2 ln ln1 ln a 1 ln1
2

2ln 2 ln a 1
2ln 2 ln a 1
ln 4 ln a 1

1
4
1
1
a 1 or
4
4
5
3
a
or
4
4
a 1

3
4
Let X be the random variable denoting the number of rotten eggs per tray.
E(X) = 0.25, Var(X) = 0.2475
Let W X1 X 2 X100 .
Since 0 < a < 1, a

W
6(i)

N 25, 24.75 approximated by CLT as n is large.

P W 15 0.0222 3 s.f.
It means that every student has equal chance of being selected to be interviewed and the
selection of every student is independent of each other.

(ii)

Year 1
Year 2

7(i)
(ii)

(iii)

Marina
barrage
12
8

Garden by the
bay
10
9

National
library
5
6

Select the number of students randomly from each strata according to the table shown
above.
10!
Method 1:
Case 1: one group has 1 female chef, the other has 3 female chefs.
C46 C14 60
Case 2: both groups have exactly 2 female chefs.
C36 C24
60
2
Total number of ways = 60 + 60 = 120
Alternative method:
C510
C56 120
2
M F M F M
or
F M F M
5! 4! 2 5760

M F M F M
M F M F

Or

C35 C24 3! 2! 2 2 2 5760


8(i)
(ii)

(i)

3 3
n 3 4
4n 3

n n 1
n n 1 n n 1
3 3

n n 1 9
4n 3
4n 3
n n 1
Let F be the random variable denoting the weight of a falafel cake and let P be the random
variable denoting the weight of a polenta cake. F ~ N(2.5, 0.12) and P ~ N(3.6, 0.22).

F F F3
Let F 1 2
. Then F
3

0.12
N 2.5,
.
3

P F 2.65 0.00469 (3 sf)

Alternatively,

Let W F1 F2 F3. Then W

N 3 2.5, 3 0.12 .

P F 2.65 P W 2.65 3 0.00469 (3 sf)

9 (ii)

Let W be the total weight of n polenta cakes. W

N 3.6n, 0.04n

P W 71 0.132
71 3.6n
1.116986727
0.04n
71 3.6n 1.116986727 0.04n 0
n 4.472106661

or

n 4.410051843 (NA)

n 20
Alternatively,
P W 71 0.132
From GC,
n = 19
n = 20
n = 21
n = 20
9 (iii)

P(W < 71) = 0.999


P(W < 71) = 0.132
P(W < 71) = 2.60 107

Regular Set: R F P ~ N (6.1, 0.05)


Special Set: S F P1 P2 ~ N (9.7, 0.09)
S1 S2 3R

N 1.1, 0.63

P S1 S2 3R 0 0.917 3 sf

10 (i)

Let X be the random variable denoting the number of days out of 60 with fewer than 10
apple strudels sold.
X B 60, 0.05
np 60 0.05 3
X

Po 3 approximately

P X 5 0.916 3 s.f.

Let Y be the random variable denoting the number of days out of 7 with more than 15
apple strudels sold.
Y B 7, 0.5

P Y 5 C57 0.55 0.52 C67 0.56 0.5 C77 0.57

C57 C67 C77 0.57


7

29
1
29
(shown)
2 128
(ii)
(iii)

29
29 29
1
0.0921 (3 s.f.)

128 128 128
Let T be the random variable denoting the number of strudel weeks in the period of 256
weeks.
29

T B 256,

128

np 58 5, nq 198 5
C13

2871

N 58,
approximately
64

P T k 0.9

P T k 0.1
P T k 0.5 0.1 (by continuity correction)
k 0.5 49.417
k 49.917
The maximum value of k is 49.

11

Unbiased estimate of population mean, x

x 86.4 9.6
12

12

1
8.91
2
0.81
x x

n 1
11
Assuming the mass of sugar in one packet follows a normal distribution.
Unbiased estimate of population variance, s 2

(a)(i)

To test H o : 9

with sig. level

Against H1 : 9 at 5% level of significance


Under H o , T

X 9
s
12

t (11)

9.6 9
2.3094
0.9
12
p-value = 0.020671 < 0.05
Reject H o .

Value of test statistic : t

(a)(ii)
(b)(i)
(b)(ii)

There is sufficient evidence at 5% level of significance that the mean mass in a packet
is underestimated.
There is a probability of 0.05 that the test shows that the mean mass in a packet is more
than 9 grams when it is in fact 9 grams.
k = 9.4

OR
2 P( X 8.6)

100
10.2%

2 P( X 9.4)

(to 3 s.f.)

100

12(i)

(i)(a)
(i)(b)
(ii)
(iii)

(iv)
(v)
(vi)

In long term predictions, a quadratic model shows that as one gets older, the cholesterol
level decreases, which is inappropriate.
As x increases, y increases at a decreasing rate. Thus it may be consistent with the model
of the form y = a + b lnx.
r = 0.950508
y = 80.263908 + 75.744677 ln x

Thus a = 80.263908 = 80.3 (3 s.f)


b = 75.744677 = 75.7 (3 s.f)
y = 80.263908 + 75.744677 ln (37)
y = 193
No.
Not reliable since it is not within the data range. (extrapolation)

Você também pode gostar